You are on page 1of 353

Constitutional LAW of INDIA

(Part-II)

(Fundamental Rights, Directive Principles, Fundamental Duties,


Civil Servants, Amendment of Constitution)

[For LL.B Students of various Universities of India; and Civil/Judicial Services


Aspirants]

Dr. ASHOK K. JAIN


LL.M; Ph.D (Delhi)

Ascent Publications
21/29, Shakti Nagar, Delhi-110007
Copyright © 1996 - ASCENT PUBLICATIONS, Delhi. First Edition, 1996 Reprints with
Supplements 2000,2006 & 2008 SECOND EDITION, 2009 [with Supplement 2010]
Reprint 2012

Price : Rs. 150.00

All Rights Reserved No part ofthis work may be copied, reproduced, adapted, abridged or
translated, stored in any computer or transmitted in any form by any means without prior written
permission of the publishers.

Published By Ascent Publications, 21 /29 , Shakti Nagar, Delhi. Printed at G.S. Offset, Delhi.
CONTENTS

CHAPTER 1. FUNDAMENTAL RIGHTS: GENERAL


Historical Background 2
Restrictions or Suspension of Fundamental Rights 2
Classification of Fundamental Rights 3 Fundamental
Rights Compared with Other Rights 4 Waiver of
Fundamental Rights 5 Article 12 [Definition of State] 7
Otlxr Authorities 7
Recent Expansion of the Term ‘State’ by Judiciary 8
BCCI Not a State 12
International Crops Research Institute Not a State 14 Is
Judiciary' included in the word ‘State’? 15 Summary: What is
‘State’ or Not? 15 Article 13 [Laws Inconsistent with Fundamental
Rights] 18 Application and Rules of Interpretation of Article 13 19 No
retrospective effect 19 Doctrine of Severability or Separability 20
Doctrine of Eclipse 22 Art. 13 and the Terms ‘Law’and ‘Laws in Force'
[A rt. 13(3)] 25 Article 31 [Right to Property and Saving of Certain
Laws] 27 Art. 31A and Saving of Laws 27 Art. 31B: Validation of Certain
Acts, etc. (9h Schedule) 28 Art. 31C: SavingofLaws giving effect to Directive
Principles 29 Art. 31D 31
Article 33 [Power of Parliament to Modify F. Rights] 31 Article 34
[Restrictions on Fundamental Rights - Martial Law] 32 Article 35
[Legislation to Give Effect to Part III] 33

CHAPTER 2. RIGHT TO EQUALITY (ARTICLES 14-18)


Article 14 [Equality before Law] 37
Article 14 permits Reasonable Classification but prohibits Class
Legislation 38
Test of reasonable classification 39 A Single
Individual may constitute a Class 41 Special Courts and
Procedural Inequality 42 New Concept of Equality:
Arbitrariness and Art. 14 47 Illustrative Cases on Art. 14
49
Article 15 [Prohibition of Discrimination against Citizens] 53 Provisos
(or Exceptions) to Art. 15(1) and (2) 54 Article 16 [Equality of
Opportunity in Public Employment] 55 Exceptions to Clauses (I) and (2)
56 Scope of Art. 16(1) ana Art. 16(4) 57 Protective Discrimination: A Facet
of Equality 59 Reservations in Employment (The Mandal Case) 60
Creamy Layer 66

(»i)
Constitutional Law o f India- II

Subversion ofMandal Rule and. Apex Court’s Rescue Efforts ('Post-


Mandal Developments) 66 fa) Non-following of
Creamy Layer Rule 66
(b) Reservation in Promotions Permitted:
Recent Amendments to Art. 16 68 fc) Catch-up Rule
Negated 69 (a) Backlog Vacancies -‘Carry Forward Rule’ Diluted 69
Constitutional Validity ofResemition in Promotions (M. Nagaraj Case) 70
Constitutional Validity ofReseroationforOBCs in Educational Institutions
(Asnoka Kumar Thakur Case) 74 No Sub-classification of Scheduled Castes
83 Reservation to Single Isolated Post 83 Reservation in Super Specialties 84
Reserved Category Candidates Competing in Open CaXegjry 84 Reservationfor
Women: Legality 85 Marriage and Status of SC/ST 86 Article 17[Abolition
of Untouchability] 87 Article 18 [Abolition of Titles] 88

CHAPTER 3. RIGHT TO FREEDOM (ARTICLE 19)


Article 19(1) 97
Need for Restrictions on Freedoms 97 [Art. 19(l)(a)] [Freedom of
Speech and Expression] 100 \a]Freedom of Speech and Expression and
Press/Print 102,125-26 Right of the Convict to Erpress Himself and
19(!)(a) 105 \b]Ereedom of Speech and Expression and Electronic Media {Ads,
Films, etc) 106 Commercial Advertisements 107 Compelled Speech
when Permissible 107 Pre-Censorship and Exhibition of Films /08 Right
to Reply 109 Right to Information 110 Trial by Press/ Media 112
[c],Freedom of Speed] and Expression and Demonstration 113 [a\ Freedom of
Speed) and Expression and Furling ofNational Flag 113
[e] Freedom of Speed) and Expression and Voters’ Right to Know 113 [/]
Freedom of Speech and Expression and Contempt ofCourt 114 Other Fun
damental Freedoms 115 An. 19 (l)fb): Freedom to Assemble 115 Art.
19 (l)(c): Freedom of Association 116 Right to Strike 117 Art. 19 (l)(d) &
(e): Freedom of Movement and Residence 118 No R ight to Call or
Enforce BA NDFI 119 Art. 19 (l)(g): Freedom of Profession and Trade
120

CHAPTER 4. PROTECTION IN RESPECT OF CONVICTION FOR OFFENCES (ARTICLE 20)


Article 20 [Protection in Respect of Conviction for Offences] 129 [A]
Art. 20 (I): Ex-Post Facto Law 129 [ZJ] A rt. 20(2): DoubleJeopardy 132
[QArt. 20(3): Protection against Self-Incrimination 134

CHAPTER 5. Protection of Life and Personal Liberty (Article 21)


Article 2 l[Protection of Life and Personal Liberty] 137
Personal Liberty:Meaningand Scope 138
Scope of ‘Personal Liberty’ (Maneka Gandhi Case) 139
Implied Fundamental Rights (Under Art. 21) 144 Right to Dignity 146
Right to Privacy 148 Right to Travel Abroad 150 Right to Food 150 Right
to Marriage 151 Right to Livelihood (Right to Work) 151 Right to Shelter
153 Right to Speedy Trial 154 Right to Legal Aid 155 Right to Clean
Environment 156 Right to Ediication[Art.21A1 158 Right to Medical Aid
and Health 161 Right to Die 133 Right to Know 162

CHAPTER 6. Safeguards against Arbitrary Arrest and Detention (Article 22)


Article 22 [Safeguards against Arbitrary Arrest and Detention] 169
\_A]Rights of Arrested Person under Ordinary Law 169
Arts. 21-22 and Police Atrocities: Custodial Violence 170
\E]PreventiveDetention Laws 173
Constitutional Safeguards 174
(a) Review by Advisory Board 174
ip) Composition and Procedure of Advisory Board 175
(c) Grounds of Detention and Representation 175

CHAPTER 7. RIGHT AGAINST EXPLOITATION (ARTICLES 23-24)


Articles 23-24 [Rights against Exploitation] 183
A rticle 23: Traffic in Human Beings, Begar, Forced Labour 18 3 Article
24:Prohibition on Employment of Children 186

CHAPTER 8. RIGHT TO FREEDOM OF RELIGION (ARTICLES 25-28)


Secularism in India 188
Article 25 [Freedom of Conscience and Free Profession Practice and
Propagation of Religion] 191 Restrictions on Freedom of Religion 192
Article 26 [Freedom to Manage Religious Affairs] 19b Religious
Denominations 1%
XVI
Constitutional Law o f India- II

Case Law: Arts. 25 and 26 199


Appointment of A rchakas (Pujan)/Priest 199 Right to Propagation and
Religious Conversion 205 Right to Excommunicate 206 Article 27
[Freedom Not to Pay T axes for Religious Promotion] 207 Article 28
[Freedom Not to Attend Religious Instructions] 208

CHAPTER 9. RIGHTS TO MINORITIES (ARTICLES 29-30)


Minorities 213
Article 29 [Protection of Interest of Minorities] 214 Article 30 [Right
of Min orities to Establish and Administer Educational Institutions]
216 Relationship Between Art. 29fl) and Art. 30(1) 217 Whether
Minority Institutions nave a Fundamental Right to Affiliation 217
Power of Government to Regulate Minority run-Instituti ons 218
General Comments 228

CHAPTER 10. RIGHT TO CONSTITUTIONAL REMEDIES (ARTICLE 32)


Judicial Review: A Basic Feature 233 Article 32 [Right to
Constitutional Remedies] 234 Importance of Art. 32/226 234 Res judicata
and Art. 32 236 Laches/Delay and Art. 32 237
Locus Standi ana Public Interest Litigation (PIL) 238 Existence of
Alternative Remedy anaArt. 32 242 Nature and Scope of Relief under
Art. 32 242 Writs, Directions or Orders 242 Art. 32(3): Powers of the
Supreme Court and Any Other Court 245 Art. 32(4): Suspension of
Rights Conferred by Art. 32 245
CHAPTER 11. DIRECTIVE PRINCIPLES AND FUNDAMENTAL DUTIES (ARTICLES
36-51)
Articles 36-51[Directive Principles] 246 Classification of tlx Directives
247 bnplementationand Utility of Directives 249 Uniform Civil Code 251
Article 51-A [Fundamental Duties] 252 Relation between Directive
Principles and Fundamental Rights 255

CHAPTER 12. CIVIL SERVANTS (ARTICLES 308-323)


Art. 309: Power to Frame Recruitment Rules 259 Arts. 310-311:
Doctrine of Pleasure & Constitutional Safeguards 262 Doctrine of
Pleasure 262 Art. 311: Constitutional Safeguards to Civil Servants
264 Procedural Requirements of Art. 311(2) 266 Exceptions to Art. 3110:
Exclusion of Inquiry and
Opportunity ofbeingHeard 267
(a) Except ion I: Conviction on a Criminal Charge 267
(b) Exception II: Inquiry not Reasonably Practicable 270
(c) Exception III: Holding of Inquiry not Expedient in
the Interest of Security of State 271 A rt.
311(3): Finality• Clause 275 Opportunity of Hearing at
Punishing Stage [Proviso 1 to Art. 311(2)] 276 Right of
Constitutional Law o f India- II vii

Government Empl oyees to go on Strike 278 All-India


Services (Art. 312) 279 Public Sendee Commissions (Arts.
315-323) 280

CHAPTER 13. AMENDMENT OF THE CONSTITUTION (ARTICLE 368)


Art. 368: Power of Parliament to Amend the Constitution 283 Are
Fundamental Rights Amendable? 284
KeshavanandBbarati's case and Theory of Basic Structure 286 42nd
Amendment and Doctrine ofBasic Structure 288 Power offudicud Review: A
Basic Feature 290 Doctrine of Basic Features as it Stands Today
292,300 Judicial Review of Legislations included in Ninth Schedule
295 Scope of Amending Power of Parliament 298 A rticle368as it Stands
Today 298
University of Madras v Shantha Bai 8
UOI v Asscn. for Democratic Reforms 113,114
UOI v Brij Lai Thakur 83
UOI v L.D. Balam Singh 31
UOI v Madhav 83
UOI v Mohd. Ramzan Khan 276,278 UOI v Motion Pictures Asscn. 107 UOI v
Virpal Singh Chauhan 69 Upendra Baxi v State of U P. 146,239,245 Usha
Mehta v State of Maharashtra 216
V
VS. Kuttan Pillai v Ramakrishnan 136 Vellore Citizen's Welfare Forum v UOI
157 Vicihtra v UOI 293
Vikram Deo Singh Tomar v State of Bihar 146 Vinay Balachandra Joshi v
Registrar General, Supreme Court 122 Virendra Kumar Srivastava v U P.
Rajya Karmchari Kalyan Nigam 11 Virudhanagar Mills v Govt of Madras 236
Vishaka v State of Rajasthan 146
W
Waman Rao v UOI 28,293,295,301
West Virginia Board of Education v Barnette 211
Y
Y. Srinivasa Rao v J. Veeraiah 49 Y. Theclamma v UOI 242 Yusuf v State of
Bombay 54
ZEE TELEFILMS LTD. v UNION OF INDIA 12
Zoroastrian Cooperative Housing Society v District Registrar 17,116
CONSTITUTIONAL LAW - II SUPPLEMENT 2010

‘State’: Art. 12

Leading Case: State of U.P. v Radhey Shyam Rai


[2009 (3) SCALE 754]
In this case, the question was whether the U.P. Ganna Kishan Sansthan, a society
registered under the Societies Registration Act, is a ‘State’ within the meaning of
Art. 12 of the Constitution of India. It was held to be a State as it was substantially
financed and controlled by the Government, managed by the Board of Directors
nominated and removable at the instance of the Government and "carrying on

X
S-2 Constitutional Law- II: Supplement 2010

functions of public interest under its control.


In Pradeep Kumar Biswas case (2002) 5 SCC 1, a Seven-Judge Bench of this
Court laid down the following tests for the purpose of determining the nature of
activities which would make the body come within the definition of ‘State": (i)
Formation of the body; (ii) Objects and functions; (iii) Management and control; (iv)
Financial aid, etc. The court observed that:
“The combination of State aid and the furnishing of an important public
service may result in a conclusion that the operation should be classified as a State
agency. If a given function is of such public importance and so closely related to
governmental functions as to be classified as a governmental agency, then even the
presence or absence of State financial aid might be irrelevant in making a finding of
State action. If the function does not fall within such a description, then mere
addition of State money would not influence the conclusion.
“The ultimate question which is relevant for our purpose is whether such a
corporation is an agency or instrumentality of the Government for carrying on a
business for the benefit of the public. In other words, the question is, for whose
benefit was the corporation carrying on the business?”
The question came up for consideration before a Constitution Bench of this
Court in Zee Telefilms Ltd. v Union of India (2005) 4 SCC 649, wherein the majority
felt itself bound by the dicta laid down in Pradeep Kumar Biswas case, ovvever, the
minority view was as under:
Broadly, there are three different concepts which exist for determining the
questions which fall within ihe expression “Other authorities”:
[S-1]
(i) The corporations and the Societies created by the State for carrying .on its
trading activities in terms of Art 298 of the Constitution whereof the
capital, infrastructure, initial investment and financial aid, provided by
the State and it also exercises regulation and control thereover.
(ii) Bodies created, for research and other developmental works which are
otherwise governmental functions but may or may not be a part of the
sovereign function.
(iii) A private body is allowed to discharge public duty or positive obligation
of public nature and furthermore is allowed to perform regulatory and
controlling functions and activities which were otherwise the job of the
Government.
The minority in Zee case also opined that “the concept that all public sector
undertakings incorporated under the Companies Act or the Societies Registration Act
or any other Act for answering the description of the State must be financed by the
Central Government and be under its de:p and pervasive control has in the past three
decades undergone a sea change. The thrust now is not upon the composition of the
body but the duties and functions performed by it. The primary question which is
required to be posed is whether the body in question exercises public function.
Tests evolved by the courts have, thus, been expanded from time to time and
Constitutional Law- II. Supplement 2010 S-3
applied having regard to the factual matrix obtaining in each case. Development in
this branch of law as in others have always found differences. Development of law
had never been an easy task and probably would never be.”
In the present case, the court observed: F or the purpose of determining the
question as to whether a society registered under the Societies Registration Act
would be a ‘State" or not, the history of its constitution plays an important role. The
functions which are being performed by the Sansthan were used to be performed by
the Government directly. The main purpose and object for which the training
institutes were established at different places in the State of U.R (by the Cane
Development Department of the Government) admittedly was to provide scientific
ways of sugarcane cultivation and management so as to improve the production of
cane with a view to achieve better production of sugar. Such a function indisputably
is a State function. The Stale established the 'Sansthan’ so as to take over its own
functions. It even transferred the entire management relating to imparting of training
in various institutes in its favour. All the assets held by it for the aforementioned
purpose including the infrastructure facilities stood transferred in favour of the
Sansthan. It was created by a Government Order. A budget of Rs. 6 lakhs was
sanctioned of which 50% was made by the Government and the remaining 50% by
the Mills run by the State Sugar Corporation, Indian Mill Association, U.R Sugarcane
Cooperative Federation and Cane Development Societies.
80 to 90% of the expenditure of Sansthan was met out of the funds made
available to it by the Government. The majority of the office bearers of the
Governing Council were holders of various offices in the Government. The Cane
Commissioner, Director and the Accounts Officer are the government servants and
the Sansthan is not free to appoint anybody on those posts who is not a government
servant. This itself clearly shows that the composition and constitution of Sansthan and
its Governing Council was nothing but a show of the Government and only a cover
of the Society was given. Rule 41 of the Rules of Sansthan provides that the
Governor shall have power to issue any directives to the Sansthan concerning any
matter of public importance and the Sansthan shall give immediate effect to the
directives so issued. The functions of the Sansthan are thus public functions.
The court concluded. There cannot be any doubt whatsoever that the State
exercises a deep and peivasive control over the affairs of the Sansthan, the Cane
Commissioner being at the helm of the affairs. Thus, the ‘Sansthan’ is a ‘State’
within the meaning of Art. 12 of the Constitution of India.
[Note: In this case, Chandra Mohan Khanna v NCERT(1991) 4 SCC 578, held
overruled on this point.]

Right to Equality (Protective Discrimination): Arts. 15-16

Leading Case: Dr. Guishan Prakash v State of Haryana


[2009 (14) SCALE 290]
Challenge in this appeal is to the judgment dated 05.02.2008 of the High Court of
S-4 Constitutional Law- II: Supplement 2010

Punjab & Haryana, dismissing the Civil Writ Petition No. 1431 of2008, filed by the
appellants herein, for quashing of the prospectus for the MD/MS/PG Diploma and
MDS Courses issued by Maharshi Dayanand University, Rohtak, Haryana for
Academic Session 2007-2008 to the extent that it does not provide any reservation of
seats for Scheduled Caste/ Schedules Tribe candidates.
According to the appellants, on 17.09.2005, all the inotitutions including All-
India Institute of Medical Sciences provided reservation in the Post-Graduate
Courses for the members of Scheduled Castes and Scheduled Tribes. The
Government Medical College, Patiala, Amritsar and Faridkot also provided
reservation in the Post-Graduate courses for the members of the SCs/STs for the
Academic session 2007. The University of Delhi is also providing reservation to the
members of the SCs/STs.'
On the other hand, counsel for the respondents submitted that Art. 15(4) *s
only an enabling provision and the State of Haryana, taking noie of various aspects,
decided not to provide reservation for the SCs and STs and OBCs in the Post-
Graduate Courses. They also pointed out that there cannot be any Mandamus
compelling the State to provide reservation for a particular class of Persons.
The court observed: The consistent view of this Court is thai Art. 15(4) is only
an enabling provision and it is for the respective States either to enact a legislation or
issue an executive instruction providing reservation in Post- Graduate Courses. In
tncb aSawhney case [(1992) Supp (3) SCC 217], the majority held:
“The aspect next to be considered is whether clause (4) is exhaustive of the
very concept of reservations? In other words, the question is whether any
reservations can be provided outside clause (4) i.e. under clause (!) of Art. 16. There
are two views on this aspect. On a fuller consideration of the matter, we are of the
opinion that clause (4) is not, and cannot be held to be, exhaustive of the concept of
reservations; it is exhaustive of reservations in favour of backward classes alone.
Merely because, one form of classification is stated as a specific clause, it does not
follow that the very concept and power of classification implicit in clause (1) is
exhausted thereby. To say so would not oe correct in principle. But, at the same time,
one thing is clear. It is in very exceptional cases that any further reservations of
whatever kind, should be provided under clause
(1) . In such cases, the State has to satisfy that making such a provision was
necessary (in public interest) to redress a specific situation. The very presence of
clause (4) should act as a damper upon the propensity to create further classes
deserving special treatment. The reason for saying so is very simple. If reservations
are made both under clause (4) as well as under clause (1), the vacancies avatiable
for free competition as well as reserved categories would be correspondingly
whittled down and that is not a reasonable thing to do.
In K. Duraisamy v State ofT.N. [(2001) 2 SCC 538), the court, while dealing
with the reservation at the Post-Graduate level and super-specialty level, observed as
follows: “That the Government possesses the right and authority to decide from what
Constitutional Law- II. Supplement 2010 S-5
sources the admissions in educational institutions or to particular disciplines and
courses therein have to be made and that too in what proportion, is well established
and by now a proposition well settled, too. It has been the consistent and
authoritatively-settled view of this Court that at the super-specialty level, in
particular, and even at the postgraduate level reservations of the kind known as
“protective discrimination” in favour of those considered to be backward should be
avoided as being not permissible. Reservation, even if it be claimed to be so in this
case, for and in favour of the in- service candidates, cannot be equated or treated on
par with communal reservations envisaged under Arts. 15(4) or 16(4) and extended
the special mechanics for their implementation to ensure such reservations to be the
minimum by not counting those selected in open competition on the basis of their
own merit as against the quota reserved on communal considerations.”
In AflMSStudent’s Union v AlIMS [(2002) 1 SCC 428], while considering the
similar issue, it was held: “When protective discrimination for promotion of
equalization is pleaded, the burden is on the party who seeks to justify the ex
facie deviation from equality. The basic rule is equality of opportunity for even'
person in the country, which is a constitutional guarantee. A candidate who gets
more marks than anotlier is entitled to preference for admission. Merit must be the
test when choosing the best, according to this rule of equal chance for equal
marks. This proposition has greater importance when we reach the higher levels
and education like postgraduate courses. Reservation, as an exception, may be
justified subject to discharging the burden of proving justification in favour of the
class which must be educationally handicapped - the reservation geared up to
getting over the handicap. The rationale of reservation in the case of medical
students must be removal of regional or class inadequacy or like disadvantage.
Even there the quantum of reservation should not be excessive or societally
injurious. The higher the level of ihe specialty the lesser the role of reservation.”
Again it was held that; “... Permissible reservation at the lowest or primary
rung is a step in the direction of assimilating the lesser fortunates in the
mainstream of society by bringing them to the level of others which they cannot
achieve unless protectively pushed. Once that is done the protection needs to be
withdrawn in the own interest of protectees so that they develop strength and feel
confident of stepping on higher rungs on their own legs shedding the crutches.
Any reservation, apart from being sustainable or, the constitutional anvil, must
also be reasonable to be permissible. In assessing the reasonability, one of the
factors to be taken into consideration would be whether the character and quantum
of reservation would stall or accelerate achieving the ultimate goal of excellence
enabling the nation constantly rising to higher levels. In the era of globalization,
where the nation as a whole has to compete with other nations of the world so as
to survive, excellence cannot be given an unreasonable go-by and certainly not
compromised in its entirety ..."
In the present case, the court observed: The principle behind Art. 15(4) is
S-6 Constitutional Law- II: Supplement 2010

that a preferential treatment can be given validly when the socially and
educationally backward classes need it. This article enables the State Government
to make provisions for upliftment of SCs and STs including reservation of seats
for admission to educational institutions. It was also held that Art. 15(4) is not an
exception but only makes a special application of the principle of reasonable
classification. Art. 15(4) does not make any mandatory provision for reservation
and the power to make reservation under Art. 15(4) is discretionary and no writ
can be issued to effect reservation. Such special provision may be made not only
by the Legislature but also by the Executive.
Learned counsel for the appellants relying on the Constitution Bench
decision of this Court in Dr. Preeti Srivasta\’a v State ofM. P. [(1999)7SCC 120],
submitted that when it is permissible to prescribe a lower minimum percentage °f
qualifying marks for the reserved category candidates, as compared to the general
category’ candidates, it is incumbent on the pari of the State Government
to prescribe certain percentage for SC/ST candidates even for the Post-Graduate
Courses. On going through the decision, we are unable to accept the said contention.
After discussing relevant aspects and earlier decisions this Court concluded:-
1. We have not examined the question whether reservations are permissible at
the postgraduate level of medical education.
2 A common entrance examination envisaged under the regulations framed by
the Medical Council of India for postgraduate medical education requires
fixing of minimum qualifying marks for passing the examination since it is
not a mere screening test.
3. Whether lower minimum qualifying marks for the resei ved category
candidates can be prescribed at the postgraduate level of medical education
is a question which must be decided by the Medical Council of India since
it affects the standards of postgraduate medical education. Even if
minimum qualifying marks can be lowered for the reserved category
candidates, there cannot be a wide disparity between the minimum
qualifying marks for the reserved category candidates and those for the
general category candidates at this level. Then percentage of 20% for the
reserved category and 45% for the general category is not permissible
under Art. 15(4), the same being unreasonable at the postgraduate level and
contrary to the public interest.
4. At the level of admission to the super specialty courses, no special
provisions are permissible, they being contrary to the national interest.
Merit alone can be the basis of selection.
Learned Counsel for the appellants next contended that, inasmuch as even in All-
India Entrance Examination for Post-Graduate Courses, the Government of India
itself has made a provision for reservation for SC/ST candidates, the State of Hary
ana is bound to follow the same and issue appropr iate orders/directions providing
reservations in the Post-Graduate Courses,
Constitutional Law- II. Supplement 2010 S-7
The court observed that this contention is also liable to be rejected. It is true
that Government of India itself has made a provision for reservation of SC/ ST
categories. This was a decision by the Government oflndia and it is applicable in
respect ofAll-lndia Enuancc Examination for MD/MS/PG Diploma and MDS
Courses, and reservation for SC/ST candidates in All-India quota for PG seats.
However, the same cannot automatically be applied in other selections where State
Governments have power to regulate. In fact, the Government of Haryana explained
their position that according to them, the matter regarding reservation of seats in the
PG courses has been considered by the State Government from time to time and it
has been decided that keeping in view the recommendations of the Medical Council
of India ( MCI) and precedents in the other States, reservation of SC/ST in PG
courses in neither feasible nor warranted, as there «s already a reservation of 50% in
the total seats in MD/MS/PG Diploma and MDS Course in the institutions of
Haryana on All-India basis entrance examination, being conducted by AIIMS, New
Delhi, and that the appellants had already availed the benefit of reservation of seats
in their qualifying examination of MBBS/BDS. They further clarified that only the
State Government is the competent authority to decide the reservation in the State.
The State Government is in a better position to determine the situation and
requirement of that particular State, as mandated by the Constitution
The directions of this Court are applicable to admission on All-India basis
whereas the same have no bearing on the admissions meant for State quota. The
clarificatory order of this Court in Abhay Nath v University of Delhi [Writ Petition
(C) No. 138 of2006] is applicable for the Institutes managed/run by the Central
Government. As the State Government is competent to make the reservation to a
particular class or category, until it is decided by the State, as being a policy matter,
there cannot be any direction to provide reservation at the PG level. After ail,
medical education is an important issue which should not have any mandatory
condition of this nature which may give rise to a situation against public interest.
The court concluded: Art. 15(4) is an enabling provision and the State
government is the best judge to grant reservation for SC/ST/Backward Class
categories at Post-Graduate level in admission and the decision of the State of
Haryana not to make any provision for reservation at that level suffers no infirmity.
Every State can take its own decision with regard to reservation depending on
various factors. In such circumstances. Courts cannot issue mandamus against the
decision of a State ]
In Nair Service Society v Dr. T. Beermasthan (2009) 5 SCC 545, it was held:
Reservation provisions are enabling provisions. The State is not bound to make a
reservation but it is empowered to do so in its own discretion. Different State
Governments in the country may have different methods for providing reservations,
and these will be valid as long as the method employed by a particular State
Government does not violate any constitutional provision or statute. It is not for the
Court to decide on the wisdom or otherwise of the method of reservation. Courts
S-8 Constitutional Law- II: Supplement 2010

should exercise judicial restraint and not interfere "ith the same unless there is some
ciear illegality.
*n Avinash Singh Bagr\ v /IT, Delhi (2009) 8 SCC 220, there was expulsion
rhe ° * ‘ ' certain SC/ST candidates for poor performance, for Cant^ates’
CanCellati n 0t ac m ss on

grievance was regarding absence of “slow track programme” ex^StUc*c"ts failing to


achieve required credits or to take up summer courses/ by 1 nnat,0ns 'n ^ ^ Delhi. The
reserved category students alleged discrimination by Delhi against them. The court
held that SCs and STs are a separate class '*ght :TrVeS anc* 'creamy layer’ principle is
not applicable to them. In the
0 rt- 46, these socially and economically backward categories are to be taken
care of at every stage even in the specialized institutions like IITs. They must make
all endeavour hy providing additional coaching and bring them on a par with gerfcral
category students.
In the present case, .there was no plausible explanation for not permitting such
students to avail summer course. The additional/extra support was not fully afforded
to these students to compete with general category students and secure minimum
required credits in the first and second year. Ihe ends of justice would be fully met by
giving them one more opportunity
In Subhash Chandra v Delhi Subordinate Services Selection Board (2009) 15
SCC 458, it was held: There is a distinction between Aft. 15(4) and Art. 16(4) of the
Constitution. The words “Backward Classes” and “Scheduled Castes and Scheduled
Tribes” find place in Art. 15(4) but only the words “backward class of citizens” find
place in Art. 16(4). The term “backward class of citizens” contained in Art. 16(4)
includes Scheduled Castes/Tribes for all intent and purport. Therefore, the protection
sought to be accorded to a section of the citizenry must not only be to Backward
Class but may also be to Scheduled Castes/Tribes for whom a special provision can
be made. Indisputably, the classes contemplated by Art. 16(4) may be wider than
those contemplated by Art. 15(4). If they are Backward Classes for the purpose of
Arts. 16(1) and (4) and not Scheduled Castes/Tribes, they will come within Ihe
purview of the reservation for Backward Classes and not the one which is exclusively
meant for Scheduled Castes/Tribes within the purview of reservation policy of the
State.
Fundamental Rights: General

Fundamental rights are rights having a noble pedigree. They are natural rights
which are in the nature of external conditions necessary for the greatest possible
unfolding of the capacities of a human being. These secured and guaranteed
conditions are called fundamental rights, it is generally agreed that these natural
I
rights are inherent in man and cannot be taken away by the State. Natural rights
command higher sanctity than other rights e.g. rights based on contract because
they exist independent of any Act. 1
Part III of the Constitution, which contains fundamental rights, has been
described as the Magna Carla of India. These fundamental rights substantially
cover all the traditional civil and political rights enumerated in the Universal
Declaration of Human Rights. Dr Ambedkar described them as “the most
criticized part” of the Constitution. Gajendragadkar, J. described them as the
"veiy foundation and cornerstone of the democratic way of life ushered in this
country by the Constitution”.
Fundamental rights were deemed essential to protect the rights and
liberties of the people against the encroachment of the power delegated by them
to their government. They are limitations upon all the powers of the government.
In Maneka Gandhi case, Bhagwati, J. observed: “These fundamental right
represent the basic values cherished by the people of this country since the
Vedic times and they are calculated to protect the dignity of the individual and
create conditions in which every human being can develop his personality to the
fullest exlent. They weave a ‘Pattern of guarantee’ on the basic structure of
human rights, and impose negative obligations on the State not to encroach on
individual liberty in its various dimensions.”
These rights are regarded as fundamental because they are most essential
for the attainment by the individual of his ful! intellectual, moral and spiritual
status. The object behind the inclusion of them in the Constitution is to establish
'a government °f law and not of man’. The object is to establish rile of law.

B.K. Sharma. introduction to toe Con.:‘itut( n if'ndia. PH!, p 59 (2007)


m
Historical Background
In England it was regarded that the Parliament suffers no limitations. There are no
restraints on its legislative powers The law made by Parliament cannot be
annulled on the ground that it violates a liberty. It does not take away a liberty not
because it does not have the power to do so but because of its long tradition of
being a champion of liberty. But things have considerably changed after the U.K.
has entered the European Community and submitted to the jurisdiction of
European Human Rights Commission In the year 1988 the Parliament in U K
enacted a Human Rights Act.
The treatment meted out by the British to the Indians was no different from
that received by the Americans. The American people fought against the British
Parliament so when they gave themselves a Constitution they built some restraints
in it so that the legislature may not become tyrannical. They made the Constitution
%
Fundamental Rights: General 3
paramount. The Declaration of American Independence drafted by Jefferson
(1786) proclaimed- "We hold these truths to be self-evident, that all men are
created equal; that they are endowed by their creator with certain inalienable
rights, that among these are life, liberty and pursuit of happiness ...” The
Constitution of U.S.A as originally framed in 1787 and brought into force in 1789
did not contain the Bill of Rights and thus lacked guarantee of inalienable rights
But soon thereafter in 1791, two years after the Constitution came into force, the
first ten amendments to the Constitution were adopted. These are called the Bill of
Rights
Our experience with British rule was painful because basic rights were
dependant on the whims of the rulers Hence our Constitution secures to the
people certain basic rights which cannot be trarnplled by the State In this respect
we have followea the American Constitution. The Constituent Assembly was
determined to safeguard certain rights and incorporated them in Part III of the
Constitution. 1

Restrictions or Suspension of Fundamental Rights


Absolute and unrestricted individual rights do not, and cannot exist in any modern
State. The Constitution permits 'reasonable' restrictions to be imposed on
individual's liberties in the interest of society. 2 In A K. Gopalan v State of Madras
{AIR 1950 SC 27), Mukherjee, J , observed “There cannot be any such thing as
absolute and uncontrolled liberty wholly freed from restraint, for that would lead to
anarchy and disorder. The possession and enjoyment of all rights are subject to
such reasonable conditions as may be deemed by the governing authority
essential to the safety, health, peace, general order and morals of the community In
fact, for the very protection of these rights the society must arm itself with certain
powers. What the Constitution therefore attempts to do by declaring the rights of the
people is to strike a balance between individual liberty and social control ’’
The courts normally would respect the legislative policy behind a valid
which imposes reasonable restrictions in the interest of the State
legislation
[P.U.C.L v Union of India (2004) 2 SCC 476}.
The Constitution also provides for the suspension of fundamental rights in
certain circumstances The U .S. and the Australian Constitution have no provision
for suspension of the fundamental rights Article 358 provides that when the
proclamation of emergency is made by the President under Art. 352, the
freedoms guaranteed by Art. 19 are automatically suspended for the period of
emergency. Art. 359 further empowers the President to suspend the right to move
any court for the enforcement of fundamental rights (except Arts. 20 and 21)
during the continuance of emergency. So even during the period a proclamation
of emergency is in operation a person has the right to seek protection of his life
and personal liberty.
Article 33 empowers Parliament to modify the application of Fundamental

2 B.K. Sharma, pp. 60-61


2 Generally fundamental rights are circumscribed ' y I'initations even though it
may not be so worded The restrictions may be express, as in Art. 19(2)-(6),
Arts 16, 23, 25 and 26 There are certain rights on which there are no
express limitations e.g. Arts 14,17,18, 20 ano 24. In U S A. and other
Constitutions the Courts had to invent limitations such as police power,
eminent domain, grave and imminent danger to limit the scope of the rights.
4 Constitutional Law o f India- II

Rights to the Armed Forces or forces charged with maintenance of public order,
etc. in the interest of discharge of duties and maintenance of discipline. Under Art.
34, Parliament may by law indemnify any person for anything done in
contravention of fundamenteKrights for maintenance of order during the operation
of martial law.

Classification of Fundamental Rights


The fundamental rights under the Constitution can be classified under the
following
six groups:
(a) Right to equality (Arts 14-18).
(b) Right to freedom (Arts. 19-22).
(c) Right against exploitation (Arts. 23-24).
(d) Right to freedom of religion (Arts. 25-28).
(e) Cultural and educational rights (Arts. 29-30).
(f) Right to Constitutional remedies (Arts. 32-35).
[Note: The 44th Amendment has abolished the ‘right to property’ as a
fundamental ^9ht guaranteed by Art. 19(1)(f) and Art. 31 of the Constitution, and
hence these rticles have been omitted. It is now only a legal right ]
fundamental rights available against State and not against private individuals - n
ividual needs constitutional protection against the State. The rights which are a'Ven
<o the citizens by way of fundamental rights are a guarantee against State as
distinguished from violation of such rights from private parties. Private Cll°n is
sufficiently protected by the ordinary law of land If the freedom of speech
given by Art. 19(1) is taken away by State action or if a person is deprived of his
personal liberty an action would lie against the State
Fundamental rights: self-executory or require legislation - Certain rights need no
legislation to make them enforceable e.g. Arts. 14, 19, and, 21, etc But there are
certain rights in Part III which are imperfect and need further legislation to make
them enforceable Such rights are Art 17 (untouchability); Art 21A (education); Art
23 (traffic in human beings; and, Art. 24 (child labour)
Fundamental rights: negative or positive - Some rights are in the form of
prohibitions; they are a mandate to the State not to do a particular type of act
(negative in form). For example, Art. 14 reads "The State shall not deny to any
person equality before the law or the equal protection of the laws.” Articles 15(1),
16(2), 18(1), 20, 21, 22(1), 27, and 28(1) are similarly worded. Some of the
articles expressly create and confer one or more rights. Art. 19(1) reads; “All
citizens shall have the right-
(a) to freedom of speech and expression.” Art. 25(1) says, ”... all persons are
equally entitled to freedom of conscience ..."Articles 21 A, 29, 30 and 32 are
other examples of positively worded rights
Fundamental rights: available to citizens or all persons - Some r ights are
available only to citizens. Such rights are-Arts 15, 16, 19, 29, and, 30 Some rights
4 . B . K Sharma, pp. 65-66
are available to all persons in India, citizens and foreigners alike (except enemy
aliens). Such rights are-Arts. 14, 20, 21, 21A, 23, 24, and, 25-28 4

Fundamental Rights Compared with Other Rights


Ordinary legal right e.g. right of a consumer, shareholder, mortgagor, etc. is
%
Fundamental Rights: General 5
protected and enforced by the ordinary law of the land. A fundamental right is
protected and enforced by the Constitution. Ordinary rights can be changed by
the common legislative process. A fundamental right can be altered only by a
constitutional amendment. A fundamental right can be suspended or abridged
only in the manner prescribed by the Constitution
An ordinary right generally imposes a corresponding duty on another
individual (and, State in some cases) but a fundamental right is a right which an
individual possesses against the State
Fundamental rights are protected against invasion by the executive,
legislature and the judiciary. All fundamental rights are limitations on legislative
power Laws and executive actions which abridge or are in conflict with such
rights are void and ineffective. Our Constitution guarantees the right to move the
Supreme Court for the enforcement of fundamental rights. Thus the remedy itself
is a fundamental right. ' This distinguishes it from other rights. The Supreme
Court is the guardian of fundamental rights.
Fundamental Rights: General 6

All constitutional rights are not fundamental rights e g. right not to be


subjected to taxation without authority of law (Art. 265), Right to property (Art.
300A), Freedom of trade (Art. 301).5
A fundamental right cannot be waived in India (see below). An ordinary legal
right can be waived by an individual.

Waiver of Fundamental Rights


Fundamental rights occupy an important place in the scheme of basic human rights.
A question arises: Can a person waive (i.e. voluntarily relinquish or abandon) any
of his Fundamental Rights? In U.S.A., a Fundamental Right can be waived [Boykin
v Alabama, 395 U.S. 238 (1969)].
In Behram v State of Maharashtra (AIR 1955 SC 123), the Fundamental
Rights were divided into two broad categories: (i) Rights conferring benefits on the
individuals, and (ii) those rights conferring benefits on the general public. The court
repudiated the doctiine of waiver saying that the Fundamental Rights were not put
in the Constitution merely for individual benefit. These Rights were there as a
matter of public policy and therefore, the doctrine of waiver could have no
application in case of Fundamental Rights. A citizen cannot invite discrimination by
telling the State 'You can discriminate’, or get convicted by waiving the protection
given to him under Arts. 20 and 21.
The question of waiver of a Fundamental Right has been discussed more
fully by the Supreme Court in Basheshar Nath v C.I.T. (AIR 1959 SC 149). After
the Commission had decided upon the amount of concealed income under Sec.
5(1) of the relevant Act, the petitioner on May 19, 1954, agreed as a settlement to
pay in monthly instalments over Rs. 3 lacs by way of tax and penalty. In 1955, the
Supreme Court declared Sec. 5(1) ultra vires Art. 14. The petitioner thereupon
challenged the settlement between him and the Commission, but the plea of
waiver was raised against him The Supreme Court however upheld his contention.
In their judgments, the learned Judges expounded several views regarding
waiver of Fundamental Rights, viz..
(1) Art. 14 cannot be waived for it is an admonition to the State as a matter
of public policy with a view to implement its object of ensuring equality.
No person can, therefore, by any act or conduct, relieve the State of the
solemn obligation imposed on it by the Constitution.
(2) A view, somewhat broader than the first, was that none of the
Fundamental Rights can be waived by a person. The Fundamental
Rights are mandatory on the State and no citizen can by his act or
conduct, relieve the State of the solemn obligation imposed on it.
The Constitution makes no distinction between Fundamental Rights enacted for
the er>efit of an individual and those enacted in public interest or on grounds of
public

5 . B.K. Sharma, pp. 60-61.

/
policy (such distinction has been attempted in USA) A large majority of the people
in India are economically poor, educationally backward and politically not yet
conscious of their rights. Individually or even collectively, they cannot be pitted
Fundamental Rights: General 7

against the State, and therefore, it is the duty of the judiciary to protect their Rights
against themselves
(3) An individual could waive a Fundamental Right which was for his benefit,
but he could not waive a Right which was for the benefit of the general
public.
In view of the majority decision in Basheshar, it is now an established proposition
that an individual cannot waive any of his Fundamental Rights This proposition
has been applied in a number of cases. "The State cannot arrogate to itself a right
to commit breach of the Fundamental Rights of any person by resorting to
principles of waiver or estoppel or other similar principles” [Yousuf AH v M.S.
Kasbekar AIR 1982 Bom 135]. Similarly, the Gauhati High Court has explained
that the Fundamental Rights have been embodied in the Constitution not merely
for the benefit of a particular individual but also as a matter of constitutional policy
and for public good, and, therefore, the doctrine of waiver or acquiescence cannot
be applied thereto (Omega Advertising Agency v State Electricity Board MR 1982
Gau 37).
In Olga Tellis case (AIR 1986 SC 180), the Apex Court asserted that the
high purpose which ‘the Constitution seeks to achieve by conferment of
fundamental rights is not only to benefit the individual but to secure the larger
interests of the community.' Therefore, even if a person says either under mistake
of law or otherwise, that he would not enforce any particular Fundamental Right, it
cannot create an estoppel against him. Such a concession, if enforced, would
defeat the purpose of the Constitution. Were the argument of estoppel valid, an all-
powerful State could easily tempt an individual to forgo his precious personal
freedoms on promise of transitory, immediate benefits In this case, the pavement
dwellers gave an undertaking that they would not claim any Fundamental Right to
put up huts on pavements or public roads and that they would not obstruct the
demolition of the huts after a certain date. Later, when the huts were sought to be
demolished after the specified date, the pavement dwellers put up the plea that
they were protected by Art. 21. It was argued in the Supreme Court that they could
not raise any such plea in view of their previous undertaking. The court overruled
the objection saying that Fundamental Rights could not be waived. There can be
no estoppel against the Constitution which is the paramount law of the land. The
Court observed: “No individual can barter away the freedoms conferred on him by
the Constitution.”
Recently, in Nar Singh Pal v Union of India (AIR 2000 SC 1401), the
Supreme Court has asserted: "Fundamental Rights under the Constitution cannot
be bartered away. They cannot be compromised nor there do any estoppel against
the exercise of Fundamental right available under the Constitution.” The doctrine
of non-waiver developed by the Supreme Court of India denotes manifestation of
its role of protector of the Fundamental Rights. 3
ARTICLE 12 [DEFINITION OF STATE]

Unless the context otherwise requires, the term State' include the following -
executive and legislature of Union and States, all local or other authorities within
territory of india or under the control of government of India."
The definition is not exhaustive but inclusive which means that apart from
those organs or bodies which have been enumerated others may also be covered

3 M.P. Jain, Constitutional Law of India pp 852-854 (2003).


XVI
Constitutional Law o f India- II
by the expression State.'Art. 12, thus, is an interpretative article. The expression
'other authorities’ has been interpreted by the courts, as it has not been defined in
the Constitution or in any statute. Generally, a liberal or extended interpretation
has been adopted by the courts.
It may be noted that this extended interpretation of the term ‘State’ is limited
in its application only to Part III (Fundamental Rights) and Part IV (Directive
Principles) and it does not extend to the other provisions of the Constitution e.g.
Arts. 309-311 (Part XIV) (Sukhdev Singh’s case, below).

(a) Executive and Legislature of Union and States


It would obviously include: Union and State Governments, and, Parliament and
State Legislatures The acting President of India and Governors of States form part
of the Executive The term 'Government' includes a Department of Government or
any institution under the control of a Government Department e g. the I T or Excise
Department; the Forest Researcn Institute, Dehradun; etc.

(b) Authorities
According to Webster’s dictionary, 'authority' means a person or body exercising
power or command. In the context of Art. 12, 'authority' means the power to make
laws, orders, regulations, bye-laws, etc., which have the force of law and power to
enforce those laws. For instance, the bye-laws made by a Municipal Committee.

(c) Local Authorities


The expression as defined in Sec 3 of the General Clauses Act refers to authorities
like Municipalities, District Boards, Panchayats, etc.

(d) Other Authorities4


The expression other authorities' in Art. 12 is used after mentioning the executive
and •egislature of Union and States, and all local authorities. Thus, it was held that
it could
only indicate authorities of a like nature i.e ejusdem generis. So construed, it could
only mean authorities exercising governmental or sovereign functions It cannot
include persons, natural or juristic e g a university unless it is 'maintained by the
State’ (University of Madras v Shantha Bai AIR 1954 Mad. 67). But, later it was
held that ejusdem generis rule could not be resorted to in interpreting this
expression, as there is no common genus running through these named bodies (in
Art. 12), nor can these bodies so placed in one single category on any rational
basis.
In Electricity Board, Rajasthan v Mohan Lai (AIR 1967 SC 1857) and
Sukhdev Singh v Bhagatram (AIR 1975 SC 1331), a very restrictive interpretation
of the expression ‘other authorities’ given by court - oniy the authorities created by
the Constitution or Statute are the ‘other authorities’, though it is not necessary
that statutory authoiity should be engaged in performing governmental or
sovereign functions Thus, Rajasthan Electricity Board, Oil & Natural Gas
Commission (ONGC), Life Insurance Corporation, Industrial Finance Corporations,

4 Write a short note on: Meaning of the term ‘Other Authorities’ under Art. 12
of the Constitution. [I.A.S.-2007\
Fundamental Rights: General 9

etc. are held to be 'other authorities'. These bodies have power to make bye-laws,
regulations, etc. for regulating conditions of services of their employees. The
employees are entitled to claim protection of Arts. 14 and 16 against these bodies.
The ONGC’ is a statutory body owned and managed by the Central
Government The structure of the ‘Life Insurance Corporation’ indicates that the
Corporation is an agency of the government carrying on the exclusive business of
life insurance. Each and every provision shows in no uncertain terms that the
voice is that of the Central Government and the hands are also of the Central
Government The provisions of the ‘Industrial Finance Corporation’ Act show that
the Corporation is in effect managed and controlled by the Central Government.
The circumstance that the ‘Rajasthan Electricity Board’ under the Electricity
Supply Act, is required to carry on some activities of the nature of trade or
commerce does not give any indication that the Board must be excluded from the
scope of the word “State."
In subsequent decisions, the Supreme Court has given a broad and liberal
interpretation to the expression ‘other authorities,’

Recent Expansion of the Term ‘State’ by Judiciary

Leading Case: som prakash rekhi v union of india (AIR 1981 SC 212)

In this case, the petitioner (Som FYakash) was a clerk in the Burmah
Shell Oil Storage Ltd. and retired after qualifying for a pension in 1973.
The undertaking of the Burmah Shell was taken over under the Burmah
Shell (Acquisition of Undertakings in India), 1976 and was vested in the
Bharat Petroleum Corporation Ltd., which became the statutory
successor of the petitioner’s employer. The petitioner challenged by a
petition under Art. 32, the deductions from his pension, as illegal and
inhuman and in the violation of his right to property, then, contained in
Art. 19( 1 )(f) of the Constitution,
Applying the test laid down in Airport Authority's case, the Supreme
Court held that the Bharat Petroleum Corporation, though a Government Company
registered under the Companies Act, by virtue of the various provisions of the
1976 Act, was transformed into an instrumentality of the Central Government with
a strong statutory flavour super added and clear indicia of power to make it an
"authority’' falling under the expression 'State' in Art. 12. The expression ‘other
authorities” in Art. 12 would include not only a statutory body but also a non-
statutory body like a Government Company if it was found that the body or
authority was an agency or instrumentality of the Government.
The Supreme Court gave a broad and liberal interpretation to the
expression ‘other authorities’ in view of the fact that in a Welfare State, a
government has to perform manifold functions for which it has to employ various
agencies or instrumentalities. Thus, such “agency or instrumentality though not
created by the statute must be subject to the same restrictions as the State.” ‘‘The
State” in Art 12 comprehends bodies created for the purpose of promoting the
educational and economic interests of the people,
in R.D. Shetty v International Airport Authority of India (AIR 1979 SC 1628)
the court laid down the following tests for determining whether a body is an agency
or instrumentality of government:
(i) financial resources of the State are the chief funding source i.e. if
XVI
Constitutional Law o f India- II
the entire share capital of corporation is held by Government.
(ii) existence of deep and pervasive State control (e g. appointment
and removal of members of a society, ruies made by society require
prior approval of government).
(iii) functional character being governmental in essence i.e. if the
functions of corporation are of public importance.
(iv) if a department of government is transferred to a corporation.
(v) whether the corporation enjoys monopoly status which is State
conferred or State protected.
However, the court said that these tests are not conclusive but illustrative only,
and will have to be used with care and caution and should not be stretched so far
as to bring in every autonomous body, which has some nexus with the
Government, within the sweep of the expression “other authorities.” In this case,
the international Airport Authority held to be ‘the State.’ The Central Government
had power to appoint chairman and other members of Board, and capital needed
by the Authority provided only by the government.
In the present case, the court observed: The expression ‘other authorities’ is
not confined only to statutory corporations alone but may include a government
company, a registered society, or bodies which have some nexus with government
If the functions of the corporation are of
public importance and closely related to government functions it would
be relevant factor in classifying the corporation as an instrumentality
'Authority' in law belongs to the province of power: Authority (in
Administrative Law) is a body having jurisdiction in certain matters of a
public nature.
A corporation constituted to carry on a commercial or other activity
is for many purposes a distinct juristic entity not drowned in the sea of
State, although, in substance, its existence may be but a projection of
the State. A juristic veil worn for certain legal purposes cannot obliterate
the true character of the entity for the purposes of constitutional law. The
corporations acting as instrumentality or agency of government would
obviously be subject to the same limitations in the field of constitutional
and administrative law as government itseif, though in the eyes of the
law, they would be distinct ana independent legal entities If government
acting through its officers is subject to certain constitutional and public
law limitations, it must follow a fortiori that government acting through the
instrumentality or agency of corporations should equally be subject to the
same limitations. 5
The crux of the matter is that public corporation is a new type of
institution which has sprung from the new socio-economic functions of
government and that it therefore does not neatly fit into old legal
categories.
The emphasis is on functionality plus State control rather than on the

5 With the introduction of new economic policy State is divesting its stake in
most of the Public Sector Unit. In some of them shares of State has been
reduced to minority or it has totally divested its interest Regarding the phrase
other authorities under the control of the government' in Art 12, the Supreme
Court has always been in favour of liberal interpretation Discuss in the light
of landmark judgments whether Art. 12 is so worded that it can take care of
new economic realities. [L C. 11-
2007]
Fundamental Rights: General 11

statutory character of the corporation. There is no reason to make


exclusions on sophisticated grounds such as that the legal person must
be statutory corporation, must have power to make law, must be created
by and not under a statute i.e. a corporation created by a statute versus
a company or society created under the statute (e.g. FCI created by
Food Corporation Act, 1964; a company registered under the Indian
Companies Act).)
In Ajay Hasia v Khalid Mujib (AIR 1981 SC 487), held that a society registered
under the Societies Registration Act, 1898, is an agency or instrumentality of the
State and hence a ‘State1 within the meaning of Art. 12. The government (State
and Central) have full control of the working of the society. The enquiry has to be
not as to how the juristic person is created but why it has been brought into
existence. It is immaterial whether the corporation is created by a statute or under
a statute
In S.M. Ilyas v ICAR [1993 (1 SCC 182)], held that the Indian Council of
Agricultural Research is a ‘State. In Tekraj Vasandi v Union of India [1988 (1 SCC
236)], held that the ‘Institute of Constitutional and Parliamentary Studies' is a
voluntary organisation. The object of society is not related to governmental
business, and in
functioning of it, government’s control is not deep and pervasive, though
gc^ernment has a say in the matter of making grant.
Following this decision, it was held in Chandra Mohan Khanna v NCERT /AIR
1" 2 SC 76^that N-C.E.R.T. is not a State’. The object of National Council of Educational
Research and Training, a society, is to assist and advice Ministry of Education in
the implementation of policies of government. These activities are not wholly related
to governmental functions. The governmental control is confined only to proper
utilization of the grant. It is an autonomous body. Art. 12 should not be stretched so
as to bring in every autonomous body which has some nexus with the
government within the sweep of the expression 'State'.
In Ashok Kumar Singh v BITCO Ltd. (AIR 1998 Pat 9), it was held that the
BiharIndustrial and Technical Consultancy Organisation Ltd. (BITCO) could be a
“State”. The objects of incorporation of BITCO as appeared from its Memorandum
and Articles of Association were to promote industrial growth in the State, a
matter of governmental concern. The entire share of the BITCO has been held by
statutory organisations which are all States’. The IDBI, a State’, has deep and
pervasive control over the functions of BITCO including controlling the
composition and management of B.O.D. of the BITCO. As a result, the BITCO
was held to be a subsidiary of the IDBI and in turn a Government company.
In Pradeep Kumar Biswas v Indian Institute of Chemical Biology (2002) 5
SCC 1, the Supreme Court by 5:2 majority held that CSIR (Council of Scientific
and Industrial Research) is an instrumentality of the State within the meaning of
Art. 12. It overruled the decision in Sabhajit Tewari case (1975). The majority held
that even though it was formed under the Registration of Societies Act, 1860, but
it is a 'State' because the government had overriding control over the
organization. The object incorporated in Memorandum of Association of CSIR
manifestly demonstrates that CSIR was set up in the national interest to further
the economic welfare of the society by fostering planned development in the
country. The Government of India has a dominant role in the governing body of
the CSIR.
The court observed: "The Constitution has to an extent defined the word
XVI
Constitutional Law o f India- II
State’ in Art. 12 itself as including “the Government under the control of the
Government of India." That an “inclusive" definition is generally not exhaustive is a
statement of the obvious and as far as Art 12 is concerned, has been so held by
the Supreme Court. The words "State" and “authority” used in Art. 12 therefore
remain among "the great generalities of the Constitution" the content of which has
sen and continues to be supplied by courts from time to time. Keeping pace with e
broad approach to the concept of equality under Arts. 14 and 16, courts have
enever possible, sought to curb an arbitrary exercise of power against individuals
centres of power", and there was correspondingly an expansion in the judicial
definition of “State” in Art. 12.”
1 _ ,n Kendra Kumar Srivastava v U.P. Rajya Karmchari Kalyan Nigam (2005) in
pr 149’the Apex Court explained the majority and minority opinions laid down t 0 radeeP
Kumar Biswas case. It observed: The multiple test which is to be applied ascertain
the character of a body as falling within Art. 12 or outside as laid down
by the majority view in the aforesaid case is to ascertain the nature of financial,
functional and administrative control of the State over it and whether it is
dominated by the State Government and the control can be said to be so deep
and pervasive as described in the minority view so as to satisfy the court "of
brooding presence of the Government” on the activities of the Corporation
In the minority view in Pradeep Kumar Biswas, different tests are required
to be applied in each particular case. The claim of a body as included within the
definition of “State” based on it being a statutory body falling in the expression
“other authorities” is to be considered differently from the claim of a body based
on the principles propounded in AjayHasia case (1981) 1 SCC 722, that it is an
“instrumentality . or agency” of the State. In the opinion of the minority, the tests
laid down in the case of Ajay Hasia are relevant only for the purpose of
determining whether an entity is “an instrumentality/ agency of the State”.
In the present case, the corporation (U.P. Rajya Karmchari Kalyan Nigam)
has not been created by any statute; it is merely a society registered under the
Societies Registration Act. The enquiry will be confined to the decision as to
whether the corporation is an “instrumentality or agency” of the State On a
detailed examination of the administrative financial and functional control of the
corporation there is no doubt that it is nothing but an “instrumentality or agency”
of the State and the control ; of the State is not only “regulatory” but it is “deep
and pervasive” in the sense that it is formed with the object of catering to the
needs of the government employees i as a supplement to their salaries and other
perks.
The court also held: Even if a body is held to be State the relief to be
granted to the aggrieved person will be determined in each case by the Court on
the basis of the structure of the Society and its financial capability

SCC/ Not a State

LEADING CASE: ZEE TELEFILMS LTD. v UNION OF INDIA


(2005) 4 SCC 649
In this landmark judgment, the Apex Court applied the tests laid down
in Pradeep Kumar Biswas (2002) 5 SCC 111, to determine whether a
body is State. In that case, the court formulated the following principles:
(i) Principles laid down in Ajay Hasia (1981) 1 SCC 722 are
not a rigid set of principles so that if a body falls within any J
Fundamental Rights: General 13

of them it must ex hypothesi, be considered to be a State


within the meaning of Art 12
(ii) The question in each case will have to be considered on the
basis of facts available as to whether in the light of the
cumulative facts as established, the body is financially, J
functionally, administratively dominated, by or under the
control of the Government.

Ii
(iii) Such control must be particular to the body in question and must be
pervasive.
(iv) Mere regulatory control whether under statute or otherwise would
not serve to make a body a part of the State.
In the present case, the BCCI (Board of Control for Cricket in India) was not found
by the court (majority opinion) to be 'State’ in view of the below- mentioned facts;
(i) The Board is not created by a statute.
(ii) No part of the share capital of the Board is held by the Government.
(iii) Practically no financial assistance is given by the Government to the
Board
(iv) The Board does enjoy a monopoly status in the field of cricket out
such status is not State-conferred or State- protected.
(v) There is no existence of a deep and pervasive State control. The
control if any is only regulatory in nature as applicable to other
similar bodies. All functions of the Board are not public functions nor
are they closely related to government functions.
(vi) The Board is not created by transfer of a government- owned
corporation. It is an autonomous body.
Thus, the facts established do not cumulatively show that the Board is financially,
functionally or administratively dominated by or is under the control of the
Government. The Government control over the activities of the Board in regard to
organizing cricket matches and travel of the Indian team abroad as aiso granting
of permission to allow the foreign teams to come to India cannot be construed as
an administrative control. At best this is purely regulatory in nature and the same
according to this Court in Pradeep Kumar Biswas case (2002) 5 SCC 1, is not a
factor indicating a pervasive State control of the Board.
The court observed' The Government has not chosen the Board to perform
duties like the selection of an Indian cricket team, controlling the activities of the
players and other involved in the game of cricket nor has it legally authorized the
Board to carry out these functions under any law or agreement. In the absence of
any authorization if a private body chooses to discharge any functions or duties
which amount to public duties or State functions which is not prohibited by any law
then it would be incorrect to hold that such action of the body would make it an
instrumentality of the State.
The court further observed: In Art 12 the term “other authorities” was introduced at
the time of framing of the Constitution with a limited objective of anting judicial
review of actions of such authorities which are created under
XVI
Constitutional Law o f India- II
by the majority view in the aforesaid case is to ascertain the nature of financial,
functional and administrative control of the State over it and whether it is
dominated by the State Government and the control can be said to be so deep
and pervasive as described in the minority view so as to satisfy the court 'ot
brooding presence of the Government” on the activities of the Corporation
In the minority view in Pradeep Kumar Biswas, different tests are required
to be applied in each particular case. The claim of a body as included within the
definition of “State" based on it being a statutory body falling in the expression
“other authorities” is to be considered differently from the claim of a body based on
the principles propounded in Ajay Hasia case (1981) 1 SCC 722, that it is an
“instrumentality or agency” of the State. In the opinion of the minority, the tests laid
down in the case of Ajay Hasia are relevant only for the purpose of determining
whether an entity is “an instrumentality/ agency of the State”.
In the present case, the corporation (U.P. Rajya Karmchari Kalyan Nigam)
has not been created by any statute; it is merely a society registered under the
Societies Registration Act. The enquiry will be confined to the decision as to
whether the corporation is an “instrumentality or agency” of the State On a
detailed examination of the administrative financial and functional control of the
corporation there is no doubt that it is nothing but an “instrumentality or agency” of
the State and the control of the State is not only “regulatory” but it is “deep and
pervasive’' in the sense that it is formed with the object of catering to the needs of
the government employees as a supplement to their salaries and other perks.
The court also held: Even if a body is held to be State the relief to be
granted to the aggrieved person will be determined in each case by the Court on
the basis of the structure of the Society and its financial capability

SCC/ Not a State

LEADING CASE: ZEE TELEFILMS LTD. v UNION OF INDIA


(2005) 4 SCC 649
In this landmark judgment, the Apex Court applied the tests laid down in
Pradeep Kumar Biswas (2002) 5 SCC 111, to determine whether a body
is State. In that case, the court formulated the following principles:
(i) Principles laid down in Ajay Hasia (1981) 1 SCC 722 are
not a rigid set of principles so that if a body falls within any 1 of
them it must ex hypothesi, be considered to be a State within
the meaning of Art 12
(ii) The question in each case will have to be considered on
the basis of facts available as to whether in the light of the
cumulative facts as established, the body is financially,
functionally, administratively dominated, by or under the
control of the Government.
(iii) Such control must be particular to the body in question and must be
pervasive.
(iv) Mere regulatory control whether under statute or otherwise would
not serve to make a body a part of the State.
In the present case, the BCCI (Board of Control for Cricket in India) was not found
by the court (majority opinion) to be 'State’ in view of the below- mentioned facts'
(i) The Board is not created by a statute.
Fundamental Rights: General 13

(ii) No p3rt of the share capital of the Board is held by the Government.
(iii) Practically no financial assistance is given by the Government to the
Board
(iv) The Beard does enjoy a monopoly status in the field of cricket but
such status is not State-conferred or State- protected.
(v) There is no existence of a deep and pervasive State control. The
control if any is only regulatory in nature as applicable to other
similar bodies. All functions of the Board are not public functions nor
are they closely related to government functions.
(vi) The Board is not created by transfer of a government- owned
corporation. It is an autonomous body.
Thus, the facts established do not cumulatively show that the Board is financially,
functionally or administratively dominated by or is under the control of the
Government. The Government control over the activities of the Board in regard to
organizing cricket matches and travel of the Indian team abroad as aiso granting
of permission to allow the foreign teams to come to India cannot be construed as
an administrative control. At best this is purely regulatory in nature and the same
according to this Court in Pradeep Kumar Biswas case (2002) 5 SCC 1, is not a
factor indicating a pervasive State control of the Board.
The court observed' The Government has not chosen the Board to perform
duties like the selection of an Indian cricket team, controlling the activities of the
players and other involved in the game of cricket nor has it legally authorized the
Board to carry out these functions under any law or agreement. In the absence of
any authorization if a private body chooses to discharge any functions or duties
which amount to public duties or State functions which is not prohibited by any law
then it would be incorrect to hold that such action of the body would make it an
instrumentality of the State.
The court further observed: In Art. 12 the term “other authorities” was
introduced at the time of framing of the Constitution with a limited objective of & 1
anting judicial review of actions of such authorities which are created under
XVI
Constitutional Law o f India- II
statute and which discharge State functions However, because of the need of
the day the Supreme Court in Rajasthan SEB (1967) 3 SCR 377 and
Sukhdev Singh (1975) 1 SCC 421, noticing the socio-economic policy of the
country thought it fit to expand the definition of the term “other authorities' to
include bodies other than statutory authorities. This development of law by -,
judicial interpretation culminated in the judgment of the seven-Judge Bench in
Pradeep Kumar Biswas case. It is to be noted that in the meantime the socio-
economic policy of the country has changed and the State is today distancing
itself from commercial activities and concentrating on governance rather than
on business. Hence, there seems to be no need to further expand the scope
of “other authorities” iri Art. 12 by judicial interpretation at least for the time
being
The minority opinion in the present case was that what must be noticed
are the functions of the body concerned. The word “State” has different
meanings in different contexts. The tests under Pradeep Kumar Biswas are
not applicable to a private body like BCCI. In view of the fact that public
interest is involved in the activities of the Board, it is a State actor.]

International Crops Research Institute Not a State

LEADING CASE: G. BASSI REDDY v INTERNATIONAL CROPS RESEARCH


INSTITUTE (AIR 2003 SC 1764)
In this case, the International Crops Research Institute (ICRISAT) was not
held to be a ’State’ within the meaning of Art. 12. The court opined that it is an
international organization and has been set up as non-profit research and
training centre to help developing countries to alleviate rural poverty and
hunger. It is not set up by the Government, and is not controlled by nor is
accountable to the Government. The government’s financial contribution to
ICRISAT is minimal. Its participation in ICRISAT’s administration is limited to
3 out of 15 members.
A writ under Art. 226 can lie against a “person' if it is a statutory body or
performs a public function or discharges a public or statutory duty. ICRISAT
has not been set up by a statute nor its activities statutorily controlled.
Although, it is not easy to define what a public function or public duty is, it can
reasonably be said that such functions are similar to or closely related to
those performable by the State in its sovereign capacity.
The object of setting up ICRISAT was to help developing countries
(India, parts of Soutn Asia, sub-Saharan, South and Eastern Africa and, parts
of Latin America) in semi-arid tropics to alieviate rural poverty and hunger in
ways that are environmentally sustainable. The primary activity of ICRISAT is
to conduct research and training programmes in the sphere of agriculture
purely on a voluntary basis A service voluntarily undertaken cannot be said to
be a public duty.
Besides ICRISAT has a role which extends beyond the territorial
boundaries of India and its activities are designed to benefit people from
all over the world. While the Indian public may be the beneficiary of the
activities of the institute, it certainly cannot be said that the ICRISAT
owes a duty to the Indian public to provide research and training
facilities ]
Fundamental Rights: General 15

Is judiciary included in the word ‘State’?


Unlike in USA, where a judicial decision implies 'State action’ for the purposes of
enforCementof fundamental rights, in India the 'judiciary' is not specifically mentioned
jn Art. 12. The judicial view is that the judgments of courts cannot be challenged
on the ground that they contravene fundamental rights.
In Naresh v State of Maharashtra (AIR 1967 SC 1), it was held that even if a
court is the State a writ under Art. 32 cannot be issued to a High Court of competent
jurisdiction against its judicial orders, because such orders cannot be said to
violate the fundamental rights. What the judicial decision purports to do is to
decide the controversy between the parties and nothing more. The court said that
the ‘judiciary’ while exercising its rule-making power under Art. 145 would be
covered by the expression 'State' within the meaning of Art. 12, but while
performing its judicial functions, it is not so included
In Rupa Ashok Hurra v Ashok Hurra (AIR 2002 SC 1771), the Apex Court
has re-affirmed and ruled that no judicial proceeding could be said to violate any
of the fundamental rights. It was said to be settled position of law that the superior
courts of justice did not fall within the ambit of State' or 'other authorities’ under
Art. 12.
Comments - Mr. H.M. Seeravi is of opinion that the judiciary should be included in
the definition of ‘the State' and a judge acting as a judge is subject to the writ-
jurisdiction of Supreme Court. In A.R. Antulayv R.S. Nayak (A\R 1988 SC 1531), it
was held that the court could not pass an order or issue a direction which would
be violative of fundamental rights, thus, it can be said that the expression State’
includes judiciary also.
•t is submitted that the judiciary, though not expressly mentioned in Art. 12, it
should be included so, since courts are set up by statute and exercise power
conferred by law It is suggested tnat discrimination may be brought about... even
y) judiciary. The courts, like any other organ of the State, are limited by the
andatory provisions of the Constitution.

^Urnmary: What is ‘State’ or Not?


he following have been held to be State':
1 Regional Engineering College established by a Society registered under
a State Act (Ajay Hasia v Khalid Mujib AIR 1981 SC 487).
2 ISI- Indian Statistical Institute (S. S. Minhas v Indian Statistical Institute
AIR 1984 SC 363).
3. ICAR- Indian Council of Agricultural Research [S.M Ilyas v ICAR (1993) 1
SCC 182]; IVRI - Indian Veterinary Research Institute, an affiliate of ICAR
(PK. Ramachandra Iyer v UOI AIR 1984 SC 541)
4. CSIR -Council of Scientific and Industrial Research [Pradeep Kumar Biswas v
Indian Institute of Chemical Biology (2002) 5 SCC 1]
5. FCI- Food Corporation of India (State of Punjab v Raja Ram AIR 1981 SC
1694).
6. SAIL- Steel Authority of India Limited (Steel Authority of India Ltd. v Shri
Ambica Mills AIR 1998 SC 418).
7. Nationalized Banks [Bank of India v O P Swarankar (2003) 2 SCC 721];
Regional Rural Banks (Chairman, Prathama Bank, Moradabad v Vijay Kumar
AIR 1989 SC 1977).
8. Rajasthan Electricity Board (Electricity Board, Rajasthan v Mohan Lal AIR
1967 SC 1857).
9. Religious Endowment Board e.g. Cochin Devasom Board (P.B.M.
Namboodripad v C D. Board AIR 1956 SC 19).
10. ONGC- Oil and Natural Gas Commission (Sukhdev Singh v Bhagatram MR
1975 SC 1331; K.C. Joshiv (70/AIR 1985 SC 1045), Indian Oil Corporation
(Mahabir Auto Stores v Indian Oil Corpn. AIR 1990 SC 1031)
11. Hyderabad Stock Exchange.
12. Council for Indian School Certificate Examinations
13. LIC- Life Insurance Corporation (Sukhdev Singh v Bhagatram AIR 1975 SC
1331).
14. United Indian Insurance Co.
15. Industrial Finance Corporations (Sukhdev Singh v Bhagatram AIR 1975 SC
1331); Gujarat State Financial Corporation (Gujarat State Financial Corpn. v
M/s Lotus Hotel MR 1983 SC 848).
16. U.P. Rajya Karmachari Kalyan Nigam [Virendra Kumar Srivastava v U.P
Rajya Karmchari Kalyan Nigam (2005) t SCC 149],
17. Export Credit Guarantee Corporation of India Ltd
18. International Airport Authority (R. D. Shetty v International Airport Authority of
India MR 1979 SC 1628).
19. Indian Banks Association.
20. Bombay Port Trust.
21. Bharat Petroleum Corporation (Som Prakash v Union of Indio AIR 1981
SC212).
22. Hindustan Steel Works Construction Ltd. (Hiv stan Steel Works Construction
Ltd. v State of Kerala AIR 1997 SC ^.75).
23. BITCO- Bihar Industrial and Technical Consultancy Organisation Ltd (AshoK
Kumar Singh v BITCO Ltd. (AIR 1998 pat 9).
24. IDE3I- Industrial and Development Bank of India.
25. Children’s Aid Society, Bombay (Sheela Barse v Secretary, Children Aid
Society AIR 1987 SC 656).
26. Delhi Transport Corporation (D.T.C. v Mazdoor Congress MR 1991 SC
101); Mysore State Road Transport Corporation (Mysore S.R.T.C. v
Devraj Urs AIR 1975 SC 1331)
27. City & Industrial Development Corporation, Maharashtra [Star
Enterprises v City & Industrial Dev. Corpn., Maharashtra (1990) 3 SCC
280].
28. NAFED - National Agricultural Co-operative Federation of India (AH
Ahamed
& Co. v 1/0/AiR 1982 Mad 247).
29. Universities (Uniesh Chandra v VN Singh AIR 1968 Pat 3); Private
educational institutions (if they fulfill the tests laid down in Ajay Hasia
case).
30. Mysore Paper Mills Ltd. [Mysore Paper Mills Ltd. v Mysore Paper Mills
Officers’ Association (2002) 2 SCC 167]
31. A non-governmental company like Shriram Food and Fertilizers Ltd.
[M.C. Mehta v Union of India (1987) 1SCC 395]
32. DDA- Delhi Development Authority (DDA v Joint Action Committee,
Allottee of SFS Flats AIR 2008 SC 1343).
The following have not been held to be 'State':
1. Institute of Constitutional and Parliamentary Studies [Tekraj Vasandi v
Fundamental Rights: General 17

Union of India (1988) 1 SCC 236],


2 NCERT- Nationai Council of Educational Research and Training
(Chandra Mohan Khanna v NCERT AIR 1992 SC 76).
3. BCCI- Board of Control for Cricket in India [Zee Telefilms Ltd v UOI
(2005) 4 SCC 649],
4. ICRISAT- International Crops Research Institute (G. Bassi Reddy v
International Crops Research Institute AIR 2003 SC 1764).
5. Superior courts of justice [Naresh v State of Maharashtra AIR 1967 SC 1:
Rupa Ashok Hurra v Ashok Hurra AIR 2002 SC 1771],
6. National Co-operative Consumers Federation (J. S. Ameja v National Co-
op Consumers Federation of India Ltd. AIR 1995 Del 44).
7 A private medical/engineering college even if recognized and affiliated to
a University [Unni Krishnan v State of A P (1993) 1 SCC 645].
8 Indian Institute of Bankers (Laxman Nath Das v Dy. Secy. (Exam) Indian
Institute of Bankers MR 1995 Ori. 277)
9 A Cooperative Society [Zoroastrian Cooperative Housing Society v
District Registrar (2005) 5 SCC 632]. Such a society is nothing but a
body created ln accordance with and governed by the provisions of the
Co-operati /e Societies Act. Certain number of persons forms a society
with certain
18 Constitutional Law o f India- II
aims and objects. It is subjected to certain rules, regulations, etc Such • a
society is not created by the provisions of the Act itself, nor the State
exercise any control over such a society.
Similarly, Co-operative Banks e g. the Punjab State Co-operative Bank
is not a ‘State’ (Satish Kumar v Punjab State Co-operative Bank Ltd AIR
1981 P&H282).

ARTICLE 13 [LAWS INCONSISTENT WITH FUNDAMENTAL


RIGHTS]

Clause (1) - All pre-Constitution or existing laws i.e. laws which were in force
immediately before the commencement of the Constitution shall be void to the
extent to which they are inconsistent with fundamental rights from the date of the
commencement of Constitution
Clause (2) - The State shall not make any law which takes away or abridges the
fundamental rights, and any law in contravention of fundamental rights shall to
the extent of contravention, be void
[Thus, Art. 13(2) applies to posf-Constitution laws ]
Clause (3) - The term ‘law’ includes any ordinance, order, bye law, rule,
regulation, I notification, custom or usage having in the territory of India the force
of law.
Article 13, in fact, provides for the 'judicial review’ of all legislations in India,
past as well as future. All laws whether made by a legislature or by a delegated
authority and all executive acts must respect and conform to the fundamental
rights. The ordinances promulgated by the President under Art. 123 or by the
Governor under Art. 213 must also not be inconsistent with the fundamental
rights Art. 13 imposes an obligation on the State to respect and implement the
fundamental rights and at the same time confers a power on the courts (Supreme
and High Courts via Art. 32 and Art. 228) to declare a law/ Act void if it infringes a
fundamental right. Art. 13, thus, provides teeth to the fundamental rights and
makes them justiciable i.e. enforceable in the courts.
The US Constitution contains no such provision but in Marbury v Madison
I (1803) 1 Cranch 137, the U.S. Supreme Court (Chief Justice Marshall) held that
it follows as a corollary of a written Constitution. A written Constitution has
superiority over ordinary laws. It may be remarked that Art. 13 has been inserted by
way abundant caution. Even in the absence of Art. 13 the result would have been
the same. 6

6 B.K. Sharma, p. 68.


Fundamental Rights: General 19

A fundamental right is a constitutional limitation upon the legislative power


of the legislatures. Art. 13(2) expressly states this limitation. Hence after the
Constitution came into force no distinction can be drawn between a law which violates
a fundamental right and a law which is beyond the legislative competence of the
legislature, in both cases the legislature goes beyond its powers by disregarding a
constitutional limitation.

Application and Rules of Interpretation of Article 1310


(1) No retrospective effect - Every pre-constitutional law must after the
commencement of the Constitution conform to the provisions of Part III.
It, otherwise, it cannot operate after 26-1-1950. Art. 13(1) (including
other fundamental rights) is prospective in nature. All inconsistent
existing laws become void only after the commencement of Constitution.
They are not void ab initio. Such laws exist for all past acts,
transactions, etc. and for enforcing all rights and liabilities accrued
before the date of the Constitution. However, no body can claim his
rights and liabilities to be enforced under a particular procedure (in
respect of pending proceedings) which becomes inconsistent with
fundamental rights.

Leading Case: keshavan madhava menon V STATE OF


BOMBAY , (AIR 1951 SC 128)
In this case, the petitioner had committed an offence punishable under
the Indian Press (Emergency Powers)Act, 1931. During the pendency of
the proceedings, the Constitution of India came into force on 26-1-1950.
It was contended on behalf of the petitioner that the provisions of the
aforesaid Act were ultra vires and void in view of Art. 19(1)(a) and Art.
13 (fundamental rights). The Act was one of the many repressive laws
enacted by an alien Government with a view to stifle the liberty of the
Indian subjects and particularly of the Indian Press. To permit pending
proceedings under a law which., after the commencement of the
Constitution had become void, to proceed further, after the Constitution
has taken effect, is to prolong the efficacy of the law notwithstanding
that it has become void on and after the date the Constitution came into
force and that is against the spirit of the Constitution.
It was held that there is no fundamental right that a person shall
not be prosecuted and punished for an offence committed before the
Constitution came into force. The Constitution is to be interpreted
according

and elucidate in the light of judicial precedents the meaning of the term ‘void’ as used in Art.
Explain
13(1) and 13(2) of the Constitution. [C.LC.-2006]

/
20 Constitutional Law o f India- II

to its language, and not according to any supposed spirit of the Constitution Every statute is
prima facie prospective unless it is expressly or by necessary implications made to have
retrospective operation There is no reason why this rule of interpretation should not be
applied for the purpose of interpreting the Constitution. If it is against the spirit of the
Constitution to continue the pending prosecutions under such a void law, surely it would be
equally repugnant to that spirit that men who have already been convicted under such
repressive law before the Constitution of India came into force should continue to rot in jail
The impugned Act was an existing law at the time when the Constitution came into
force. That existing law imposed on the exercise of the right guaranteed to the citizens of
India by Art. 19( 1 )(a) restrictions which could not be justified as reasonable under Art
19(6) as it then stood and consequently under Art. 13(1) that existing law became void “to
the extent of such inconsistency.”
The law became void not in toto or for all purposes or for all times or for all persons
but only “to the extent of such inconsistency” i.e. to the extent it became inconsistent with
the provisions of Part III which conferred the fundamental rights on citizens It did not
become void independently of the existence of the rights guaranteed by Part III In other
words, on and after the commencement of the Constitution, the existing law as a result of
its becoming inconsistent with the provisions of Art. 19(1 )(a) read with clause (6) as it then
stood could not be permitted to stand in the way of the exercise of that fundamental right.
Art. 13(1) by reason of its language cannot be read as having obliterated the entire
operation of the inconsistent law or having wiped out it altogether from the statute book.
Such law existed for all past transactions and for the enforcement of rights and liabilities
accrued before the date of the Constitution. Art. 13(1) is entirely prospective in operation
arid as it was not intended to have any retrospective effect there was no necessity at all for
inserting in that article any such saving clause. The effect of Ait. 13(1) is quite different from
the effect of the expiry of a temporary statute or the repeal of a statute by a subsequent
statute ]
(2) Doctrine of Severability or Separability7 - It means that if an offending provision can be
separated from that which is constitutional then only that part which is offending is to be
declared as void and not the entire statute (State of Bombay v F.N Balsara AIR 1951 SC
318. the case arose under the Bombay Prohibition Act, 1949) The words “to the extent of
such inconsistency be void” in Art. 13 means that only the repugnant provisions of the law
in question shall be treated by courts as void and not the entire statute.

[LC H 2006/20071

7 Mention the doctrine of severability


Fundamental Rights: General 21

The primary test is whether what remains is so inextricably mixed with the part
declared invalid that what remains cannot survive independently. Doctrine of
severability is applicable to legislation which is partly ultra vires that is beyond the
legislative competence of the legislature This is contemplated by Art. 254, which is
app!'cable t0 an Act wflich IS generally within the legislative competence of a legislature but
certain parts are outside the scope of legislative entry.

Leading Case: R.m.d.c. v union of INDIA (AIR 1957


SC 628)
in this case, Sec. 2{d) of the Prize Competition Act was broad enough to
include competitions of a gambling nature as well as competitions
involving skill. Held that the provisions of the Act were severable and
struck down those provisions which related to competition involving skill.
The court observed that the intention of the legislature is a
determining factor i.e whether the legislature would have enacted the
valid part if it had known that the rest of the statute was invalid. If the
valid and invalid parts are so distinct and separate that after striking out
what is invalid what remains is, in itself, a complete code independent of
the rest, then it will be upheld. However, if the valid and invalid parts are
so inextricably mixed up that they cannot be separated or if what
remains on the statute cannot be enforced without making alterations
and modifications the wnole Act to be declared void, as otherwise, it will
amount to judicial legislation.
Even when the valid provisions are distinct and separate from the
invalid parts but they all form part of a single scheme which is intended
to be operated as a whole, then the whole Act must fail. Similarly, when
the valid and invalid parts are independent and do not form part of a
scheme, but what is left after omitting the invalid part, is so thin and
truncated, so as to be in substance different from the original legislation,
then also the Act must fail. Severability is the question of substance and
not of form.
In determining the legislative intent on the question of separability,
it will be legitimate to take into account the history of the legislation, its
object, the title and the preamble to it. The impugned Act, in the present
cas6, was
passed to control end regulate prize competitions of a 'gambling'
character. The application of impugned provisions of the Act could be
restricted to the competitions of gambling character. The provisions of
the '"'Pugnea Act were thus held severable in their application to
competitions,
•fth n W,1'Ch does not piay any subs,anlial Part)
of Cor ls or‘e provision which covers restrictions both within and without the limits ,he
con ! Utl°nality' 's not P°ssib,e t0 uphold the law which may be applied within its b S
^ *' * limits because the two types are inseparable. If there be possibility e,ng
u ona

applied to purposes not sanctioned by the Constitution then the whole


22 Constitutional Law o f India- II

Act must be held to be wholly invalid and void (Chintaman Rao v State of M. P
AIR 1951 SC 118).
In A.K. Gopalan v State of Madras (AIR 1950 SC 27), the court struck
down Sec 14 of the Preventive Detention Act, 1950, as violative of the
fundamental right under Art. 22. The rest of the Act was held to be valid The
doctrine of severability has been applied by the Supreme Court in cases of
challenge to the validity of a constitutional amendment. In Minerva Mills Ltd. v
UOI (AIR 1980 SC 1789), the court struck down certain sections of the
Constitution (42nd Amendment) Act, 1976, as beyond the amending power of the
Parliament. The rest of the Act was held to be constitutionally valid.
In Kihota Hollohan v Zachithu (AIR 1993 SC 412), it was held that a
composite amendment (containing amendments in provisions requiring ratification
by States as well as provisions not requiring such ratification) is covered by the
rule of severability. Held, that Sec. 10 of the Tenth Schedule minus para 7
remains valid and constitutional. Para 7 which has been declared unconstitutional
is severable from the main provision of Tenth Schedule. The remaining provisions
of the Schedule stands independent of Para 7 and are complete in themselves
and workable Para
7 provided that the speaker’s decision regarding the disqualification shall be final
and no court could examine its validity.
(3) Doctrine of Eclipse8 - It provides for validation of void laws. It is based
on the principle that a law which violates fundamental rights is not
nullity or void ab initio but becomes only unenforceable i.e. remains in a
moribund condition, it is over-shadowed or eclipsed by the fundamental
rights and remains dormant, but it is not dead."

LEAoiNG CASE: BHIKAJI NARAIN DHAKRAS V STATE OF M.P.


(AIR 1955 SC 781)
in this case, the provision of C.P. and Berar Motor Vehicles Act 1947
authorized the State government to take up the entire motor transport
business in the province to the exclusion of motor transport operators.
This provision, though valid when enacted, became void on the coming
into force of the Constitution in 1950 as they violated Art 19(1 )(g) of
Constitution. However, in 1951, by the Constitution (First Amendment)
Act, clause (6) of Art. 19 was amended so as to authorise the
government to monopolise any business. Held that the effect of the
amendment was to remove the shadow and to make the impugned Act
free from all blemish or infirmity... thus making it enforceable.
The court observed: The doctrine of eclipse is applied in relation to
a pre-Constitution law, which was valid when it was enacted.
Subsequently when the Constitution came into force a shadow falls on it
because it is

[LC11-2007]
8 What do you mean by Doctrine of Eclipse?
inconsistent with the Constitution. The Act is eclipsed. When the
shadow is removed (the fundamental right was amended to remove the
conflict
between the law and the right) the pre-Constitution law becomes fully
applicable and is free from infirmity. However, between the 26-1-1950
(when the Constitution came into force) and the 18-6-1951 (when the
First Amendment was made), the impugned Act could not stand in the
way of the exercise of the fundamental right of a citizen under Art. 19(1
)(g)
All laws, existing or future, which are inconsistent with the
provisions of f>art III of Constitution, are, by the expr ess provisions of
Art.
13, rendered void “to the extent of such inconsistency." Such laws exist
for all past transactions, for rights and liabilities incurred before the
Constitution came into force, and for the determination of rights of
persons who haven’t been given fundamental rights by the Constitution
e.g. noncitizens. It is only as against the citizens that they remained in a
dormant or moribund condition. After 18-6-1951, the impugned Act
ceased to be unconstitutional and became revivified and enforceable
against citizens as well as against non-citizens.
Thus, the petitioner's contention that the law having become void
for unconstitutionality was dead and could not be vitalized by a
subsequent amendment of the Constitution removing the constitutional
objection, unless it was re-enacted, could not be sustained in view of the
aforesaid position of the iaw]

Does the doctrine of eclipse apply to a post-Constitution law?


Art. 13(2) deals with post-Constitution or future laws [while clause (1) deals wjth
pre-Constitution or existing laws] As distinguished from clause (1), clause (2)
makes the inconsistent laws void ab initio (void from their verv inception or still-
born law) and even convictions made under such laws shall have to be set aside.
Also, such laws cannot be revived by subsequent amendment of the Constitution.
It is to be noted that in both clause (1) and (2) of Art. 13, a declaration by the
court of their invalidity will be necessary, to make the laws invalid

Leading Case: state of gujarat V SHRI


AMBICA MILLS LTD. 9
(AIR 1974 SC 1300)
In this case, held that a posf-Constitution law which is inconsistent with
fundamental rights is not nullity or non-existent in all cases and for all
Purposes. After the Ambica Mills judgment, the doctrine of eclipse

Mention the views expressed by the Supreme Court in State of Gujarat v


Ambica Mills. tL.C. 11-
2006]
Constitutional Law o f India- II

applies to both pre- and post-Constitution laws.


In this case, the provisions of the Bombay Labour Welfare Fund Act were
challenged by the Ambica Mills on the ground that the impugned provisions
(which provided for the payment of unpaid accumulations to the employees)
infringes its fundamental right to property [Art 19(l)(f)] the Act being a post-
Constitution law. The High Court favoured Ambica Mills holding impugned
provisions unconstitutional and void The Supreme Court, however, took a
contrary view.
The Apex Court held that just as a pre-Constitution law abridging or taking
away fundamental rights remained after the Constitution came into force as
respect non-citizens' as it was not inconsistent with their fundamental rights, so
also a post-Constitution law which is inconsistent with fundamental rights is not
nullity or non-existent in all cases and for all purposes. A post-Ccnstitution law
which, takes away or abridges rights conferred by Art. 19, wiil be operative as
regards to non-citizens, because fundamental rights are not available to them.
Such a law will become void only against citizens because fundamental rights are
conferred on them In the present case, the Ambica Mills, being a company, was a
non-citizen for the purposes of Art. 19. Thus, the Bombay Labour Welfare Fund
Act was valid in respect of non-citizens.
In Deep Chand v State cfU.P (AIR 1959 SC 648) and Mahendra Lai Jain v
State of U.P. (AIR 1963 SC 1019), held that the doctnne of eclipse applies only to
pre-Constitution law, and not to post-Constitution law, because voidness of latter
is from its very inception and such a law cannot exist for any purpose. A pre-
Constitution law which violates fundamental rights remains valid till the
commencement of the Constitution. The minority (in Deep Chand case) however,
expressed the view that the doctrine of eclipse is applicable even to post-
Constitution law against non-citizens
The court, in the present case, observed: The meaning of the word 'void' is
same both in Art. 13(1) and (2), and for that reason a post- Constitution law which
takes away or abridges rights conferred by Art. 19 should be operative as regards
non-citizens as it is void only to the extent of contravention of rights conferred on
citizens under Art. 19. In other words, voidness is not in rem, but to the extent of
inconsistency or contravention of fundamental rights; the phrase to the extent of
contravention’ can mean only to the extent of the contravention of rights
conferred. When Art 13(2) uses the expression ‘void’, it can only mean, void as
against persons whose fundamental rights are taken away cr abridged by a law.
There are many fundamental rights and they inhere in diverse types of persons,
minorities, or denominations and if a law abridges fundamental right of one class
of persons, or minorities, it should not be void as against others who have no
such fundamental right
In M.P.V. Sundararamaiei v State ofA.P. (AIR 1958 SC 468), it was held
that a law if it lacks legislative competence was absolutely null and void, it could
not be revived and have to be re-enacted But a law within
the legislative competence but violative of constitutional limitation was
unenforceable but once the limitation was removed, the law became
effective The court, in the present case, rejected this distinction. It
observed that ifF the
u n dlaw
a menacted
e n t a l Rbyi gah legislature
t s : G e n e having
ral no capacity in 2 5
the former sense would be void in rem, there is no reason why a law
passed by a legislature having no legislative capacity in the latter sense
is void only qua persons whose fundamental rights are taken away or
abridged.
In Jsgannath v Authorized Officer, Land Reform (1971) 2 SCC
893, this Court has said that a post-Constitution Act which has been
struck down for violating the fundamental rights and was therefore still-
born, has still an existence without re-enactment, for being put in the
Ninth Schedule. That only illustrates that any statement that a law which
takes away or abridges fundamental rights is still-born or null and void
requires qualifications in certain situations. Although the general rule is
that a statute declared unconstitutional is void at all times and that its
invalidity must be recognized and acknowledged for all purposes and is
no law and a nullity, this is neither universally nor absolutely true and
there are many exceptions to it.
The court concluded It is enough to say that if a law is otherwise
good and does not contravene any of their fundamental nghts, non-
citizens cannot take advantage of the voidness of the law for the reason
that it contravenes the fundamental right of citizens and claim that there
is no law at all ]
In Dulare Lodh v Additional District Judge, Kanpur case (AIR 1984 SC 1260), the
Supreme Court applied the doctrine of eclipse to post-Constitution law even
against citizens. By virtue of the Amendment Act, 1976, the decree which was
lying dormant and could not be executed became executable. A 'void' statute can
be revived by constitutional amendment . by virtue of Art. 31-B such curing of
defect took place with retrospective operation from the date on which the Acts
were put on the statute-book.
In IR Coelho v State of Tamil Nadu (AIR 2007 SC 861), the Apex Court
observed that even though an Act is put in the Ninth Schedule by a constitutional
amendment, its provisions would be open to challenge on the ground that they
destroy or damage the basic feature or structure of the Constitution, for instance,
if the fundamental rights are taken away or abrogated. If the validity of any Ninth
Schedule law has already been upheld by this court, it would not be open to
challenge such law again on the principles declared by this judgment. However, if
a law held to be violative of any rights in Part III is subsequently incorporated in
the Ninth Schedule after 24th April, 1973, such a violation/infraction shall be open
to challenge on the ground that it destroys or damages the basic structure as
indicated in Art. 21 read with Art. 14, Art. 19 and the principles underlying
thereunder.

Art. 13 and the Terms ‘Law’ and ‘Laws in Force’ [Art. 13(3)]
Article 13(3) (a) gives the term ‘laW a very bread connotation which includes any
ordinance, order, bye-law. rule, regulation, notification, custom or usage having
the force °t law. This is necessary to protect the violation or infringement of
fundamental nghts

\
The term “law”, thus, includes enacted law or legislation; ordinances issued
by
2 6 the President/Governor;
C o n s t i tadministrative
u t i o n a l L a /executive
w o f I n orders
d i a - I including
I delegated
legislation; statutory rules, government notifications, bye-laws or regulations made
by a municipal or statutory body, custom or usage having the force of law etc
It may be noted that administrative directions or instructions issueo for the
guidance of its officers which are not intended to be legal obligations, are outside
the purview of the term ‘law under Art. 13. Similarly, the provisions of a Manual
(compilation of executive orders or circulars) issued by the Government are not
‘law1. Circulars issued by the Central Board of Direct Taxes or by the Central
Board of Excise and Customs are not law. The Flag Code which contains
guidelines regarding flying of our national flag is not law. 14
Personal laws - Hindu or Muslim law - are not covered by the term 'law”
under Art. 13. An amendment of the Constitution enacted under Art 368 is outside
the purview of Art. 13 (See under the chapter on ‘Amendment of the
Constitution’).
Article 13(3) (b) lays down that the expression “laws in force” includes "laws
passed or made by a Legislature or other competent authority in the territory of
India before the commencement of this Constitution and not previously repealed,
notwithstanding that any such law or any part thereof may not be then in operation
either at all or in particular areas.” It, therefore, include laws which were on the
Statute Book, though not in use or operation e.g the Fugitive Offenders Act, 1881,
passed by the British Parliament and applicable to India.

Art. 13 is not applicable to law declared by the Supreme Court u/Arts. 141-142
Art. 13(2) of the Constitution envisages a situation where the State action, be it
legislative or executive, violates the fundamental rights; such law is declared as
void but when the previous overruled decision and the new rule laid down by the
Court as a stare decisis operates prospectively from a given date (namely, either
the date of judgment or extended date), such order/judgment is not a legislative
Act which is void under Art 13(2) but a judicial tool by which the effect of the
judgment was given.
Therefore, the judgment of the Supreme Court in Mandal case declaring
that Rangachari ratio did not correctly interpret Arts. 16(1) and 16(4) is a
declaratory law under Art. 141 of the Constitution. Art. 13(1) and (2) deal with the
statute law and not the law declared by the Supreme Court under Art. 141 and
directions/orders under Art. 142. The question is whether such a declaration is
inconsistent with the Constitution or in derogation of the fundamental rights.
The Court in Mandal case (holding reservation in promotion to be
unconstitutional) haa obviously recognised the need to adjust the competing rights
of both the disadvantaged and advantaged sections of citizens and, therefore, it
postponed the operation of that judgment for 5 years from that date giving an
option

1 4 . B.K. Sharma, p. 70.


the executive to have the law amended appropriately [Ashok Kumar Gupta v
Fundamental Rights: General 27
State of U P. (1997) 5 SCC 201],

ARTICLE 31 [RIGHT TO PROPERTY AND SAVING OF CERTAIN LAWS]

The 'Right to Property' has proved to be the most complicated and controversial.
The Constitution (44th Amendment) Act, 1978 omitted Art. 19( 1 )(f) (Right to
acquire, hold and dispose of property), the sub-heading Right to Property (above
Art. 31) and Art. 31 (Compulsory acquisition of private property for a public
purpose - concept of 'eminent domain'). The effect of this change is that the Right
to Property is no more a fundamental right.
A new chapter IV has been inserted in Part XII of the Constitution and the
provision in Art 31 has been transferred there as Art. 300A (‘no person shall be
deprived of his property save Dy authority of law’). Thus the Right to Property
though a constitutional /legal right is not a fundamental right. If this right is
infringed the aggrieved person cannot access the Supreme Court directly under
Art. 32. Art. 300-A gives protection against executive action but not against
legislative action.
Notwithstanding the repeal of Art 31, Art. 31-A (added by the 1st
Amendment, 1951 and amended by the 4th, 17th, and 44th Amendment), Art. 31-
B (added by the 1st Amendment 1951), and.. Art 31-C (added by the 25th
Amendment, 1971 and amended by 42nd and 44th Amendments) remains part of
the Fundamental Rights. Art. 31-A saves certain laws providing for acquisition of
estates etc. from being questioned on grounds of inconsistency with Arts. 14 and
19. Art. 31-B saves certain Acts and Regulations listed in the 9th Schedule from
being challenged on the ground of inconsistency with any of the Fundamental
Rights. Art. 31-C saves certain laws giving effect to Directive Principles from being
questioned and invalidated °n grounds of inconsistency with Arts. 14 and 19.

Art. 31A and Saving of Laws


Article 31A was originally enacted by way of exception to the right to property (Art.
31
). Art. 14 and 19(1)(g). Even though right to property had been abolished this
ex
ception stands by way of abundant caution. This article was inserted by the 1 st
Amendment to the Constitution made in 1951. The object was to protect and
validate the laws extinguishing the Zamindari and similar grants and rights
(notwithstanding ^ything contained in Art. 13). The amendment was given
retrospective effect from ® January 1950. It was further amended by the 4 th and
17th Amendments to ®n|arge the scope of the exception and to take away the
agrarian reform and other ^islation from the purview of the courts.
The legislation that are provided protection by all 31A are
(a) Acquisition of any estate by the State Estate' means any jagir, iriam,
muafi, or similar grant of land held for agricultural purposes etc
(b) Taking over management of any property
(c) Amalgamation of companies
(d) Extinction for modification of rights of directors or shareholders.
(e) Extinction or modification of mining leases 15

Art. 31B: Validation of Certain Acts, etc. (9th Schedule)


Article 31B constitutes a draftsman’s device to shield legislation from being
declared
28 unconstitutional
C o n s tand
i t u tvoid
i o n Ita lalso
Lawbreaths
o f Innew
d i alife
- I Iin the enactments
declared void by a court of law. In other words, it validates such laws with
retrospective effect from the date of its enactment.
It says: "Without prejudice to the generality of the provisions contained in
Art. 31A, none of the Acts and Regulations specified in the Ninth Schedule nor
any of the provisions thereof shall be deemed to be void, or ever to have become
void, on the ground that such Act Regulation or Provision is inconsistent with, or
takes away, or abridges any of the rights conferred by any provisions of this Part
and. notwithstanding any judgment, decree, or order of any court/tribunal to the
contrary, each of the said Acts and Regulations shall, subject to the power of any
competent Legislature to repeal or amend it, continue In force ”
It follows that laws providing for acquisition of estates and takeover of
corporations etc. have been saved against challenge on the ground of alleged
infringement with, taking away or abridging any fundamental right Thus, the Ninth
Schedule was inserted by the 1st Amendment 1951, and it consisted of 13 entries.
The number has since been increased by successive amendments (4 th, 17th, 39th,
42nd, 44th, 47th, 66 th, 75th, 78th etc.). On 31st May 2007, it had 282 entries
Article 31B uses the expression ‘Acts and Regulations ’ On this basis it has
been held that the Acts specified in the 9th Schedule are entitled to protection. But
the protection does not extend to orders or notifications subsequently issued in
exercise of the powers conferred by an Act included in the 9 th Schedule (Prag Ice
Mills v UOI AIR 1978 SC 1298) The Court has not accepted the submission that
when an Act has been protected all subordinate legislation (mles orders etc)
would be equally entitled to protection. In other words the theory of derivative
immunity has been rejected.
Article 31B Acts included in the 9th Schedule subsequent to the date of
decision of Keshavanand Bharati (24-4-1973) are open to challenge on the
ground of being opposed to the basic structure and to the mandate of Equality
and Fundamental Rights (Waman Rao v UOI MR 1981 SC 271)
The protection given by Art. 31B covers an Act as it stood on the date it was |Uded
in the 9m Schedule. It would include all amendments made in the Act till 'h°t date.
Subsequent amendments have no claim to immunity. They must be in nformity
with the Fundamental Rights and other Constitutional limitations (Raman °°gl v state
of Gujarat AIR 1969 SC 168)
Misuse of Art. 31B - The object of insertion of Art. 31B and the 9th Schedule
originally was promotion of agrarian reforms and other similar welfare measures.
The Congress during the Emergency of 1975 inserted the Representation of
People Act, 1951, its amending Act of 1974 and the Election Laws (Amendment)
Act, 1975 in the 9th Schedule in order that the validity of election of Smt Indira
Gandhi, Prime Minister, may not be questioned in a court of law. This was done
by the 39th Amendment. The 44th Amendment brought by the Janata government
omitted these entries.
The Constitution (76th Amendment) Act, 1994 seeks to provide protection to
a Tamil Nadu Act which reserves 69 per cent seats for backward classes in
educational institutions and for appointments under the State. This is to
circumvent the ceiling of 50% allowed by the Supreme Court Such tinkering with
the Constitution for personal or party benefit generates disrespect towards the
1 5 . B K . Sharma, p. 112.
Constitution and is misuse 10
F u n d aofmconstituent
e n t a l R i power.
ghts: General 29

Art. 31C: Saving of Laws giving effect to Directive Principles11


Article 31C was introduced in the Constitution by the 25th Amendment with effect
from 20-4-1972. The object was to take away the jurisdiction of the courts to
determine adequacy of compensation on acquisition of property and to add a new
Art. 31C to lay down that no law which declared that it was for giving effect to the
principles specified in Clauses (b) and (c) of Art. 39 would be called in question on
the ground that it is inconsistent with the Fundamental Rights.
By the 42nd Amendment Act, 1976 the scope of protection was sought to be
enlarged by substituting the words "All or any of the principles laid down in Part IV’
for the words “the principles specified in Cl. (b) or Cl. (c) of Art. 39.”
This Article (31C) consisting of one clause, two sentences and spread over
ten lines has generated so much litigation that it must have consumed a few
thousand man days of the Supreme Court (not counting the High Courts). It was
considered by a 13 judge Bench (the largest so far constituted) in Keshavanand
(AIR 1973 SC 1461) and followed by Minerva Mills (1980) 3 SCC 625, Waman
Rao, and Sanjeev Coke (1983) 1 SCC 147, to mention the oft quoted. But the full
meaning and the effect of Keshavanand and other decisions is still a matter of
difference °f opinion. In 1996, the matter was referred to a Bench of 5 judges. A
Bench of
9 judges in 2007 answered the reference (I R. Coelho v State of T.N. AIR 2007 ^
861; discussed later in the present book).

\
B.K Sharma, pp. 112-113.
11 Explain the provisions of Art. 31-C of the Constitution of India. [D.U.-2008]
1
Art. 31-C and Keshavanand - Art. 31C contains the words that no law containing
30 a Constitutional Law o f India- II declaration that it
is for giving effect to such policy shall be called in question in any court on
the ground that it does not give effect to such policy. By the 25*h Amendment, Art
31-C containing this declaration ousting the jurisdiction of the courts was made a
part of the Constitution. The effect of the declaration clause was that if an Act
contained a declaration that is was to give effect to the policy of the State towards
securing the directive principles mentioned therein then the courts would be
debarred from examining the constitutionality of the Act or even that the
declaration is not founded on facts and is colourable.
The Supreme Court in Keshavanand struck down the ouster clause on the
ground that it destroys judicial review which is a basic feature of the Constitution.
It held the earlier part to be valid The part which said that if it is to implement the
policy contained in Art. 39 (b) and (c) then it shall not be declared to be void on
the -ground of violation of Arts. 14, 19 or 31. So after Keshavanand the court
continued to have the power of judicial review in relation to an Act covered by Art.
31-C.
Art. 31-C and Minerva Mills - The 42nd Amendment Act, 1976 (Sec. 4) sought to
extend the protection of Art. 31C to all those Acts which were meant to implement
any of the Directive Principles of State Policy. The effect of this extension was far
reaching and practically accorded primacy to the Directive Principles over the
Fundamental Rights.
In Minerva Mills the Supreme Court held Sec. 4 as unconstitutional and
consequently void. The Act destroyed the basic feature of the Constitution by
excluding from challenge all such laws which give effect to a Directive Principle.
Harmony and balance between Fundamental Rights and Directive Principles is a
basic feature of the Constitution. To give primacy to one over the other is to
disturb the harmony. The Court however declared that Art. 31-C as originally
introduced by the 25th Amendment was valid [Protecting Acts to implement Art
39(b) and (c) only].
The declaration made by legislature is not conclusive. It is justiciable. The
court may tear the veil to examine whether the declaration is colourable or is an
abuse of power, or whether it has direct and reasonable nexus with directive
principles (Bhimsingji v UOI AIR 1981 SC 234). But the economic considerations
for nationalization, the quantum of compensation, and necessity for nationalization
will not be judged by a court.
Art. 31-C and Sanjeev Coke - In Sanjeev Coke doubts were expressed on the
ratio of Minerva Mills and the main decision holding the extended declaration
clause as void was regarded as obiter. More areas of darkness and shadows
clamouring for light from the Supreme Court have emerged. 18

1 8 . B.K. Sharma, pp. 113-115.


Art. 31-D
Fund
Article 31-D was inserted bya 42
m endnAmendment
t a l R i g h t sand
: Gomitted
e n e r aby 31
l the 43rd Amendment.
It sought to save laws which provided for prevention or prohibition of anti-national
activities or the prevention of formation of or the prohibition of anti-national
associations. Such laws were to be immune from challenge on the ground of
violation of Arts. 14, 19 or 31. The immunity was conferred only to Central laws.
Thus the majority at Centre could easily pave the path to dictatorship and one party
rule by outlawing other parties This gift of emergency of 1975 was omitted by the
43 rd Amendment. 1 9

ARTICLE 33 [POWER OF PARLIAMENT TO MODIFY


FUNDAMENTAL RIGHTS]

Article 33 empowers Parliament to modify the application of Fundamental Rights


to the Armed Forces or forces charged with maintenance of public order, etc. in
the interest of discharge of duties and maintenance of discipline. There are certain
arms of the executive where freedom has to be controlled.
Art. 33 provides that “Parliament may by law, determine to what extent any of
the fundamental rights shall, in their application to-
(a) the members of the Armed Forces; or
(b) the members of the Forces charged with the maintenance of public
order; or
(c) persons employed in any bureau or other organization established by
the State for purposes of intelligence or counter intelligence; or
(d) persons employed in, or in connection with, the telecommunication
systems set up for the purposes of any Force bureau or organization
referred to in clauses (a), (b) and (c),
be restricted or abrogated so as to ensure the proper discharge of their
duties and the maintenance of discipline among them.”
Thus, Art. 33 confers on the Parliament power to make law setting out the limits
within which the fundamental rights would be available to the members of the
organizations specified in Art. 33 [UOI v LD. Balam Singh (2002) 9 SCC 73].
Though the appellate jurisdiction of courts is excluded in cases of court martial,
the writ jurisdiction remains (unless taken away by law under Art. 33).
Article 33 is an exception to the fundamental rights conferred by Part III of
the Constitution. This Article is an exception to the operation of Art. 13(2), which
prohibits
taking away or abridgement of the rights guaranteed by Part III of the Constitution.
Hence, a law passed under Art 33 cannot be challenge under Art. 13(2). The
power under Art. 33 is only exercisable by Parliament and not by State
Legislatures.
The Army Act, The Navy Act, The Air Force Act. The Border Security Force
Act, etc. curtail the right of association under Art 19(1 )(c) The Police Forces
(Restriction of Rights)
1 9 . B.K. Sharma, p. Act,
115. 1966 declares that a member of police force cannot be
a member of trade union or labour union or political association
Certain civil employees working as cooks! chowkidars, mechanics, tailors,
etc. attached to defence establishments claimed the right to form associations as
they were \ non-combatants. The Supreme Court rejected their contention and held
that they are members of the defence services within the meaning of Art. 33.
32 Constitutional Law o f India I!
Consequently their right can be curtailed (Achudan v UOI AIR 1976 SC 1179).

ARTICLE 34
[RESTRICTIONS ON FUNDAMENTAL RIGHTS DURING OPERATION OF
MARTIAL LAW]

Article 34 provides' ‘ Notwithstanding anything in the foregoing provisions of this


Part, Parliament may by law indemnify any person in the service of the Union or of
a State or any other person <n respect of any act done by him in connection with
the maintenance or restoration of order in any area within the territory of India
where martial law was in force or validate any sentence passed, punishment
inflicted, forfeiture ordered or other act done under martial law in such area."
Under Art. 34, Parliament may by law indemnify any person for anything
done in contravention of fundamental rights for maintenance of order during the
operation of martial law. In other words, if during martial law any illegalities are
committed in the restoration of order the Parliament is given the power to pass an
Act of Indemnity. This power of Parliament is subject to Mo conditions viz :
(1) the act must have been done in connection with the maintenance or
restoration of order; and
(2) martial law must be in force in the area where the act was done
In the proper sense of the term, martial lav/ means “the suspension of ordinary
law and the government of country or part of it by military tribunals” [Wharton Law
Lexion (1966) 18 Law Ed. 281, 295] The expression is also used to denote
‘military law.’ Martial Law, thus, is the action of the military authorities, imposing
restrictions and regulations on civilians, in order to deal with a situation amounting
to war or insurrection or rebellion in any part of the country
The Constitution does not have a provision authorizing proclamation of
martial law However, it is implicit in the text of Art 34 that the Government may
declare martial law in any part of the territory of India. Also, the President may
issue such a proclamation under Art. 53(2) as the Supreme Commander of the
Defence Forces of the Union.
It is the events, which result in a martial law. If a proclamation is so made H
is merely a notification to the people that the military authorities have assumed
control of the area. Art. 34 visualises a situation of martial law. It envisages that
fundamental rights will have to be infringed during such a situation

Martial Law and Emergency - Distinguished


“MartialLaw" and “Emergency” as envisaged under Art. 352 of the Constitution,
may be distinguished as two different concepts. While, a Proclamation under Art.
352 may be made in anticipation of breakdown of the law and order, the basis of
martial law is actual rebellion or insurrection. During an Emergency (which
depends on the satisfaction of the Executive regarding threat to the security of
India), there may be no breakdown of law and order, the Government and the
courts continue to function normally and the military authorities do not take charge
of maintaining law and order. On the other hand, during martial law, which
depends on the factual existence of insurrection or rebellion, military authorities
take charge of the situation, the court's interference with military action
automatically comes to an end and they do not function to enforce the right of the
inhabitants.
While martial law can come into being only in the actual area of rebellion,
Fundamental Rights: General 33
Emergency under Art. 352 may be imposed throughout the territory of the
country. While martial law relates to and affects the maintenance of law and
order, Emergency may affect many other aspects of the Government. For
example, the distribution of powers undergoes change during Emergency while,
martial law does not affect this aspect of the Constitution.

ARTICLE 35 [LEGISLATION TO GIVE EFFECT TO THE PROVISIONS


OF PART III]

Art. 35 lays down that the power to make laws to give effect to certain specified
fundamental rights shall vest only in the Parliament and not in State Legislatures.
The object is that fundamental rights, both as to their nature and as to the
punishment involved in their violation, shall be uniform throughout India.
Parliament may exercise this power even thought that matter might, fall in the
exclusive field of the State Legislatures.
Article 35(a)(i) empowers Parliament to make laws with respect to any of i ic
Matters which under Cl. (3) of Art. 16, Cl (3) of Art. 32, Art 33 and Art. 34 may
provided for by law made by Parliament. Art 35(a)(ii) further confers on
Parliament, power to make laws for prescribing punishments for those acts which
ar
5 declared offences under P3rt lit relating to fundamental rights For -^xampie. Art
,

/
17
3 4declares that practice
C o n s of'
t i tUntouchability”
utional Law in any
o f of
I nits
d iform
a - I shall
I be an offence
punishable by law. Likewise, Art. 23(1) prohibits ' traffic in human beings” “begar"
and “forced labour” and declares the contravention of this prohibition as an
offence punishable by law.
Art. 35(b) lays down that any law existing on the date of the commencement
of the Constitution and dealing with the matters mentioned in sub-clauses (i) and
(ii) Art. 35(a), shall remain in force until repealed, altered or abrogated by
Parliament For example, the Mulki Rules prescribing the condition of residence
within the whole of Hyderabad for appointment to offices in that area are saved
and continued in force by Art. 35(b) (State of UP v. Banwari AIR 1951 SC 615)

QUESTIONS AND A N S W E R S

Q.1 Regarding the expression "other authorities under the control of the
Government' in Art. 12, the Supreme Court in recent times has
been adopting a liberal interpretation. Thus, in Som Prakash \i
Union of India, it observed that the emphasis is on functionality plus
State control, rather than on the statutory character of the
corporation There is no reason to make exclusion on sophisticated
grounds such as that the legal person must be a statutory
corporation, must have power to make law, must be created by and
not under a statute and so on ... Discuss with the help of important
decisions. [C.L C.-94/95]

Write a short note on meaning of the term 'State under Art 12.
(L C l/L C 11-20062007]
A.1 Article 12 reads: “Unless the context otherwise requires, the term 'State’
include the following - executive and legislature of Union and States; all
local or other authorities within territory of India or under the control of
government’'
In the context of Art. 12, 'authority' means the power to make laws, bye-laws, etc.,
which have the force of law and power to enforce those laws
In Electricity Board, Rajasthan and Sukhdev Singh v Bhagatram cases, the
Supreme Court held that only the authorities created by the Constitution or Statute
are the ‘other authorities', though it is not necessary that statutory authority should
be engaged in performing governmental or sovereign functions Thus, Rajasthan
Electricity Board, ONGC, LIC, etc are held to be ‘other authorities' These bodies
have power to make bye-laws, regulations, etc. for regulating service conditions.
But in subsequent decisions, the Supreme Court has given a broad and
liberal interpretation to the expression 'other authorities’ in view of the fact that in a
welfare State, a government has to perform manifold functions for which it has to
employ various agencies or instrumentalities (which are not created by statute).
Tests to determine whether a body is an agency or instrumentality of government
(International Airport Authority case) -
(i) State mainly or chiefly contributed to the financial resources.
(jj) Deep and pervasive State control e g appointment and removal of
members
of a society, etc.
(ijj) Functions of corporation are of public importance i.e. governmental in
essence,
(iv) If a government department is transferred to a corporation,
(V) Corporation enjoys monopoly status which is State-conferred
Applying this test in Som Prakash v Union of India, the court held that a
government company (Bharat Petroleum Corporation) is ‘State’. The expression
‘other authorities' is not confined only to statutory corporation alone, but may
include a government company, a registered society, or bodies which have some
Fundamental Rights: General 35
nexus with the government. The emphasis is on ‘functionality plus State control’ rather
than the statutory character of the corporation.
In Ajay Hasia case, held that a registered society is-an agency or
instrumentality of the State. The enquiry has to be not as to how the juristic
person is created but why it has been brought into existence. It is immaterial
whether the corporation is created by or under a statute e.g. FCI created by Food
Corporation Act. 1964 and a company registered under the Indian Companies
Act. In this case held that the Regional Engineering College established and
administered by a registered society is a 'State'. The composition of the society
was dominated by the representatives of the Central and State Governments, the
rules to be made by the society had to be approved by the Central Government
and the accounts of the society were required to be submitted for government
approval and scrutiny.
In J.S. Ameja v National Co-op. Consumers Federation of India Ltd. (AIR
1995 Del 44) held that the National Co-op. Consumers Federation is not a ‘State’
under Art. 12. The entire share capital of Co-operative is not held by the
Government nor does it receive any financial assistance or subsidy from
Government but runs its own business on commercial principles by raising its own
funds, resources and obtaining loans from banks. It does not enjoy monopoly
status, it is an independent body functioning according to its bye-laws, according
to which the Board of Directors is the authority responsible for the management,
administration, business and functioning of Federation.
In Tekraj \/asandi v Union of India, held that the 'Institute of Constitutional
and Parliamentary Studies' is not a ‘State’ as the object of society is not related to
governmental business and in the functioning of it, government's control is not
deep and pervasive though government has a say in the matter of making grant.
Similarly in Chandra Mohan Khanna v NCERT, held that N.C.E.R.T. is not a State
as the governmental control is confined only to proper utilization of the grant. It is
an autonomous body. Art. 12 should n°t be stretched so as to bring in every
autonomous body which has some nexus with the government within the sweep
of the expression 'State'.
In the final analysis, it could be said that the Supreme Court has widened
the net of the expression “other authorities" to cover bodies and institutions
irrespective of the fact that they are created by or under a statute, with a view to
bringing under the discipline of Fundamental Rights (It is important to note that the
fundamental r'9hts are available only against the 'State' and not private individuals
or institutions).
Q.2 (a) Critically examine the concept of "State” as defined in Art. 12 of the
Constitution. What is the test to determine whether an institution or
corporation is an agency or instrumentality of the State?
[I. A S -92/95]
(b) Can a non-governmental company be called a “State" within the
meaning of Art. 12 of the Constitution?
Discuss and state whether Art. 12 also includes private persons
[I A.S -97]
A.2 (a) Please refer to answer to Q.1.
(b) In MC. Mehta v Union of India (1987) 1SCC 395, the question was
whether a private corporation (Shriram Food and Fertilizers Ltd.) is
a “State” under Art. 12 It was contended that Shriram Company is
State’ because it is carrying on an industry vital to the public interest
with a potential to affect the life and health of the people under the
active control of the government. Furthermore it is carrying on an
industry which according to the declared policy of the government
was ultimately intended to be carried out by the government itself It
XVI was also pointed
C o n s tout
i t uthat
tiona asizable
l L a wfinancial
o f Ind aid
i acomes
- II from the
government. Taking aid of the American State Action doctrine, it
was also argued that a private activity if supported, controlled or
regulated by State may get so intertwined with governmental activity
as to be termed State action and it would then be subject to the
same constitutional restraints as the State. The court observed that
a non-governmental company can be placed within the meaning of
Art 12, if for reasons of State control and regulations and kind of
public function they are performing, they satisfy the tests of
instrumentality or agency of government The Corporate device will
not be allowed to be used as a barrier for ousting the Constitutional
control of fundamental rights
A non-governmental company like the United India Insurance Co. which is
given monopoly rights on general insurance business under the General
Insurance Business (Nationalization) Act, 1972 has been held ‘State”, as it has
the trappings of the State.
In Unni Krishnan v State of A P. (1993) 1 SCC 645. the court observed that
the term ‘authority’ used in Art. 226 must receive a liberal meaning unlike the term
in Art. 12 because Art. 226 confers powers on High Court not only for the
enforcement of fundamental rights but for the other rights aiso Therefore, the term
'authority' as used in Art. 226 must not be confined only to statutory authorities or
instrumentalities of the State. It may cover any other person or body performing
public duty’. Thus, a private medical/engineering college comes within the writ
jurisdiction of the court irrespective of the question of aid and affiliation However,
it would not be ‘State’ within the meaning of Art. 12.

Right to Equality (Articles 14-18)

'Right to Equality is the first fundamental right assured to the people of India.
Article 14 embodies the idea of equality expressed in the Preamble. The
succeeding Arts. 15, 16 and 17 lay down specific applications of the general rules
laid down in Art. 14. Art. 14 is the equality clause because of its wide ambit and
applicability. It applies to all persons while Art. 15 and others cover only citizens.
In the second India Sawhney case it has been held that the principle of equality
enshrined in Art.
14 is a basic feature of the Constitution.

ARTICLE 14 [EQUALITY BEFORE LAW]

“The State shall not deny to any person equality before the law or the
equal protection of the laws within the territory of India.’’
Equality is a cherished ideal of humanity. The famous author A. V. Dicey, an
authority on the British Constitution, had propounded his principles of Rule of
Law. ‘Equality before the law’ is part of that enduring concept The words ‘Equal
protection of the law’ is part of the U.S. Constitution 14thAmendment. The same
Right to Equality 37

phrase occurs in Art. 7 of the Universal Declaration of Human Rights, 1948. Even
the Preamble to the U.N, Charter has reference to equality.
While “equality before law" is a somewhat negative concept implying the
absence of any special privilege in favour of individuals and the equal subject of all
classes to the ordinary law. It means no man is above the law and that every
Person, high or low, is subject to the ordinary law of the land and amenable to the
Jurisdiction of the ordinary courts.
“Equal protection of iaw" is a more positive concept (as it expects a positive
a
ction from the State) implying equality of treatment in equal circumstances. In
other Words, all persons who are in the same circumstances will be governed by
the
same set of rules. It is a guarantee of equal treatment Equal law should be applies j
with an equal hand to all persons who are equals. The rule is that like should be
treated alike and not that unlike should be treated alike.
In State of W.B. v Anwar Ali Sarkar (discussed below), the Court rightly
observed that the second expression is the corollary of the first and it is difficult to
imagine a situation in which violation of equal protection of laws will not be the
violation of the equality before law. Thus, in substance the two expressions mean
one and the same thing
In Re Special Courts Bill, 1978 (AIR 1979 SC 478), Chandrachud, J.
observed: ] "The underlying principle of the guarantee of Art. 14 was that all
persons similarly circumstanced should be treated alike both in privileges conferred
and liabilities j imposed.”
Availability of right to equality - The words “any person” in Art. 14 denotes that
equal protection of law is available to any person which includes any Company or
association, etc. Art. 14 extends to both citizens and non-citizens and to natural
persons as well as legal persons. The equality before law is guaranteed to all
without regard to race, colour or nationality. However, an alien (foreign national)
cannot claim equal rights (under Art. 14) with that of an Indian national, in relation
:
to the grant of Indian citizenship.
Exceptions to rule of equality- Under Art. 359, when the proclamation of emergency
1 is in operation the enforcement of Art. 14 may be suspended during that period. -
Art. 361 provides that President and Governors shall not be answerable to any ;
court for the exercise and performance of the powers and duties of office. They 1
also enjoy immunity from criminal and civil proceedings until certain conditions are
fulfilled.
Members of Parliament and of State legislatures are not liable in respect of
anything done or said within the House (Arts. 105 and 194). Foreign Diplomats are

/
XVI
Constitutional Law o f India- II
immune from the jurisdiction of courts. Art. 31C forms an exception by excluding 1
some laws [for implementing any of the directive principles specified in Art. 39(b) or
(c)] from the purview of Art. 14.

Article 14 permits Reasonable Classification but prohibits Class


Legislation12
Article 14 does not mean that all laws must be general in character or that the same
1 laws should apply to all persons or that every law must have universal application,
I for, all persons are not, by nature, attainment or circumstances, in the same
positions The State can treat different persons differently if circumstances justify 1
such treatment. In fact, identical treatment in unequal circumstances would amount
to inequality The legislature must possess power to group persons, objects and 1
transactions with a view to attain specific aims. So, a reasonable classification is n0t
permitted but necessary if society is to progress
By the process of classification, the State had the power of determining who
should be regarded as a class for purposes of legislation and in relation to a law
enacted on a particular subject. Classification meant segregation in classes which had
a systematic relation, usually found in common properties and characteristics. It
postulated a rational basis and did not mean herding together of certain persons
and classes arbitrarily [Re Special Courts Bill, 1978 AIR 1979 SC 478].
Class legislation is that which makes an impropsi discrimination by conferring
particular privileges upon a class of persons arbitrarily selected. And no reasonable
distinction can be found justifying the inclusion of one and exclusion of other from
such privilege. While Art. 14 forbids class legislation, it permits reasonable
classifications of persons, objects and transactions by the legislature for the
purpose of achieving specific ends. In other words, what Art. 14 prohibits is class
legislation and not classification for the purpose of legislation [Sfafe of A P. v N.R,
Reddi (2001) 7 SCC 708],

Test of reasonable classification13


The classification must not be “arbitrary, artificial or evasive”, and it must fulfill
following two conditions -
(a) it must be founded on an intelligible differentia which distinguishes
persons or things that are grouped together from others left out of the
group, and
(b) the differentia must have a rational relation to the object sought to be
achieved by the Act (Anwar Ali case, below).
The differentia which is the basis of classification and the object of the Act are two

12 What is prohibited is hostile discrimination, and not reasonable


classification? Discuss. [/.AS -99]
13 Write a short note on: 'Test of reasonable classification’. [1C. 11-2006]
“The extensive use of the device of ‘reasonable classification’ by State and its
approval by the Supreme Court has rendered the guarantee of 'fair and
equitable treatment' under Art. 14 largely illusory." Discuss making clear the
constitutional and popular concepts of 'right to equality' in India. [I.A.S.-2004]
Right to Equality 39

distinct things. What is necessary is that there must be a nexus between the basis
of classification and the object of the Act which makes the classification. There
should be a reasonable basis for a classification
To illustrate by an example: Under Sec. 11 of Indian Contract Act, persons
who haven’t attained majority can’t enter into a contract. The two categories are
adults and minors. The basis of classification is the age, which obviously has a
relation to the capacity to enter into contract. The section, therefore, satisfies both
requirements of a valid classification. But suppose that the basis of classification is
coiour of hair e.g. all persons having black hair may contract. This classification
would fail because the differentiation has no relation to the object of legislation - the
capacity to contract.

/
Where a Special
C o n sAct
t i t uprovides
t i o n a la Lsummary/
a w o f Ispeedier
n d i a - Iprocedure
I (but harsher
and more onerous than the ordinary civil procedure) for eviction of a person from
unauthorized occupation of government premises, in N.l Caterer's case (AIR 1967
SC 1581), held that the Act is discriminatory. But, in M Chagganlal v Greater
Bombay Municipality (AIR 1974 SC 2009), it held that “government premises” is a
sufficient guidance for authorities to take action under special procedure It was laid
down:
(i) Where a statute providing for a more drastic procedure different from the
ordinary procedure covers the whole field covered by the ordinary
procedure, without any guidelines, as to the class of cases in which either
procedure is to be resorted to, the statute will be hit by Art. 14 (Anwar Ali
case, below).
(ii) However, if from the preamble and surrounding circumstances, as well as
the provisions of statute themselves explained and amplified by affidavits,
necessary guidelines could be inferred, the statute will not be hit by Art.
14 (Kathi Raning v State of Saurashtra AIR 1952 SC 123).
Various decisions have established important guidelines or principles which further
clarify the “scope of permissible classification” [Ramkrishna Dalmia v Justice
Tendolkar AIR 1958 SC 538; Re Special Courts Bill. 1978 AIR 1979 SC 478; R.K.
Garg v UOI AIR 1981 SC 2138; D.S. Nakara v UOI AIR 1983 SC 130]:
(1) A law may be constitutional even though it relates to a single individual if
on account of some special circumstances that single individual may be
treated as a class by itself.
(2) There is always a presumption in favour of the constitutionality of a
statute (the courts does not favour a construction which reduces the
statute to a futility). However, the presumption may be rebutted in certain
cases by showing that there is no classification at all and no difference
peculiar to any individual or class and not applicable to any other
individual or class, and yet the law hits only a p?rticular individual or
class.
(3) In order to sustain the presumption of constitutionality, the court may take
into consideration matters of common knowledge, the history of the times
and may assume every state of facts which can be conceived existing at
the time of the legislation. However, the presumption cannot be earned to
the extent of holding that there must be some undisclosed or unknown
reasons for subjecting certain individuals to the discriminating legislation.
(4) It must be presumed that, while making classification, the legislature
understands and correctly appreciates the needs of its own people, that
its laws are directed towards problems made manifest by experience and
that its discriminations are based on adequate ground
(5) The legislature is free to recognize degrees of harm and may confine its
restrictions to those cases where the need is deemed to be the clearest.
(6) Mere inequality in no manner would determine the matter of
constitutionality, for the very idea of classification implied inequality.
(7) The classification made by a legislature need not be scientifically perfect or
logically complete. Mathematical nicety and perfect equality are not
required. Similarity, and not identity of treatment, is enough. Equal
treatment does not mean identical treatment. Mere differentiation or
inequality of treatment, does not perse amount to discrimination. It does
not matter that there are some categories distributed across the dividing
line, as long as, there is a broad discernible classification based on
intelligible differentia. The over- or under-inclusiveness of the
classification, if marginal, is permissible.
(8) The question whether a classification is reasonable and proper or not,
must, however, be judged more on common-sense than on legal
subtleties.
(9) If the legislative policy was clear and as an effective method of carrying
out that policy, a discretion was vested by the statute upon a body of
administrators or officers to selectively apply the law to certain classes or
groups of persons, the statute itself could not be condemned as a piece of
discriminatory legislation.
(10) A legislation may not be amenable to challenge on ground of violation of
Art. 14, when it is intended to give effect to principles specified under Arts.
15 or 16 or when the differentiation is not unreasonable. But, when a
classification is made, which is perse violative of constitutional provisions,
the same cannot be upheld.
(11) Substantive law and procedure law are both subject to the mandate of Art.
14. Thus, classification should be reasonable both from substantive and
procedural stand points.

Basis of Classification
The basis of classification may be different e g. geographical, vocational, difference
in time, difference in nature of persons, trade or occupation, etc.
Examples. A tenancy law may be necessary only for a part of the State because the
conditions of tenants vary from locality to locality. Similarly, a rule is not violative of
Art. 14 if it imposes a capitation fee on the non-resident students of a State and
exempts the resident students, because the State has to contribute for the upkeep
and running of its educational institutions.

A Single Individual may constitute a Class14

Leading Case: chiranjit lal chaudhary V UOl (AIR 1950 SC 41)


In this case, it was held that a single individual may constitute a class for
the purpose of Art. 14. In this case, the management of a company in

[L.C.1-2006]
14 Write a short note on: Single person laws under Art. 14.
XVI
Constitutional Law o f India- II
Sholapur was taken over by the Government by passing the Sholapur Spinning
and Weaving Co. (Emergency Provision) Act The Act was challenged by a
shareholder of the company on the ground that a single company and its
shareholder was being denied equality before the law, because the Act treated
him differently vis-a-vis other companies and their shareholders. In other
words, law had selected one particular company and its shareholders and had
taken away from them their r ight to manage their own affairs but the same
treatment had not been meted out to all other companies or shareholders in an
identical manner.
Holding the Act to be valid, the Apex Court said that a law may be
constitutional even though it applies to a single individual if, on account of
some special circumstances or reasons applicable to him and not applicable to
others, that single individual may be treated as a class itself, unless it is shown
that there are others who are similarly circumstanced. The Legislature is free to
recognize the degree of harm and it may confine its restriction to those cases
where the need is deemed to be the clearest.
In the present case the Sholapur Company formed a class by itself
because the mismanagement of the company's affairs prejuaiaally affected the
production of an essential commodity and had caused a serious
unemployment amongst labourers. A corporation which is engaged in
production of a commodity vitally essential to the community, has a social
character of its own and it must not be regarded as the concern primarily or
only of those who invest their money in it. It is in the interest of the community
at large that the Legislature has treated it as a class by itself and made special
legislation applicable to it alone.]

Special Courts and Procedural Inequality

Leading Case: state of w.b. V anwar ali sarkar (AIR 1952 SC 75)

Facts and Issue - IP this case, the validity of the respondents, Anwar Ali and
others, by the special courts established under Sec 5(1) of West Bengal
Special Courts Act, 1850, was at issue. The Act was passed ' to provide for the
speedier trial of certain offences" as stated in Preamble It prescribed a special
procedure which courts had to follow in the trial of cases referred to it.
Observations - The Supreme Court observed that such a law will not be
violative of Art. 14 if it lays down proper guidelines for classifying “offences",
“classes of offences", or “cases” or "classes of cases" to be tried by special
courts. But the special procedure prescribed by such a law should not be
substantially different from the procedure under ordinary law.
Right to Equality 43

The differentia which is the basis of the classification and the object of the
Act are two different things. Thus, the object by itself cannot be the basis of
classification of offences or cases, for in the absence of any special circumstances
which may distinguish one offence or case from another offence or case, the
speedier trial would be desirable in disposal of all offences or cases. On the other
hand, there are certain offences e g frequent incidents of theft, dacoity, kidnapping,
etc. in an area or arson, loot, rape, etc. during communal riots in particular localities
which legitimately call for a speedier trial in the interest of the community. Similarly
political murders or crimes against the State may require speedier trial. These
special circumstances add a peculiar quality to these offences, or classes of
offences, or classes of cases which distinguish them from stray or isolated cases of
similar crimes.
The court said that the Act did not lays down any basis for classification nor
did it mention clearly what kind of cases were to be directed for trial by special
courts. The object as stated in the Preamble, "speedier trial of certain offences” was
too vague, uncertain and illusive to afford a basis for reasonable classification Sec.
5(1) which authorises Government to direct "cases” to be tried by the special court,
contravened the Art. 14 as it conferred arbitrary powers on the Government to
classify offences or cases at its pleasure.
Decision - The majority held that the procedure laid down by the Act varied
substantially from the procedure laid down for trial by Criminal Procedure Code Art.
14 applies to both, the substantive as well as procedural law, as discrimination can
occur in both.]

Leading Case: kathi raning rawat V STATE OF


SAURASHTRA (AIR 1952 SC 123)

In this case, the object as mentioned in the Ordinance wss to provide for “public
safety, public order and preservation of peace and tranquility” ir the State of
Saurashtra. Thus, four distinct categories of “offences” (viz offences or classes of
offences or cases or classes of cases) could be directed by the Government to be
tried by the Special Courts established under the Ordinance The procedure under
the Ordinance was harsher than that provided under the Criminal Procedure Code
Notwithstanding this Ordinance, the crimes went on increasing and there
occurred numerous cases of dacoity, murder, nose-cutting, ear- cutting, etc for
some of which certain notorious gangs were responsible, and hence the Ordinance
was amended to constitute Special Courts for the speedy trial of cases arising out
of the activities of the dacoits and other criminals guilty of violent crimes in certain
areas. The amended Ordinance thus had the same object as the original
Ordinance.
It was heldCthat
o n sint comparison
i t u t i o n a l toLthe
a wWest
o f Bengal
I n d i a Ordinance
- II (State of W.B. v
Anwar Ali Sarkar case), the Saurashtra Ordinance clearly laid down the guiding
principle i.e. to provide for public safety, public order and preservation of peace and
tranquility in the State. The mere mention of the speedier trial in the West Bengal
Act did not cure the defect because the expression afforded no help in determining
what cases or class of cases or offences or class of offences required speedier trial
The court observed: A distinction should be drawn between 'discrimination
without reason’ and ‘discrimination with reason' The whole doctrine of classification
is based on this distinction and on the well- known facts that the circumstances
which govern one set of persons or objects may not necessarily be the same as
those governing another set of persons or objects, so that the question of unequal
treatment does not really arise as between persons governed by different conditions
and different set of circumstances. The main objection to the West Bengal Act was
that it permitted 'discrimination without reason' or without any rational basis. In the
present case, one could discover a nexus (which was missing in the W.B. Act)
between offences of a particular category and the object with which the Ordinance
was promulgated, as should suffice to repel the charge of discrimination and furnish
some justification for the special treatment of those offences.
In the present case, the Notification also specifies certain areas in the State
over which only the Special Courts is to exercise jurisdiction. There can be no
dispute that if the State Legislature finds that lawlessness and crime are rampant
and there is a direct threat to peace and tranquility in certain areas within the State,
it is competent to deal with offences which affect the maintenance of public order
and preservation of peace and tranquility in certain areas as a class by themselves
and to provide that such offences shall be tried as expeditiously as possible in
accordance with a special procedure devised for the purpose
The court observed: If from the preamble and surrounding circumstances, as
well as the provisions of statute themselves explained and amplified by affidavits,
necessary guidelines could be inferred, the statute will not be hit by Art. 14. The
legislature is given the utmost latitude in making the classification and it is only
when there is a palpable abuse of power and the differences made have no rational
relation to the objectives of the legislation, that necessity of judicial interference
arises.
The court further observed: If the impugned legislation indicated the policy
which inspired it and the object which it seeks to attain, the mere fact that the
legislation does not itself make a complete and precise classification, but have the
selective application of the law to be made by the executive authority in accordance
with the policy/object, is not a sufficient ground for condemning it as arbitrary
It was argued that the list of classification of offences mentions
offences of a particular character but excludes other offences of a cognate
character. For example, while Sec. 183, IPC is mentioned in the list, Secs.
184, 186 and 188 which deal with similar offences were excluded. Similarly,
the list does not mention Sec. 308, IPC (attempt to commit culpable
homicide), though it mentions Sec. 307 (attempt to commit murder). The
court observed: A meticulous examination of the various offences specified
in the list with regard to jeir nature and punishment is not necessary for
Right to Equality

purposes of this case. Simply because certain offences which could have
been mentioned along with similar other in a particular group have been
omitted therefrom, it cannot be said that the ■sole list is bad. The appellant
is accused of murder and dacoity and no offences of a similar nature are
excluded from the list.

Leading Case: m. chhagganlal v greater BOMBAY MUNICIPALITY 15


(AIR 1974 SC 2009)

Facts and Issue - In this case, the question was to the validity under Art. 14
of the Acts, which conferred on the authorities the power to initiate eviction
proceedings against unauthorised occupants of Corporation and
Government premises The two Acts in question were: The Bombay
Municipal Corporation Act, 1888, and, The Bombay Government Premises
(Eviction) Act, 1S55. The procedure under these Acts were alleged to be
harsher and more onerous than that available by way of a suit under the
ordinary law i.e. Civil Procedure Code (C.P.C.). The Supreme Court upheld
the validity of the impugned Acts.
The N.I. Caterers Lt Governor of Delhi (AIR 1967 SC 1581) was
examined afresh in this case. In that case, Sec. 5 of Punjab Public
Premises and Land (Eviction & Rent Recovery Act) Act of 1959 which
provided for a speedy remedy to the Government for evicting unauthorised
tenants, was questioned under Art. 14, as two processes were available to
Government for evicting a tenant, one under the CPC., and another which is
more drastic, under Sec 5 of said Act, and the Government could choose to
proceed under either of them The Supreme Court held that section to be
discriminatory and so void, as two tenants of the Government could be
exposed i differential treatment and there were no intelligible guidelines for
exei sc of Government's discretion.

15 Evaluate the relevance of nexus test" in determining vioiation of Art. l-» in


instances where the statutes themselves make a classification and in those
instances, which authonzes the executive to make the classification Discuss in the light
of relevant case lav/.
XVI
Constitutional Law o f India- II
Following the N.l Caterer’s case, it was argued that on the availability of
procedures, one more onerous and harsher than *he other, the former is hit by Art.
14 in the absence of any guidelines as to which procedure might be adopted.
Observations and Decision - The court took a different approach in present case.
The court said that where a statute providing for a more drastic procedure different
from the ordinary procedure covers the whole field covered by the ordinary
procedure, without any guidelines as to the class of cases in which either procedure
is to be restored to, the statute will be hit by Art. 14 (Anwar Ali case, Suraj Mall
Mohta case - AIR 1954 SC 545). Even there, a provision for appeal may cure the
defect. Further, in such cases if from the preamble and surrounding circumstances,
as well as the provisions of statute themselves explained and amplified by affidavits 1
necessary guidelines could be inferred the statute wili not be hit by Art. 14. Then
again where the statute itself covers only a class of cases the statute will not be bad
(Kathi Ranning v State of Saurashtra AIR 1952 SC 123, Jyoti Pershad case AIR
1961 SC 1602)
The court observed... the Saurashtra case would seem to lay down the
principle that if the impugned legislation indicated the policy which inspired it and
the object which it seeks to attain, the mere fact that the legislation does not itself
make a complete and precise classification, but have the selective application of the
law to be made by the executive authority in accordance with the policy/object, is
not a sufficient ground for condemning it as arbitrary.
In considenng the validity of impugned legislation, it would first be necessary
to ascertain the policy and the object of statute Then, court should apply the dual
test i.e. is classification based on intelligible differentia and has the basis of
classification any rational nexus with policy/object. If both these tests satisfied, the
statute will be valid, and the consideration as to whether the same result couldn’t
have been better achieved by adopting a different classification would be outside
the scope of judicial inquiry.
In the present case, the statutes lay down the purpose behind them i.e
premises belonging to Corporation and Government should be subjected to speedy
procedure in evicting unauthorised occupants. This is a sufficient guidance for
authorities to take action under special procedure and not resort to the time
consuming ordinary procedure of C.P.C. It would be extremely unreal to hold that
an administrative officer would in taking proceeding for eviction of unauthorized
occupants of Government/Municipal property resort to the procedure prescribed by
the impugned Acts in one case and to the ordinary civil court in the other. In
considering whether the officers would be discriminating between one set of
persons and another, one has got to take into account normal human behaviour
and not behaviour which is aonormal It is not every fancied possibility of
discrimination, but the real risk of discrimination that we must take into
account. This is not one of those cases wheie discrimination is writ large on the
face of the statute.
Furthermore, the fact that the Legislature considered that the
ordinary procedure is insufficient or ineffective in evicting unauthorized
occupants of Government/Municipal property and provided a special
speedy procedure therefor is a clear guideline for the authorities
Right to Equality
charged with the duty of evicting unauthorized occupants. It was further
observed that minor differences between the special procedure and
ordinary procedure were not sufficient in invoking the inhibition of
equality clause In the latter, two appeals were available, but in the
former the provision for second appeal were taken care of by giving an
option to the aggrieved party to resort to the High Court.)
In Re Special Courts Bill, 1978 (AIR 1979 SC 478), the Parliament set up special
courts for the speedy trial of offences committed by the holders of high public
offices during the emergency period (1975-1977). The Apex Court held that the
classification made by the Bill was valid and it did not infringe Art 14, as it
classified both "offences” and “class of offenders”, the former in relation to the
period and in relation to objective that it was imperative to decide such cases
speedily, and the latter in relation to their status i.e. holders of high public offices
When both these conditions existed, it was only then the prosecution could be
instituted in the special courts. The offences alleged to have been committed
during the emergency period constitute a ‘class’ by themselves and so do the
persons who have utilized the high public offices for the purpose of committing
those "offences”. Thus, there was a close relationship between the basis of
classification and the object of the Bill, which is speedier trial.

New Concept of Equality: Arbitrariness and Art. 145


While the traditional concept of equality is based on the doctrine of classification,
the new concept is based on the doctrine of arbitrariness Art 14 had a highly
activist magnitude and it embodies a guarantee against arbitrariness Arbitrariness
is the very antithesis of equality [E.P Royappa v State of T.N AIR 1974 SC 555].
Non-arbitranness ls an essential facet of Art. 14 pervading the entire realm of State
action governed by Art, 14. Natural justice in turn is an antithesis of arbitrariness.
It therefore follows

*• “The horizons of equality as embodied in Art. 14 have been expanding as a


result of judicial pronouncements and Art. 14 has now come to have a highly
activist magnitude”. Critically examine. [L.C. 1-
2006]
".... Art. 14 strikes at arbitrariness in State action and ensures fairness and
equality of treatment’. The principle element of equality or non-arbitrariness
pervades Art. 14 like brooding omnipresence." Comment and cite case-laws.
[C.L. C.-2007]
“In fact, equality and arbitrariness are sworn enemies; one belongs to the
rule °f law in a republic while the other, to the whim and caprice of an
absolute monarch.” Elucidate with the help of constitutional provisions and
judicial decisions. [I A.S -
2006]
that audi4 8alteram partemC(ofair n s thearing’)
i t u t i o nwhich
a l L aisw facet
o f of natural
Ind i a - I Ijustice is a
requirement of Art. 14 [Basudeo Tiwary v Sido Kanhu University (1998) 8 SCC
194] Any action that does not conform to the rules of natural justice infringes Art
14
In E.P Royappa, held that equality is a dynamic concept with many aspects
and dimensions and it cannot be imprisoned within traditional and doctrinaire
limits. From a positivistic point of view, equality is antithetic to arbitrariness, one
belong to the rule of law in a republic while the other, to the whim and caprice of
an absolute monarch. Equality and arbitrariness are sworn enemies Where an act
is arbitrary, it is implicit in it that it is unequal both according to political logic and
constitutional law and is therefore violative of Art. 14.
In Maneka Gandhi case, the court observed that principle of
reasonableness, which legally as well as philosophically, is an essential element
of equality or nonarbitrariness, pervades Art. 14 like a brooding omnipresence.
Art 14 strikes at arbitrariness in State action and ensures fairness and equality of
treatment Where an act is arbitrary, it is implicit in it that it is unequal and so
violative of Art. 14 The conclusion is that if the action of State is arbitrary, it
cannot be justified even on the basis of doctrine of classification. In International
Airport Authority case, the court observed that the doctrine of classification is
merely a judicial formula for determining whether the legislative or executive
action is arbitrary and so constitutes a denial of equality
The new concept of equality has been criticized by Seervai as illogical,
inadequate and unnecessary. In the Bearer Bonds case (R K Garg v UOI AIR
1981 SC 2138), a Supreme Court Judge (Gupta J.) remarked: “To pass the test
of reasonableness if it was enough that there should be differentia which should
have some connection with the object of the Act, then these observations made
in Maneka Gandhi and Royappa would be so much wasted eloquence" Thus the
test of classification is still recognized and has not been thrown out 6
In A.L. Kalra v Project & Equipment Corpn. (1984) SCC 999, the court
widened the scope of application of Art. 14 Now, for the application of Art. 14 it
need not allege any discrimination vis-a-vis others. Arbitrary or unreasonable
actions, according to court, are perse discriminatory. What actions are arbitrary or
unreasonable and what shall be the standards, other than the classification
doctrine, for testing reasonableness of such actions are matters yet to be settled
(and it depends on facts and circumstances of each case). An obvious test to
apply is to see whether there is any discernible principle emerging from impugned
Act and if so, does it satisfy test of reasonableness (an uninformed by reason is
arbitrary).
In Krishnan Kakkanth v Govt of Kerala (AIR 1997 SC 128), the court
observed: To ascertain unreasonableness and arbitrariness in the context of Art.
14, it is not necessary to enter upon any exercise for finding out the wisdom in the
policy decision of the State government It is immaterial if it can be demonstrated
that the policy decision is unwise and is likely to defeat the purpose for which
such decision has beer taken (unless the policy decision is demonstrably
capricious or

6 . B K . Sharma, p. 74
Right to Equality
arbitrary and not informed by any reason whatsoever), the policy decision
cannot be struck down. It should be borne in mind that except for the limited
purpose of testing a public policy in the context of illegality and unconstitutionality,
the courts should avoid “embarking on uncharted ocean of public policy”
A legislation which confers on the executive or administrative authority an
unguided or uncontrolled discretionary power in the matter of application of law or
allows it to select cases violates Art. 14. Exercise of discretion must be
transparent, just, fair and non-arbitrary [Common Cause v UOI (1996) 6 SCC
530],

Illustrative Cases on Art. 14


(1) In Sunil Jaitely v State of Haryana (1984) 4 SCC 296, the reservation of
some seats for admission to M.B.B.S. course for students who were
educated from class I to VIII in common rural schools was held to be
violative of Art. 14, as the classification between rural educated and
urban educated students for this purpose was wholly arbitrary having no
nexus to the object i.e. providing extra facilities to students coming from
rural schools to enter medical college.
(2) In Ajay Hasia v Khalid Mujib (AIR 1981 SC 487), the Regional
Engineering College made admissions on the basis of oral interview
after a written test. The court held that allocation of one-third of total
marks for interview was plainly arbitrary and violative of Art. 14. The oral
interview can't be regarded as a very satisfactory test for evaluating the
caliber, as it is subjective and capable of abuse. It can’t be the exclusive
test. It should be resorted to only as an additional or supplementary test
(3) In Air India v Nargesh Meerza (AIR 1981 SC 1829), the petitioner
challenged the validity of the regulations under which they could he
retired at the age of 35 years or if t! ey got married within 4 years of their
service or on first pregnancy on the ground that they were discriminatory
and violative of Arts. 14, 15 and 16 Held that the termination of service
on pregnancy was manifestly unreasonable and arbitrary and was,
therefore, clearly violative of Art. 14. Further, the provision for extension
of service of Air Hostess “at the option" of Managing Director confers a
discretionary power without laying down any guidelines or principles and
liable to be struck down as unconstitutional.
(4) In Y Srinivasa Rao v J Veeraiah (AIR 1993 SC 929), held that
Government’s policy to give preference to less educated persons over
more educated persons in granting licence for running fair price shop
was arbitrary and liable to be set aside. The policy of the government to
prefer an uneducated person over an educated person amounts to
allowing premium on ignorance, incompetence and consequently
inefficiency and therefore unconstitutional.
(5) In Radhey Shyam Singh v UOI (AIR 1997 SC 1610), the preparation of
zone- wise separate merit lists by the Subordinate Services Commission
on the basis of the same examination albeit conducted in various zones,
was held
XVI
to be violative of Arts C
14o and
n s t i16.
t u tThe
i o nselection
a l L a wmust
o f be
Ind i a - I based
strictly I on
merit. The objective sought to be achieved by this process or method of
selection was to enable the candidates from different regions to be
absorbed ' in the vacancies arising within the respective regions. But, the
Couit held that there exists no nexus between the process of zone-wise
selection and the object to be achieved i.e the selection of the best
candidates
(6) In Mithu v State of Punjab (AIR 1983 SC 473), the court struck down Sec.
303 of I.PC. as unconstitutional on the ground that the classification
between persons who commit murders whilst under the sentence of life-
imprisonment and those who commit murders whilst under no
imprisonment, for the purpose of making death sentence mandatory in case
of fcrner class and optional in latter class, was not based on any rational
principle. The discretion as to which sentence is to be awarded is to be
exercised by courts which will determine the matter on the nature of offence
committed by an accused. This judicial discretion is not available to a life
convict under Sec. 303 I.P.C.
(7) In BALCO Employees Union v UOI (AIR 2002 SC 350), the apex court held
(hat there can be no judicial review of economic policy of the Government
unless there is violation of the Constitution or any Act, The process of
disinvestments is a policy decision involving complex economic factors. The
petitioners have failed to prove that decision to disinvest in BALCO is any
way capricious, arbitrary, illegal or uninformed.
(8) In Sarbanand Sonowal v UOI (AIR 2005 SC 3920). the validity of Illegal
Migrants (Determination by Tribunal) Act, 1983 was in question Held that
the IMDT Act, which applied only to the State of Assam and not the other
States similarly situated is discriminatory as the classification based on
'geographical basis' contains provisions and prescribes such procedure that
it virtually becomes impossible to detect and deport foreigners (illegal
Bangladesh immigrants). Instead of achieving the object of detecting and
deporting foreigners it defeats the purpose. For satisfying the test of Art. 14,
the ‘geographical factor’ alone in making classification is not enough but
there must be a nexus with the object sought to be achieved. If
geographical consideration becomes the sole criteria, it would be open to
the legislature to apply enactments made by it to any sub-division or district
within the State and leaving others at their sweet will.
(9) In Javed v State of Haryana [JT 2003 (6) SC 283], the apex court upheld
the constitutional validity of certain provisions of Haryana Panchayati Raj
Act, 1994, which disqualified a person for holding office of Sarpanch or a
Panch of a Gram Panchayat, etc. if he had more than two living children. ,
The provision was held to be not discriminatory and the classification made
by it (viz. persons having more than two children and persons having not
more than two children) is based on intelligible differentia having nexus with
the object of popularization of family planning programme. The provision
also does not violate Art 25 of the Constitution.
r
Right to Equality 5]
^ 1 o) In Danial Latifi v Union of India (2001) 7 SCC 740, the petitioners
challenged the constitutional validity of the Muslim Women (Protection of
Rights on Divorce) Act, 1986, under which Sec. 125, Cr.P.C., providing for
maintenance of wives (including divorced women) by their former
husbands, was made inapplicable to divorced Muslim women. The Act was
said to be violative of Arts. 14, 15 and 21. Upholding the Act, the Court said
that a careful reading of the provisions of the Act would indicate that a
Muslim husband is liable to make reasonable and fair provision for the
future of the divorced wife which obviously includes her maintenance as
well. Even under the Act, the provisions of Sec. 125, Cr.P.C. would still be
attracted
(11) In John Vallamatton v UOI (AIR 2003 SC 2902), the Supreme Court held
that Sec. 118 of the Indian Succession Act, 1926 is discriminatory and thus
violative of Art. 14. The impugned provision places certain restrictions upon
an Indian Christian to bequeath his property for religious or charitable
purpose.
(12) In State of Haryana v Jai Singh (AIR 2003 SC 1696), the Government of
Haryana issued a Notification granting remission of prison sentence to all
convicts except those excluded in the said Notification. Held that the
offences excluded had been properly classified (based on the nature of
offence), and, there is no violation of Art 14. Likewise, in S. Satyanaryan v
Government of A P. (AIR 2003 SC 3074), the court upheld the classification
to keep away convicts for crimes against women from the benefits of
remission, as reasonable, proper and not violative of Art. 14
(13) In K.H. Siraj v High Court of Kerala (AIR 2006 SC 2339), the appellants
challenged the validity of Kerala Judicial Service Rules 14 and 15 fixing the
minimum marks for interview for selection of Munsif Magistrate. The
Supreme Court held that the Rules made by the Kerala High Court for the
selection of judicial officers is valid. It also held that interview is the best
method of assessing the suitability of a candidate. It brings out overall
intellectual and personal qualities of candidates while written examination
will testify the candidate’s academic knowledge.
(14) In E.V. Chinnaiah v State of A.P. (2005) 1 SCC 394, it was held that it is
not open to a State to sub-dassify the Scheduled Castes and apportion the
seats/quota already reserved for Scheduled Castes as a whole, amongst
such sub-classes. Such sub-classification or micro-classification would be
violative of Art. 14 and of the doctrine of reasonableness. Except, to the
extent the Constitution itself makes a sub-classification, there cannot be
grouping of Scheduled Cates into different categories for differential
treatment. The Constitution itself has kept the Scheduled Castes and
Scheduled Tribes List out of interference by the State Governments
(15) in A.PB.C. Sangh v Jharkhand State Vaishya Federation (AIR 2006 SC
2814), the appellants challenged the validity of the State Government order
amalgamating ‘Extremely Backward Class’ and ‘Backward Class’ into one
group for the purpose of reservation in professional and educational
institutions as violative of Art. 14. It was held that the State Government’s
52 Constitutional Law o f India- II
decision to amalgamate the two classes is unreasonable as two
different classes are treated similarly Treating unequal as equals is
violative of Art 14. In the absence of new circumstances justifying the
afotesaid amalgamation, the government’s decision is arbitrary and
unreasonable.
(16) Kanhaiya Lai Sethia v UOI (1997) 6 SCC 573 - In this case, the
petitioner contended that non-inclusion of Rajasthani language in Sch.
VIII of the Constitution, when Manipuri, Konkani and Nepali have been
included tnerein, is violative of the basic structure of the Constitution
viz. equality. Held that to include a particular language in Sch. VIII is a
“policy matter1 of the Union.

Examples of violation of Art. 14


- Classification between pensioners retiring before a particular date and
after that date.
- Reservation of seats in Medical Colleges on the basis of domicile or
institutional preference regardless of merit.
- The rule framed by Bar Council of India barring the enrolment of
persons who have crossed the age of 45 years as advocates.
- Denial of grants to private law colleges while giving grants to colleges
teaching others subjects.

Examples of permissible classification


- Reduction of age of retirement from 58 years to 55 years.
- Debarring persons who are in prison/ police custody to vote in an
election and not those who are in preventive detention.
- Differentiating employees of State Bank of India from those of its
subsidiary banks.
- An Act which applies to Marwar portion of Rajasthan.
- An act creating a monopoly in favour of the State.
- Sales tax on Virginia tobacco but not on country tobacco
- Higher tax on cinema houses situated in posh localities and having
larger seating capacities and lower tax on cinemas located in poor
localities and having small seating capacities.
- Imposing higher rate of road-tax on company owned cars and motor
cycles than individual owned.
- Difference in rates of sales-tax on cooked food sold by luxury hotels
and modest hotels.
- Picking up a minister for the purpose of enquiry and setting up a
commission to go into the charges of corruption 7
ARTICLE 15 [PROHIBITION OF DISCRIMINATION AGAINST
CITIZENS]

7 . B K Sharma, pp 75-76.
Right to Equality 53

Article 15 provides for a particular application of the general principle embodied in ^


14. Hence, if a discriminatory action does not fall within the purview of Art. 15 (or
Art. 16), it may yet be avoided if it is obnoxious to Art. 14, which has a wider
coverage. When a law comes within the prohibition of Art. 15 (or Art. 16) it cannot
be validated by recourse to Art. 14 by applying the principle of reasonable
classification. It may be noted that, unlike Art. 14 : the guarantee under Art. 15 (or
Art. 16) is available to citizens only.
Clause (1) - “The State shall not discriminate against any citizen on grounds only of
religion, caste, sex, place of birth or any of them "
The prohibition is against the State but not against private persons. The
word 'discrimination' means to make an adverse distinction with regard to’, 'to
distinguish unfavorably from others’. The constitutional prohibition is directed to all
State action- legislative or executive. It may be political, civil or otherwise. A law
which provides for election on the basis or separate electorates based on the
religion of the voters offends against this clause.
It must be noted that discrimination based on any of these grounds and
also on other grounds is not hit by Art. 15(1). In other words, the discrimination
should be based solely or only on a ground mentioned in Art. 15(1) and no other
ground. Thus, a law which discriminates on the ground of residence does not
violate Art 15(1) (Place of birth is different from residence). Similarly, the
requirement of a test in regional language for State employment doesn't
contravene Art. 15 as the test is made compulsory for all persons seeking
employment.
The expression “place of birth’’ is not synonymous with the expression
“domicile". Thus, reservation in Government run medical colleges based on
domicile was not prohibited by Art. 15(1) [DP Joshi case AIR 1955 SC 334,
Saurabh Chaudari v UOI]
Clause (2) - “No citizen shall, on ground only of religion, race, caste, sex, place of
birth or any of them, be subject to any disability, liability, restriction or condition
with regard to - (a) access to shops, public restaurants, hotels and places of
Public entertainment; or (b) the use of wells, tanks, bathing ghats, reads and
Places of public resort maintained wholly or partly cut of State funds or dedicated
to the use of general public ’
‘Places of public resort’ includes public parks, buses, railway, hospitals, etc
It •s to be noted that while clause (1) prohibits discrimination by State, clause (2)
includes Private persons also from making any discrimination. Thus, clause (2) is
more general ir> nature or wider in operation. However, otherwise, clause (2) is a
particular application °fthe general principle against discrimination embodied in
clause (1), because the latter forbids discrimination against citizens in all matters,
the former deals only with discrimination
XVI
Constitutional Law o f India- II

as regards to use or access to public places. Clause (2) is quite relevant in


the context of the practice of untouchaoility' prevalent in India on a large scale

Provisos (or Exceptions) to Art. 15(1) and (2)


Clause (3) - “Nothing in this article shall prevent State from making any special
provision for women and children
Thus, reservation of seats for women in a college does not offend against
Art. 15(1) Provisions can be made for maternity benefit or free education of
women and children. On this basis, the Supreme Court has upheld the validity of
Sec. 497 of the Indian Penal Code under which a man is punishable for adultery
but not a woman (Yusuf v State of Bombay AIR 1954 SC 321); a woman is
considered victim in such cases. Similarly special treatment for women and
children in Sec 437 of Code of Criminal Procedure in granting bail is valid
Sections 14 and 15 of Hindu Succession Act which favour a Hindu woman is
constitutional.
It may be noted that under clause (3), special provisions can be made in
favour of and not against women. Art. 15(1 )-(3) would imply that State can
discriminate in favour of women against men but not vice versa (Revathi v 'JO/AIR
1998 SC 835)
Clause (4) - “Nothing in this article or clause (2) of Art 29 shall prevent the State
from making any special provision for advancement o f any socially and
educationally backward classes of citizen or for the Scheduled Castes and
Scheduled Tribes."
Clause (4j was added by the Constitution (First Amendment) Act, 1951. It
was also to override the judgment in Champakam v State of Madras (AIR 1951
Mad. 120) in which caste-wise distribution of seats was struck down as violating
Art. 15(1)
Under Art. 15(4), two things are to be determined while providing for
reservation of seats in public educational institutions: (i) Who are socially and
educationally backward classes? (ii) What is the limit of reservation? It may be
noted that the expression “special provisions for advancement" is of wide import
and not necessarily confined to reservation of seats in educational institutions.
Thus, it may include financial assistance, free medicai facilities, concessional oi
fiee housing, etc.
The scope of Art. 15(4) was discussed in Balaji v State of Mysore (AIR
1963 SC 649). The court observed that Art. 15(4) only enable the State to make
special and not exclusive provisions for backward classes Clause (4) is only an
enabling provision and does not irroose any obligation on the State to take any
special action under it. It merely confers discretion to act, if necessary
Clause (5) - The Constitution 93rd Amendment Act, 2005 (w.e f 20th January
2006), has added a new Clause (5) to Art. 15 Cl. (5) provides that nothing in Art.
15 or in Art 19(1)(g) prevent the State in making any special provision, by lav;, for
the advancement of socially and educationally backward classes of citizens or for
SCs/ STs in so far as such special provision relate to admission to educational
institutions, including private ones, whether aided or unaided by the State, other
than minority educational institutions referred to in Art. 30(1).
Right to Equality 55

This amendment is intended to nuliify the effect of the Supreme Court's


judgment in T.M.A Pai Foundation and PA. inamdar cases, in which it was held
That the government cannot make reservat.on of seats for Backward Classes and
SCs and STs in private educational institutions. But the 93rd amendment exempts
the minority educational institutions established under Art. 30 fiom its purview.
Art 15(5) amplify what is stated in Art 15(4), by specifically mentioning,
admission to educational institutions Further, The State could do so only by law,
and not by executive action.

ARTICLE 16 [EQUALITY OF OPPORTUNITY IN PIJEIIC


EMPLOYMENT]

Under Article 16, the guarantee agamst discrimination is limited to employment


and appointment’ under the Slate. Art. 15 is mov general and deals with all cases
of discrimination which do not fall under Art 18. Thus, Art 15 is wider in operation.
However, while Art. 15 prohibits discrimination on five grounds (viz. religion, race,
caste, sex or place of birth), Art. 16 contains seven prohibited grounds (viz.
religion, race, caste, sex, descent, place of birth or residence). Both these articles
can be invoked only by citizens.
Clause (1) - There shall be equality of opportunity for all citizens in matters
relating to ‘employment or appointment to any office under the State.
Clause (2) - No citizen shall, on grounds only of religion, race, caste, sex, descent,
place of birth, residence or any of them, be ineligible for, or discriminated against
in respect of any employment or office under the State.
Clauses (1) and (2) of Art 16 guarantee equality of opportunity to all citizens
in the matter of appointment to any office or any other employment under the
State. It covers not only initial appointment but also promotion age of
superannuation, seniority, etc. [U.RS.C. v Girish Jayantilal Vagheia (2006) 2 SCC
482]. Art. 16, however, does not prevent the State from prescribing the necessary
qualifications anc selective tests for recruitment for government services The
selective test, however, must not be arbitrary, and must have nexus between the
qualifications and the object i.e post or nature of the service.
The ‘equality of opportunity1 under Art. 16(1) means equality between
members °f the same class of employees, and not equality between members of
separate ^dependent classes
In Pradeep Jain v Union of India (1984) 4 SCC 654 held that although in
view of Art. 15(2) and earlier decisions of the court the residential requirement for
admission to a medical college in a State is valid and constitutional but its validity
030 be tested on the touchstone of Art. 14 and if it violates Art. 14 it will be

^constitutional.
In Randhir Singh v Union of India (AIR 1982 SC 879), it has been held that
'equal pay for equal work’, although not expressly to be a fundamental right is
clearly a constitutional goal under Arts 14, 16 and 39(c) of Constitution and can be
enforced by the courts in cases of unequal scales of pay based on irrational
XVI
Constitutional Law o f India- II

classification. This principle has been followed in a number of cases and has
virtually become a fundamental right.
It may be noted that, under Art. 16(2), where there is discrimination only on
any of the grounds mentioned therein, this clause will get attracted. Where
discrimination is based partly on the grounds contained in Art. 16(2) and partly on
other grounds, or, where discrimination is based on grounds other then those
mentioned in Art 16(2). this clause would not be attracted But, the case will have
to be judged in the light of general principles laid down in Art 16(1). Thus, if
discrimination is based on the ground of backwardness, clause (2) will not be
attracted (State of Kerala v N.M. Thomas, discussed below)

Exceptions to Clauses (1) and (2)


Clause (3) - It empowers Parliament to make any law prescribing in regard to a
class or classes of employment or appointment to an office under the Government
of, or any local/other authority within a State/UT, any requirement as to residence
within that State/UT, prior to such employment or appointment.
Residence may be laid down as a condition for particular classes of
employment but the power to do so is conferred on the Parliament and not on the
States. In Kaiiash Chandra Sharma v State of Rajasthan (2002) 6 SCC 562, it
was held that giving employment to the residents of a particular district on
preferential basis is violative of Art. 16. Similarly, addition of bonus marks to the
applicants of the rural areas of a district (preferring them over candidates living in
nearby towns) would amount to discrimination which falls foul of Arts 14 and 16
[Harshendra Choubisa v State of Rajasthan (2002) 6 SCC 393).
Clause (4) - It empowers the State to make special provision for the reservation of
appointments or posts in favour of any backward classes of citizens, which in the
opinion of the State are not adequately represented in the sen/ices under the
State
Art 16(4) applies only if two conditions are satisfied:
(i) the class of citizens is backward i.e. mainly social (and, therefore
educationally and economically),
(ii) the said class is not adequately represented in service of the State.
The second test cannot be the sole criterion. “Backward classes of citizens"
(though not defined in the Constitution) includes SCs and STs. Posts cannot be
reserved on communal lines e g. 10% for Brahmins, 20% for Muslims etc ‘Women’
as a class also cannot be regarded as backward. There cannot be relaxation of
rules of recruitment or in regard to qualifications favouring the women.
Art. 16(4), like Art. 15(4), is only an enabling provision. It merely confers
discretion to act, if necessary Reservation for backward classes is not a mandate.
It is a matter of policy. No right is conferred on the citizens to claim reservation.
Thus, Art. 16(4) has been held to be net mandatory [£. V. Chinnaiah v State ofA.P
(2005) 1 SCC 394],
Art. 16(4) and maintenance of efficiency of administration {Art. 335)- Art. 16(4)
must be interpreted in the light of Art. 335 which says that claims of SC and ST
Right to Equality 57

shall be taken into consideration constituently with the maintenance of efficiency


of administration. The reservations for backward classes should not be
unreasonable. It should be considered having regard to the employment
opportunities of the general public. However, the 82nd Amendment Act, 2000
restored the relaxation in qualifying marks and standards of evaluation in both, job
reservation and promotions to SC/STs oy adding a proviso to this effect in Art.
335. This Amendment makes provision for no interference to be made in making
of any provision in favour of the numbers of SCs and STs for relaxation in
qualifying marks in any examination for lowering the standards of evaluation for
reservation in matters of promotion to any class or classes or services of posts in
connection with the affairs of the Union or of a State.
Clause (5) - Nothing in this Article shall affect the operation of any law, which
provides that the incumbent of an office in connection with the affairs of any
religions institution or any member of the governing body thereof shall be a person
pro?essing a particular religion.
Thus, a Hindu religious institution could be governed oniy by Hindus and a
Muslim institution on!y by the Muslims. This exception is in consonance with the
fundamental right to freedom of religion contained in Arts. 25-28, and, the rights of
the minorities under Arts. 29-30.

Scope of Art. 16(1) and Art. 16(4)

Leading Case: state of kerala V n.m. thomas16 (AIR 1976 SC


490)

Facts and Issue - The important question which came up for


consideration of the court was whether it was permissible to give
preferential treatment to SC and STs under clause (1) of Art. 16 i.e.
outside the exception clause (4) of Art. 16.
The point at issue was whether Art. 16(1) protected exemption of
Lower Division Clerks belonging to SC and STs from passing the
special tests for promotion as Upper Division Clerks The impugned
order granted

16 “Article 16(4) is by itself a rule of equality rather than an exception to the


rule What is your view? Discuss with the help of case law. [i.A S -
2005\
Constitutional Law o f India- II
exemption for two years more to SC and STs This exemption was challenged as
discriminatory under Art. 16(1)
Observations and Decision - The majority consisting of five judges, Ray (C.J.),
Mathew, Beg, Krishna Iyer and Fazal Alt, J. held these exemptions and
promotions as valid, whereas Khanna and Gupta JJ. dissented Chief Justice Ray
stated categorically that Arts 14 15 and 16 form part of a string of constitutionally
guaranteed rights supplementing each other. Art. 16 was explained as an incident
of guarantee of equality contained in Art 14 and therefore, pennitted reasonable
classification of employees in matters relating to employment or appointment Art
16(1) using the expression “equality ’ makes it reiatable to all matters of
employment from appointment through promotion and termination to payment of
pension and gratuity. Art. 16(1) permits classification on the basis of object and
purpose of law or State action, except classification involving discrimination
prohibited by Art. 1S(2).
Art 16(4) indicated one of the methods of achieving equality embooed in
Art. 16(1), and Art 16(4) do not exhausts the equality of opportunity which can be
made available to the backward classes. The guarantee of equality before law is
something more than is required by formal equality, and Art. 16(1) means effective
material or real equality, and that Art 16(4) is not to be read by way of any
exception to Art. 16(1).
Equality of opportunity for unequals can only mean aggravation of inequality
Equality of opportunity admits discrimination with reason, which means rational
classification for differential treatment having nexus to the constitutionally
permissible object Preferential treatment for the backward classes in service with
due regard to administrative efficiency is permissible object. The quality and
concept of equality is that if persons are dissimilarly placed, they cannot be made
equal by having the same treatment.
Thus, in the present case, the classification of employees belonging to SC
and STs was a just and reasonable classification 'having rational nexus to the
object of providing equal opportunities for all citizens in matters relating to
employment and appointment.' The temporary relaxation of rules m favour of them
was wari anted in the services in view of their over-all backwardness. T he above
Rules do riot impair the test of efficiency in administration in as much as they have
to pass the test ultimately (the exemption granted to them only for limited period)
The dissenting judges followed Balaji and argued that carving out classes of
citizens for favoured treatment in marteis of public employment, except in cases
for which there is an express provision in Art 16(4), would in the very nature of
things run counter to the principle of equality of opportunity in Art. 16(1).
Nevertheless, a new interpretation of Art 16(1) was made by the majority
decision in this case ]
Protective Discrimination: A Facet of Equality17

17 Does reservation under Art. 15(4) and Art. 16(4) lead to any infraction of
right to equality guaranteed under Art 14? Illustrate vvitn reference to
decided cases.
[L.C.II-2006\
"Equality is the basic feature of the Constitution of India arid any treatment
XVI
Constitutional Law o f India- II

New development in equality also includes increased emphasis on positive


equality or affirmative action. In several decisions (e g Mandal case), the court
has emphasised that equality is a positive right and requires the State to minimise
existing inequalities and to treat unequal or underprivileged with special care.
As heid in Jagdish L a l v State of Haryana (AIR 1997 SC 2366): Equality
must not remain mere idle incantation but must become a vibrant living reality for
the large masses of people In a hierarchical society with an indelible feudal st3mp
and incurable actual inequality, it is absurd to suggest that progressive measures
to eliminate group disabilities and promote collective equality are antagonistic to
and anathema of equality on the ground that every individual is entitled to equality
of opportunity based purely on the “merit mantra”
Equality of opportunity is not simply a matter of legal equality. Its existence
depends not merely on the absence of disabilities but on the presence of abilities
and opportunity for excellence in each cadre/grade. Where is inequality in fact,
legal equality always tends to accentuate inequality
It is, therefore necessary to take into account do facto inequalities which
exist in the society. In order to bring about real equality, affirmative action fills the
bill and allows to give preference to the socially and economically disadvantaged
groups, i e. placing the weaker sections on a footing of equaiity with the stronger
and more powerful so that each member of the community may enjoy equal
opportunity of using to the full, his natural endowments of physique, character and
intelligence.
Protective discrimination is a facet of equality under Arts. 14, 15 and 16.
When, therefore, competing rights between general and reserved candidates
require adjudication and adjustment with the right of general candidates, the
doctrine of violation of Art. 14 have no role to play since protective discrimination
itself is a facet of Art. 14 and it does not again deny equality to the reserved
candidates.
Reservations in Employment

LEADING CASE: INDRA SAWHNEY V UNION OF INDIA


(THE MANDAL CASE) 18 (AIR 1993 SC 477)
The scope and extent of Art. 16(4) has been examined thoroughly by
the Supreme Court in this historic case The court by 6-3 majority upheld
the decision of Union government to reserve 27% Government jobs for
OBCs (Other Backward Classes) provided “creamy layer" among them
are eliminated and reservation only confined to initial appointments and

of equals unequally or unequais as equals will be violation of basic structure


of the Constitution." Explain. [I A.S.-2008\
[Note: Also see M. Nagaraj case and Ashoka Kumar Thakur case discussed
below]
18 In the light of Indra Sawhney v Union of India (AIR 1993 SC 477) and
subsequent developments thereto, examine the extent and scope of
reservations in State services under Art. 16 of the Constitution of India.
[L C1-2006]
Right to Equality 61

not promotions, and, the total reservation shall not exceed 50% (There
is already a 221/2% reseivation for SC arid STs in gcvernment jobs)
However, the court struck down the government notification reserving
10% government jobs for economically backward classes among higher
castes
The Mandal Commission was appointed to investigate the
condition of socially and educationally backward classes. Cne of the
major recommendations of the Commission was that, excluding the
SCs/STs, Other Backward Classes (OBCs) constitute nearly 52% of the
population and therefore 27% government jobs could be reserved for
them so that total reseivation for all (SCs/STs/OBCs) amounts to 50% In
1990, *he Centra1 Government Memorandum announced 27%
reservation for socially and educationally backward classes in the
government posts/services In 1991, the Memorandum was modified:
first, the poorer sections among the backward classes to get preference;
secondly, 10% vacancies to be reserved for the other economically
backward sections of the people uncovered by any existing reservation
scheme The constitutional validity of the Memorandum was considered
by the Bench of S Judges.
The majority held that the Mandal Report is valid however, no
opinion is expressed by it Also, held that a provision under Art 16(4) can
be made by the legislature by enacting a law or by an ‘executive’ order 19
The court, however, observed: Reservation by an executive order may
not be invalid, but since it was being made for the first time in services
under the Union, propriety demands that it should have been laid before
the
Parliament not only to lay down a healthy convention but also to consider the
change in social, economic and political conditions of the country.
A permanent statutory body be appointed to examine complaints of over-
inclusion and under-inclusion. All disputes regarding new criteria evolved by the
Central or State Governments can be raised only in the Supreme Court and not
before any High Court or tribunal.
The majority opinion may be summarised as follows (Readers must note
carefully the comparison of this case with Balaji, Thomas and other cases):
(I) Backward class of citizens in Article 16(4) can be identified on the basis of
caste and not only on economic basis -
The word 'class' in Art. 16(4) is used in the sense of social class. It is not
“antithetical to caste". The term ‘class’ was used in Art. 16(4) with a view to
secure the protection of Art. 16(4) to all communities, groups, classes, sections

19 The Government of India by an office memorandum reserved 27% of posts


for appointment under the State in favour of socially and educationally
backward classes. A petition is filed in the Supreme Court challenging its
validity on the ground that it could only be done by an Act of Parliament and
not by an executive order. Decide. [/.AS •98)
XVI
Constitutional Law o f India- II

found to be backward.
A caste can be quite often is a social class in India an;) if it is backward
socially, it would be a backward class for the purpose of Art. 16(4). Though caste
alone can’t be taken into consideration for purposes of identification of backward
classes (occupation groups, classes and sections of people are other important
criteria). There are classes among non-Hindus, Muslims, Christians, etc. and if
they are backward socially, they are entitled to reservation.
In Balaji case, court observed that the term ‘backward class' is not defined
in Constitution, and government’s decision in this regard is justiciable. A class
implies a division of society according to status, rank or caste; in India, caste
plays an important role in determining status However, caste is not the sole
criterion ... Muslims, Christians, Jains do not recognise castes, how one decide
social backwardness in them.
(II) Article 16(4) is not an exception to Article 16(1), but an independent clause.
Reservation can be made under Article 16(1) on the basis of reasonable
classification -
The court thus overruled Balaji case and approved Thomas case Further, court
said that Art. 16(4) is exhaustive of the subject of reservation in favour of
backward classes, though it may not be exhaustive of the very concept of
reservation. Reservation for other classes can be made under Art. 16(1). Art.
16(4) is an instance of classification implicit in and permitted by Art. 16(1). In
other words, Art. 16(4) did not cover the entire field covered by Art. 16(1) and (2).
Art. 16(1) permitted the making of reservation of appointment/posts which
should be made only in exceptional situations and wherein the State is called
upon to do so in public interest. For instance, reservations for the wards of military
personnel or political sufferers or any other class except for backward classes.
The court also explained that Art 16(4) permitted not only reservation of
appointment/posts which was the highest form’ of special provision, but also
preferences concessions and exemptions which are ‘lesser forms’ of special
provision.
(III) Backward classes in Article 16(4) are not similar to as socially and
educaticnaliy backward in Article 15(4) -
It is much wider and takes in SC/STs and OBCs including socially and
educationally backward classes. It followed that certain classes might not qualify
for the protection of Art. 15(4) but might qualify for the protection of Art. 16(4}
Thus, court overruled Balaji case on this point.
Also, backwardness contemplated by Art 16(4) is mainly social. It need not
be both social and educational as required by Art 15(4).
(IV) Creamy layer (socially advanced persons) can be and must be excluded
from Backward Classes -
The 'means test' signifies imposition of an income limit Persons whose income
exceeds that limit are excluded from backward class. Such persons are called the
creamy layer. Exclusion of the creamy layer makes a backward class truly
backward. For excluding creamy layer, economic criteria can be adopted as an
indicium or measure of social advancement. There are certain positions e.g. IAS,
etc. which can be treated as advanced without further inquiry.
Right to Equality 63

In Balaji case, the court observed that reservation of seats shouldn't be


allowed to become vested interest (once a caste is considered backward it
shouldn’t continue to be backward for all the time). Government should review
test and if a class reaches state of progress where no reservation is necessary,
that class should be deleted.
(V) Article 16(4) permits classification of Backward classes into backward and
more backward classes -
This is necessary so that advanced sections of backward classes might not take
all the benefits of reservation. The court approved State of A. P. v U.S.V. Balram
(AIR 1972 SC 1375).
On the contrary, in Balaji case, court observed that Backward Classes’ are
one homogenous group, and no further sub-classification can be done. Under Art.
15(1), concept of backwardness was not intended to be relative in the sense that
any classes who are less advanced in relation to most advanced classes should
be included in it.
(VI) A backward class of citizens cannot be identified only and exclusively with
reference to economic criteria -
Art. 16(4) is not aimed at economic upliftment or alleviation of poverty Mainly
social, and therefore educational and economic backwardness has to be taken
into account Economic backwardness may give jurisdiction to State to reserve,
provided it can find mechanism to ascertain inadequacy of representation of such
classes. But such a group or collectivity does not fall under Art 16(1)
In State of UP. v Pradeep Tandon (AIR 1675 SC 563), the U.P.
Government made reservation of seats for admission to Medical Colleges in the
State in favour of candidates coming from rural areas, Hill and Uttarakhand areas
The Supreme Court held that the Hill and Uttarakhand areas of U.P. are
instances of socially and educationally backward classes of citizens, while rural
areas not. Backwardness is judged by the economic basis that each reg;on has
its own measurable possibilities for the maintenance of human members’
standards of living and fixed property. In Balaji case, court observed that from an
economic point of view, classes are backward when they do not make effective
use of resources... cannot maintain standard of living, etc. (e.g. in Hilly areas).
However, rural areas are not a homogenous class by itself, as
standards/occupations of people are different Poverty alone can’t be basis of
classification, as poverty is found in all the parts of India
(VII) Reservation shall not exceed 50 percent -
Reservation should not as a rule exceed 50%. The rule should be applied each
year It cannot be related to the total strength. However, it may be relaxed in
favour of people living in far flung and remote areas because of their peculiar
conditions, but in doing so extreme caution is to be exercised.
The Court thus affirmed Balaji case and overruled Thomas case. In Bala/i
case, court observed that Art. 15(4) only enables the State to make special and
not exclusive provisions for backward classes Clause
(4) is only an enabling provision and doesn t impose any obligation on the State
to take any special action It merely confers discretion to act, if necessary.
Advancement of the rest of society can't be ignored altogether in.... zeal to
XVI
Constitutional Law o f India- II

promote welfare of backward classes.


The court in Mandat case further observed that Art. 16(4) speaks of
adequate representation and not the proportionate representation The adequacy
of representation is not to be determined merely on the basis of the overall
numerical strength of the Backward Classes in the services. For determining this
adequacy, their representation at different levels of administration and in different
grades has to be taken into consideration.
Rule of 50% applicable to reservation proper only not to exemptions,
concessions or relaxations provided to backward classes under Art. 16(4).
Further, for the purpose of applying the rule of 50%, a year should be taken as a
unit and not the entire strength of the cadre, service or the unit, as the case may
be.
(VIII) Carry forward rule is valid but subject to 50 percent -
The court also held that 'carry forward rule' (to carry forward the unfilied
vacancies in the next year) is valid provided it shouldn’t result in breach of 50%
rule. The court thus approved T. Devadashan v UOI (AIR 1964 SC 179), and,
overruled A.B.S.K. Sangh Rly. v UOI (AIR 1981 SC 298).
65 Constitutional Law o f India- II

In T. Devadashan case, the carry forward rule regulating reservation of


vacancies for SCs/STs, resulted in 64% reservation for them in a particular year.
It was held that reservation exceeding 50% in a single year would be
unconstitutional and invalid Such exceesive reservation had the effect of
destroying the guarantee of equality of opportunity contained in Art. 16(1).
In A.B.S K. Sangh Rly. case, the Circulars issued by the Railway Board
provided reservation of 17.5% posts in railway services for SCs/ STs. The
Circulars also contained the rule to cariy forward the unfilled reservation quota for
3 years. As a result of it, the reservation quota for these categories in the 3 rd year
came to about 65%. The Supreme Court upheld it by ruling that mathematical
precision could not be applied while dealing with human affairs.
(IX) No reservations in promotions -
Art. 16(4) is confined to initial appointment only and cannot extend to reservation
in the matter of promotions, though the expression ‘appointment’ in Art. 16(4)
included appointment by direct recruitment or by promotion or by transfer. The
court thus overruled Rangachari case. In the latter case, it was held that
reservation under Art. 16(4) could be made, not only at the initial stage of
recruitment, but even in the matter of promotion from a lower to a higher
post/cadre (Manager, S. Riy. v Rangachari AIR 1962 SC 36).
The court in the Mandal case observed that it would be a serious and
unacceptable inroad into the rule of equality of opportunity to say that a handicap
should be provided to the backward classes of citizens at every stage of
promotion throughout their career Once the advantaged and disadvantaged are
made equal and are brought into one class or group, any further benefit would
amount to treating equals unequally. The court reasoned that it creates a lot of
resentment among persons who are denied promotions and so affects efficiency
in administration. Art. 16(4) has to be read with Art. 335. The State, however,
could extend concessions and relaxations to the members of reserved categories
in the matters of promotion without compromising the efficiency of the
administration.
(X) Merit when necessary and when not -
The court observed that the rule of reservation cannot be called anti- meritarian.
The court said that mere getting of high marks in examinations doesn’t make a
good administrator. An efficient administrator has capacity to understand with
sympathy and to tackle bravely the problems of weaker sections. Thus, a person
belonging to these very sections is appropriate. The court in this case, also
observed that there are certain services where reservation should not be applied
e.g. Professors, Pilots, Scientists, Technicians, Technical Posts, Super
specialties, Defence Services, etc. In these services and positions, the merit
alone, counts.]
Comments - The determination of backwardness on the sole ground of caste has
been criticised by Nani A. Palkhivala, an eminent jurist, on the ground that it will
revive casteism which the Constitution emphatically intended to end The decision

would revitalize casteism, cleave the nation into two - forward and backward - and
open up new vistas for conflicts and fissiparous forces, and make backwardness a
vested interest. This is against the call of the Preamble - unity and integrity and
XVI
Constitutional Law o f India- II

opposed to the basic structure of the Constitution.


The minority judgment of justices Thomas, Kuldip Singh and R M. Sahai
according to Palkhivala state the correct law. They had held that casteism is
anathema to the Constitution and caste can never be the basis of reservations for
employment under the Government. It may be noted that social reformers like
Swami Dayanand, Swami Vivekanand, Mahatma Gandhi, and Dr. Ambedkar
regarded caste as the bane of and curse on society. In earlier judgments the
Supreme Court had refused to treat caste as class.
The Government had not accepted the Report of the Backward Class
Commission headed by Kalelkar because it was not prepared to list the castes as
backward. The majority struck down provision for additional reservation of 10% for
other economically backward communities. It held that caste backwardness is
relevant while holding that economic backwardness is not. It is submitted that the
rejection of economic criterion for determining backwardness will create serious
problems in identifying the backward sections (among backwards) for whom the
court wants jobs to be reserved. It fact, this can only be done on the basis of
economic yardsticks.
Despite certain drawbacks the court's decision is to be welcomed as it has
been able to clarify certain important points on reservation issue. Firstly, it has put
a limit on the percentage of reservation which can’t be more ths.i 50%. Secondly, it
has held that there will be no reservation in promotion posts. This would help
reduce caste tension within offices. Thirdly, while upholding the government's
decision reserving 27% jobs for backward classes it has put a rider that it can be
implemented only when “creamy layer” among them are eliminated. This is aimed
at giving benefit of reservation to the poorest among backward. Thus, the court
has made a bold attempt to strike a balance between the interests of society and
educationally backward classes and persons belonging to the general category in
matters of government employment.
In pursuance of the directions of the Supreme Court in Mandal Case the
Parliament enacted the National Commission for Backward Classes Act, 1993. Its
function is to examine requests for inclusion of any class of citizens as a backward
c|
ass and hear complaints of over-inclusion and under-inclusion of any backward
cla
ss. It may be noted that in the Mandal Case, the court opined that for the
lc|
entification of backward classes, one has to begin somewhere with some group,
cla
ss or section. Neither the Constitution nor the law prescribes the procedure or
method of identification of backward classes. Nor is it possible or advisable for the
c
°urt to lay down any such procedure or method. It must be left to the authority
a
Ppointed to identify and adopt such method/procedure as it thinks convenient.
Creamy Layer
In the Mandal case, the Supreme Court held that the 'creamy layer’ among the
backward classes of citizens must be excluded by fixation of proper income,
property or status criteria In accordance with the direction given by the Supreme
Court, the Union Government appointed an expert committee known as Justice
Ram Nandan Committee to identify the Creamy Layer1 among the soctaliy and
educationally backward classes (SEBC).
Right to Equality 67

The report (submitted in 1993) has been accepted by the Government of


India. The report identifies the 'creamy layer’ among the SEBC for excluding it
from the list of Mandal beneficiaries The committee report states that only when
the creamy layer is substantially and stably formed after crossing the Rubicon
limit of social backwardness, then and then alone can it be made the basis for
disentitlemenL
The report says that certain constitutional posts qualify for the rule of
exclusion e.g. posts of President, Vice President, Judges of Supreme and High
Courts, Chairman and members of UPSC and State PCS, Chief Election
Commissioners, CAG, Governors, Ministers and Membership of Legislatures.
The rule of exclusion covers class I officers of Central and State Services (direct
recruits) public sector undertakings, armed forces, professional class including
trade, business and industry and property owners. It excludes those having gross
annual income of Rupee one lakh and above.
The report says that if both spouses are class II officers then the rule of
exclusion would apply to their offspring The rule of exclusion will apply if either
the husband or wife is a class I officer. For property owner, the report says that if
a person belongs to a family which owns irrigated land equal to or more than
65% of the statutory ceiling area the rule of exclusion would apply.

Subversion of Mandal Rule and Apex Court’s Rescue Efforts (POST-


MANDAL DEVELOPMENTS)
Unfortunately, the politicians in India have started flouting the Mandal judgment
with a view to keeping vote bank intact in their favour. In Tamil Nadu, a law has
been enacted to give sanction to 69% reservation for backward classes (By the
Constitution ; 76m Amendment Act. this Act was placed in the 9th Schedule to
protect it from attack). Following Tamil Nadu, Karnataka sought to introduce a Bill
to provide 80% reservations It was later deferred. j

(a) Non-following of Creamy Layer Rule


The Creamy layer rule excluding the affluent and well placed members of a caste
was made ineffective by several States. States like U.P and Bihar were using
Mandal not as an engine for social justice to make up for past inequities against
OBCs and to give them a socio-economic head start, but as a tool for re-
enforcing the vested interests of those belonging to these castes who had
already entered
tfie realm of India's elite (‘creamy layer'). Both States allowed reservations to apply,
for example, to IAS officers earning up to Rs. 10,000 a month - an act of defiance
that would place only the chief secretary and the likes in the ‘creamy layer 1. Similar
concessions were made for professionals and industrialists.
The Supreme Court’s judgment in Ashoka Kumar Thakur v State of Bihar
(1995) 5 SCC 403, reiterating that the ‘creamy layer 1 among OBCs cannot qualify
for job reservations under its interpretation of the Mandal Commission
recommendations _ is nothing short of a knockout punch to the States like U P. and
XVI
Constitutional Law o f India- II

Bihar. Held, that such criterion for identification of ‘creamy layer’ is violative of Art.
14 and Art. 16(1) and against the Mandal judgment.
If these absurdities were allowed to continue, Mandal would have been
stood on its h^ad: the elite, purely on the basis of caste, and using the power
already at their command, would have simply continued hogging the reservations
that should have gone to the genuinely deserving. If the rich and the poor treated
alike in the matter of job^servation, they are bound to benefit at the cost of the
poor. The Supreme Court by stnking down the creamy layer test by the two
Governments has contributed considerably for social justice

LEADING CASE: INDRA SAWHNEY ll v UNION OF INDIA (AIR 2000 SC 498)


In Indra Sawhney II case, the State of Kerala followed the footsteps of
Bihar and created farcical criteria which would circumvent the
requirement of excluding the creamy layer. It passed an Act [The Kerala
State Backward Classes (Reservation of Appointments or Posts in
Services) Act, 1995] which laid down that as the facts existed in the
State, there are no socially advanced categories in any Backward
Classes and they would continue to be entitled to reservation under Art.
16(4).
The Act was passed in 1995 but it was given retrospective effect
from 1992. The State sought for the extension of time for setting up a
Commission for identifying the creamy layer but did not do so for three
years. The Supreme Court directed the Kerala High Court to appoint a
committee to identify creamy layer under the chairmanship of a retired
High Court Judge and also directed the State Government to extend
cooperation to the committee.
The Apex Court was forced to observe that the discriminatory law
made by Kerala was in virtual defiance of the Rule of law and an
indefensible breach of equality principle which is the basic structure of
the Constitution. The Act was violative of Arts. 14, 16(1) and 16(4) of the
Constitution and, therefore, unconstitutional and invalid. The ‘creamy
layer1 in the backward classes is to be treated on par with the forward
classes; thus, disentitled for reservation, otherwise there will be
discrimination and violation of Arts. 14 and 16(1), as equals - creamy
layer of backward classes and forward classes - cannot be treated
unequally. Non-exclusion of creamy layer will also be violative of Arts.
16(1) and 16(4), since unequal (creamy layer) cannot be treated as
equals i.e. equal to the rest of the backward class.
The court stated that non-exclusion of creamy layer or inclusion of
forward castes in the list of backward classes is totally illegal It warned
that the illegality attacking the root of the Constitution cannot be allowed
to be perpetuated even by a Constitutional Amendment
Comments - Thus within a decade of allowing caste-based reservation
the Supreme Court is finding that it has left the door open for a flood of
Right to Equality 69

constitutional violations of the guarantee of equality.] 20

(b) Reservation in Promotions Permitted: Recent Amendments to Art. 1621


The Constitution (77th Amendment) Act, 1995 - This amendment has been passed
to remove the difficulty created by the Mandal case in which the court had held that
reservation could not be made to promotions in jobs. This amendment has added
a new clause (4-A) to Art 16 which provides that "Nothing in this article shall
prevent the State from making any provision for reservations in the matters of
promotion to any class or classes of posts in the services of the State in favour of
SCs and STs which, in the opinion of the State, are not adequately represented in
the services under the State." This means that the reservation in promotions in
government jobs will continue in favour of SCs and STs. Further, the State may
provide for reservation i.e. the extent and nature of reservation is a matter for the
State to decide.
The evil of reservation in promotions was abolished by the Supreme Court
as it caused lot of bitterness and disappointment among employees of the same
category, who were by-passed by their colleagues having less merits. There was
no demand for it from any section of SCs and STs. In view of this, the amendment
for reservation in promotions is hardly justifiable. The haste in which the
Government had brought the Amending Bill shows that it was passed for political
considerations. It has its own dangers. Later, demand for such reservation can be
n ,ade for OBCs also.
‘Right to promotion’ is ordinarily a statutory right. It is not a fundamental
right. The right to promotion to a post/class of posts depends upon the operation of
the conditions of service. Art. 16(4-A) read with Arts. 16(1) and 14 guarantees a
right to promotion to SCs/ STs as a fundamental right where they do not have
adequate representation consistently with the efficiency in administration [Ashok
Kumar Gupta v State of UP. (1997) 5 SCC 201],

20 B.K. Sharma, p. 81.


21 Describe the Constitutional provisions relating to reservation in promotions.
[D.U.-2008]
Reservations should be made only in respect of direct recruitment at any
level but not in respect of promotions.” [I A S -97]
Right to Equality

(C) Catch-up Rule Negated


l=ighty-fifth Amendment (2001): Amended Clause (4-A) of Art. 16 and substitutes
for the words “in matter of promotion to any class” the words “in matters of
promotion with consequential seniority, to any class”. It means that the promotion
will be given to these classes with retrospective date i.e. 17th June 1995. It
nullifies the decision in the Mandal case, wherein it was held that there can be no
reservation in promotions.
The Amendment also nullifies the effect of the Supreme Court decisions in
UOI v Virpal Singh Chauhan (1995) 6 SCC 684, and, Ajit Singh Juneja v State of
Punjab (AIR 1996 SC 1189) The Supreme Court had held that on promotion the
government servants belonging to SCs/STs will not have the benefit of
consequential seniority. By the 85th Amendment it has been ensured that such
government servants will be entitled to consequential seniority on being promoted.
In Ajit Singh case, the court observed that the rule of reservation gave
accelerated promotion but it did not give the accelerated consequential seniority. A
reasonable balancing of the rights of general candidates and roster (reserved)
candidates would be achieved by following the 'catch-up rule’. According to it, if a
senior general candidate at a particular level reaches the ‘next level’, before the
reserved candidate at 'that next level' goes further up to next higher level, in that
case the seniority at ‘that next level' has to be modified by placing such a general
candidate above the roster point promotee i.e. reserved candidate.
The effect of the 85th Amendment is that when reserved candidates are
promoted earlier to general candidates, their seniority in the new cadre would rank
from the date of their joining on promotion and this seniority would not be wiped
out after the promotion of general candidates from their respective dates of
promotion and that the general candidates would remain junior to the reserved
candidates.
This is another instance of national interest and efficiency in administration
being sacrificed to gain transitory political support. Also, as noted earlier, the 82nd
Amendment (2000), inserted a proviso in Art. 335, relaxing qualifying marks of
promotion to SCI ST candidates. This reversed Mandal and Vinod Kumar, in S.
Vinod Kumar v UOI (1995) 6 SCC 580, held that the lowering standards and
qualifying marks for reserved categories is contrary to Art. 335, and, Mandal.

(d) Backlog Vacancies -‘Carry Forward Rule’ Diluted


The Constitution (81st Amendment) Act, 2000 - This amendment has added a new
clause (4-B) to Art. 16 which ends the 50% ceiling on reservation for SC/STs and
OBCs in backlog vacancies which could not be filled due to the non-availability of
e|
igible candidates of these classes in the previous year or years. The new clause
Provides that the unfilled vacancies would be treated as a separate class and
w
°uld be filled in succeeding year or years and will not be considered together with
XVI
Constitutional Law o f India- II

vacancies of the year in which they are being filled up, even if the 50% limit
imposed in the Mandal case is crossed. 22

Comments
The aforesaid four amendments (77th, 81st, 82nd and 85th Amendments) were
intended to nullify the Mandal Case. In M. Nagaraj (below-discussed), the Supreme
Court upheld the validity of these amendments on the ground that the amendments
are enabling in nature. Parliament has not provided content to the right If a State
enacts a law in regard to reservation which goes beyond the parameters set by Art.
16(4) and Art. 335 then the Court will set aside the legislation. Also, the State will
have to introduce ‘time-cap’ in the carry over rule.

Constitutional Validity of Reservation in Promotions

Leading Case: M. nagaraj V union of INDIA (AIR 2007 SC 71)

In this case, the petitioner invoked Art. 32 for a writ in the nature of
certiorari to quash the Constitution (85th Amendment) Act, 2001 inserting
Art. 16(4A) retrospectively from 17-6-1995 providing ‘reservation in
promotion with consequential seniority’ for SCs and STs as being
unconstitutional and violative of basic sfructure. The main issue
concerned the “extent of reservation” (reservation within reservation). A
five-judge Bench unanimously held that Clause (4A) of Art 16 is an
enabling provision. It applies only to SCs and STs. The said clause is
carved out of Art. 16(4). Therefore, clause (4A) will be governed by the
two compelling reasons “backwardness” and ‘inadequacy of
representation” as mentioned in Art. 16(4) If the said two reasons do not
exist then the enabling provision cannot come into force The State can
make provision for reservation only if the above two circumstances exist.
The court further held that clause (4B) of Art. 16 [inserted in Art.
16 by the Constitution 81st Amendment Act, 2000], treating unfilled
reserved vacancies as separate class (could go beyond the 50% limit), is
also an enabling provision. The court also upheld the constitutional
validity of the Constitution 82nd Amendment Act, 2000, which inserted a
proviso in Art. 335 relaxing qualifying marks of promotion to SCI ST
candidates.
The Apex Court held that they are enabling provisions and do not
obliterate constitutional requirements namely, ceiling limit of 50%
(quantitative limitation), the concept of creamy layer (qualitative
limitation), the concept of sub-classification between OBCs and SCs and
STs, and, do not change the equality code indicated in Arts. 14,15 and
16. They retain the controlling

22 Examine the constitutional validity of the following Government order


Carrying forward of unfilled posts to the next year. [C LC-2007\[L.C. 1-2007]
Right to Equality 71

factors namely- backwardness and inadequacy of representation which enables


the States to provide for reservation keeping in mind the overall efficiency of the
State administration under Art. 335. Thus, the parameters mentioned in Art. 16(4)
are retained These amendments do not change the identity of the Constitution,
and, are not beyond amending power of Parliament.
The court observed that the theory of basic structure is the only theory to
judge validity of constitutional amendment. In the matter of application of this
theory, twin tests have to be satisfied, namely, the ‘width test' and the ‘test of
identity’. Neither of these tests has been violated in the present case. Hence there
is no violation of basic structure of the Constitution by any of the impugned
legislation. For instance, constitutional limitation has been relaxed but not
obliterated by the 82nd Amendment Act Subject to the constitutional limitations
(50% ceiling, creamy layer, efficiency in administration), the impugned
amendments are valid.
The Apex Court observed and held as follows:
(i) Equality of opportunity has two different and distinct concepts.
There is a conceptual distinction between a nondiscrimination
principle and affirmative action under which the State is obliged to
provide a level playing field to the oppressed classes. Affirmative
action in the above sense seeks to move beyond the concept of
non-discrimination towards equalizing results with respect to various
groups. Both the conceptions constitute “equality of opportunity".
(ii) Social justice is one of the sub-divisions of the concept of justice. It
is concerned with the distribution of benefits and burdens. The basis
of distribution is the area of conflict between rights, needs and
means. These three criteria can be put under two concepts of
equality, namely “formal equality" and “proportional equality".
Formal equality means that the law treats everyone equal. Thus, it
exists in the rule of law. Concept of egalitanan equality is the
concept of proportional equality and it expects the States to take
affirmative action in favour of disadvantaged sections of society
within the framework of democratic polity. Proportional equality is
equality "in fact” whereas formal equality is equality “in law”.
(iii) The gravamen of Art. 14 is equality of treatment Art 14 confer a
personal right by enacting a prohibition which is absolute. By judicial
decisions, the doctrine of classification is read into Art. 14. Equality
of treatment under Art. 14 is an objective test. It is not the test of
intention. Therefore, the basic principle underlying Art. 14 is that the
law must operate equally on all persons under like circumstances.
Every discretionary power is not necessarily discriminatory.
According to the Constitutional
Law of India, by H M Seervai, 4th Edn, p 546, equality is not violated by mere
conferment of discretionary power It is violated by arbitrary exercise by those
on whom it is conferred This is the theory of "guided power”. This theory is
based on the assumption that in the event of arbitrary exerase by those on
XVI
Constitutional Law o f India- II

whom the power is conferred, it would be corrected by the courts. This is the
basic principle behind the enabling provisions which are incorporated in Arts.
16(4-A) and 16(4- B). Enabling provisions are permissive in nature. They are
enacted to balance equality with positive discrimination.
(iv) The constitutional law is the law of evolving concepts Some of them are
generic, others have to be identified and valued. The enabling provisions deal
with the concept, which has to be identified and valued as in the case of
access vis-ci-vis efficiency which depends on the fact Situation only and not
abstract principle of equality in Art 14 as spelt out in detail in Arts. 15 and 16.
Equality before the law, guaranteed by the first part of Art. 14, is a negative
concept while the second part is a positive concept which is enough to
validate equalizing measures depending upon the fact situation.
(v) It is important to bear in mind the nature of constitutional amendments They
are curative by nature. Art 16(4) provides for reservation for Backward
Classes in cases of inadequate representation in public employment. Art.
16(4) is enacted as a remedy for the past historical discriminations against a
social class. Art. 16(4) did not confer any fundamental right to reservation; it is
only an enabling provision. The object in enacting the enabling provisions like
Arts. 16(4), 16(4-A) and 16(4-B) is that the State is empowered to identify and
recognize the compelling interests. If the State has quantifiable data to show
backwardness and inadequacy then the State can make reservations in
promotions keeping in mind maintenance of efficiency which is held to be a
constitutional limitation on the discretion of the State in making reservation as
indicated by Art. 335. The concepts of efficiency, backwardness, inadequacy
of representation are required to be identified and measured. That exercise
depends on availability of data and on numerous factors. It is for this reason
that enabling provisions are required to be made because each competing
claim seeks to achieve certain goals.
(vi) How best one should optimize these conflicting claims can only be done by
the administration in the context of local prevailing conditions in public
employment. Therefore, there is a basic difference between “equality in law"
and “equality in fact". If Arts 16(4-A) and 16(4-B) flow from Art. 16(4). and, if
Art 16(4) is an enabling provision then Arts. 16(4-A) and 16(4-B) are also
enabling provisions. As long as the boundaries mentioned in Art. 16(4),
namely, backwardness, inadequacy and efficiency of administration are
retained in Arts. 16(4-A) and 16(4-B) as controlling factors, we cannot
attrioute constitutional invalidity to these enabling provisions. However, when
the State fails to identify and implement the controlling factors then
excessiveness comes in, which is to be decided on the facts of each case. In
a given case, where excessiveness results in reverse discrimination, this court
has to examine individual cases and decide the matter in accordance with
law. This is the theory of “guided power”.
(vii) In Indra Sawhney case, the equality which was protected by the rule of 50%,
was by baiancing the rights of the general category vis-&-vis the rights of BCs
Right to Equality 73

en bloc consisting of OBCs, SCs and STs On the other hand, in the present
case, the question which we are required to answer is: whether within the
egalitarian equality, indicated by Art. 16(4), the sub classification in favour of
SCs and STs is in principle constitutionally valid. Art. 16(4-A) is inspired by
the observation in Indra Sawhney in which this court has unequivocally
observed that in order to avoid lumping of OBCs, SCs and STs which would
make OBCs take away all the vacancies leaving SCs and STs high and dry.
the State concerned was entitled to categorize and sub-classify the SCs and
STs on one hand vis-^-vis OBCs on the other hand.
(viii) The test for judging the width of the power and the test for adjudicating the
exercise of power by the State concerned are two different tests which
warrant two different judicial approaches. In the present case, as stated
above, we are required to test the width of the power under the impugned
amendments. Therefore, we have to apply “the width test”. In applying “the
width test’ we have to see whether the impugned amendments obliterate the
constitutional limitations mentioned in Art. 16(4), namely, backwardness and
inadequacy of representation. As stated above, these limitations are not
obliterated by the impugned amendments. Applying the test of "identity ’, we
do not find any alteration in the existing structure of the equality code (Arts.
14, 15 and 16) by the impugned amendments.
The boundaries of the ‘width of power are: (1) The ceiling limit of 50%
(quantitative limitation); (2) The principle of
creamy layer (qualitative exclusion); (3) The compelling
reasons viz. backwardness, inadequacy of
representation, etc. (4) The overall administrative
efficiency.
(ix) The so-called "catch up'' rule is that a reserved category
candidate promoted on the basis of reservation earlier
than his senior general category candidates in the feeder
grade, shall necessarily be junior in the promoted
category to such general category candidates This rule is
not implicit in Arts. 16(1) to
(4) . The concept of the “catch up” rule and
“consequential seniority" are not constitutional
requirements or limitations. They are judicially evolved
concepts to control the extent of reservation, derived
from service jurisprudence They are not constitutional
principles so as to be beyond the amending power of
Parliament It cannot be said that by insertion of the
concept of "consequential seniority" the structure of Art.
16(1) stands destroyed or abrogated. Obliteration of
these concepts or insertion of these concepts does not
change the equality code indicated by Arts. 14, 15 and
16.]
Constitutional Law o f India- II

Constitutional Validity of Reservations for OBCs in Educational Institutions

Leading Case: ASHOKA KUMAR THAKUR v UOI [(2008)


6 SCC 1]
Facts and Issue - In this case, the reservation of 27% seats for OBCs in
specified State-aided Central Universities under the Central Educational
Institutions (Reservation in Admission) Act, .2006 (Act 5 of 2007) was in
issue. The Constitution (Ninety-Third Amendment) Act, 2005, by which
Art. 15(5) was inserted in the Constitution, was basically challenged in
these petitions on various grounds. The Act of 2007 was passed by the
Parliament consequent upon the 93 rd Amendment Act Thus, the validity
of the Act of 2007 depends upon the fact whether the Constitution
(Ninety- Third Amendment) Act, 2005 itself is valid or not.
The Constitution (Ninety-Third Amendment) Act, 2005, is
challenged on many grounds, viz. if it is allowed to stand it would be
against the “basic structure” of the Constitution itself and this
Amendment seriousiy abridges the equality principles guaranteed under
Art. 15 and other provisions of the Constitution; the Golden Triangle' of
Arts. 14,19 and 21 is not to be altered and the balance and structure of
these constitutional provisions has been ousted by the Amendment Act,
2005
It was contended that the Union Government has faiied in
performing the constitutional and legal duties towards the citizenry and
its resultant
Right to Equality 75

effect. Consequentially the Act of 2007 shall have the effect and wide ramifications
and ultimately it shall have the result in dividing the country on caste basis.
Besides affecting the peaceful atmosphere in the educational and other institutions,
it would seriously affect social and communal harmony. The constitutional
guarantee of equality and equal opportunity shall be seriously prejudiced.
Observations and Decision - The court observed: Reservation is one of the many
tools that are used to preserve and promote the essence of equality, so that
disadvantaged groups can be brought to the forefront of civil life. It is the duty of
State to promote positive measures to remove barriers of inequality. To cope with
the modern world and its complexities and turbulent problems, education is a must
and it cannot remain cloistered for the benefit of a privileged few. Reservations
provide that extra advantage to persons who, without such support, can forever
dream of university education, without ever being able to realise it This advantage
is necessary. However, the fact remains that any reservation or preference shall
not lead to 'reverse discrimination.'
The various questions and the court’s answer to them could be summarized
as below:
(I) Whether 93rd Amendment Act of the Constitution is against the basic structure
of the Constitution?
The Apex Court held: The Constitution (Ninety-Third Amendment) Act, 2005, does
not violate the basic structure of the Constitution so far as it relates to State-
maintained institutions and aided educational institutions (question as to validity of
the said amendment with respect to private unaided institutions left open).
Equality is a multi-ccloured concept incapable of a single definition. The
pnnciple of equality is a delicate, vulnerable and supremely precious concept for
our society and has embraced a critical and essential component of constitutional
identity. The larger principles of equality as stated in Arts 14,
15 and 16 may be understood as an element of the basic structure of the
Constitution and rnay not be subject to amendment, although, these provisions,
intended to configure these rights in a particular way, may be changed within the
constraints of the broader principles. The variability of changing conditions may
necessitate the modifications in the structure and design of these rights, but the
transient characters of formal arrangements must reflect the larger purpose and
principles that are the continuous and unalterable thread of constitutional identity. It
is not the introduction of significant and far-reaching change that is objectionable
rather it is the content of this change insofar as it implicates the question of
constitutional identity.
The principles of equality cannot be completely taken away so as to leave
citizens in a state of lawlessness. But the facets of the principles of equality can
always be altered, especially to carry out the directive
Constitutional Law o f India- II

principles of State policy If any constitutional amendment is made which


moderately abridges the principle of equality, it cannot be said that it violates the
basic structure of the Constitution It has not been held in Keshavananda (1973) 4
SCC 225 that all facets of Art. 14 or any of the fundamental rights would form part
of the basic structure. If any one of the provisions relating to a particular basic
feature of the Constitution is altered or modified, that does not amount to the
alteration of the basic structure.
The court also observed: Affirmative action is nothing but a crucial
component of social justice in the constitutional dispensation But at the same time
it has to be kept in view that affirmative action does not infringe the principles of
equality and/or unreasonably restrain or restrict other fundamental freedoms and
that it does not violate the basic structure of the Constitution. It needs no emphasis
that individual rights are superior to the social rights When fundamental rights are
at stake, they must be harmonized with, not made subject to, the directive
principles.
(II) Whether Arts. 15(4) and 15(5) are mutually contradictory, hence Art 15(5) is to
be held ultra vires? Whether exclusion of minority educational institutions from Art.
15(5) is violative of Art. 14 of the Constitution?
It was argued that Art. 15(4) and Art. 15(5) are mutually exclusive and under Art.
15(5), the minority educational institutions are excluded, this is a ciear
contravention of the secular and equality principles. It was contended that minority
educational institutions (which are getting aid from the States) are not severable
from the purview of Art 15(5)
The court held that Art, 15(5) is constitutionally valid and Arts. 15(4) and
15(5) are not mutually contradictory
The court reasoned: Art. 15(5) does not exclude Art. 15(4). It is a well settled
principle of constitutional interpretation that while interpreting the provisions of
Constitution, effect shall be given to all the provisions of the Constitution and no
provision shall be interpreted in a manner as to make any other provision
inoperative or otiose. If the intention of the Parliament was to exclude Art. 15(4),
they could have very well deleted it Minority institutions are also entitled to the
exercise of fundamental rights under Art. 19(1 )(9). whether they be aided or
unaided But, in the case of Art 15(5), the minority educational institutions (whether
aided or unaided) are excluded from the purview of Art. 15(5). Both, Arts 15(4) and
15(5), being enabling provisions, would operate in their own field and the validity of
any legislation made on the basis of Art. 15(4) and Art. 15(5) have to be examined
on the basis of provisions contained in such legislation or the special provision that
may be made under Arts. 15(4) and 15(5)
As has been held in N.M. Thomas case and Indra Sawhney case, Art. 15(4)
and Art. 16(4) are not exceptions to Art. 15(1) and Art. 16(1) but independent
enabling provisions. Art. 15(5) also to be taken as an enabling provision to carry
out certain constitutional mandate and thus it is constitutionally valid.
Right to Equality 77

The court also held that exclusion of minority educational institutions


from Art. 15(5) is not violative of Art. 14 as the minority institutions, by
themselves, are a separate class and their rights are protected by other
constitutional provisions like Art. 30. In fact, exemption of minority institutions
has been allowed to conform Art 15(5) with the mandate of Art. 30.
(III) Whether the 93rd Constitutional Amendment followed the procedure
prescribed under Art. 369 of the Constitution?
The court held that the 93ra Amendment does not affect the executive power of
the State under Art. 162 of the Constitution and hence, procedure prescribed
under Proviso to Art. 368(2) is not required to be followed. The executive
power of the States under Art. 162 has been made subject to the executive
power expressly conferred by the Constitution or by any law made by
Parliament upon the Union authorities thereof (Proviso to Art. 162).
(IV) Whether the Act of 2007 is constitutionally invalid in view of definition of
“Backward Class" and whether the identification of such "Backward Class’’
based on “caste” is constitutionally valid?
The court observed: Classification of SEBCs cannot be done exclusively on the
basis of caste as it would violate Art 15(1). Poverty, social backwardness,
economic backwardness, all are criteria for determination of backwardness.
However, following Indra Sawhney case, the court observed that the ideal and
wise method would be to mark out various occupations, which on the lower
level in many cases amongst Hindus would be their caste itself and find out
their social acceptability and educational standard, weigh them in the balance
of economic conditions, and the result would be an eligible 'backward class’ of
citizens
Caste plays an important role in determining the backwardness of the
individual In the present case, it was clear from the material on record that the
lists of SEBCs were being prepared validly, not solely on the basis of caste, but
after consideration of detailed data with regard to social, educational and
economic cnteria. Thus, other parameters are followed in identifying the
backward class. Therefore, Act 5 of 2007 is not invalid for this reason. Thus, if
the real backwardness is found in a caste, it can be considered backward.
Similarly if real backwardness is found in any other group, section or class they
too can be treated as backward. It is clear from Indra Sawhney case that caste
itself is not the final destination, that is, caste by itself cannot be determinative
of social and educational backwardness, though caste can be the starting point
for determination of backwardness.
If 49 5% caste-based reservation was upheld in Indra Sawhney case for
government employment, it follows that 49.5% caste-based reservation is
permitted in aided educational institutions. Indra Sawhney ' case compels that
conclusion that use of caste is valid. Hence, there is no choice but to uphold
the impugned legislation by which the Government may still identify SEBCs, in
part, by using caste.
Social and educational backwardness if may determined purety on 'economic
criteria’ (per Pasayat and Thakker, JJ ; Bhandari, J impliedly in concurrence)
- Passage of time shows that the occupational label has lost much of its
XVI
Constitutional Law o f India- II

significance A blind eye cannot be turned to the poor, those covered by the all-
encompassing expression "economically backward classes.” The poor have no
caste A person belonging to a higher caste should not be made to suffer for what
his forefathers had done several generations back If the creamy layer has to be
excluded the economically backward classes have to be included. That would be
social balancing and would be giving true meaning to the objectives of the
Constitution Data shows that income is a better determinant of educational
achievement than caste. Therefore, economic criteria should be the sole means by
which beneficiaries of special provisions under Art. 15(5) are to be identified.
Economic criteria however must include occupation and landholdings, because
income alone is insufficient. Using purely economic criteria would lighten the
identification load, as ascertaining caste would no longer be required. The problem
of too many people becoming eligible on the basis of economic criteria would only
arise if reservation was made based on the group's proportion of the total
population. If economic reservation were limited to a reasonable number, it could
be upheld Hence, the Government is urged that for a period of 10 years, caste and
other factors such as occupation/income/property holdings or similar measures of
economic power may be taken into consideration and thereafter only economic
criteria should prevail.
(V) Whether “Creamy Layer" is to be excluded from SEBCsl
The court observed: Act 5 of 2007 is constitutionally valid subject to the definition
of “Other Backward Classes” in Sec 2(g) of Act of 2007 being clarified as follows; If
the determination of “Other Backward Classes” by the Central Government is with
reference to a caste, it shail exclude the “creamy layer' among such caste Hence,
“creamy layer” is to be excluded from SEBCs. The identification of SEBCs will not
be complete and without the exclusion of “creamy layer” such identification may not
be valid under Art. 15(1) The “Other Backward Classes” defined in Sec 2(g) of Act
5 of 2007 is to be read as "Socially and Educationally Backward Classes” other
than Scheduled Castes and Scheduled Tribes, determined as ‘Other Backward
Classes” by the Central Government Hence, for implementation of the impugned
statute, creamy layer must be excluded.
Had Parliament insisted on creamy layer inclusion, it could have said as
much in the text of Art. 15(5). Instead, Parliament left the text of Art. 15(5) silent on
the issue, delegating the issue of OBCs identification to the executive in Sec 2(g).
It seems unlikely that it would have been an all or nothing proposition for
Parliament, when the very goal of the impugned legislation of promoting OBC
educational advancement does not depend on creamy layer inclusion Not even
Parliament, by constitutional amendment, could dismantle the basic structure by
including the creamy layer in reservation. For these
Right to Equality

reasons,the implied inclusion of the creamy layer in Art. 15(5) is severed or


excluded The court also said that the "creamy layer rule” is a necessary bargain
between the competing ends of caste-based reservations and the principle of
secularism. It is a part of constitutional scheme.
When socially and educationally backward classes are determined by
giving importance to caste, the segment of that caste that is economically
advanced does not deserve any sort of reservation for further progress in life.
They are socially and educationally advanced enough to compete for the
general seats along with other candidates The contention is rejected that if the
“creamy layer” is excluded there may practically be no, representation for a
particular backward class in educational institutions because the remaining
members, namely, the non-creamy layer, may not have risen to the level or
standard necessary to qualify or get admission even within the reserved quota.
By inclusion of the creamy layer a fresh lease of life is given to those who
should have been left out. When Indra Sawhney case has held that the creamy
iayer should be excluded for purposes of Art. 16(4), dealing with “backward
class” which is much wider than “socially and educationally backward class"
occurring in Arts. 15(4) and 15(5), it goes without saying that without the
removal of the creamy layer there cannot be a socially and educationally
backward class.
In Indra Sawhney case, it has been aptly observed than reservation is
given to backward classes until they cease to be backward, and not indefinitely.
Creamy layer inclusion robs the poor and gives to the rich. With the creamy
iayer excluded, poor OBCs would compete with poor OBCs: the playing field
levelled. Lastly, exclusion of the creamy layer does not violate the group rights
of the classes sought to be benefited by Art. 15(5) for the group must contain
only those individuals that belong to the group.
The court, in the present case, observed: The principle of creamy layer
emanates from the broad doctrine of equality itself Arts. 15(4) and 15(5) are
designed to provide opportunities in education thereby raising educational,
social and economic levels of those who are lagging behind and once this
progress is achieved by this section, any legislation passed thereunder should
be deemed to have served its purpose. By excluding those who have already
attained economic advancement, the special benefits provided under these
clauses cannot be further extended to them and, if done so, it would be
unreasonable, discriminatory or arbitrary, resulting in ‘reverse discrimination.’
(VI) What should be the parameters for determining the “Creamy Layer' group?
The court held: The parameters contained in the Office Memorandum issued by
the Government of India, Ministry of Personnel, Public Grievances and
Pensions (Department of Personnel and Training) on 08.09.1993 may be
applied. This was issued after the decision in Indra Sawhney case
XVI
Constitutional Law o f India- II

The court, however, made it clear that same principle of determining the
creamy layer for providing 27% reservation for backward classes for appointment
need not be strictly followed in case of reservation envisaged under Art. 15(5). For
example, if a strict income restriction is made for identifying the “creamy layer”,
those who are left in the particular caste may not be able to have a sufficient
number of candidates for getting admission in the Central Institutions as per Act of
2007. The Government can make a relaxation to some extent so that sufficient
number of candidates may be available for the purpose of filling up the 27%
reservation. It is for the Union and State Governments to issue appropriate
guidelines to identify the "creamy layer” so that SEBCs are properly determined in
accordance with the guidelines given by this court.
(VH) Whether the “creamy layer” principle is applicable to SCs and STs?
It was held in Indra Sawhney case that “creamy layer” principle is not applicable to
SCs/STs. In the instant case, the entire discussion was confined only to OBCs.
Therefore, no opinion is expressed with regard to applicability of exclusion of
creamy layer to SCs and STs. However, according to M Nagaraj case, reservation
in promotion for SCs and STs is contingent on exclusion of the creamy layer It was
held therein that if the State fails to exclude SC/ST creamy layer, the reservation
must fail.
Nevertheless, the court, in the present case held: Right from the beginning,
the SCs and STs were treated as a separate category and nobody ever disputed
identification of such classes. So long as “creamy layer" is not applied as one of
the principles of equality, it cannot be applied to SCs and STs. So far, it is applied
only to identify the 'socially and educationally backward classes ”
(VIII) Whether the principles laid down by the United States Supreme Court for
affirmative action such as “suspect legislation”, “strict scrutiny” and"compelling
State necessity” are applicable to principles of reservation or other affirmative
action contemplated under Art. 15(5)7
It was contended that the classification of OBCs was not properly done and it is not
clear as to whose benefit the legislation itself is made therefore, it is a “suspect
legislation.” Thus, such legislation should be subjected to “strict scrutiny" test as
laid down by the U S Supreme Court. According to this test, the University, whose
affirmative action programme is in question is required to prove that its programme
has been designed in the narrowest possible manner, in order to benefit only those
specific people who are to be benefited, thus serving the “compelling purposes” of
the affirmative action programme. Otherwise, it may be possible that the rights of
other people may be infringed upon, which would make the affirmative action
programme unconstitutional
The court noted that the decisions of the U.S. Supreme Court were not
applied in the Indian context as it was felt that the structure of the provisions under
the two Constitutions and the social conditions as well aS other factors are widely
different in both the countries. The gamut of affirmative action in India is fully
supported by constitutional provisions and the principles of "suspect legislation”
have not been applied. The country has been following the doctrine that every
legislation passed by the Parliament is presumed to be constitutionally valid unless
otherwise proved.
(IX) Whether delegation of power to the Union Government to determine as to who
Right to Equality 81

shall be the backward class is constitutionally valid?


It was held that such delegation is constitutionally valid: it is not excessive
delegation. The determination of backward classes itself is a laborious task and the
Parliament cannot do it by itself. There are sufficient guidelines (parameters to be
used) to determine the backward classes. If any undeserving caste or group of
persons are included in the backward class, it is open to any person to challenge
the same through judicial review.
(X) Whether the Act of 2007 is invalid as there is no time limit prescribed for the
operation and no periodical review is contemplated!
It was held that Act of 2007 is not invalid for the reason that there is no time limit
prescribed for its operation, but a review can be made after a period of 10 years.
And the Parliament could examine whether the reservation has worked for the
good of the country.
“Reservation will have to stop some day...after some years or decades. If it
is perpetual, the entire objective is defeated" The court was reacting to the
argument that there had to be proper monitoring and periodic review of the
reservation policy to see if it has achieved the desired objective.
The court also observed. Classification on the basis of castes in the long run
has the tendency of inherently becoming pernicious. Therefore, the test of
reasonableness has to apply. When the object is elimination of castes and not
perpetuation thereof, to achieve the goal of a casteless society and a society free
from discrimination based on caste, judicial review within permissible limits is not
ruled out If there is no review of caste-based reservation, the country will become a
caste divided society permanently. While affirmative action is a road to equality,
care should be taken that the road does not become a rut in which the vehicle of
progress gets entrenched and stuck Reservation is a temporary crutch and should
not become a permanent liability.
(XI) What shall be the educational standard to be prescribed to find out whether
any class is educationally backward?
While determining educational backwardness, graduation (not technical
graduation) or professional shall be the standard test/ yardstick for measuring
backwardness. Once a candidate has graduated from a university, the said
candidate is educationally forward and is ineligible for special benefits under Art.
15(5) for postgraduate and any further studies.
The contention that educational standard of matriculation or (10+2)
should be the benchmark to find out whether any class is educationally
backward is rejected.
(XII) Whether the quantum of reservation provided for in the Act is valid
and whether 27% of seats for SEBCs was required to be reserved?
It was held that 27% of seats for other backward classes is not illegal and
the Parliament must be deemed to have taken into consideration all
relevant circumstances when fixing the 27% reservation.
Government can make relaxation to some extent, reasonable cutoff
marks should be set so that standards of excellence are not greatly
affected. Whenever the non-creamy layer OBCs fail to fill the 27%
reservation after adopting cut-off marks below that of the cut-off for the
XVI
Constitutional Law o f India- II

General Category, the unfilled seats shall revert to and be filled up by


candidates from the General Category.]
Comments - In Jagdish Negi, President, Uttarakhand Jan Morcha v State of U.P.
(AIR 1997 SC 3505), it was held that reservation is subject to periodical review by
the Government. Reservation : Subject to periodical review by GovemmentThe
court observed: “It is not possible to agree that reservation should continue without
any limitation i.e. indefinitely, or there cannot be periodical review about the said
reservation policy. The statutory scheme of 27% reservation may continue
indefinitely till the Reservation Act continues to operate but a given category of
citizens which may form a part and parcel of that socially and educationally
backward class of citizens as on that date may in future cease to belong to that
class. The State cannot be bound in perpetuity to treat such classes of citizens for
all time as socially and educationally backward class of citizens. It is open to the
State to review the situation from time to time and to take its own policy decision in
the light of relevant material available to it. Thus, reservation on year to year basis
is valid."
The concept of equality enshrined in Part III and Part IV of the Constitution
has two different dimensions. It embodies principle of non-discrimination [Articles
14, 15(1), (2) and 16(2)]. At the same time it obligates the State to take affirmative
action for ensuring that unequals (downtrodden, oppressed and have-nots) in the
society are brought at a level where they can compete with others (haves of the
society) [Articles 15(3), (4), (5), 16(4), (4-A), (4-B), 39, 39-A and 41] [UOI v Pushpa
Rani (2008) 9 SCC 242],
The court further observed: Legislative and administrative measures taken by
the State for providing reservation of seats and posts in the field of education and
employment are reflective of affirmative action taken for achieving the goal of real
equality. However, implementation and execution of such actions have
continuously faced roadblocks at several stages. Those who had not been
benefited by the existing system cried foul and created the bogey of violation of
their legal and constitutional rights. Almost all the actions taken by the State and its
agencies for ameliorating conditions of have-nots of the society by providing
reservation were subjected to periodical judicial scrutiny. By and large, the courts
approved the affjrmative actions of the State but on some occasions the policy of
reservation or implementation thereof was found to be faulty and actions taken by the
Government nave been nullified or sliced by judicial intervention.

fjo Sub-classification of Scheduled Castes


Backward class of citizens may be classified into four categories (i) backward (ii)
more backward (iii) Scheduled Castes and (iv) Scheduled Tribes. But Scheduled
Castes cannot be further divided into different categories. Seats reserved for SCs
cannot be divided among them on the basis that some of them are more backward
[E.V. Chinnaiah v State of A. P. (2005) 1 SCC 394],

Reservation to Single Isolated Post


In UOI v Madhav (AIR 1997 SC 3074), it was held that reservation could be
Right to Equality 83

provided even to the isolated posts on the basis of the rule of rotation. Extension
of reservation in such cases is not unconstitutional. Similarly, held in UOI v Brij l.al
Thakur (AIR 1997 SC 2101).
In State of Punjab v G.S. Gill (AIR 1997 SC 2324), it was held that ceiling of
50% reservation in a recruitment year does not apply where there is only one post.
With a view to give adequate representation in public service to reserved category
candidates, the opportunity given to them is not violative of Arts. 14 and 16(1).
!n Post Graduate Institute of Med. Ed. & Res. v K.L. Narasimhan (1997) 6
SCC 283, there was one post each in various disciplines of medicines in the
institute. All posts carried same pay scale but were not interchangeable. The
institute clubbed all the posts together for the purpose of applying roster (backlog
vacancies for SC/STs). Held that reservation to a single cadre post and the
applicability of the roster to it, is constitutionally valid. Aggrieved by that decision,
the Faculty Association of the P.Gl., Chandigarh moved a review petition. Allowing
it, the Supreme Court held that any attempt at reservation, by whatever means, in
a single-post cadre (even through the device of rotation of a roster) was “bound to
create 100% reservation in such cadre” [Posf Graduate Institute of Med. Ed. &
Res. v Faculty Association [JT 1998(3) SC 223],
The Constitution Bench heid: “In a single post-cadre, reservation at any
point °f time on account of rotation of roster is bound to bring a situation where
such single post in the cadre will be kept reserved exclusively for the members of
backward classes and in total exclusion of general members of the public, and
cent Percent reservation for backward classes is not permissible within the
constitutional framework. Until there is “plurality of posts” in a cadre, the question
of reservation W 'H not arise”. The Court further held that Arts. 14, 15 and 16
including Art. 16(4) and 16(4A) should be applied in such a manner so that a
balance is struck in the Matter of appointments by creating reasonable
opportunities for reserved classes ar>d also for other members (non-reserved) of
the community.
8 4 Constitutional Law o f India- li

Reservation in Super Specialties


In State of M P. v Nivedita Jain (AIR 1981 SC 2045) the Supreme Court upheld the
validity of an Executive Order of State Government which had completely relaxed
minimum qualifying marks in Pre-medical examination for selection of students to
Medical Colleges of the State in respect of SC & STs. Under Art. 15(4), the State is
obliged to do everything possible for upliftment of SCs/STs and other backward
communities.
In Dr. Priti Srivastava v State of I/I.P. (AIR 1999 SC 2894) held that lowering
of the minimum qualifying marks for admission to "super-specialty” post-graduate
medical courses in favour of the reserved category candidates is unconstitutional
and violative of Art. 15(4). This is in agreement with the Mandal judgment
(discussed below) in which it was held that merit aione would count in super
specialties Admissions to the highest available medical courses in the country at
the super specialty levels, where even the facilities for training are limited must be
given on the basis of competitive merit. ‘Merit alone can be the criterion for
selecting students to the super specialty courses'. The object of Art. 15(4) is to
advance the equality principle by providing for protective discrimination in favour of
the weaker sections of society so that they may become stronger and be able to
compete equally with others, one cannot ignore the wider interests of society while
devising such special provision
However, in State of Punjab v Dayanand Medical College & Hospital ( 2001)
3 SCC 664, upholding d notification of the State providing for reservation under Art.
15(4), the Supreme Court has narrowed down the application of Priti Srivastava
case to the extent that the State’s power under Art. 15(4) is not subject to the
power of the Medical Council of India to prescribe standards for medical education
The State, however, must coordinate its admission policy with that of the Medical
Council.
In K. Duraisamy v State of T.N. (AIR 2001 SC 718), the Supreme Court laid
down that it is now a proposition well settled that at the super specialty level in
particular and at the Post-graduate level, reservation in favour of backward classes
should be avoided as being not permissible.
Institutional reservation not permissible
In AIIMS Students’ Union v AIIMS (AIR 2001 SC 3262), the Supreme Court while
emphasizing “merit-based” selection, struck down the institutional reservation (i.e.
for students of AIIMS) of 33% coupled with 50% reservation discipline-wise in the
Institute’s post-graduate courses, as violative of Art. 14 The court observed: Any
reservation, apart from being sustainable on the constitutional anvil, must also be
reasonable to be permissible. Reservation other than constitutional reservation is
subversion of fraternity, unity, integrity and dignity of individual. Institutional
reservation is not supported by the Constitution.

Reserved Category Candidates Competing in Open Category


In Manjit Singh v State of Punjab (AIR 1997 SC 318), the Supreme Court ruled that
if a person from a reserved class did well on merit and secured a higher position
Right to Equality 85

than the general category candidates, he should not be denied to take advantage
of ftis merit and compete for the seats meant for the general candidates. The policy
of the Government according to which only such number of candidates for the
reserved category would be allowed admission as were equal to the number of
reserved seats, even though large number from that class might have secured more
marks than the general candidates, the Court held, would be arbitrary and violative
of Art. 14.
In Mandal case also, the Supreme Court said that “reserved category
candidates getting selected in open competition on the basis of their merit should
net be counted against the quota reserved for them”. Further, in P.GI. of Med. Ed.
& Res. case (19S7) 6 SCC 283, it was heid that if a reserved category candidate
gets selected for admission to a course, or appointment to a post, on the basis of
merit as a general candidate, he should not be treated as reserved candidate.
Only one who does get admission/appointment by virtue of relaxation of eligibility
criteria should be treated as a 'reserved" candidate.

Inter-changeability of reserved/unreserved seats


In Manjit Singh v State of Punjab (AIR 1997 SC 318), the Supreme Court directed
that if the seats reserved for SC/ST/OBC candidates could not be filled up on
account of failure of the candidates belonging to these categories to obtain
minimum qualifying marks, such seats should be made available to the candidates
belonging to the general category. Keeping such seats unfilled, the Court,
borrowing the language used in Jagdish Seiran case (AIR 1980 SC 820), said,
would be a "national loss1’.
In Superintending Engineer, Public Health. Union Territory Chandigarh v
Kuideep Singh [JT 1997 (2) SC 508], the Supreme Court ruled that as per principle
of alternative exchange, where no Scheduled Tribe was there, Scheduled Caste
had to be considered for promotion.
In Post Graduate institute of Med. Ed. & Res. v K.L. Narasimha (1997) 6
SCC 283, the Supreme Court observed: Whether or not reserved vacancies
should be de-reserved, is a matter falling primarily within the administrative
discretion of the government. There is no right in general candidates to seek filling
up of the reserved vacancies and to insist on de-reservation of such vacancies so
long as it is possible in law to fill up the reserved vacancies. Carry-forward
(unfiiled) vacancies reserved for SCs/STs should be filled up cnly by the reserved
candidates and general candidates have no right to seek direction for de-
reservation thereof for filling up of the same by general candidates [S(ate of
Punjab v G.S G/7/AIR 1997 SC 2324],

Reservation for Women: Legality


•n Union of India v K.P Prabhakaran (1997) 11 SCC 638, validity of carving out a
separate cadre for women was challenged on the ground of violation of Arts. 15(1)
& 3, 16(1) & (2) and 14 The Railway Administration decided to reserve the posts °f
Enquiry-cum-Reservation Clerks in Reservation Offices in the metropolitan cities of
XVI
Constitutional Law o f India- II

Delhi. Madras, Bombay and Calcutta exclusively for women. The Madras High
Court held that Railway Administration’s decision is violative of Arts 14 and 16(1)
and (2) and is not protected by Art. 15(3). The Supreme Court, however, upheld
the validity of Railway Administration’s decision
The Court held: Art. 15 deals with every kind of State action in relation to the
citizens of India and every sphere of activity of the State is controlled by Art. 15(1)
and therefore, there is no reason to exclude from the ambit of Art 15(1)
employment under the State. Since Arts. 15(1) and (3) go together, the protection
of Art. 15(3) would be applicable to employment under the State falling under Arts
16(1) and
(2) .Therefore, the impugned judgment of the High Court holding that
Art. 15(3) has no application in matters relating to employment under the State
falling under Arts. 16(1) and (2), cannot be upheld. ‘

Marriage and Status of SC/ST


In Neelima v Dean, PG. Studies, A P Agriculture University (AIR 1993 SC 229), it
has been held that a high caste girl marrying a boy belonging to ST is not entitled
to the benefit of reservation available to STs. In Sandhya Thaxur v Vimla Devi
Kushwah (2005) 2 SCC 731, held that the appellant, who by birth did not belong to
a backward class or community, would not be entitled to contest a seat reserved
for a backward class or community merely on the basis of her marriage to a male
of that community.
Aiso, in Meera Kanwaria v Sunita (AIR 2006 SC 597), it has been heid that if
a female of high caste Hindu marries a person belonging to Scheduled Caste she
is not entitled to take the benefit of reservation under Arts. 15(4) and 16(4) of the
Constitution. The court observed and held that a person who is a high caste Hindu
and not subjected to any social or educationai backwardness in her life, by reason
of marriage alone cannot ipso facto become a member of SC/ST In absence of any
strict proof, she cannot be allowed to defeat the very provisions made by the State
for reserving certain seats for disadvantaged people. A similar view was taken in
Sobha Hymavathi Devi v Setti Gangadhar Swamy (AIR 2005 SC 800)
In M.C. Valsala v State of Kerala (AIR 2006 Ker 1), the issue was whether
children born out of inter-caste married couple could claim the status of SCs/STs
for the benefit of reservation in admission to educational institutions and in public
employment on the mere fact that one of their parents belonged to SCs /STs. It
was observed and held that a particular community, caste, race or tribe or part of
group thereof can be declared as SC or ST initially by the President of India and
new groups can be added to the lists only by a Parliamentary legislation The State
Government cannot, by an executive fiat, direct to treat any particular group of
persons as SC or ST. The caste of person depends on birth. The burden is on the
person who claims the benefit to establish that he/she is subjected to the same
hardship and disadvantages having been born as a member of SC/ST.
Right to Equality 87

ARTICLE 17 [ABOLITION OF UNTOUCHABILITY]

Article 17 “abolishes untouchability and forbids its practice in any form. The
enforcement of any disability arising out of untouchability shall be an offence
punishable in accordance with law.” The ‘law’ includes a law passed before the
coming into force of the Constitution.
Untouchability is neither defined in the Constitution nor in the Act. It refers to
a social practice which looks down upon certain depressed classes solely on
account of their birth and makes any discrimination against them on this ground.
Their physical touch was considered to pollute others. Such castes which were
called untouchables were not to draw water from the same wells, or use the pond/
tank which is being used by the higher castes. They were not allowed to enter
some temples and suffered many otner disabilities.
Inclusion of this provision in the Constitution shows the importance attached
by the Constituent Assembly towards eradication of this evil. Art. 15(2) of the
Constitution also help in the eradication of untouchability by prohibiting access to
shops, etc. on grounds only of caste. Art. 17 is also a significant provision from the
point of view of equality before law (Art. 14). It guarantees social justice and dignity
of man, the twin privileges which were denied to a vast section of the Indian society
for centuries together.
This right is directed against private persons. The nature of untouchability is
such that it is not possible to conceive where the State may practice untouchability.
In People's Union for Democratic Rights v UOI (Asiad Project Workers’ case AIR
1982 SC 1473), the Supreme Court held that whenever a fundamental right
contained in Arts. 17, 23 or 24 was being violated by a private individual, it would
be the constitutional obligation of the State to take necessary steps to interdict
such violation and ensure that such person should respect the right. Merely
because the aggrieved person could himself protect or enforce his invaded
fundamental rights, did not absolve the State from its constitutional obligations.
Article 35 read with Art. 17 confer on the Parliament power to make laws
prescribing punishment for practicing untouchability. The Parliament enacted the
Untouchability (Offences) Act, 1955. In 1976, it was made more stringent and was
renamed' The Protection of Civil Rights Act, 1955. It defines ‘Civil Right’ as ‘any
right accruing to a person by reason of the abolition of untouchability by Art. 17 of
the Constitution.’All offences under the Act have been made non-compoundable.
The Act prescribes punishment (1-2 years imprisonment) for preventing any person
from entering any place of public worship or from worshipping or denying access to
any shop, public restaurants, hotels or places of public entertainment or refusing to
admit persons to hospitals and refusing to sell goods or render services to any
person. Also, insulting a member of Scheduled Caste on the ground of
untouchability or preaching untouchability or justifying it on historical, philosophical,
religious or other grounds is a crime.
Under the Act, it has been declared a duty of the public servants to
investigate offences relating to untouchability. If a public servant wilfully neglects
the investigation of any offence punishable under this Act, he shall be deemed to
XVI
Constitutional Law o f India- II

be an abettor The State Governments have been empowered to impose


collective fines' on the inhabitants of an area involved in or abetting the
commission of offences under the Act.
To prevent the commission of offences or atrocities against the members of
the Scheduled Castes and Scheduled Tribes, the Parliament also enacted the
‘Scheduled Castes and Scheduled Tribes (Prevention of Atrocities) Act, 1989.'
The Act provides for special courts for the trial of offences under the Act and for
the relief and rehabilitation of the victims of such offences The Act excludes the
application of Sec. 438, CrPC (granting of anticipatory bail in case of anticipated
accusation of non-bailable offences) to cases arising under the Act. Atrocities
committed against a Hindu SC or ST, who had converted to another religion, can
be prosecuted under the Act, if the victim is still suffering from social disability.
In State of Karnataka v Appa Balu Ingale (AIR 1993 SC 1126), the
Supreme Court expressing its concern on the continuance of the practice of
untouchability, held that it was an indirect form of slavery and only extension of
caste system. Caste system and untouchability has stood together and would fall
together. It would be imperative to broach the problem not merely from the
perspectives of criminal jurisprudence, but more also from sociological and
constitutional angulations.
Article 17, more or less, has succeeded in its objective - to liberate the
society from a traditional practice which treated certain sections of the Hindu
society as sub-humans. It leads to an egalitarian society.

ARTICLE 18 [ABOLITION OF TITLES]

A democracy should not create titles and titular glories. This will go against the
realisation of social equality. England is a monarchy where one finds Marquis,
Barons, Lords and Knights, etc. Prior to Independence, India also had His
Hignness, Raja, Dewan Bahadur and similar titles some of which were hereditary
Art 18(1) abolishes all titles. It prohibits the State to confer titles on any
body whether a citizen or a non-citizen. Military and academic distinctions are,
however,' exempted from the prohibition. Thus, a university can give title or
honour on a man of merit. Clause (2) prohibits a citizen of India from accepting
any title from any foreign State Clause (3) prohibits a person not being a citizen of
India, but holding any office of profit or trust under the State, from accepting any
title from any foreign State without the consent of the President. Clause (4) further
prohibits such a person from accepting present, emolument or office of any kind
from or under any foreign State without the consent of the President Clauses (3)
and (4) have been
added to ensure that a non-citizen should remain loyal to the State i.e. do not
commit the breach of trust reposed in him.
It may be noted that Art. 18 do not secure any fundamental right but imposes
a restriction on executive and legislative power. Further, conferring of titles
offended against the fundamental principle of equality of all citizens guaranteed by
Art. 14
Right to Equality 89

Art. 18 is merely directory as there is no penalty prescribed for infringement


of the prohibition. However, the Parliament would have constitutional power to
make a law prescribing what should be done if there is violation of the provisions
contained in Art. 18 by any individual Dr. Ambedkar has opined that one of the
penalties might be loss of the right of citizenship by that individual
A title' is something that hangs to one’s name, as an appendage (either
prefix or suffix e.g Sir, Nawab, Maharaja, etc.). It means a title of honour, rank,
function or office in which there is a distinctive appellation. It is these appellations
that appear as prefixes or suffixes which are sought to be interdicted by Art. 18(1).
The recent conferment of titles of “Bharat Ratna”, "Padma Vibhushan", “Padma
Shri", etc. (introduced in 1954) are said to be not prohibited under Art 18 as they
merely denote State recognition of good work by citizens in the various fields of
activity. The opposite view is that these awards though may not be used as titles,
the decorations’ tend to make distinctions according to rank, contrary to the
Preamble which promises equality of status. Morarji Desai during his prime
ministership (1977- 79) discontinued these decorations
In Dr Dasarathi v State of A P. (AIR 1985 A P.), the Andhra Pradesh High
Court opined that constitutional government consistent with its obligations under
Arts. 14 and 18 could not confer the title of Poet Laureateship’ on any one. It was
heid that the institution of Poet Laureateship could not be regarded as consistent
with the nature of poetry or with the democratic polity. Conferment of Poet
Laureateship, which more or less, looked like conferment of a title, the court held,
might be a constitutional anathema.
In Balaji Raghavan v UOI (AIR 1995 SC 770), the Supreme Court upheld the
validity of civilian honours but criticised the government for not exercising restraint
in awarding these. A 5-judge bench, headed by Chief Justice Ahmadi, held that the
national awards were not meant to be used as titles and those who have done so
should forfeit the award. In this case, the petitioners challenged the conferment of
the awards on the ground that it was violat: 2 of Art. 18(1). They were of the view
that the word 'title' should be given the widest possible meaning and amplitude in
order to give effect to the legislative intent since the oniy exception to this rule has
been carved out in respect of military and academic distinctions. The contention of
the Union government was that since the national awards are not titles of nobility
a
nd are not to be used as suffixes or prefixes, they are not prohibited by Art. 13.
further, almost every country in the world follows the practice of conferring awards
f
or meritorious services rendered by its citizens.
The court observed that during British rule, power to confer titles was being
a
bused for imperialistic purposes and for corrupting public life. In recent years, the
Constitutional Law o f India- II

conferment of these awards without any firm guidelines and fool-proof


method of selection is bound to breed nepotism, favouritism, patronage and even
corruption Thus, an extremely high standard should be prescribed for those awards
and the total number of such awards should not exceed 50 every year "The
exercise of such restraint is absolutely necessary to safeguard the importance of
these awards” said Justice Ahmadi. To curb the misuse of this power, the court
suggested that a high level committee may be appointed by the Prime Minister in
consultation with the President who will ensure that only deserving persons
bestowed the decorations. Justice Kuldip Singh in his separate but concurring
judgment suggested that the committee may include the Speaker of Lok Sabha, the
Chief Justice of India and the leader of the Opposition in the Lok Sabha.
Subsequent to this decision, the Union Government appointed a ‘High Level
Review Committee,’ chaired by the Vice-President, to go into the existing
guidelines and fix the criteria for the selection of persons for 'Padma awards' so
that it would enhance respect for these awards. This Committee suggested the
setting up of State-level Committees to forward recommendations to the Centre.
The recommended names are to be reviewed by the Centre, by a Committee
including the Cabinet Secretary, the Home Secretary and Secretary to the
President of India. Thereafter, the finalized names are to be submitted to the Prime
Minister’s office and finally sent to the President. There are no guidelines for
'Bharat Ratna' awards.

QUESTIONS AND ANSWERS

Q. 1 The Preamble to the W.B. Special Criminal Courts Act states that with
a view “to provide for the speedier trial of certain offences, the State
Government may set up special criminal courts for the ‘speedier
trial’ of offences.” Sec. 5(1) of the Act provides: “A special criminal
court shall try such cases as the State Government may direct.”

The State Government directed the trial of two dacoits, A and B,


before the special criminal court. They challenge the validity of the
Act - which prescribed a speedier procedure which was materially
different from that laid down in Cr. P.C. - cn the ground of violation
of Art. 14. Alternatively they challenge the validity of the State
Government's direction for their trial before the special court as
being ultra vires the Act as well as violative of Art 14.

How will you decide? Will your answer be different if the Preamble
to the Act reads - "...with a view to maintaining peace, public order
and tranquility in the State, the State Government may set up...”
And, if the Sec 5 of the Act reads - “...shall try such offences or
classes of offences or cases or classes of cases ...?”
[C.L.C.-94; L.C.I.-94J

In order to curb the offences such as nose-cutting, leg-cutting, a


State enacts a legislation “to provide for speedier trial of the
offences" described above by curtailing some of the rights of the
accused
Right to Equality 91

permissible under ordinary law. Under the legislation so enacted,


special courts were set up. An accused prosecuted under the newly
enacted legislation challenges the validity of the legislation and his
prosecution. Decide under Art. 14 of the Constitution. [D.U.-2008]
^ 1 Art. 14 reads 'The State shall not deny to any person equality before the law
or the equal protection of the laws within the territory of India."
Equality before law implies the absence of any special privilege in favour of
individual3 and the equal subject of all classes to the ordinary law. Equal protection
of law implies equality of treatment in equal circumstances.
Art. 14 does not mean that all laws must be general in character or that the
same laws should apply to all persons, for all persons are not, by nature,
attainment or circumstances, in the same positions. While Art. 14 forbids class
legislation (improper discrimination by conferring particular privileges upon a class of
persons arbitrarily selected), it permits reasonable classification of persons,
objects, etc. by the legislature for achievement of specific ends.
Test of reasonable classification - (a) It must be founded on an intelligible
differentia which distinguishes persons or things that are grouped together from
others left out of the group, and, (b) the differentia must have a rational relation to
the object sought to be achieved by the Act.
In State of W.B. v Anwar Ali, the court observed that the differentia which is
the basis of classification and the object of the Act are two different things. Thus
the object by itself cannot be the basis of classification of offences or cases, for in
the absence of any special circumstances which may distinguish one offence or
case from another offence or case, the speedier trial would be desirable in disposal
of all offences or cases.
The W.B. Special Courts Act did not lays down any basis for classification
nor did it mention clearly what kind of cases were to be directed for special trial.
The object as stated in the Preamble, “speedier trial of certain offences" was too
vague, uncertain and illusive to afford a basis for classification. Sec. 5(1) of the Act
which authorises the State Government to direct “cases” to be tried by special
court, contravened the Art. 14 as it conferred arbitrary powers on the government
to classify offences or cases at its pleasure.
Procedural inequality - Art. 14 applies to both, the substantive as well as
procedural law, as discrimination can occur in both. In the present case, as the
procedure laid down for trial in the Act is materially different from that laid down in
Cr.P.C., it is violative of Art. 14.
$scond/third part of the question - The object as stated In the Preamble, “to
maintain Peace, public order and tranquility in the State," gives proper guidelines
for exercise of discretion by the executive to refer cases to special courts, thus not
violative of Art 14.
Further, Sec. 5 of the Act which authorised Government to direct “offences 0r
classes of offences, or cases or classes of cases gives an idea of classification
(Kathi Ranning v State of Saurashtra). Similarly, the last (third) part of the question
ls
not violative of Art. 14.
XVI
Constitutional Law o f India- II

Q. 2 The State Z passed an Act authorising the government to set up


‘Special Criminal Courts’ in order to check the frequent incidents of
offences of kidnapping in the dacoit infested area The procedure, to
be followed by these courts, is harsher and more onerous for the
accused than the procedure prescribed under the Cr PC.

Y, a dacoit, who was tried and convicted by the 'Special Criminal


Court’ for the offence of kidnapping, challenges the validity of the Act
under Art. 14. Will he succeed? [L.C.I-91/95]
A. 2 The Act gives proper guidelines for exercise of discretion by the executive
to refer cases to special courts
The procedure to be followed by these courts could be speedier
(summary) and thus harsher for the accused but the procedure must not
be materially different from that laid down in Cr PC Y will not succeed.
Q. 3 The Bihar Government Premises (Eviction) Act, 1955 (Special Act)
provides a summary/speedier procedure for eviction of a person from
unauthorised occupation of any government premises. The
procedure under the Special Act is harsher and more onerous than
the ordinary civil procedure for the same purpose. Ihe Act empowers
the Secretary, Housing and Development, to set up Special Eviction
Courts and vests him with power to refer to such courts such cases
for eviction as the Secretary may in his discretion direct.

X, a former minister, who continues to occupy the government


bungalow, challenges the validity of the Act on the ground that it
confers unfettered discretion on the government to choose either
procedure and therefore violates Art. 14. Decide, citing the relevant
case law.

Will your answer be different if the Special Act makes prevision for
one appeal to the High Court against the decision of the Government
under the said Act? [C.L.C.-92/95; L.C.I.-93/94;L.C. 11-95]
A. 3 Case-Law - in N.I. Caterers case, the Supreme Court held (on the similar
facts as in the present case) that the Act is discriminatory as two tenants
of the Government could be exposed to a differential treatment and there
were nc intelligible guidelines for exercise of Government’s discretion.
But, in M. Chagganlal v Greater Bombay Municipality, the Supreme Court
took a different approach. It laid down the following guidelines -
(i) Where a statute providing for a more drastic procedure different from
the ordinary procedure covers the whole field covered by the
ordinary procedure, without any guidelines as to the class of cases in
which either procedure is to be resorted to, the statute will be hit by
Art. 14 (Anwar AH case).
(ii) However, if from the preamble and surrounding circumstances, as
well as the provisions of statute themselves explained and amplified
by affidavits, necessary guidelines could be inferred, the statute will
not be hit by Art 14 (Saurashtra case).
(iii) If the impugned legislation indicates the policy which inspired it and
)
Right to Equality 93

the object which it seeks to attain, the mere fact that the legislation
does not itself make a complete and precise classification but left it
to the discretion of executive authority... is not a sufficient ground for
labelling it as arbitrary (Saurashtra case).
In the present case, the statute lays down the purpose behind it i.e."government
premises' should be subjected to speedy procedure in evicting unauthorised
occupants. This is a sufficient guidance for authorities to take action under special
procedure and not resort to the ordinary procedure which is time-consuming. Thus,
X cannot succeed
Second part of the question - The provision for one appeal to the High Court in the
Special Act wou!d cure the defect in the Act when it lacks any guidelines to be
followed by the authorities.
However, in the present case, as the Act already lays down sufficient
guidance for the authorities, the provision for appeal would not make any
difference in the answer.

Q. 4 (a) In State of Kerala v N.M. Thomas, the Supreme Court (majority)


observed: “The rule of equality within Art. 16(1) will not be violated
by a measure which ensures equality of representation in the
services for the unrepresented classes after satisfying the basic
needs of efficiency of administration.”

Comment on the above statement. Do you agree with the view that
Art. 16(4) is not an exception to Art. 16(1). Can the members of tha
SC, STs and other backward classes be given concessions, test
exemptions and other employment preferences (not involving
reservations), under Art. 16(1). Give reasoned arguments.
(L C. 11-94/95]

(b) A State Subordinate Service Rule provides for passing of a


departmental test for promotion of clerks from the L.D.C. to the
U.D.C. within 2 years but grants temporary exemption for another 2
years to L.D.Cs. belonging to SC and STs. Examine the
constitutional validity of the Rule. [L.C.1-93/94/2007]
A.4 (a) Art. 16(1) reads: “There shall be equality of opportunity for all citizens in
matters relating to employment or appointment to any office under the
State."
Art. 16(4) empowers the State to make special provision for the reservation
of appointments or posts in favour of any backward classes of citizens, which in the
opinion of the State are not adequately represented in the services under the
State.
In State of Kerala v N.M Thomas, the majority stated that Arts. 14, 15 and
16 form part of a string of constitutionally guaranteed rights supplementing each
other. Art. 16 was explained as an incidence of guarantee of equality contained in
Art. 14 and, therefore, permitted reasonable classification of employees in matters
relating to employment or appointment.
Art. 16(4) indicated one of the methods of achieving equality embodied in
Art. 16(1), and Art. 16(4) do not exhausts the equality of opportunity which can be
94 Constitutional Law o f India- II

made available to the backward classes. The guarantee of equality before law is
something more than is required by formal equality, and Art. 16(1) means effective
material or real equality Thus, Art. 16(4) is not to be read by way of any exception
to Art. 16(1).
Equality of opportunity for unequals can only mean aggravation of inequality.
Equality of opportunity admits discrimination with reason, which means rational
classification for differential treatment having nexus to the constitutionally
permissible object. Preferential treatment for the backward classes in service with
due regard to administrative efficiency is a permissible object. The quality and
concept of equality is that if persons are dissimilarly placed, they cannot be made
equal by having the same treatment.
(b) The Rule is constitutionally valid. The classification of employees belonging
to SC and STs was a just and reasonable classification 'having rational
nexus to the object of providing equal opportunities for all citizens in
matters relating to employment and appointment’. The temporary
relaxation of rules in favour of them was warranted in the services in view
of their overall backwardness. The above Rule does not impair the test of
efficiency in administration in as much as they have to pass the test
ultimately (the exemption granted to them only for a limited period) (See
N.M. Thomas Case).
Art. 16(4) must be interpreted in the light of Art. 335 which says that claims
of SC and ST shall be taken into consideration constituency with the maintenance
of efficiency of administration. However, the 82nd Amendment Act, 2000 restored
the relaxation in qualifying marks and standards of evaluation in both, job
reservation and promotions to SC/STs by adding a proviso to this effect in Art. 335.
This Amendment makes provision for no interference to be made in making of any
provision in favour of the numbers of SCs and STs for relaxation in qualifying
marks in any examination for lowering the standards of evaluation for reservation in
matters of promotion to any class or classes or services of posts in connection with
the affairs of the Union or of a State.

Q. 5 'The reservation policy adopted in India in the last tour decades has
failed to promote social justice. On the contrary it has led to social
friction, conflict and disharmony. It has been called a case of right
goal, wrong strategy.’ Critically examine this statement and suggest
measures to protect the interests of socially, educationally and
economically backward classes of people. [I.A S.-92]
Right to Equality

^ 5 As rightly held in PGI v Faculty Asscn. (AIR 1998 SC 1767) by a five judge
bench of the Supreme Court: “In making reservation for backward
classes, the State cannot ignore the fundamental rights of the rest of the
citizens”. "The doctrine of equality of opportunity under Art. 16(1) is to be
reconciled with concessions in favour of backward classes under Art.
16(4) in such a manner that the latter while serving the cause of
backward classes, shall not unreasonably encroach upon the field of
equality."

Causes
(i) Politicisation of reservation issue - reservations was made more for
political benefits (‘vote bank’) than for upliftment of backward classes.
(ii) Casteism - determination of backwardness on the basis of caste has
divided the society and increased the conflicts between forward and
backward classes. Lack of opportunities in the country and stiff
competition has greatly aggravated the problem.
(iii) Backwardness has become a vested interest - once a caste is considered
backward class then it continues to fcf backward for all the time. The elite
classes (creamy layer) among the, ' ,kward classes continue to grab the
benefits of reservation, leaving ii te '-est among backward classes
struggling.

Measures
(i) Adoption of economic criteria for determining backwardness.
(ii) Government should review its reservation policy from time to time to check
that reservation of seats does not become a vested interest.
(iii) A limit on the percentage of reservation.
(iv) Head-start programme - The government must provide basic facilities
(food, health, education etc.) to backward classes so as to improve their
position right from the childhood, and thus to make them compete of their
own in the society rather than relying on the clutches of reservation.
Q. 6 (a) ‘Arbitrariness is the very antithesis of equality'. Discuss.
[I.A.S.-92]

(b) Examine the Constitutional validity of the following under Art. 14:

(i) A rule framed by Bar Council of India debarring persons aged above
45 years of age from enrolment as an advocate.
(ii) Allocation of one-third of total marks for interview in admission to a
regional engineering college.
A- 6 (a) Art. 14 have an activist magnitude and it embodies a guarantee against
arbitrariness. Please refer to the text (New Concept of Equality).
(b) (i) The rule is discriminatory, arbitrary and unreasonable. In Indian
Council of Legal Aid and Advice v Bar Council of India (Al R 1995 SC
691), it was contended by Bar Council that rationale for the rule is to
maintain the dignity and purity of profession by keeping out those who
retire from various government, quasi-government and other institutions
Constitutional Law o f India- II

since they on being enrolled as advocates use their past contacts to


canvass for cases and thereby bring the profession into disrepute and
also pollute the minds of young advocates. Held that the rule is
discriminatory as it ooes not debar only such persons from entry into
profession but those who have completed 45 years of age on the date of
seeking enrolment The rule is unreasonable and discriminatory as there
may be a large number of persons who initially entered the profession but
later took up government jobs or entered any other gainful occupation,
who revert to practice at a later date even after they’ve crossed the age of
45 years.
(ii) Allocation of one third of total marks for interview is arbitrary and violative
of Art. 14 The oral interview can’t be regarded as a very satisfactory test
for evaluating the caliber, as it is subjective and capable of abuse It can't
be the exclusive test and should be resorted to only as an additional/
supplementary test (Ajay Hasia case). The court observed that allocation
of more than 15% marks to interview will be arbitrary and unreasonable
In. Art/ Sapru v J. & K. (AIR 1981 SC 1009), held that the allocation of 30%
marks for the viva-vo'ce for admission to the medical college was excessive But in
Lila Dharv Rajasthan (AIR 1981 SC 1777), where 25% of marks were allotted for
interview for the selection of munsifs in Rajasthan Judicial Service it was held that
the selection were not illegal
However, in D. V. Bakshi v UOI, the Supreme Court upheld the validity of the
rule allotting 100 marks with 50 pass marks for oral test. The test which may be
valid for competitive exams or admissions to educational institutions may not hold
good where it concerns selection for appointments in public services. In the case of
educational institutions, the candidates are young and their personality has yet to
develop, so greater weight has to be given to their performance at the written
examinations rather than at the oral examinations The above rule cannot be
applied to selection for public services where the duties, responsibilities, and
functions are very special demanding a high degree of probity and integrity, and
intellectual skills, adaptability, judgment and capacity to take prompt decisions In
case of selection of professionals higher marks for oral test can be allotted.
Right to Freedom
(ARTICLE 19)

Personal liberty is the most important of all fundamental rights Arts. 19-22 deal
with different aspects of this basic right.
Article 19(1) - Article 19 guarantees to the citizens of India the following six
fundamental freedoms:
(a) Freedom of Speech and Expression
(b) Freedom of Assembly
98 Constitutional Law o f India- II

(c) Freedom to form Associations


(d) Freedom of movement
(e) Freedom to reside and to settle
(g) Freedom of profession, occupation, trade or business.
[Note: Clause (f) - Right to acquire, hold and dispose of property- was abolished
by the 44th Amendment Act, 1978 with effect from 19-6-1979. In the result the
citizen has no fundamental right to acquire, hold and dispose of property. By the
same amendment, Art. 300A has been inserted which safeguards that a person
can be deprived of his property only by a law made by the legislature and not by
an executive fiat.]

Need for Restrictions on Freedoms


In U.S. the Constitution guarantees the rights in unqualified terms in the first ten
amendments But the necessity to put restraint on them has compelled the U.S.
Supreme Court to invent the doctrine of ‘police power.' It has been described as
the governmental power of self-protection and permits reasonable regulation of
rights and property in particulars essential to the preservation of the community
from injury [Penhandle v State Highway Commission (1935) 19 L.Ed. 1090], Thus,
under Police powers the State can restrict the use of loudspeakers, speed on
highways, sale of polluted food and compel people to take fire prevention
measures, etc.
Our Constitution recognizes that liberty has to be limited in order to be
effectively exercised. Dr. Ambedkar in his speech in the Constituent Assembly
emphasized the need for the restrictions when he said:"... (I)nstead of formulating
fundamental rights in absolute terms and depending upon our Supreme Court to
come to the rescue of Parliament by inventing the doctrine of police power, it (the
Constitution) permits the State directly to impose limitations upon the fundamental
rights.
Article 29 of the Universal Declaration of Human Rights balances rights with
duties to the community and permits restriction of rights for the purpose of
securing due recognition and respect for the rights and freedoms of others and of
meeting the just requirements of morality, public order and general welfare in a
democratic society.1
In A.K. Gopalan v State of Madras (AIR 1951 SC 21), the court observed:
“Man as a rational being desires to do many things, but in a civil society his
desires have to be controlled, regulated and reconciled with the exercise of similar
desires by other individuals.”
The freedoms enumerated in Art. 19(1) are those great and basic rights
which are recognised as the natural rights inherent in the status of a citizen. But
none of these freedoms is absolute or uncontrolled, for each is liable to be
curtailed by laws made or to be made by the State to the extent mentioned in

1. B.K. Sharma, p. 85.


99 Constitutional Law o f India- II
clauses (2) to (6) of Art. 19. A restriction to be constitutionally valid, must satisfy
the following two tests -
(i) the restriction must be for the purposes mentioned in clauses (2) to (6)
of Art. 19.
(ii) the restriction must be reasonable.
Limitations/Restrictions are exhaustive - The limitations enumerated in Art 19(2)
to (6) are intended to be exhaustive. They are the only grounds available to the
State to control a freedom given in the Article. A restriction on Freedom of Press
can be held valid only on the grounds contained in Cl. (2) and not under any other
clause. The courts always construe the restrictions strictly They lean in favour of
the rights and against the controls. Because the limitations are considered
exhaustive the State had to amend Arts. 19(2) and (6) to increase and modify
them to permit it to impose restrictions on new grounds.

Tests of ‘Reasonable Restriction’


The Supreme Court has laid down the following guidelines for determining the
reasonableness of restrictions:
(i) The term “reasonable restriction” connotes that the limitation imposed
on a person in the enjoyment of his right should not be arbitrary or of an
excessive nature beyond what is actually required in the interest of
public.
The word “reasonable" implies intelligent care and deliberation, i.e. choice of
a course which reason dictates.
(ii) Reasonableness demands proper balancing - The expression “reasonable
restriction" seeks to strike a balance between the individual rights
guaranteed by Art. 19 and social control permitted by Art. 19(2)-(6).
(iii) The restrictions on the rights can only be imposed by a “law” and not
executive or departmental instructions.
(iv) It is the courts and not the legislature which has to judge finally the
reasonableness of restriction. The court is called upon to ascertain the
reasonableness of restriction and not of the law which permits the restriction.
A law may be reasonable, but restriction imposed by it on the exercise of
freedom may not be reasonable. If the court finds a restriction to be
unreasonable, it may declare the law or order containing it to be
unconstitutional and void.
(v) Reasonableness: both substantive and procedural - The court would see
both to the nature of restriction and the procedure prescribed by statute for
enforcing restrictions. Prohibiting all civil servants from forming an
association would be a restriction that is unreasonable from substantive
stand-point. Allowing civil servants to form an association only, if permitted
by the government, is bad from procedural viewpoint. Procedural
reasonableness generally requires following rules of natural justice e.g.
opportunity to be heard, absence of bias, etc.

1. B.K. Sharma, p. 85.


Right to Freedom
(vi) Validity of total prohibition - Total prohibition in some cases would be prima
facie unconstitutional e.g. total prohibition upon the freedom of speech and
expression or forbidding all people from working as brokers in the sale and
purchase of immoveable property. In case of dangerous or noxious trades or
harmful goods or immoral or illegal occupations the restriction may extend to
complete prohibition. Examples are, trading in liquors, cultivation of narcotic
drugs, trafficking in women, gambling, and occupation of a tout. But where a
restriction becomes total prohibition the court has to exercise special care to
see that the test of reasonableness is justified
(vii) A restriction which is imposed for securing the objects laid down in Directive
Principles of State Policy may be regarded as reasonable. For example, a
law imposing restriction on the sale, possession and consumption of liquor is
valid in view of Art. 47.
(viii) Reasonableness and objective concept - The reasonableness has to be
determined in an objective manner and from the standpoint of the interests of
general public and not from the point of view of the persons upon whom the
restrictions are imposed. Thus, prohibition of advertisements by a person
claiming to cure a disease by magic is a ‘reasonable restriction.’ Similarly,
pre-censorship of films.
(ix) No exact standard or general pattern of reasonableness - Each case is to be
judged on its own merit. The standard varies with the nature of the
XVI
Constitutional Law o f India- II
right infringed, the underlying purpose of the restrictions imposed, the
extent and the urgency of the evil sought to be remedied, the prevailing
conditions at the time.
Rights available to ‘citizens’ only - The object of Art. 19 is to provide protection to
the citizens (Indian) from State action The action may be legislative or executive.
This article affords no protection from invasion of a right by a private person unless
the private action is supported by the State Citizens', under Art 19, mean only
natural persons and not legal persons, such as corporations or companies, deities
or municipal committees. But, the Bank Nationalization Case has established the
view that the fundamental rights of shareholders, editors, printers, as citizens are
not lost when they associate to form a company. The reason is that the
shareholders rights are equally and necessarily affected if the rights of the
company are affected. In Bennett Coleman v UOI (AIR 1973 SC 106), a'so, it was
held that a shareholder dees not lose his right by forming a company.

[ART. 19(l)(a)J [FREEDOM OF SPEECH AND EXPRESSION] 23

Article 19(1)(a) says that all citizens shall have the right to freedom of speech and
expression. But, this right is subject to limitations imposed under Art. 19(2) which
empowers the State to put 'reasonable' restrictions on the following grounds -
security of the State, friendly relations with foreign States, public order; decency
and morality, contempt of court, defamation, incitement to offence, and integrity
and sovereignty of India.24
The freedom of speech and expression means the right to express one's
convictions, and opinions freely by word of mouth, writing, printing, pictures,
electronic media or any other mode (addressed to the eyes or the ears). It also
includes the right to propagate or publish the views of other people, otherwise this
freedom would not include the freedom of press.
A democratic government attaches great importance to this freedom
because without freedom of speech the appeal to reason which is the basis of
democracy

2. What included within the right to freedom of speech and expression?


[/.AS.-*
“The fundamental right to freedom of speech and expression has in recent
times seen expanding horizons." Discuss and point out the judicial approach
w this regard. [I.A.S.-20051
24 Mention the grounds on which restrictions can be imposed on ‘freedom 0 speech
and expression’ as guaranteed under Art. 19(1)(a) of the Constitut'0^ of India.
lLCM-20071
can’t be made. Freedom R i gofh speech
t t o F and
r e e of
d othe
m press lay at the foundation
1 0 1 of
all democratic organisation, tor without free political discussion, no public
education, so essential for the proper tunctionmg of the process of popular
government, is possible (lustice Patanjali Shastri in Romesh Thapper v State of
Madras AIR 1950 SC 70).
When on the New Year day of 1941, President F D. Roosevelt made his historic
broadcast in the midst of the Second World War he listed out four objectives for
attainment by markind They were-Freedom of speech, Freedom to worship,
freedom from want and Freedom from fear. Freedom of speech is regarded as the
matrix, the indispensable condition, of nearly every other freedom.4
Freedom of expression has four broad special purposes to serve - (i) it helps
an individual to attain self-fuifilrnent; (ii) it assist in the discovery of truth; (iii) it
strengthens the capacity of an individual in participating in decision-making; and,
(iv) it provides a mechanism by which it would be possible to establish a reasonable
balance between stability and social change.
In sum, the fundamental principle involved here is the people’s right to know.
In National Anthem case (1936) 3 SCC 615, held that the freedom of speech and
expression under Art. 19(1)(a) also includes the freedom of silence. Again, in Noise
Foilution (V), In re (2005) 5 SCC 733, it was held that the right to speech implies
the 'right to siience’. It implies freedom, not to listen, and not to be forced to listen
The right comprehends the freedom to be free from what one desires to be free
from. Free speech is not to be treated as a promise to everyone with opinions and
beliefs, to gather at any place and at any time and express their views in any
manner. The right is subordinate to peace and order. A person can decline to read
a publication, or switch off a radio or television set But, he cannot prevent the
sound from a loudspeaker reaching him. He could be forced to hear what he wishes
not to hear. No one has a right to trespass on the mind or ear of another and
commit auricular or visual aggression.
Instances of reasonable restrictions are.
(i) restrictions on the use of loud speakers likely to cause nuisance and harm
public health.
(ii) law which prohibits obscenity.
(iii) causing contempt of court by interfering with the course of justice
(iv) instigating persons to commit serious offences like murder.
(v) censorship of films.
(vi) Application of labour laws and Factories Act to the Press.
Instances of unreasonable restrictions are:
(i) Punishing sedition or merely creating disaffection or bad feelings towards
♦he government.
(ii) Punishing criticism or vulgar abuse of a minister.
__ (iii) Prohibiting peaceful demonstrations.
(iv) Not permitting a prisoner to publish a book authored by him.
(v) To require a newspaper to reduce its space for advertisements.

4 B.K. Sharma, p. 87.


XVI
Constitutional Law o f India- II
(vi) To impose a tax on the press calculated to limit its circulation.
(vii) Pre-censorship of or any form of previous restraint on the newspap ers
(except in emergent circumstances to prevent a breach of peace for a
limited period).
(viii) Telephone tapping unless it falls within the grounds given in Art 19(2) 25
[Note: These instances of reasonable and unreasonable restrictions have been
discussed below with the help of case law ]

[a] Freedom of Speech and Expression and the Press/Print26


The “liberty of the press,” as defined by Lord Mansfield, consists in printing without
any licence subject to the consequences of law. Unlike the American Constitution,
Art. 19(1)(a) does not expressly mention the liberty of press i.e. the freedom to print
and to publish what one pleases without previous permission. But it is settled law
that the right to freedom of speech and expression includes the liberty of the press.
Press is supposed to guard public interest by bringing to fore the misdeeds, failings
and lapses of the government and other bodies exercising governing power.
Rightly, therefore, it has been described as the Fourth Estate
The Press has the same rights as those of an individual. It cannot claim better
rights. The freedom of press is not confined to newspapers and periodicals. It
includes also pamphlets, circulars, and every sort of information which affords a
vehicle of information and opinion [Lowell v Griffin (1938) 303 US 444],

Leading Case: BENNETT COLEMAN & CO. v UNION OF INDIA (AIR 1973
SC 106)
Facts and Issue - In this case, the validity of the Import policy for
Newsprint for 1972-73, along with the Newsprint Control Order, 1962, was
questioned, which imposed various restrictions, viz.
(i) bar on starting newspapers or editions by common
ownership units,
(ii) rigid limitation to 10 pages,
(iii) bar on interchangeability within common ownership unit, and
(iv) allowance of 20% page increase only to newspaper below 10 pages.
The Government defended the measure on the ground that it would help small
newspapers to grow and to prevent a monopolistic combination of big newspapers.
Decision - The Court held that the newsprint policy was not reasonable restriction
within the ambit of Art. 19(2). The newspapers operating above
10 pages level (and newspapers operating below 10 pages) have been treated
equally for assessing the needs and requirements of newspapers which are not

5. B.K. Sharma, p. 88.


26 What is the meaning of ‘Freedom of Press under the Indian Constitution?
[I. A. S.-2007]
“Freedom of speech and expression has been very broadly and courageously
interpreted by the Supreme Court." Discuss briefly with reference to judicial
precedents with regard to both print as well as electronic media [C L.C -?006]
[Note: Electronic media has been discussed later ]
Right to Freedom 103
their equals. Once the quota is fixed and directions to use the quota in accordance
with the newsprint policy made applicable, the big newspapers are prevented from
determining their pages, their circulation and their new editions. The individual
requirements of the different dailies render it eminently desirable in some cases to
increase the number of pages than circulation. Such adjustment is necessary to
maintain the quality and the range of readers in question. The denial of this
flexibility hampers- the quality, range and standard of the dailies and affect the
freedom of the press.
The test in determining the question whether a legislation or executive action
infringes the fundamental rights is to examine its 'effects’ and not its object or
subject-matter. It was contended on behalf of the Government that the subject-
matter of the newsprint policy was to regulate and control the newsprint and not to
control newspapers. The subject-matter of the import policy was rationing and
equitable distribution of newsprint. The Supreme Court rejected this contention and
approved the ”effect” test viz, if the direct effect of the law is to abridge a
fundamental right, its object or subject-matter will be irrelevant The direct effect, in
present case, was the newspaper control, as restrictions placed on their growth
and circulation.
In Sakai Papers/ Union of India, (AIR 1962 SC 305), the Daily Newspaper
(Price & Control) Order, 1960, which fixed a minimum price and number of pages
which a newspaper was entitled to publish, was held unconstitutional, as it infringed
the liberty of the press. An increase in price without any increase in the number of
pages would reduce circulation. On the other hand, any decrease in the number of
pages would reduce the column, space for news The order, therefore, acted as a
'double-edged knife’. The State justified the law as a reasonable restriction on the
business activity of a newspaper in the interest of general public. The court rejected
this contention, and said that freedom of speech can only be restricted on the
grounds mentioned in Art 19(2). It cannot, like the freedom to carry on business, be
curtailed in the interest
XVI
Constitutional Law o f India- II
of general public The freedom of speech could not be restricted for the
purpose of regulating the commercial aspects of activities of the
newspapers. Freedom of press is both quantitative and qualitative.
Freedom lies both in circulation and its content (news and views).
Dissenting opinion: Drawing inspiration from the American First
Amendment, Mathew J., was of the opinion that there might be an
abridgement of speech, but not an abridgement of the freedom of speech,
and observed that if the scheme of distribution is calculated to prevent
even an oligopoly ruling the market and thus check the tendency to
monopoly in the market, that will not be open to any objection on the
ground that the scheme involved a regulation of the press which would
amount to abridgement of the freedom of speech.27
However, the majority pointed out that the American First
Amendment contained no exception like Art. 19(1) and (2) of Indian
Constitution, and the American decisions establish that a governmental
regulation was justified in America as an important or essential
Government interest which v/as unrelated to the suppression of free
expression. The Supreme Court in India has established that the freedom
of press is to speak and express and it cannot be taken away in the
manner the impugned Import Policy had done.]
In Express Newspapers v Union of India (AIR 1958 SC 578) the validity of the
Working Journalists Act, 1955 was challenged The Act was enacted to regulate
conditions of services of persons employed in newspaper industry e g hours of
work, leave, fixation of wages, etc. It was contended that the Act would adversely
affect financial position of newspaper, and would curtail circulation, arid hence the
Act was violative of Art. 19(1 )(a). The Court held the Act valid It said that press
was not immune from laws of general application or ordinary forms of taxation, or
laws of industrial relations The Act was passed to improve the service condition of
workmen in the newspaper industry, and, therefore, impose reasonable restriction
on the right guaranteed by Art. 19( l)(a)28

27 “.... There may be an abridgement of speech, but not an abridgement of


freedom
of speech" (Bennett Coleman & Co v UOI AIR 1973 SC 106). Make a critical
appraisal of the above statement, giving clearly the scope of freedom of
speech and expression for the press under the Constitution of India. [LC.1-
2007]
28 In order to make the best use of the limited resources in the country and to
meet certain demands of the journalists and other workers in the newspaper
industry, the Government plans to enact a law which (i) fixes the minimum
wages for all the journalists and workers, (ii) declares Sunday and all other
national holidays as compulsory holidays for the newspaper industry. Discuss
the constitutional validity of such legislation vis-^-vis Art. 19(1) (a).
. [I.A. S. -2001/2007]
Right to Freedom 105
^eP the Freedom of Press is Violated - Test to be Applied
Sushii Choudhury v State of Tripura (AIR 1998 Gau 28), the State government's
'olicy was t0 allot 3°0/,° ®ovt Advertisements to a category of newspapers A Memorandum
dated 14-12-1993, however, allotted 24% of the ads to only one daily nd remaining
6% allotted to other dailies belonging to the same class. It was held t^at the
Memorandum is violative of freedom of speech guaranteed under Art. 19(1)( 3)'ancl is
also vio ative
' of equality clause. The policy contained in Memorandum affects formation
of healthy public opinion necessary for good democracy.
The High Court observed: “Freedom of speech and press are not so much
fQC the benefit of the press as for the benefit of general community and the
Government’s obligation to ensure circulation of the newspaper does not arise from
any duty of the Govt, to ensure that any particular press does not gain or lose, but
because the community has a right to be supplied with information and
Government owes a duty to educate the people within the limits of its resources.
Therefore, the test that has to be applied in determining whether the impugned
Memorandum is violative of Arts. 19(1 )(a) and 14 is not as to whether it equitably
takes care of the losses and gains suffered by different newspapers depending on
Govt, ads, but as to whether the community in general will suffer by the said
Memorandum”.
In Ajay Goswami v UOI (AIR 2007 SC 493), the freedom of press’ was in
issue. The court recognized the right of adults to entertainment within the
acceptable level of decency on the ground that it may not be appropriate for
children However, a blanket ban on publication of obscene materials to shield
juvenile innocence cannot be imposed No news item should be viewed In isolation.
“Fertile imagination” of minors snould not be a matter that should be agitated in
court of law The court considered the applicability of American test of "clear and
present danger". The court observed that nudity alone is not enough to make
material legally obscene’. The standard for judging cbscenity should be that of an
ordinary man of common sense and prudence and not that of a hypersensitive man
A publication, whether obscene, should be viewed as a whole. A culture of
'responsible reading’ should be inculcated among the readers of any news articles.3

Right of the Convict to Express Himself whether Covered by Art. 19(l)(a)


'n State v Charita (AIR 1999 SC 1379), the Supreme Court held that the press does
not have an ’unfettered’ right to interview an undertrial prisoner in a jail. The Court
while granting permission will have to weigh the competing interest between the
r
'9ht of the press and the right of the authorities prohibiting such interview in the
'nterest of administration of justice. The permission granted by the court would be
subject to the relevant rules/regulations contained in the Jail Manual
XVI
Constitutional Law o f Indi a- II
Earlier, in M. Hasan v Govt, of A.P. (AIR 1998 A.P. 35), the Andhra Pradesh
High Court held that refusal to journalist and videographer seeking interview wjjj.
condemned prisoners amounted to deprivation of citizens' fundamental right to
freedom of speech and expression under Art. 19(1)(a). A condemned prisoner, |||<e
a free citizen, had a right to give his ideas and was entitled to be interviewed o r
televised. The apprehension of the jail authorities that such reporting or videograpty
is not reasonable and in the interest of safety and security was unfounded.
In R. Rajagopat v State of T.N.'° (1994) 6 SCC 632, the editors of the Tamil
magazine “Nakkheeran” published from Madras moved the Supreme Court and
asked for a writ restraining government officials from interfering with their right to
publish the autobiography of Auto Shanker who had been convicted for several
murders and awarded death sentence. Auto Shanker had written his autobiography
in Jail which depicted close relationship between the prisoner and several IAS, IPS
and other officials, some of whom were partners in several crimes The
announcement by the Magazine that very soon a sensational life history of Auto
Shanker would be published created panic amo`ng several police officials that they
might be exposed. The IG wrote the publisher that the autobiography was false and
should not be published.
The Supreme Court held that the petitioners have a right to publish the
autobiography of Auto Shanker in so far as it is based upon public records, even
without his consent or authorisation. But if they go beyond that and publish his life
history they may be invading his right to privacy. Once a matter becomes a matter
of public record, the 'right to privacy' no longer subsists and it becomes a legitimate
subject for comment by Press and Media. It is enough for the press to prove that he
acted for a reasonable verification of the facts; it is not necessary for the press to
prove that what is published was true.
But in the interest of decency [Art. 19(2)], the name of a female who was
victim of sexual assault, kidnapping, etc. should not be published in the Press. The
right of privacy of public servants, however, stands on a different footing. In a free
democratic society public officials must always be open to criticism. The court held
that the State or its officials have no authority in law to impose prior restraint on
publication of defamatory matter. The public officials can take action only after the
publication if it is found to be false.

[b] Freedom of Speech and Expression and Electronic Media (Ads, Films,
etc.)
Advertisement is undoubtedly a form of speech. But every form of ad is not a form
of speech or expression of ideas e g social, political, literary, etc. An ad of a
'commercial' nature is not protected under Art. 19(1 )(a). Such ad has an element of
trade and commerce. Professionals like doctors, lawyers and C.A. are legally
forbidden to advertise their services.

[M.S.-90
Right to Freedom 107
£ 0fJ imercial Advertisements 29
Tata Press v M.T.N.L. (1995) 5 SCC 139, the Supreme Court declared that the
^flht to “Commercial Speech” or advertisement is part of the fundamental right to
freedom of speech and expression guaranteed by Art. 19(1 )(a). A private agency l^e
Tata Press is, therefore, entitled to bring out yellow pages comprising advertisements,
the annual buyer’s guide for Bombay. The larger implication of the eight to
advertisement being elevated to the status of a fundamental right is that from now
on it can be restricted only on the grounds specified in Art. 19(2). Mahanagar
Telephone Nigam Limited (MTNL) cannot come in the way of Tata Press Yellow
pages in "public interest’’ as no such ground is mentioned in Art. 19(2). Commercial
speech now enjoys as much protection as any other speech.
Kuldip Singh, J. however, made it cioar that the commercial advertisements
which are deceptive, unfair, misleading and untruthful could be regulated by the
government. The Court referred to the judgment in Hamdard Dawakhana v Union of
India (AIR 1960 SC 554) in which it was held that an obnoxious advertisement
(advertisements of prohibited drugs having magic qualities for curing diseases)
wouldn’t come within the scope of Art, 19(1 )(a).
The Court said advertisement as a “Commercial Speech” has two facts.
Advertising which is no more than a commercial transaction is nonetheless
dissemination of information regarding the product - advertised. Public at large is
benefited by the information made available through the advertisement. In a
democratic economy, free flow of commercial information is indispensable. There
can’t be honest and economical marketing by the public at large without being
educated by the information disseminated through ads. Examined from another
angle, the court said that the public at large has a ‘rignt to receive’ the “Commercial
Speech”. Art. 19(1 )(a) not only guaranteed freedom of speech and expression but
also protects the rights of an individual to iisten, read and receive the said speech.

Compelled Speech when Permissible


In UOI v Motion Pictures Asscn. (AIR 1999 SC 2334), the issue was whether the
compulsion for cinema owners to show scientific/educational films infringes their
freedom of speech guaranteed under Art. 19(1)(a). Held that the requirement under
the various cinematograph legislations that in each cinema hall the exhibitor of
films Must show a film which may be educational or scientific, a documentary film,
or a film carrying news or current events, along with the other films does not violate
Ar
t 19(1 )(a).
The Supreme Court observed: ‘Compelled speech’, often known as a ‘must
carry' provision in a Statute, Rule or Regulation, can be an infringement of the right
to
free speech, except to the extent permitted under Art. 19(2). The nature of a
‘must carry' provision is the determining factor. If such a provision furthers informed
decision making which is the essence of the fundamental freedom of speech and
expression, it will not amount to any violation of Art 19(1 )(a). If, however, such a
provision compels a person to carry out propaganda or project a partisan or

29 Are commercial advertisements covered by the protection of Art. 19(1) (a)?


Discuss referring to decided cases. [/.A. S.-99]
XVI
Constitutional Law o f Indi a- II
distorted point of view, contrary to his wish, it may amount to a restraint on his
freedom of speech and expression.
Thus, the obligation that any food product should carry on its package the
list of ingredients used in its preparation, or must print its weight, or the obligation
that the cigarettes cartons should carry a statutory warning that "cigarette smoking
is harmful to health," are meant to further the basic purpose of imparting relevant
information which will enable the user to make a correct decision. Such mandatory
provisions although they compel speech cannot be viewed as a restraint on the
freedom of speech and expression. The social context' of any such legislation
cannot be ignored. When a substantially significant population body is illiterate or
does not have easy access to ideas or information, it is important that all available
means of communication, particularly audio-visual, are utilized not just for
entertainment but also for education, information, etc

Pre-Censorship and Exhibition of Films


It is justified under Art. 19(2) on the ground that films have to be treated separately
from other forms of art and expression because a motion picture was able to stir up
emotions more deeply. Hence, classification of films into ‘A’ & ‘IT categories held to
be valid in K.A. Abbas v Union of India (AIR 1971 SC 481).
In L/C v Manubhai D. Shah (1992) 3 SCC 637, held that prior restraints are
threats to freedom of expression, because of their potential for imposing arbitrary
and irrational decisions. However, in case of films, censorship by prior restraint is
justified for the protection of society from the ill effects that the films may produce if
unrestricted exhibition is allowed. Censorship is thus permitted to protect social
interests enumerated in Art. 19(2), but such censorship must be reasonable and
must answer the test of Art. 14.
In S. Rangarajan v P. Jagjivan Ram (1989) 2 SCC 574, the Madras High
Court revoked the 'U' certificate issued to a film on the ground that exhibition of film
was likely to cause public disorder and violence On appeal by the proaucer of the
film and Union of India, the Supreme Court reversed. The Court not only reiterated
the importance of the freedom of speech and expression and the role of films a
legitimate media for its existence, but also held that if exhibition of films cannot be
validly restricted under Art. 19(2), it cannot be suppressed on account of the threat
of demonstration and processions or threat of violence. The court added: “It is the
duty of State to protect freedom of expression since it is a liberty guaranteed
against the State. The State cannot plead its inability to handle the hostile audience
problem. It is its obligatory duty to prevent it and protect the freedom of
expression”. This positive aspect of the right of freedom of speech and expression
is an important innovation by the Court in this case.
In Bobby Art International v Om Pal Singh Hoon (1996) 4 SCC 1 (“Bandit
Queen Case”), a writ petition was filed by the respondents in the court tor quashing
Right to Freedom 1 09
the certificate of exhibition given to the film 'Bandit Queen’ and restraining its
exhibition in India. The film is the story of Phoolan Devi exposed to from an early age
to the brutaiity and lust of man. The film was granted 'A certificate by the Censor
Board and the Appellate Tribunal. The High Court held that the film was obscene and
quashed the order of the Tribunal, thus restraining the exhibition of the film
altogether.
The contention of the respondents was that though the audiences were led to
beiieve that the film depicted the character of a former queen of ravines (Phoolan
Devi), the depiction was “abhorrent and unconscionable and a slur on the
womanhood of India”. The respondents and his community had been depicted in a
most depraved Way especially in the rape scene, which scene was suggestive of the
moral depravity of the Gujjar community. The Supreme Court held that the
certificate issued to the film upon conditions imposed by the Tribunal is valid and is,
therefore, restored. Held that the film must be judged in its entirety from the point of
view of its overall impact. The story of the film is a serious and sad stcry of a village
bom female child becoming a dreaded dacoit. The scene where she is humiliated
stripped and paraded naked does not arouse the cinemagoer’s lust but to arouse in
him sympathy for the victim and disgust for the perpetrators. In fact, that scene is
central to the story. A film that carries the message that the social evil is evil cannot
be impermissible on the ground that it depicts the social evil.

Right to Reply
In LIC v Manubhai D. Shah (1992) 3 SCC 637, the respondent, an executive
trustee of the Consumer and Research Centre, published a study paper entitled
“Fraud on the Policy Holders' - A shocking story, which depicted the discriminatory
practice adopted by the LIC (Life Insurance Corporation) which adversely affected
the interest of a large number of policy holders. A member of the LIC published a
counter in the magazine called ‘ Yogakshma' published by the LIC. The respondent
requested the LIC to publish his rejoinder to the said article in the said magazine,
but his request was not accepted. The LIC contended that the magazine is a house
magazine (meant only for informing its members, staff, etc.) and not put in the
market for sale to the general public. The respondent contended that refusal to
publish his rejoinder, violated his fundamental rights under Art. 14 and Art. 19(1 )(a).
The High Court directed LIC to publish the article in next issue, as the
magazine is available to any one on payment of subscnption and articles are
invited for magazine from public. The Supreme Court also held that the
respondents' fundamental right of speech and expression entitled him to insist that
his views on the subject should reach those who read the magazine. The Court
explaining the scope of the freedom said “the words freedom of speech and
expression must be broadly construed to include the freedom to circulate one’s
views by words of mouth, or in 'writing, or through audio visual media. These
communication channels are great purveyors of news and views and make
considerable impact on the minds of the readers and viewers and are known mould
public opinion on vital issues of national importance."
The freedom of speech and expression includes freedom of circulation and
Propagation of ideas and ‘therefore’ the right extends to the citizen to use the
XVI
Constitutional Law o f India- II
media
to answer the criticism levelled against his views propagated by him. A “right to
reply” (by a dissonant note) is implied in the system of Freedom of Expression
Every free citizen has undoubted right to lay what sentiments he pleases subject
to reasonable restrictions.

Right to Information

Leading Case: Secretary, Ministry of I & B v Cricket Association,

Bengal30 (AIR 1995 SC 1236)

This is a landmark judgment as it recognises the 'right to information' as


part of the fundamental right to speech and expression under Art. 19(1
)(a).
Facts and Issue - The Cricket Association of Bengal (CAB) and Board
of Cricket Control of India (BCCI) have sought a licence to telecast their
matches through a foreign agency and through telecasting equipment
brought in by such foreign agency. In the case of "Hero Cup” matches
organised by CAB, they wanted uplinking facility to INTELSAT through
the Government agency VSNL also. They wanted the permission to
import the telecasting equipment along with the personnel to operate it
by moving it to places all over the country wherever the matches to be
played. They claimed this licence (or permission) as a matter of right
said to be flowing from Art. 19(1)(a).
In late 1993, challenging a decision of CAB to award exclusive
rights to an international broadcasting company (TWI) to cover Hero
Cup tournament, the Ministry of I & B directed VSNL (the government’s
international telecom monopoly) to deny uplinking facilities. The CAB
petitioned the High Court, which permitted the telecast. In turn, the
ministry appealed against the decision in the Supreme Court
contending that it had monopoly over it under the Telegraph Act, 1885.
The word ‘telegraph’ includes telecast. The Supreme Court turned
aside the appeal and delivered a landmark judgment.
Decision - The ratio of Supreme Court's judgment is as follows:
(1) The court allowed the private broadcasters the right to
airwaves. The Court ruled that airwaves are public
property and as such they must be regulated by an
independent,

30 The expression 'freedom of speech and expression' in Art. 19(1)(a) has


been held to include the right to acquire information and disseminate the
same. Elucidate with the help of judicial precedents. [L.C.I-
2006\
Whether a legislation creating State monopoly in electronic media due to
availability of limited number of frequencies is constitutionally valid?
Discuss.
[C.L.C-2007]
Right to Freedom 111
autonomous public authority representative of all sections and
interests in society.
(2) This autonomous authority or body is to decide who really should
have the right to airwaves. It should rescue the electronic media
from government monopoly and bureaucratic control. The court
directed the government to set up such autonomous body
immediately.
(3) The court expanded the scope of the fundamental right to speech
and expression (Art. 19(1)(a)). The court observed that a citizen has
a fundamental right to use the best means of imparting and receiving
information and as such have access to telecasting for the purpose.
(In other words, the government has no exclusive right to use
broadcast media or deny the use by others).
(4) The court declared that freedom of speech applied not just to the
print media but also to the electronic media. Albeit with a caveat: the
airwaves are a public resource and must, therefore, be regulated in
the public interest.
(5) The court ruled that the freedom of speech and expression includes
the right to educate, inform and entertain. According to court, sport is
an expression of self. Therefore, the right to telecast sporting event
also includes the right to educate and inform present and
prospective sportsmen interested in a particular sport and also to
inform and entertain the lovers of the game. Hence, when a
telecaster desires to telecast a sporting event, it is incorrect to say
that the ‘free speech' element is absent from his right.
(6) Freedom of speech and expression includes the right to
communicate effectively. A citizen has under Art. 19(1 )(a) a right to
telecast/broadcast to the viewers/listeners through electronic media
(Radio and TV.) any important event. The Government can impose
restrictions on such a right only on grounds specified in Art. 19(2).
State monopoly on electronic media is not mentioned in Art. 19(2).
(7) The Telegraph Act of 1885 is wholly inadequate and unsuited to
radio and television i.e. broadcasting media. The Parliament should
soon enact law or laws to fill the void. (Reddy J.).
Comments - The Government of India has so far thwarted private, foreign and
Indian companies from broadcasting sports, news, etc. by claiming that the
Telegraph Act gives it exclusive powers to uplink - telecast direct - from Indian soil.
By this judgment, the apex court has paved the way for a revamp of the country’s
archaic broadcasting policy. This media analysts feel, is long overdue in view of the
fast - changing television
scenario in India, where the private sector is now coming into its own.
The judgment has allowed private broadcasters to buy rights from sporta
organisations, and to present live news and current affairs. The
autonomous body, being a regulating authority can effectively be used to
check anti-national propaganda through satellite networks
However, it remains to be seen how much feet-dragging the
government will do to comply with court's directive of enacting a liberal
XVI
Constitutional Law o f India- II
new broadcasting policy. The sensible thing for the government would be
to establish an independent authority consisting of media representatives
and public figures in order to fashion a broadcasting code covering issues
such as fairness, equal time for opposing points of view, obscenity,
advertising standards and so on. And just as important, in consultation
with the media groups, norms and exacting qualification standards should
be laid down for giving broadcasting rights. Further, the government \
should act with a sense of urgency, otherwise there will be another Prasar
Bharati]
Trial by Press/ Media Not Covered by Arts. 19(l)(a) and (2)
In State of Maharashtra v Rajendra J. Gandhi (1997) 8 SCC 386, relating to the
‘rape’ of a girl, unnecessary publicity and taking out of morcha by the public caused
great harm to the girl. Even the case had to be transferred from Kolhapur to Satara
under the Supreme Court's orders. While not denying the freedoms guaranteed by
Arts. 19(1)(a) and (2), the Supreme Court cautioned that there is procedure
established by law governing the conduct of trial of a person accused of an offence.
A trial by press, electronic media or public agitation Is the very antithesis of rule of
law. It can well lead to miscarriage of justice A judge has to guard himself against
any such pressure and is to be guided strictly by rules of law.
It is submitted that the trial by press or electronic media is unavoidable in the
present-day society clouded by various information channels In the name of
’freedom of speech and expression’, the press/electronic media often intrudes into
the private lives of people especially the big celebrities, and more often than not
causes undue harm viz. the trial by press/media of Princess Diana-Prince Charles
divorce case, and the trial by press/media of US President Bill Clinton- Monica
Lewinsky “sex scandal.”
^ The public sentiments/opinions created by such ‘trials’ may influence the
Court's judgment. Thus, the Supreme Court awarded Death sentence to a woman'
(Ramshree) for the first time, after independence, but suspended it, being possibly
influenced by press/media attention and agitation by the women’s groups.
Ultimately the Supreme Court (in its judgment dated 19th August, 1998) converted
the death sentence of Ramshree into the life-imprisonment.
Right to Freedom 113
[C] Freedom of Speech and Expression and Demonstration or ‘Picketing’
These are visible manifestation of one’s ideas and in effect a form of speech and
expression. They are protected under Art. 19(1)(a) provided they are not violent
and disorderly. Right to strike is not included within the ambit of freedom of speech.

[d] Freedom of Speech and Expression and Furling of National Flag


In Union of India v Navin Jindal (2004) 2 SCC 410, the Supreme Court made
certain important observations in respect to the flying of the National Flag:
(i) Right to fly the National Flag freely with respect and dignity is a
fundamental right of a citizen within the meaning of Art. 19(1)(a) being an
expression and manifestation of his allegiance and feelings and
sentiments of pride for the nation. However, such right is subject to
reasonable restrictions under Art. 19(2).
(ii) The Emblems and Names (Prevention of Improper Use) Act, 1950 and the
Prevention of Insults to National Honour Act, 1971 regulate the use of the
National Flag.
(iii) Flag Code although is not a law within the meaning of Art. 13(3) of the
Constitution of India, for the purpose of Art. 19(2) it would not restrictively
regulate the free exercise of the right of flying the National Flag. However,
the Flag Code to the extent it provides for preserving respect and dignity
of the National Flag, the same deserves to be followed.

[e] Freedom of Speech and Expression and Voters’ Right to Know

Leading Case: people’s union for CIVIL liberties V UOI (AIR 2003 3C
2363)
In this case, the Apex Court struck down Sec. 33-B of the Representation
of People (3rd Amendment) Act, 2002 which had the effect of rendering
certain directions issued by the court in UOI v Asscn. for Democratic
Reforms (AIR 2002 SC 2112) as not binding and therefore void.
Reiterating the law laid down in Asscn. for Democratic Reforms case,
the Supreme Court ruled that the right of a voter to know the biodata and
antecedents of a candidate is the foundation of democracy, a facet of the
right to freedom of speech and expression guaranteed under Art. 19(1)(a).
it would be the basis for free and fair election' which was a basic structure
of the Constitution. The "freedom of speech and expression’ includes right
to impart and receive information [Secretary,
Ministry of I & B, Govt, of India v Cricket Association, State of W B
(1995) 2 SCC 161]. Restriction to the said right could be only as
provided in Art. 19(2).
In Asscn. for Democratic Reforms case, this court observed The
people of the country have a right to know every public act, everything
that is done in a public way by the public functionaries. MPs or MLAs
are undoubtedly public functionaries. People have a right to know about
their educational qualifications, assets held by them, and. antecedents
XVI
Constitutional Law o f India- II
of their life including a past criminal record.
In this case, it was contended that as there is no specific
fundamental right conferred on a voter by any statutory provision to
know the antecedents of a candidate, the directions given by this Court
are against the statutory provisions. The court rejecting it, observed
that in an election petition challenging the validity of an election of a
particular candidate, the statutory provisions would govern respective
rights of the parties. However, voters’ fundamental right to know the
antecedents of a candidate is independent of statutory rights under the
election law. Members of a democratic society should be sufficiently
informed so that they may cast their votes intelligently in favour of
persons who are to govern them Right to vote would be meaningless
unless the citizens are well informed about the antecedents of a
candidate.
The court further observed: It is established that fundamental
rights themselves have no fixed content, most of them are empty
vessels into which each generation must pour its content in the light of
its experience.
The attempt of the court should be to expand the reach and ambit of
the fundamental rights by process of judicial interpretation. During the
last more than half a decade, it has been so done by this court
consistently. There cannot be any distinction between the fundamental
rights mentioned in Chapter III of the Constitution and the declaration of
such rights on the basis of the judgments rendered by this court ]

[f] Freedom of Speech and Expression and Contempt of Court


The right to freedom of speech and expression does not entitle a person to
commit ‘contempt of court' [In Re Arundhati Roy AIR 2002 SC 1375; Radha
Mohan Lai v Rajasthan High Court AIR 2003 SC 1467]. The law relating to the
Contempt of Courts imposes reasonable restrictions on the freedom and within
the ambit of Art. 19(2) [Narmada Bachao Andolan v UOI AIR 1999 SC 3345].
In Re Arundhati Roy, as a mark of protest against the Narmada Bachao
Andolan judgment, the respondents led a huge crowd and held a ‘dharna’ in front
of the Supreme Court shouting abusive slogans against the Court including
slogans ascribing lack of integrity and dishonesty to this institution. The Court
observed: The Constitution of India has guaranteed freedom of speech and
expression to every citizen as a fundamental right. While guaranteeing such
freedom, it has also

rs^
provided under Art. 129 that the Supreme Court shall be a court of record and shall have all the powers of
such a court including the power to punish for contempt of itself. Under the Constitution, there is no separate
guarantee of the freedom of the press and it is the same freedom of expression, which is conferred on all
citizens under Art 19(1). Any expression of opinion would, therefore, be not immune from the liability for
exceeding the limits, either under the law of defamation or contempt of court or the other constitutional
limitations under Art. 19(2). If a citizen, therefore, in the grab of exercising right of free expression under Art.
19(1), tries to scandalize the Court or undermines the dignity of the Court, then the Court would be entitled to
Right to Freedom 115

exercise power under Art. 129.

OTHER FUNDAMENTAL FREEDOMS

Fundamental freedoms enumerated under Art. 19 are not necessarily and in all circumstances mutually
supportive, although taken together they weave a fabric of a free and equai democratic society. Thus,
freedom of speech of one affects the freedom of movement of another. Exercising the right to form an
association may curtail the freedom to express views against its activities. Some restriction on one’s rights
may be necessary to protect another's right in a given situation. The rights must be harmoniously construed
so that they are properly promoted with the minimum of such implied and necessary restrictions [M.H.
Devendrappa v Karnataka State Small Industries Dev. Corpn. AIR 1998 SC 1064],

Art. 19 (l)(b): Freedom to Assemble


Art. 19(1)(b) secures to all citizens of India the right "to assemble peaoeably and without arms". Thus, there
is a right to hold public meetings and demonstrations and take out processions peacefully. Under Art 19(3),
reasonable restrictions can be imposed on the right by the State by law “in the interests of the sovereignty
and integrity of India or public order'. Thus, an assembly declared unlawful can be validly banned
Meetings are the most common form of coming together. In fact, the purpose of holding an assembly
is to educate the public and the formation of opinion on religious, political, economic or social problems.
That is why the 'right of assembly' guaranteed under Art. 19(1)(b) is a corollary of the ‘right to freedom of
speech and expression’ guaranteed under Art. 19(1 )(a). The very idea of a government, republican in form,
implies a right on the part of citizens to meet peaceably for consultation in respect of public affairs.
It does not imply that a citizen may hold meeting anywhere on anybody’s Property. The government
has a right to control the time and place of a meeting The right to take out procession on a public street is
subject to the order of the authority regulating traffic. This applies to all processions whether religious or
non- religious. The right is not affected by playing music in the procession. A procession
116 Constitutional Law o f India- II
Right to Freedom 117
is a mobile assembly. A meeting is stationary. Religious processions may be recognized by it and to withdraw from an association which was derecognized.
further protected by Art. 25. Anybody who resists the exercise of a fundamental The rule was struck down gS unreasonable restriction because it made the right
right is a wrongdoer. It is the duty of the State to suppress such wrongdoers. dependant on recognition of the association by the government. It rendered the
Art. 19(1 )(b) has been held to cover the right to hold “hunger strikes’’ so right illusory and ineffective (O.K. Ghosh v F.X. Joseph AIR 1963 SC 812).
long as it is assured to be peaceful without arms and not against any individual or In Dharam Dutt v UOI (AIR 2004 SC 1295), the Supreme Court held that a
group or community. right to form associations or unions does not include within its ken as a
This article does not afford protection to a violent or riotous assembly or to fundamental right a right to form associations or unions for achieving a particular
one which bears arms. Section 144 of the Code of Criminal Procedure, 1973 has object or running a particular institution, the same being a concomitant or
been held to be valid because it authorizes the making of temporary orders for the concomitant to a concomitant of a fundamental right, but not the fundamental right
prohibition of meetings or processions to prevent an imminent breach of peace itself. The associations or unions of citizens cannot further claim as a fundamental
(Madhu Limaye S.D.M. AIR 1971 SC 1762). A rule which prohibited a government right that they must also be able to achieve the purpose for which they have come
servant from taking part in any demonstration with regard to any matter pertaining into existence so that any interference with such achievement by law shall be
to his conditions of service was held to be violative of Art. 19(2) (Kameshwarsi unconstitutional, unless the same could be justified under Art. 19(4).
State of Bihar MR 1962 SC 1166).
Right to Strike31

Art. 19 (l)(c): Freedom of Association It has been held that this Article does not guarantee a 'right to strike’. Even a
Art. 19(1 )(c) guarantee to all citizens the right “to form associations and unions for liberal interpretation of Art. 19(1)(c) dees not lead to the conclusion that the trade
pursuing lawful purposes’. It includes the right to start an association and to unions have a right to strike (Dharam Dutt v UOI AIR 2004 SC 1295).
continue it. Under Art. 19(4), however, reasonable restrictions can be imposed by In T.K. Rangarajan v State of T.N. (AIR 2003 SC 3032), a two-Judge bench
the State “in the interests of the sovereignty and integrity of India or public order or of the Supreme Court held that Government servants have no legal or statutory
morality.” right to go on strike. In the year 2002, the Government of Tamil Nadu terminated
An “association” means a collection of persons who have joined together for the services of 2 lakh employees under the T.N Essential Services Maintenance
a certain object, which may be for the benefit of the members or the welfare or Act, 2002; the government employees had gone on strike for their demands.
advantage of the public. The associations so formed would include political The court said that “Government employees cannot hold society to ransom
parties, societies, clubs, companies, organizations, partnership firms, trade unions b
y going on strike”. If the employees felt aggrieved by any Government action,
and indeed any body of persons. Thus, like, the ‘freedom of assembly', the they should seek redressal from the statutory machinery provided under different
freedom of association' is essential for the proper functioning of parliamentary statutory Provisions. The court said strike as a weapon is mostly misused which
democracy or for the protection of other rights guaranteed by the Constitution. results in chaos and total mal-administration. Though the trade unions have a
An association that claims the benefit of the right of association must be guaranteed “right for collective bargaining’’ on behalf of employees, they have no
such whose object is lawful. Under the Criminal Law Amendment Act, 1908 (as fundamental right to strike under Art. 19(1)(c).
amended in Madras) it was left to the Government to determine in a subjective Recently, a Constitution Bench of the Supreme Court in Harish Uppal v UOI
manner whether an association constituted danger to public peace. The grounds (AIR 2003 SC 739), categorically proncunced that the lawyers had no right to go
of forming the opinion could not be tested in a judicial inquiry. The court held that °n strike or give a call for boycott, not even a token strike. It has been suggested
this provision imposed a restriction on the exercise of the right and declared it void h
at the Advocates can get redressal of their grievances by passing resolutions,
(State of Madras v V.G. Row MR 1952 SC 196). However, declaring an taking representations and taking out silent processions, holding dharnas or to
organization as a terrorist organization under the Prevention of Terrorism Act, re
sort to relay fast, having discussion by giving TV. interviews and press
2002, would be permissible (People’s Union for Civil Liberties v UOI AIR 2004 SC statements.
456).
A cooperative society may restrict its membership to a particular religion,
community, belief or trade etc. A citizen has no fundamental right to become its
member [Zoroastrian Cooperative Housing Society v District Registrar (2005) 5
SCC 632], An Act which alters the composition of a society introducing new
members was declared unconstitutional as violative of Art. 19(1)(c). The government
cannot compel a citizen to withdraw from the membership of a union.
The right to form an association cannot be construed to mean that it
extends to obtaining recognition of the association or continuation of recognition.
Recognition of association is not a fundamental right. A rule framed by the
government required Its servants not to join an association which was not
31 Describe the right to call ‘hartal’ (strike).
[L. C.l1-
116 Constitutional Law o f India- II 2007]
Right to Freedom 118
119 Constitutional Law o f India- II

Comments - "Right to strike’’ also affects other’s rights. In recent times, strikes by
teachers/lecturers are on the increase, thus affecting the students’ interest to a
large extent. When the 'right to education’ has been recognised as a fundamental
right, such strikes by teachers seem to be quite unjustified even if the teachers
have a fundamental 'right to strike’. A conflict between two fundamental rights
has to be resolved by seeing the exercise of which fundamental right is in
societal interest. Though the right to strike/right to protest cannot and should not
be eliminated altogether, it could be controlled or regulated, viz restrictions on the
number of days in a strike, or strikes in a phased manner, etc.

Art. 19 (l)(d) & (e): Freedom of Movement and Residence


The right of every citizen of India “to move freely throughout the territory of India"
and his right “to reside and settle in any part of the territory of India” guaranteed
under clauses (d) and (e) respectively of Art. 19(1) are really interlinked Both the
rights lay stress on the oneness of the territory of India. Any citizen can travel to
or reside in any part of India. This is in consonance with the ‘single citizenship’
provided by Art 5 of the Constitution.
Art. 19(5), however, provides for imposition of reasonable restrictions on
the exercise of these rights by law “in the interests of the general public or for the
protection of the interests of any Scheduled Tribe.” Generally, the protection
afforded by these rights is invoked to challenge the validity of extemment or
deportation orders which go to curtail the two freedoms.
The ‘freedom of movement' has three aspects- (a) The right to move inside
the country; (b) The right to move out of the country; (c) The right to come back
to the country. It is the first aspect which is covered by Art. 19(1)(d). The second
has been protected by Art. 21. In Kharak Singh v State of U P. (AIR 1963 SC
1295), it was held that the right to move freely throughout the territory of India
mean the right of locomotion’ which connotes the right to move wherever one
likes, whenever one likes, and however one likes.
An order by the District Magistrate or a Police Commissioner to a person to
remove himself from a district for a period of 3 months would be valid because
the duration of the order is reasonable. Since the Act- East Punjab Public Safety
Act, 1949, was to have a limited duration, there was no possibility of an order of
externment being made for an indefinite period (N.B. Khare v State of Delhi AIR
1950 SC 211). A watch by the police on the movement of a suspect does not
infringe the right (Malak v State of Punjab MR 1931 SC 760).
Restrictions imposed on prostitutes to carry on their trade within a
specified area and to reside in or move from particular areas have been held to
be valid (State of U P. v Kaushalya AIR 1964 SC 416). Likewise, restrictions on
residence imposed on habitual offenders have been upheld Restrictions on the
movement of persons afflicted by AIDS have also been upheld. Restrictions on
the movement in a tribal area are within the permissible limits. The requirement
that all persons who ride a two wheeler must wear a helmet is an instance of a
restriction in the interest of the general public [Ajay v Union of India (1988) 4 SCC
156].

( 1
XVI
Constitutional Law o f India- II

Article 19(1 )(e) guarantees theTight to reside and settle' in any part of India. To some extent
this right and the freedom of movement are overlapping The object of both is to remove barriers
within India This freedom can be subjected to the same restrictions as the 'freedom of movement’
because the grounds of restrictions are contained in Art. 19(5) for both the freedoms.
An Act which empowers the State Government to issue an order requiring a person to reside
and remain in the specified place or an order directing a person to leave a place and go to another
place imposes unreasonable restrictions (State of M.P. v Bharat Singh MR 1967 SC 1170).
In U.P. Avas Evam Vikas Parishad case (AIR 1996 SC 114), it was held that the right to
residence assured in Art. 19(i)(e) includes the 'right to shelter’. In Ahmedabad Municipal Corpn. v
Nawab Khan Gulab Khan (AIR 1997 SC 152), the Court observed [in the context of Art. 19(1 )(e)]:
The policy or principle should be such that everyone should have the opportunity to migrate and
settle down in any part of India where opportunity for employment or better living conditions are
available and, therefore, it would be unconstitutional and impermissible to prevent the persons from
migrating arid settling at places where they find their livelihood and means of avocation. The
fundamental rights and the directive principles and the Preamble being “trinity” of the Constitution,
the right to residence and to settle in any part of the country is assured to every citizen.

No Right to Call or Enforce BANDH14


No political party has the right to call for ‘bandh’ on the plea that it is a part of its fundamental right of
speech and expression under Art. 19(1 )(a). The calling for and holding of 'bandh’ by a political
party,'organization involved a threat, express or implied, to citizen not to carry on his activities or to
practice his avocation on the day of the ‘bandh’ and hence violative of Arts. 19(1)(a) (b) (c) (d) and
21. Calling for a bandh is different from calling for hartal or strike having regard to the effect of bandh
on the fundamental right of the people ‘as a whole' [Communist Party of India (M) v Bharat Kumar
MR 1998 SC 184],
The respondent’s plea that a bandh could be peaceful or violent and even •f the Court were to
act it could act only to curtail violent bandhs and not peaceful bandhs, is not very relevant. A call for
a bandh holds out a warning to the citizen that if he were to go out for his work or to open his shop,
he would be prevented and his attempt to take his vehicle on the road will also be dealt with. It is true
that theoretically it is for the State to control any possible violence or to ensure that a bandh is not
accompanied by violence. But in our present set up, the reluctance and sometimes the political
subservience of the law-enforcing agencies and the absence of “political will” exhibited by those in
power at the relevant time, has really to a situation where there is no effective attempt made by the
law-enforcing a9encies either to prevent violence or to ensure that those citizens
who do not want

[L.C.11-
14. Write a note on Right to call and enforce bandh’. 2006]
Right to Freedom 121

to participate in bandh are given the opportunity to exercise their right to work, their right to trade or
their right to study.
In James Martin v State of Kerala (2004) 2 SCC 203, it was held that no person has any right
to destroy another’s property in the guise of bandh or hartal or strike, irrespective of the proclaimed
reasonableness of the cause oi the question whether there is or was any legal sanction for the same.
There is need to control such acts with an iron hand.
Comments - ‘Bandhs’, though are a legitimate weapon in the hands of the financially weaker
segments, have over the years become a nuisance to the majority of the common man. ’Bandhs’ are
now a political weapon used by various political parties to highlight their personal grievances and
bringing life of common man to a grinding halt. It leaves a daily worker with no bread and butter.
Could a developing country like ours think of wasting its precious manpower and resources on such
events which usually takes place to the whims and fancies of few individuals who are more
concerned in highlighting their individual names rather than the real needs of the group to which they
claims to be representative of.
Exercise of a “right to protest’’ should not affect the fundamental right of another citizen who is
equally entitled to exercise his rights. One has to lay down a border mark at which the right to
express one's voice encroaches or starts to inroad into other's right. A conflict between two
fundamental rights has to be resolved by seeing the exercise of which fundamental right is in societal
interest.

Art. 19 (l)(g): Freedom of Profession and Trade 32


Under Art. 19(1)(g) every citizen of India has the right “to practice any profession or to carry on any
occupation, trade or business”. This right is also not absolute and the State can impose reasonable
restrictions “in the interest of the general public.” The expression “in the interest of the general public"
is of wide import comprehending public order, public health, public security, morals, economic
welfare of the community and the objects mentioned in Part IV of the Constitution,
Under Art. 19(6), the State has also been empowered to prescribe professional or technical
qualifications necessary for practicing any profession or carrying on any occupation, trade or
business, as well as for enabling the State to carry on any trade or business to the exclusion of
citizens wholly or partially. In fact the State is competent to nationalize any trade or to monopolize a
trade
Art. 19(1 )(g) uses four expressions which have similar connotations -not identical The
expressions are 'profession, occupation, trade and business.’ The object is to make the right
comprehensive and wide. It covers all the avenues and methods by which a person earns his
livelihood. 'Profession' means an occupation carried on by a person by virtue of his personal and
specialized qualifications,

32 Describe the nature of restrictions which can be imposed on the freedom, of trade and
profession. [D.U-20081
Right to Freedom 122

training or skill. The word ‘occupation’ has a wide meaning such as any regular work,
profession, job, principal activity, employment, business, or a calling in which an individual is
engaged. ‘Trade’ includes any bargain or sale, any occupation or j, usiness carried on for subsistence
or profit, etc. ‘Business’ is a very wide term and would include anything which occupies the time,
attention and labour of a man f0r the purpose of profit. Thus, trade, profession, industrial and
commercial occupations, sale and purchase of goods, etc. are included in it [Sodan Singh v H.D.M-
C. (1992) 5 SCC 52],
The right to carry on a business includes the right not to enter a business and the right to close
the business. The first negative right is absolute but second may be subjected to restrictions in the
interest of public e.g. requirement to pay compensation to workers. The Industrial Disputes Act
requires the employer to obtain permission of the State government before closure; it is not
unreasonable. But, refusal to give permission even when; the employer is not able to pay minimum
wages is not a reasonable restriction (Excel Wearv UOI AIR 1979 SC 25).
In Narendra Kumar v UOI (AIR 1960 SC 430), it was held that restrictions may also amount to
'prohibition' under certain circumstances. Thus, a law depriving a citizen of his fundamental right may
be regarded as reasonable restriction, if it prohibits him to carry out dangerous trade such as that of
trade in liquor or cultivation of narcotic plants or trafficking in women. The right of every citizen to
pursue any iawful trade or business is obviously subject to such reasonable conditions as may be
deemed proper by the government essential to the safety, health, peace, decency or morals of the
community. But where a restriction reaches the stage of prohibition, special care has to be taken by
the court to see that the test of reasonableness is satisfied. In this case, the Non-Ferrous Metal
Order, 1958, which completely excluded the dealers in a trade of imported copper, was held valid as
imposing reasonable restriction in the interest of general public. The court said that prohibition was
only a kind of restriction provided it satisfied the test of reasonableness.
The court ruled that the reasonableness of a restriction has to be considered In the
background of the facts and circumstances under which the order was made, taking into account the
nature of the evil that was sought to be remedied by such •aw, the ratio of the harm caused to
individual citizens by the proposed remedy, to the beneficial effect reasonably expected to result to
the general public, and, whether the restraint caused by the law was more than what was necessary
in the interests of the general public
In Chintaman Rao v State of M P. (AIR 1951 SC 118), the prohibition was, however, held to be
unreasonable because it was in excess of the object in view and was drastic in nature. In this case, a
State law prohibited the manufacture of bidis in the villages during the agricultural season. The object
was to ensure adequate supply of labour for agricultural purposes. Even villagers incapable of
6r|
gaging in agriculture, like old people, women and children, etc. who supplemented their income by
making bidis in their spare time, were prohibited from engaging ’hemselves in bidi manufacture
without any reason.
The court laid down the test for a "reasonable restriction” as follows The phrase 'reasonable
restriction' connotes that the limitation imposed on a person in enjoyment of the right should not be
arbitrary or of an excessive nature beyond what is required in the interest of the public. The courts
are thus entitled to consider the “proportionality” of these restrictions. The word 'reasonable’ implies
intelligent care and deliberation i.e the choice of a course which reason dictat»s.‘Legis!ation which
arbitrarily or excessively invades the right cannot be said to contain the quality of reasonableness
and unless it strikes a proper balance between the freedom guaranteed in Art. 19(1 )(g) and the
social control permitted by Art. 19(6), it must be held to be wanting in that quality.”
In B.R. Enterprises v State of U.P (AIR 1999 SC 1667), it was held that the right of sale of
lottery tickets is not a right under Art. 301 or Art. 19(1)(g). Lottery has been held to be gambling. In it,
there is no skill but only an element of chance, it falls outside the realm of res commercium (things in
Right to Freedom 123

which there is commerce). Even State lotteries cannot be said to constitute “trade” as contemplated
by Art 301.
A person has no fundamental right to carry on trade to the detriment of others (viz. causing of
pollution, noise, etc.) (Abhiiash Textiles Ltd. v Rajkot Municipal Corporation AIR 1988 Guj 57). In
State of Gujarat v Mirzapur Moti Qureshi Kassab Jamet (AIR 2006 SC 212), it was held that the
prohibition of slaughter of cows and her progeny does not amount to total ban on activity of butchers.
It is not a prohibition but only a restriction They can slaughter animals other than cow progeny and
carry on their activity. Cow and her progeny are the backbone of Indian agriculture and economy.
Thus, the ban on slaughter of cow progeny, as imposed by the ‘Bombay Animal Preservation Gujarat
Amendment Act 1994' (as applicable to the Gujarat State) is in the interest of general public and is
reasonable restriction within the meaning of Art. 19(6) of the Constitution. It is not violative of the
fundamental right to carry on trade and business under Art. 19(1 )(g) of the Constitution.
Complete prohibition of sale of eggs within the limits of Rishikesh (a holy city on the banks of
Ganga river) is not an unreasonable restriction [Omprakash v State of U.P. (2004) 3 SCC 402]. A
government rule whish prohibits private practice by doctors engaged in teaching in a government run
medical college does not violate the freedom. Those who take up government service consent to
abide by the rules of service [Sukumar Mukherjee v State of W.B. (1993) 3 SCC 724], The hawkers
have a right to carry on trade on the pavements but the State may regulate the places where and the
time when they may be permitted to do so [Soda/i Singh v N.D.M.C. (1992) 5 SCC 52]. A temporary
law controlling the supply and distribution of an essential commodity is constitutionally valid.
In Vinay Baiachandra Joshi v Registrar General, Supreme Court (1998) 7 SCC 461, the
allotment of chambers within the compound of the Supreme Court Bar Association was in issue The
Advocates-on-Record claimed that not giving a chamber to an advocate is violative of Art. 19(1)(g) of
the Constitution. The Supreme Court held that the members do not have any fundamental right to such
allotment even proceeding on the basis that to practice as an advocate is a fundamental right under
Art. 19(1 )(g). The Court observed: A legal practltioner/an advocate can carry 0n his legal profession
without a chamber. It is not necessary that he should have a chamber within the Court premises. That
which merely facilitates the exercise of the fundamental right cannot be regarded as an integral part
of that fundamental right. Far from being a fundamental right it does not even have the status of a
right. No law confers such a right on a member of a legal profession nor has such a facility been
accepted as a right even otherwise.
In T.M.A. Pai Foundation v State of Karnataka (2002) 8 SCC 481, the Supreme Court for the
first time brought into existence the concept of education as an “occupation"’, a term used in Art.
19(1)(g) of the Constitution. The fundamental right to establish and run educational institutions is
thus subject to reasonable restrictions under Art 19(6).

QUESTIONS AND ANSWERS

Q. 1 In order to secure an equitable distribution of the imported newsprint and to curb the
monopolistic tendency in newspaper industry, the Central Government issues the
Newsprint Control Order under the Essential Commodities Act. The Order provides as
follows -

(a) No person other than the Central Government shall import Newsprint.
(b) No person shall use newsprint except, under a licence given by the government.

(c) No Newspaper establishment will publish more than 16 pages.


(d) Not more than 1/5th of the space shall be utilised for commercial advertisement.
XVI
Constitutional Law o f India- II

(e) Newspapers containing 12 to 16 pages will be priced at Rs. 1.50.


(f) No new edition or adjustments between different publications will be resorted to.

(g) A certain percentage of news space will be allocated to pressing socio-economic


issues facing the country.
Z & Co., publishers of a national daily The Times’, its editor and a reader challenge the
validity of the above conditions as violative of Art. 19(1 )(a). How will you decide?
What would be the status of an ordinance which restricts the number of
advertisements by television per episode?
What Preliminary objections do you expect the government to raise?
[C.L. C.-93/94/95/2000/2006/2.007: L. C.l - 94/99/2000/2004;
L C. ll-95l[I.A. S. -2001]
125 Constitutional Law o f India- II

A. 1 Freedom of speech and expression and Press - Article 19(1 )(a) says that all citizens shali
have the right to freedom of speech and expression. This right includes the liberty of the
Press. The right under Art. 19(1)(a) empowers the State to put ‘reasonable' restrictions on
various grounds, viz. security of the State, friendly relations with foreign States, public
order, decency and morality, contempt of court, defamation, incitement to offence, and
integrity and sovereignty of India.
it is now a well-settled law that freedom of press is both quantitative and qualitative. Freedom
lies both in circuiation and its content (news and views). The freedom of speech could not be
restricted for the purpose of regulating the commercial aspects of activities of the newspapers. It
cannot, like the freedom to carry on business, be curtailed in the interest of general public. It can be
restricted only on the grounds mentioned in Art. 19(2).
In Bennett Coleman & Co. v Union of India, the Supreme Court observed that once the quota
is fixed and directions to use it in accordance with the newsprint policy made applicable, the
newspapers are prevented from determining their pages, circulation, and new editions by the
Government Order. The individual requirements of the different dailies render it eminently desirable
in some cases to increase the number of pages than circulation. Such adjustment is necessary to
maintain the quality and the range of readers in question. The denial of this flexibility hampers the
quality, range and standard of the dailies and affects the freedom of press.
in Sakai Paper’s case, the Newspaper Order which fixed a minimum price and number of
pages which a newspaper was entitled to publish, was held unconstitutional, as it infringed the liberty
of press. An increase in price without any increase in the number of pages would reduce circulation.
On the other hand, any decrease in the number of pages would reduce the column, space for news.
In Bennett Coleman case, held that if as a result of page reduction, newspaper will have to
depend on advertisements as their main source of income, they will be denied dissemination of
news. On the other hand, if they have to sacrifice advertisements and thus weaken the link of
financial strength, the organisation will crumble.
Decision - In the present case, except measures (a) and (b) which are necessary in the public
interest, measures (c), (d), (e), (f) and (g) are violative of Art. 19(1 )(a) as they restrict the liberty of
the press. The ordinance which restricts the number of advertisements by television per episode is
also restrictive and thus invalid.

Preliminary objections of the Government


(i) The measure would help curb the monopolistic tendency in newspaper industry.
(ii) There would be no abridgement of fundamental right of the press if as a result of regulation
of newspaper business there was the incidental effect of curtailing circulations. The press is
not immune from laws of general application, or laws of industrial relations (Express
Newspaper’s case and Hamdard Dawakhana v Union of India).
(iii) The subject-matter of the newsprint order was to regulate and control the newsprint and not
to control newspapers.
(iv) The regulation is a ‘reasonable’ restriction on the business activity of a newspaper in the
interest of general public.
(v) Reduction in the page level is justified on the ground that the big dailies devote high
percentage of space to advertisements and so cut in pages will not be felt by them if they
adjusted their advertisement space.
(vi) If a certain quantity of steel was allotted the government could insist as to how it was going
to be used. Thus, output could be controlled. Similarly, newsprint could be controlled.

Q. 2 The freedom of press is not expressly mentioned in Art. 19(1 )(a), but the Supreme Court
XVI
Constitutional Law o f India- II

has ensured it in its various decisions. Discuss. What you think is the scope of freedom
of press in India? Is it permissible to impose pre-censorship on the newspapers in
India?
[L.C.II-95\

“Liberty of Press consists in laying no prior restraints upon publications and not in
freedom from censure for matters when published.” Explain and indicate how far this
liberty of Press is protected under the Indian Constitution. [I.A. S. -2004]

Write a short note on: Rights of the Press and the problem of Precensorship.
[I.A.S.-2008]
A. 2 Freedom of Press - Unlike the American Constitution, Art. 19(1 )(a) does not expressly
mention the liberty of press i.e. the freedom to print and to publish what one pleases without
previous permission. But it is settled law that the right to freedom of speech and expression
under Art. 19(1 )(a) includes the liberty of the press. The following cases illustrate the point.
In Bennett Coleman’s case, the Supreme Court held that the freedom of newspapers to
publish any number of pages or to circulate it to any number of Persons is each an integral part of the
freedom of speech and expression. Freedom of press is both quantitative and qualitative Freedom
lies both in circulation and its content (news and views). In Sakai Paper’s case, held that the freedom
of speech could not be restricted for the purpose of regulating the commercial aspects of activities of
the newspapers. ‘'
In Indian Express Newspaper’s case, held that the press industry is not free from taxation, but
tax should be within reasonable limits and does not impede freedom of expression i.e. circulation.
The imposition of tax like the custom duty on newsprint is an imposition on knowledge and would
amount to a burden imposed on a man for being literate and for being conscious of his duty as a
citizen to inform himself about the world around him.
In Printers (Mysore) Ltd v Asst. Commr. Tax Officer (1995) AIR SCW 204 on press freedom,
the Supreme Court held that no sales Tax can be imposed ori sale of newspapers in the country.
However, the press is not immune frorn taxation... the prohibition is upon the imposition of any
restriction to disseminate information and to the circulation of newspapers. Freedom of press has
always been a cherished right of all democratic countries. The newspapers not only purvey news but
also ideas, opinions and ideologies. They are supposed to guard public interest by bringing to fore
the misdeeds, failings and lapses of the government and other bodies exercising governing power.
Rightly, therefore, it has been described as the Fourth Estate, The court held that the freedom of
press stands at a higher footing than other enterprise. In view of this, the test for determining the
vires of a statute taxing newsprint have, therefore, to be different from the test usually adopted for
testing the vires of other taxing statute.
In Rajagopal v State of T.N. (1994) 6 SCC 632, held that the press have the right to publish an
unauthorised account of a citizen's life in so far as it is based upon public records. Freedom of the
press, the court said, means absence of interference by the State except insofar as it is authorised
by the Constitution and by laws.
Scope of freedom - It is clear from the above discussion, that press in India enjoys sufficient
freedom. However, it must be pointed out that the press is not immune from laws of general
application or ordinary forms of taxation, or laws of industrial relations (Express Newspaper v Union
of India).
Pre-Censorship on Newspapers - It is not permissible. In Brij Bhushan v State of Delhi (AIR 1950 SC
129), the Chief Commissioner of Delhi issued an order against the printer, publisher, etc. of a
English Weekly, directing them to submit for scrutiny before publication till further orders, all
communal matter and news and views about Pakistan other than those derived from official sources
Right to Freedom 127

or supplied by the news agencies. The court struck down the order, and observed that the imposition
of precensorship of a journal is a restriction on the liberty of the press which is an essential pa/t of
the freedom of speech and expression declared by Art. 19(1 )(a).
In Express Newspaper v Union of India, held that a law which imposes precensorship
(censorship prior to publication) or prevents newspapers from being started or require them to seek
government aid in order to survive was violative of Art. 19(1 )(a). In Rajagopai v State of T.N. held
that the State or its officials have no authority in law to impose prior restraint on publication of
defamatory matter. The public officials can take action only after the publication if it is found to be
false. In LIC v Manubhai D. Shah, held that prior restraints are threats to freedom of expression,
because of their potential for imposing arbitrary and irrational decisions.

Q. 3(a) Discuss citizen's right to show films on T.V. and Doordarshan, making reference to
decided cases. [I.A S. -90j

(b) Doordarshan invited X, a lady journalist, for an interview to be telecast subsequently.


The theme of the interview was “Laws relating
Right to Freedom 128

to women.” In the course of the interview X referred to a bill on women's rights pending
in Parliament and criticized that some of its provisions are unjustified arid
unconstitutional. This portion of the interview was arbitrarily deleted when the interview
was telecast. X wants to challenge with deletion. Discuss the questions involved.
[I. A. S. -95]
A. 3(a) Right of citizens to exhibit films on Doordarshan subject to the terms and conditions to be
imposed by the Doordarshan is a part of the fundamental right of freedom of expression
guaranteed under Art. 19(1 )(a). The right is similar to the right of citizen to publish his
views through any other media such as newspapers, magazines, advertisement hoardings
etc. subject to the terms and conditions of the owner of the media. The freedom of
expression is a preferred right which is always very zealously guarded by the Supreme
Court (Odyssey Communications Pvt. Ltd. v Lokvidayan Sangathan AIR 1988 SC 1642).
In the aforesaid case, telecasting of a serial 'Honi Anhoni’ was resisted by a social
organization on the ground that it was likely to spread false and blind beliefs and
superstitions amongst the members of the public. However, the respondent organization
failed to show that the exhibition of the serial was prima facie prejudicial to the community.
In LIC v Manubhai D. Shah (1992) 3 SCC 637, the managing trustee of the respondent trust has
produced documentary film on the Bhopal Gas disaster titled “Beyond Genocide”, which was
awarded (best non-feature film of 1987). I & B Ministry, at the time of presentation of awards, had
made a declaration that award- winning short films will be telecast on doordarshan. But,
respondent’s film refused to be telecast by doordarshan on ground that content being outdated, and
also that fiim is long. Held that a film maker has a fundamental right under Art. 19(1 )(a) to exhibit his
film, and the party which claims the right to refuse the telecast under Art. 19(2) on the ground that
the film did not conform to the requirements of law made under Art. 19(2) has the onus to prove that
he is entitled to refuse this right. Doordarshan being a State controlled agency founded by public
was not justified to refuse the film’s telecast merely because the film was critical of State
Government.
(b) The issues in the present case relates to the violation of a citizen’s rights under Arts. 14
and 19 of the Constitution. Doordarshan's action is unreasonable and unjust thus violative
of Art. 14. It also violates right to freedom of speech and expression guaranteed under Art.
19(1)(a). The freedom of speech and expression means the right to express one’s
convictions, and opinions freely by word of mouth, writing, printing, pictures or any other
mode. Freedom of speech lay at the foundation of all democratic organisations, for without
free political discussion, no pubiic education, so essential for the proper functioning of the
process of popular government, is possible.
!h LIC v Manubhai D. Shah, the court held that merely the film was critical of the State government
was no reason to deny telecasting of film. These communication channels are great purveyors of
news and views and make considerable impact on the minds of viewers and are known to mould
public opinion on vital national issues. Prior restraints are threats to freedom of speech and
expression, because of their potential for imposing arbitrary and irrational decisions. In R. Rajagopal
v State of T.N., the court observed that in a free democratic society public officials must always be
open to criticism. In Secy., Min. of l&B v Cricket Asscn., Bengal the court observed that a citizen has
a fundamental right to use the best means of imparting and receiving information and as such have
access to telecasting fQr the purpose. The government has no exclusive right to use broadcast media
or deny the use by others.
Protection in Respect of
Conviction for Offences
(Article 20)
4
ARTICLE 20
[PROTECTION IN RESPECT OF
CONVICTION FOR OFFENCES]

Under Art. 20, the Constitution of India has taken care to safeguard the rights of
persons accused of crimes (citizens or non-citizens, including a corporation). This
Article cannot be suspended even during an emergency by an order under Art.
359. Art. 20 constitutes a limitation on the legislative powers of the Union and
State Legislatures.
Art. 20 has three clauses. Each of these clauses gives protection in respect
of conviction for offences:
(a) Ex-post facto laws [Art. 20(1)].
(b) Double jeopardy [Art. 20(2)].
(c) Self-incrimination [Art. 20(3)].

[A] Art. 20 (1): Ex-Post Facto Law


According to Art. 20(1), “no person shall be convicted of any offence except for
violation of law in force at the time of commission of the act charged as an
offence, nor be subjected to a penalty greater than that which might ha ;e been
inflicted under the law in force at the time of commission of offence”. Thus, the
legislature is Prohibited to make criminal laws having retrospective effect. Art.
20(1) is not applicable t° a trial or a civil liability or preventive detention case or
disciplinary proceedings.
Generally, all statutes must be interpreted as prospective in operation.
However, retrospectivity may be expressly declared by the statute or couid be
inferred from the language used therein. Art. 20(1) spells out the prohibition
against retrospective Penal laws. A legislature has the authority to make laws.
Such laws may be given

[1 2 9 ]
130 C oan date
effect from s t i tanterior
u t i o n atol itsL making.
a w o fAnI nAct
d iwhich
a - I Ioperates to cover pa§{
transactions is called retroactive or retrospective. Thus, an ex-post facto law is a
law which is enacted subsequent to some occurrence i.e. the commission of some
act or omission. In other words, it declares some act or omission as an offence for
the first time after the completion of the offence. Such a law may also enhance the
punishment/penalty for an offence subsequent to the commission of that offence.
Further, it may prescribe a new and different procedure for the prosecution of an
offence subsequent to the commission of that offence. Art. 20(1) does not cover
the last category of ex-post facto laws.
Since Art. 20(1) is concerned with the liberty of a person, a liberal
interpretation had to be given to its language (Transmission Corpn., A.P. v Ch
PrabhakarAIR 2004 SC 3365). Art. 20(1) lays down certain iimitations on the
legislature/courts:
(a) Prohibition against enacting ex post facto penal law. In other words, if an act
or omission was innocent when done the legislature cannot make a law which
declares such act or omission a crime. The legislature cannot make law which
provides for punishment of acts or omissions which were committed prior to the
date when the Act came into force. If bringing gold into India was never an offence
and a law is made in 1970 making import of gold into India an offence and
applying the law from 1960 onwards then such law is a retrospective criminal law
not permitted by the Constitution.
' In short, a new law cannot punish an old act. An act which is not an
offence on the date of its commission (as per the actual factual law existing on
that date) cannot be made an offence on a subsequent date. Thus, a penal law
which creates new offences is always prospective (Dayal Singh v State of
Rajasthan AIR 2004 SC 2608).
Thus, something would be an offence only if that thing is made punishable
by a ‘law in force’. That also means that it is the knowledge of only those laws,
which are in force, at the time at which a person does some act, which is made
punishable by the law. The ‘knowledge of the future laws’ cannot be imputed to
any person. The principle that ‘ignorance of law is no excuse’ is not applicable tn
such situations.
Section 304-B inserted in the Indian Penal Code on November 19, 1986,
creating a distinct offence of 'dowry death’ and providing a minimum sentence of
seven years’ imprisonment could not be applied to such death caused before the
insertion of the section (Soni Devrajbhai Babubhai v State of Gujarat AIR 1991 SC
2173). Similar would be the case of the offence of‘bribery’ inserted in I.PC. in
1952.
(b) A taw cannot aggravate the crime. A law cannot change the punishment and
make it greater and apply it to previous offences. The new punishment (if greater)
may be applied only from a future date. ‘A’ commits an offence in 1970. The
offence is punishable with imprisonment for 3 years, in 1972, the legislature
amends the law and increases the punishment to 7 years. The revised
punishment cannot be inflicted on ‘A because the offence was committed before
the date of the passing of the Act. Art. 20(1) does not permit retrospective
application of increased penalty-
However, where a person undergoing life-imprisonment for an offence f°r
which death was one of the punishments or where the death sentence wa s
commuted to life-imprisonment, and the law was amended to provide that such

^4
P r ot e cJV « "3K!
t i o n iV
n R e s pe c t of C o n vi c t i o n f or O f f e n c e s 1 3 1
person would not be released from prison without serving at least 14 years
imprisonment, it was held that the amended law did not enlarge punishment
retroactively, it merely prescribed a minimum sentence of 14 years for a murderer
(,Maru Ram v UOI AIR 1980 SC 2147).
The prescription of a minimum sentence of fine where the law in force does
not prescribe any such limitation is valid; however, an additional fine may amount
to enhanced punishment. Thus, where a person accused under the Prevention of
Corruption Act was convicted, and later the Act was amended to provide for an
additional fine to be equivalent to the amount of money found to have been
procured by the offender, Art. 20(1) applied (Kedar Nath v State of W.B. AIR 1953
SC 404). But, where the ex-post facto law did not impose a penalty but provided a
speedier remedy for the recovery of the embezzled money, it did not amount to
enhanced punishment. Thus, where the amended law provided for the
confiscation of the property of a person convicted for embezzlement of
government money, to set off the embezzled money was held valid (State of W.B.
v S.K. Ghosh AIR 1962 SC 255).
In Transmission Corpn., A.P. v Ch. Prabhakar, a Division Bench of the
Supreme Court referred the following question for the decision of the larger
Bench: Whether constitutional guarantee enshrined in Art. 20(1)... also prohibits
legislation which aggravates the degree of crime or makes it possible for the
accused to receive greater punishment even though it is also possible for him to
receive the same punishment under the new law as could have been imposed
under the prior law or deprives the accused of any substantial right or immunity
possessed at the time of the commission of the offence charged ...
(c) Prohibition against conviction - Art. 20(1) not only prohibits the legislature from
enacting ex post facto laws depriving a person of the protection given by this
article but it also lays down that no person shall be convicted of an offence. This
indicates that the courts too are forbidden to pronounce conviction on the basis of
a law violating the two protections set out in Art. 20(1).
However, it may be noted that the interpretation of a provision of law by the
courts do not create any hew offence. For instance, the decision in the Sarla
Mudgal v UOI (AIR 1995 SC 1531), laying down that the second marriage of a
Hindu husband after conversion to Islam without having his first marriage
dissolved, would be an offence under Sec. 494, IPC; this judicial pronouncement
did not lay down any new law or offence, thus, Art. 20(1) has no application (Lily
Thomas v UCI AIR 2000 SC 1650).

When Art. 20(1) does not apply


(>) No prohibition for change in procedural laws - A person does not have the
fundamental right to be tried by a particular procedure. Art 20(1) does not prohibit
,h
e ‘trial’ of offences under the ex-post facto laws. This is unlike the American law,
w
nere a trial under the ex-post facto law is barred.
A law which retrospectively changes the venue of a trial or an amended law
which provides for setting up of a Special Tribunal to try summarily the offences
Under the Act in question have been upheld by the court as not violating the Art.
2°0) (Transmission Corpn., A.P. v Ch. Prabhakar AIR 2004 SC 3368).

(ii) Benefit of reduction in punishment - Art. 20(1) does not bar the accused from
taking benefit of the reduction in punishment (i.e. modifications of the rigour of a
criminal law). The rule of beneficial construction required that an ex-post facto
law could be applied to reduce the punishment. If any subsequent legislation
tones down punishment for an offence, legislative benevolence can be extended
to the accused who awaits judicial verdict regarding sentence [Supdit, Narcotic
Control Bureau v Parash Singh (2008) 13 SCC 499],
X VWhere a boy of C
16oyears
I
n s t i tofuage
tionwas
a l undergoing
L a w o f rigorous
I n d i a - imprisonment
II for
six months for house trespass and outraging the modesty of a girl, and
meanwhile the Probation of Offenders Act, 1958 was passed which provided that
a person below the age of 21 years should not ordinarily be sentenced to
imprisonment, it was held' that the ex-post facto law, which was beneficial to the
accused did not fall within the prohibition of Art. 20(1) (Rattan Lalv State of
Punjab AIR 1965 SC 444) It may be noted that the Probation of Offenders Act
was not a penal statute; it was a social welfare legislation aiming to reform the
offenders. It is the penal laws which have a prospective operation. In that case,
the accused boy could not have the benefit of the legislation.
(iii) No application to civil laws - There is no limitation on retrospective operation
of civil laws including tax laws or disciplinary proceedings. The Government could
lay down, under an ex-post facto law, any manner, for the recovery of its dues.
Art 20(1), also, does not apply to a law punishing continuing offence or a case of
preventive detention.
Non-fulfilment of civil liability may entail penal damages or imprisonment.
But, that does not make out a case under Art. 20(1). Thus, where the non-
payment of compensation by the employers closing their undertaking was made
punishable (via imprisonment) retrospectively by the Act, the Apex Court
upholding the constitutionality of the impugned Act heid that the liability imposed
was civil liability and since the failure to discharge a civil liability was not an
offence, Art. 20(1) would have no application (Hathisingh Mfg. Co. v UOi AIR
1960 SC 923). Similarly, imposition of penal damages for unauthorized
possession of village common land had been held to be a civil liability.

[B] Art. 20(2): Double Jeopardy33


Art. 20(2) provides that “no person shall be prosecuted and punished for the
same offence more than once.” The principle of double jeopardy has been
already recognized in the Sec. 26, General Clauses Act and Sec. 300, Cr. PC
The object is to avoid the harassment, which must be caused to a person for
successive criminal proceedings where only one crime has been committed by
him
This embodies the common law (English) maxim nemo debet bis vexari-
‘no man shall be put twice in peril for the same offence.’ The U.S. Constitution
also contains this provision- ‘no person shall be twice put in jeopardy of life or
limb.’ This is commonly called ‘double jeopardy.’ Double jeopardy has two
aspects: (a) autrefois convict and (b) autrefois acquit. The plea of autrefois
convict avers that the defendant has been previously convicted in respect of the
same offence and autrefois acquit is the plea that the accused has been acquitted
on a charge for the same offence for which he is being prosecuted. Under the
U.S. law, the protection is available not only against a second punishment but
even against the second trial for the same offence, irrespective of whether the
accused was acquitted or convicted in the first trial.
The Constitution of India protects a person from being prosecuted and
convicted more than once for the same offence. Both prosecution and punishment
must co-exist for the operation of Art. 20(2). Where a person having been
prosecuted is acquitted, he can be prosecuted for the same offence again. In other
words, Art. 20(2) contains the principle of autrefois convict only and does not
include autrefois acquit. It is narrower than the American and English doctrine of
double jeopardy. Protection against autrefois acquit is to be found in Sec. 300(1) of

Explain the doctrine of double jeopardy.


the Code of Criminal Procedure, 1973. It is a statutory right; not a fundamental
P r ot e c t i o n i n R e s pe c t of C o n vi c t i o n f or O f f e n c e s 1 3 3
right.
Art. 20(2) would have no application to a continuing offence viz. pollution
activities (nuisance), continued unauthorized possession of land, violation of
copyright, etc.
Appeal does not amount to second prosecution - An 'appeal’ against an acquittal
does not amount to second prosecution; it is oniy a continuation of the original
prosecution and therefore Art. 20(2) would not be attracted. Further, there was no
punishment for the offence in the earlier prosecution (Kaiawati v State ofH.P. AIR
1953 SC 131). An ‘appeal’ against the conviction also does not amount to second
prosecution. Similarly, where the prosecution was a nullity or void (e.g. due to the
lack of sanction of a competent authority for the prosecution of a government
servant), and the accused was discharged, a fresh prosecution for the same
offence would not be violative of Art. 20(2) (Brij Nath v State of Bhopal AIR 1957
SC 494). An enhancement of punishment by the revising authority has been held
not a second punishment.
Administrative and Departmental Proceedings - The protection afforded by Art.
20(2) is attracted only in respect of punishment inflicted by court of law or judicial
tribunal. In other words, the term “prosecution”, in the context of Art 20(2), means
initiation or starting of any proceeding, criminal in nature, before a court/judicial
tribunal. Thus, if the proceedings are held under any revenue authorities, Art.
20(2) has no application.
Thus, a Government servant prosecuted and convicted by a court of law
can be punished under departmental proceedings for the same offence, A person
who has been fined under the Customs Act can still be prosecuted under the
Foreign Exchange Regulation Act because the customs authority is not a court
(Maqbool Hussain v State of Bombay AIR 1953 SC 325). An inquiry and
subsequent dismissal of a government servant is no bar to prosecution for an
offence under the Indian Penal Code and Prevention of Corruption Act. The
inquiry is not prosecution in a court and disciplinary action is not a punishment
given by a court (S. A. Venkataraman v UOI AIR 1954 SC 375).
Similarly, Art. 20(2) do not ban proceedings before a civil court for
disobedience °f an injunction along with criminal proceedings.
Same offence - The Constitution bars double punishment for the same offence.
Conviction for one offence does not bar subsequent trial and conviction for
another offence. It does not matter that some ingredients of the two offences are
common. If the same act constitutes offences under different sections or Acts the
Constitution does not bar separate trial and punishment.
Thus, where a person was prosecuted under the Indian Penal Code for
conspiracy
XVI to commit the act for which he was already convicted under the Sea
Constitutional Law o f India- II
Customs Act, it was held that the second prosecution was not barred by Art. 20(2).
Committing an offence and conspiracy to commit that offence are two distinct
offences (Leo Roy Frey v Superintendent, District Jail AIR 1958 SC 119).
Double Jeopardy and Res Judicata/Issue Estoppel - Res judicata means the thing
has already been decided'. The principle is embodied in Sec. 11 of the Code of
Civil Procedure. Res judicata rests on the principle that where an issue of fact has
been tried by a court on. a former occasion the finding is final and binding on both
the parties and cannot be raised again. The principle is applicable to both Civil
and Criminal trials Since the doctrine rests on the identity of issues at the two trials
it is known as the ‘doctrine of Issue Estoppel.’ It precludes evidence being led to
prove a fact in issue as regards which evidence has already been led and a
specific finding recorded at an earlier criminal trial.
Art. 20(2) bars 'double punishment’, the rule of Issue Estoppel bars
'reception of evidence’ on an issue on which the finding was in the favour of the
accused at a previous trial. Art. 20(2) has no direct bearing on the question at
issue: it would be attracted only if the “offence” is the same in both the
prosecutions.

[C] Art. 20(3): Protection against Self-Incrimination2


Art. 20(3) lays down that “no person accused of any offence shall be compelled to
be a witness against himself. In other words, the accused person is protected
against incriminating himself under compulsion e g. 'making a statement which
makes the case against the accused person at least probable considered by itself.
It can be claimed by natural persons as well as by corporations, etc
This clause is based on the maxim nemo tenetur prodere accussare
seipsum, which means that 'no man is bound to accuse himself. The accused is
presumed to be innocent till his guilt is proved and it is the duty of the prosecution
to establish his guilt. Thus, the accused need not make any admission or
statement against his free will.
Clause (3) of Art. 20 is an attempt to prevent torture of the accused by
investigating agencies for the purpose of extracting confession from him. In the
18th century in England and even in the 20 fh century in communist Russia, Nazi
Germany and many other countries torture was a legal procedure. The Universal
Declaration of Human Rights proclaims: 'No one shall be subjected to torture or to
cruel, inhuman or degrading treatment or punishment’ (Art. 5).
Person accused of an offence - This immunity is available only to a person who is
'accused of an offence’ (M R Sharma v Satish Chandra AIR 1954 SC 300) A
person cannot claim the protection if at the time he made the statement, he was
not an accused but becomes an accused thereafter. Further, unlike USA and
England, the protection in India is confined to 'accused’ only and
not other witnesses.

[D.U.-2008\
2. Explain the doctrine of self incrimination.’

MINI
A person against whom a formal accusation relating to the commission of an
P r ot e c t i o n i n R e s pe c t of C o n vi c t i o n f or O f f e n c e s 1 3 5
0ff<=nce (viz. FIR, Complaint, Criminal prosecution before the court) has been
levelled is covered by Art. 20(3). However, it is not necessary that to avail the
privilege contained in Art. 20(3), the actual trial or inquiry should have commenced.
Where a person was arrested on suspicion of having committed an offence, eliciting
^formation from him would not be protected by Art. 20(3), when neither the case is
registered nor an FIR is recorded.
In Nandini Sathpathy v PL Dani (AIR 1978 SC 1025), during the course of
the investigation, the accused was interrogated with reference to a long list of
questions given to her in writing. She refused to answer those questions claiming
the protection of Art. 20(3). It was held that the protection contained in A>1 20(3)
extends back to the stage of police investigation not commencing in court only,
since such inquiry was of an accusatory nature Further, the ban on self-accusation
and the 'right to silence’ extends beyond that case and protects the accused in
regard to other offences, pending or imminent, which might deter him or her from
voluntary disclosure of incnminatory nature.
Article 20(3) provides that no person accused of any offence shall be
compelled to be a witness against himself. The 'right to remain silent’ is an
extension of the rule of civil liberty enjoined by our Constitution. Considering the
guarantee under Art. 20(3) and also humanizing standards under Art 21, court is
required to tread cautiously while construing retracted confession. To withdraw
from what has been said previously needs to be interpreted in vein of right to
remain silent as an extension of this civil liberty [Aloke Nath Dutta v State of W B.
(2007) 12 SCC 230 ]
Protection against criminal proceedings only - The protection is confined to
criminal proceedings before a court of law or a judicial tribunal. It does not extend
to civil proceedings or such proceeding which are not of a criminal nature e g.
under the Customs Act or the Foreign Exchange Regulation Act. In these
proceedings, a person cannot refuse to answer on the ground that it might
incriminate him.
Notice or pendency of contemot proceedings does not attract Art 20(3)
[Delhi Judicial Service Asscn. v State of Gujarat (1991) 4 SCC 406J. The
contemners are not accused of an offence. Contempt proceedings are not in the
nature of criminal proceedings for an offence. Art. 2C(3) also do not apply to
departmental •nquiries into allegations against a government servant, as there is
no accusation of any offence within the meaning of Art. 20(3).
Protection against compulsion to be a witness - The protection is against
compulsion ,0 he a wrtness. The expression covers oral evidence and written
statements (documentary anc< testimonial evidence) conveying the accused
person s personal knowledge relating *° the charge against him. The protection
contained in Art. 20(3) covered testimonial ^Pulsion in the court as well as
compeiied testimony previously obtained (viz. production of evidentiary documents)
(M.P Sharrna v Satish Chandra AIR 1954 SC 300)
In State of Bombay v Kathi Kalu Oghad (AIR 1961 SC 1808), the Apex Court,
°Wever, held that the immunity granted to the accused does not extend to
compulsory I Auction of material objects or compulsion to give specimen writing/
signature, finger 'Pression or giving of blood specimens. In these cases, the
accused is not giving y Personal testimony. They are merely materials for
comparison, in order to lend
assurance to the court that its inference based on other pieces of evidence is
reliable. The court distinguished 'to be a witness' from 'furnishing evidence’.
Compulsion for production of documents is prohibited only if the documents
convey the personal knowledge of the accused relating to the charge (‘to be a
witness’) Where an accused is compelled to produce a document in his
possession, which is not based on his personal knowledge (‘furnishing evidence’),
thereX is
V I no violation of Art. 20(3), because he does not become a witness by the
Constitutional Law o f India- II
mere fact that he has produced it it does not amount to testimonial compulsion
offending Art. 20(3).
Search of the premises of a person accused of an offence under a search
warrant and seizure of the documents are not violative of Art. 20(3). In such
cases, it is not the act of the accused but that of a police officer to which the
accused is obliged to submit and is therefore, not his testimonial act in any sense
It cannot be treated as a compelled production of a thing/document. However, if
the accused is compelled to be a party to a search, it would be violative of Art.
20(3) (V. S. Kuttan Pillai v Ramakrishnan AIR 1980 SC 185).
> Likewise, tape-recording of statements made by the accused, without his
knowledge, but without force or oppression, is not violative of Art. 20(3), as there
is no compulsion involved.
Compelled testimony - The provisions of Sec. 27 of the Evidence Act, 1872 (under
which a statement made by the accused leading to the discovery of some facts is
admissible in evidence) are not within the prohibition of Art 20(3), unless the
compulsion had been used in obtaining the information from the accused person.
Compulsion in this context would mean “duress” (State of Bombay v Kathi
Kolu Oghad). The Supreme Court in Nandini Sathpathy case held that “relevant
replies which furnish a real and clear link in the chain of evidence indeed to bind
down the accused with the crime become incriminatory and offend Art. 20(3) if
elicited by pressure from the mouth of the accused”. It widened the scope of
compulsion and heid that “compelled testimony" is evidently procured not merely
by physical threats or violence but also by psychic torture, atmospheric pressure,
environmental coercion, tiring interrogative proximity, overbearing and htimidating
methods and the like.
In Sampath Kumar v £.0. Enforcement Directorate, Madras (AIR 1998 SC
16), held that administration of “caution” to person summoned that not making
truthful statement was an offence, did not amount to use of pressure within the
meaning of Art. 20(3). Protection against self-incrimination secured by Art. 20(3) is
available only if the accused is compelled to give evidence against himself.
Administration of such a caution, the court said, was, in fact, in the interest of the
person who was making the statement.
Waiver of privilege - The privilege given by Art. 20(3) to the accused person may
be waived by him. Thus, this Article is not violated when he volunteers evidence
against himself. Art. 20(3) would not apply to a case where the confession was
made by the accused without any inducement, threat or promise (Kalawati v State
of H.P AIR 1983 SC 131).

Protection of Life and Personal Liberty


(ARTICLE 21)

ARTICLE 21 [PROTECTION OF LIFE AND PERSONAL LIBERTY]

"No person shall be deprived of his life or personal liberty except


according to the procedure established by law' (Art. 21).
The State shall provide free and compulsory education to all children
between the age of 6 and 14 years in such manner as the State may. by
P r ot e c t i o n o f L i f e a n d P e r s o n al L i be r t y 137

law, determine" (Art. 21-A).


Article 21 is one article which has been so transformed by the Supreme Court that
it now encompasses all conceivable human rights within its ambit. On a plain
reading it is a directive to the State to refrain from infringing the right to life or
personal liberty of a person. The courts have taken a very liberal view and
transformed the negative injunction to a positive mandate to do aii things which
will make life worth living. It is now well-settled that Art. 21 has both negative and
affirmative dimension.
After the Maneka Gandhi’s decision, Art. 21 now protects the right of life and
personal liberty not only from the executive action but from the legislative action
also. Prior to it, the State could interfere with the liberty of citizens if it could
support its action by a valid law. The right guaranteed in Art. 21 is available to
citizens as well as non-citizens (Chairman, Railway Board v Chandrima Das AIR
2000 SC 988).
The word ‘deprived’ does not mean total loss. Any restriction imposed on the
liberty is a deprivation. The court may even act when there is an imminent threat.
Personal Liberty: Meaning and Scope1
In A.K. Gopalan v State of Madras (AIR 1950 SC 27), the petitioner challenged
the validity of his detention under the Preventive Detention Act, 1950, on the
ground that it was violative of his right to freedom of movement under Art. 19(1
)(d) which is the very essence of personal liberty guaranteed by Art, 21 The court
took the view that since the word ‘liberty’ is qualified by the word 'personal’, which
is a narrower concept and so it does not include all that is implied in the term
‘liberty’ (i.e. all the freedoms). Thus, personal liberty’ only means ‘liberty relating to
or concerning the person or body of the individual.’
Fazal Ali, J., however, in his dissenting opinion, gave a wide and
comprehensive meaning to the words 'personal liberty’ as consisting of freedom of
movement and locomotion. Thus, any law which deprives a person of his personal
liberty must satisfy the requirements of Arts 19 and 21 both.
in England, the expression has been understood in the restricted sense.
Dicey has said, “right to personal liberty means a person's right not to be
subjected to imprisonment, arrest, or other physical coercion in any manner that
does not admit of legal justification”.
XVI
Constitutional Law o f India- II

But, in Kharak Singh v State of Punjab (AIR 1963 SC 1295), the term
‘personal liberty' was interpreted to be a compendious term including within itself
all the varieties of rights which go to make up the personal liberty of man other
than those dealt with in Art. 19(1) While Art. 19(1) deals with particular species or
attributes of that freedom, ‘personal liberty' in Art. 21 takes in and comprises the
residue. It is true that in Art. 21, the word ‘liberty' is qualified by a word ‘personal’,
but this qualification is employed in order to avoid overlapping between those
incidents of liberty which are mentioned in Art. 19(1).
In that case. Police regulations authorising domiciliary visits (i.e. visits in the
night to a private house to make sure whether the suspect is staying at home or
not) against bad characters, were held to be violative of Art 21. By the term ‘life’
as used here, something more is meant than mere animal existence. The
inhibition against its deprivation extends to all those limits and faculties by which
life is enjoyed.
However, in Govind v State of M.P. (AIR 1975 SC 1379), held that
depending upon the character and antecedents of the persons subjected to
surveillance, and the object and limitations under which surveillance is made, it
cannot be said that

Discuss the circumstances under which a person can be deprived of his


personal liberty. [LC. 1-
20061
Explain the meaning of expression "personal liberty” in Art. 21 as
interpreted by the Supreme Court. [C.L.C.-
2006]
Evolution of Art. 21 is in essence journey from A.K Gopalan v State of
Madras (AIR 1950 SC 27) to Maneka Gandhi v UOI (AIR 1978 SC 597).
Comment.
[L.C.11-20071
Critically examine the decisions of the Supreme Court from A.K. Gopalan to
Maneka Gandhi cases with special reference to “procedure established by
law’ [I.A. S.-
2003]
urveillance by domiciliary visits would always be unreasonable restriction upon the
5 ht of privacy. The petitioner in this case was shown to be a dangerous criminal

Vvh0se conduct showed that he was determined to lead a criminal life. The
regUlations by police, upheld by court, as they had the force of law.

In Malak Singh v State of Punjab (AIR 1981 SC 760), held that under Sec 23
of the Punjab Police Act it was the duty of the police officers to keep surveillance
over bad characters, and habitual offenders for the purposes of preventing crimes.
So long as the surveillance is for the purpose of prevention of crimes and confined
to the limits prescribed by Rule 23(7) of the Punjab Police Rules, a person cannot
complain against the inclusion of his name in the surveillance register. But, if it is
excessive and goes beyond the limits prescribed by Rule, i.e. surveillance of a
person who does not belong to above categories, its validity may be challenged
as infringing the right of privacy of a citizen as his fundamental right to personal
liberty under Art. 21, and freedom of movement in Art. 19(1 )(d).
P r ot e c t i o n o f L i f e a n d P e r s o n al L i be r t y 139

“The Constitution does not put an absolute embargo on the deprivation of


life or personal liberty but such a deprivation must be according to the procedure,
in the given circumstances, fair and reasonable. To become fair, just and
reasonable, it would not be enough that the procedure prescribed in law is a
formality. It must be pragmatic and realistic to meet the given fact-sityation. No
inflexible rule of hearing and due application of mind can be insisted upon in
every, or, all cases. Each case depends upon its own backdrop” (Ahmedabad
Municipal Corpn. v Nawab Khan Gulab Khan AIR 1997 SC 152).
in Maneka Gandhi case, the Supreme Court has given the widest possible
interpretation to the word personal liberty'. A valid law interfering with personal
liberty must satisfy a ‘triple test (i) It must prescribe a procedure (ii) the procedure
must withstand the test prescribed in Art. 19 (iii) it must not infringe Art. 14. Thus,
the procedure must be just, fair and reasonable,

Scope of ‘Personal Liberty’

Leading Case: maneka GANDHi v UNION OF INDIA2


(AIR 1978 SC 597)

Facts - In this case, the petitioner’s passport was impounded by the


Central Government under Sec. 10(3)(c) of the Passport Act, 1967. The
Act authorised the Government to do so if it was ‘necessary in the
interest of general public’. Though sub clause (5) of this section required
the passport authority to record reasons for impounding passport and
furnish to the holder of the passport a copy of the same; but this sub sec,
further provides that if passport authority is of the opinion that it will not
be in the interest of sovereignty and integrity of or security of India, or in
the

Maneka Gandhi’s case is said to have put life into the “Life and Liberty
clause” of the Constitution of India. Comment. lC.LC.-2007\[L.C.I-2007l[!,A.S,-
2OQO]
interest of general public, it may decline to furnish a copy, and which was
done so in the present case.
The petitioner challenged the validity of the said order on the
following grounds:
(i) Sec. 10(3)(c) was violative of Art. 14 as conferring an
arbitrary power, since it did not provide for a hearing of
passport holder before impounding passport.
(ii) Sec. 10(3)(c) was violative of Art. 19(1)(a) and (g) since it
permitted imposition of restrictions not provided in clause
(2) or (6) of Art. 19.
(iii) Sec. 1 G(3)(c) was violative of Art. 21, since it did not
prescribe ’procedure’within the meaning of the Art. 21. The
reason for the order were, however, disclosed in the
affidavit filed on behalf of the Government which stated that
XVI
Constitutional Law o f India- II

the petitioner’s presence was likely to be required in


connection with the proceedings before a Commission of
Inquiry
Issues and Observations - Various issues considered in the present case,
were as follows:
(a) Inter-relation of Arts. 14, 19 and 21 34 - In Gopalan's case,
the Supreme Court held that the Art. 19 had no application
to laws depriving a person of his life and personal liberty
enacted under Art. 21. It was held that Arts. 19 and 21 dealt
with different subjects. Thus, so long as a saw of preventive
detention satisfied the requirements of Art. 22, it would not
be required to meet the challenges of Art. 19. This view
proceeded on the assumption that certain articles in the
Constitution exclusively deal with specific matters. Thus,
Art. 22 is a self-contained code.
In R.C. Cooper v Union of India (AIR 1970 SC 564), the doctrine of
exclusiveness was seriously questioned. It was held that each freedom
has different dimensions and there may be overlapping between different
fundamental rights and therefore it is not a valid argument to say that the
expression ‘personal liberty’ in Art. 21 must be so interpreted as to avoid
overlapping between that Article and Art. 19(1).
In the present case, the Court overruled Gopalan’s view, and held
that Art. 21 is controlled by Art. 19 i.e. it must satisfy the Art. 19
requirements also. Art. 21 does not exclude Art. 19, and even when there
is ‘no infringement of fundamental rights under Art, 21, such a law in so
far as it abridges or takes away any right under Art. 19, would have to meet
the challenges of Art. 19, and ex-hypothesi of Art. 14.
Bhagwati J., observed: The expression ‘personal liberty’ is of widest
amplitude and it covers a variety of rights which go to constitute the personal
liberty of man and some of them have raised to the status of distinct
fundamental rights and given additional protection under Art. 19. The correct
way of interpreting the provisions conferring fundamental rights is that, ‘‘the
attempt of the court should be to expand the reach and ambit of the
fundamental rights rather than to attenuate their meaning and content by
judicial construction”.
(b) Procedure established by lavi/35 - In A.K. Gopaian’s case, it

34 “Procedure established by Saw under Art. 21 should be just, fair and reasonable’-
Discuss referring to interrelationship between Arts. 14, 19 and 21 of
Constitution. [C. L. C./L. C.1-
2006/2007]
Critically examine: Arts. 14, 19 and 21 are mutually exclusive.
“A law establishing procedure for curtailing personal liberty of a citizen must be in
consonance with the rule of justice, ;airness and reasonableness.” Critically
examine. What would be your answer if such a law also violates equality clause"?
[D.U.-2008]
“Right to equality and protection to personal liberty are no longer mutually
exclusive.” Discuss. [M.S.-95]
P r ot e c t i o n o f L i f e a n d P e r s o n al L i be r t y 141

was argued that the word law did not mean enacted law, but it
meant principles of natural justice; and the expression
“procedure established by law” meant the same thing as the
phrase “due process of law” in the American Constitution.
(Note: “due process of law” is a wider term, which covers both
substantive and procedural law). The omission of the word
‘due’ made no difference to the interpretation of Art. 21. But,
Supreme Court rejected these contentions.
Pointing out the difference in the use of words the Supreme Court held in
Gopalan that the difference is deliberate and material. In effect, Art. 21 only
requires that there must be a valid enacted (State) law and not the law in the
abstract or general sense embodying the principles of natural justice as
interpreted by the U.S. Supreme Court. The court will not examine the
content of the law and will not judge its fairness or reasonability. This view is
supported by the history of the Article. In the draft Constitution the clause was
as under: 'No person shall be deprived of his life or liberty without due
process of law.’ After discussion, the expression ‘without due process of iaw’
was substituted by the expression ‘except according to procedure established
by law.’
In the present case (Maneka Gandhi), Bhagwati J. did not apply the
concept of “due process” as such in the context of Art. 21, when he stated
that the protection secured by Art. 21 is a limited one in as much as Art. 21
safeguards the right to 'personal liberty’ against executive interference which
is not supported by law, and law here means ‘enacted law’ or ‘State law’.
However, he departed from the Gopalan approach and read ‘procedure in Art.
21 not as prescribing any specific procedure, but
only that procedure which conforms to the principles of natural justice i.e
procedure has to be 'right, just and fair’. It was explained that the principle
of reasonableness, which is an essential element of equality or non-
arbitrariness pervading Art. 14, must also apply with equal force to the
'procedure' contemplated by Art. 21, so procedure must not be arbitrary,
fanciful or oppressive. Thus ‘due process' has been imported and
established as a part of the Constitution.
However, Krishna Iyer J., in his separate but concurring opinion,
extended the guarantee in Art. 21 not only to 'procedure' as being fair and
just, but also to ‘law’ as being ‘reasonable’; it meant in a way reading
‘procedure established by law” as almost amounting to ‘due process of
law1 in the American sense.
Decision - Held that any procedure which permits impairment of the
Constitutional right to go abroad (vide Satwant Singh’s case AIR 1967 SC
1836) without giving a reasonable opportunity to show cause cannot but
be condemned as unfair and unjust.
In the present case, it was held, however, that Sec. 10(3)(c) of
Passport Act is not violative of Art. 21 as it is implied in the provisions that

Critically examine Procedure established by law under Art. 21 need not comply
with the principles of natural justice.
XVI
Constitutional Law o f India- II

the rules of natural justice would be applicable in the exercise of the power
of impounding a passport. The power conferred on the Passport Authority
is not unguided. The grounds denoted by the words in the interest of
general public' are clearly defined. In view of the statement of the
Attorney-General that the Government was agreeable to consider the
representation of the petitioner, it was held by majority that the defect of
the order was removed and order was passed in accordance with the
procedure established by law
In the present case, the court also considered the question that
what is the test or yardstick to be applied for determining whether a statute
infringes a particular fundamental right? Decisions in R.C. Cooper's case
and Bennett Coleman's case shows that the tests of pith and substance of
the subject-matter and of the direct object and of incidental effect of
legislation are relevant to the questions of legislative competence but they
are irrelevant to question of infringement of fundamental rights. The true
test is whether the effect (direct) of the impugned action is to take away or
abridge fundamental rights. If the effect of State action on fundamental
rights is direct and inevitable, than a fortiori it must be presumed to have
been intended by the authority taking the action and hence this doctrine of
direct and inevitable effect has been described by some jurists as the
doctrine of intended and real effect.36
Comments - The interaction of Arts 14, 19 and 21 analysed by the Court
in the present case, demonstrates that the requirement of
reasonableness’ runs like a golden thread through the entire fabric of
fundamental rights.
Arts 14, 19 and 21 are not mutually exclusive and there is overlapping in
the contents of rights given under these articles
The Court created a protective shield against any type of arbitrary,
unreasonable and unjust actions which seeks to violate the individual
freedom and liberties of people, by inter-mixing and co-relating the
provisions of Arts. 14, 19 and 21.
In later cases, the Maneka Gandhi’s decision has been followed.
In Mohd. Sabir v State ofJ&K (AIR 1989 SC 1899), the court similarly held.
In Sunil Batra v Delhi Administration (AIR 1978 SC 1675), Krishna Iyer,
J. said: “True cur Constitution has no due process clause, but after
Cooper and Maneka Gandni, the consequence is the same. Art. 21 is
the counterpart of the procedural due process in the United States.’’]
In Francis Coralie v Delhi Administration (AIR 1981 SC 746), the validity of the

36 “A law establishing procedure for curtailing personal liberty of a citizen must be


in consonance with the rule of justice, fairness and reasonableness.” Critically
examine. What would be your answer if such a law also violates equality
clause? [D.U -2008\
“Right to equality and protection to personal liberty are no longer mutually
exclusive.” Discuss. [I.A.S.-98\
Critically examine: Procedure established by law under Art. 21 need not
c.ompty with the principles of natural justice.
P r ot e c t i o n o f L i f e a n d P e r s o n al L i be r t y 143

provisions of the COFEPOSA which provided that a detenu (a person placed


under preventive detention) can have interview with his lawyer only after obtaining
permission of the District Magistrate, and that too, in the presence of the custom
officer, and, permitted interview of the family members only once in the month,
were challenged on the ground that they are arbitrary, unreasonable and violative
of Arts.
14 and 21. The Supreme Court held that the detenu’s right to have interview with
his lawyer and family member is part of his 'personal liberty guaranteed by Art. 21,
and cannot be interfered with except in accordance with reasonable and just
procedure established by law The word ‘personal liberty' in Art. 21 is of the widest
amplitude and it includes the “right to socialise'' with members of family and
friends, subject of course, to any valid reasonable prison regulations.
The right of detenu to consult a legal advisor of his choice for any purpose
including securing release from preventive detention is included in the right to Hve
with human dignity and is also part of personal liberty. Right to 'live' is not merely
confined to physical existence but it includes within its ambit the right to live with
human dignity (vide Maneka Gandhi case), and all that goes along with it, namely,
the bare necessities of life such as adequate nutrition, clothing, and shelter, and
facilities for reading, writing, education and expressing ourselves in diverse forms,
freely moving about and mixing and commingling with fellow human beings.
In Bandhua Mukti Morcha v UOI (AIR 1984 SC 802), an organisation
dedicated to the cause of release of bonded labours informed the Supreme Court
about a large number of labourers working in some stone-quarries under
“inhuman and intolerable conditions”. The court observed that right to live with
human dignity, derives its life breath from the Directive Principles of State Policy,
and therefore, it must include protection of health and strength of workers,
opportunities and facilities for children to develop in a healthy manner and in
conditions of freedom and dignity, just and humane conditions of work. Since the
Directive Principles are not enforceable in a court of law, it may not be possible to
compel the State through the judicial process
Constitutional Law o f India- II
to make provision by statutory enactment or executive fiat for ensuring these
basic essentials, but where iegislation is already enacted providing these basic
requirements to workmen, the State can certainly be obligated to ensure
observance of such legislation, for inaction on the part of the State would amount
to denial of the right to live with human dignity enshrined in Art. 21.

IMPLIED FUNDAMENTAL RIGHTS (UNDER ART. 21)

A new judicial trend has emerged from the cases of Maneka Gandhi, Sunil Batra,
Hoskot and Hussainara Khatoon, in which the Supreme Court has taken the view
that the provisions of Part III of the Constitution of India should be given widest
possible interpretation. In Maneka Gandhi’s case, Bhagwati J., said, the correct
way of interpreting the provisions of Part III is that attempt of the court should be
to expand the reach and ambit of the fundamental rights rather than to attenuate
their meaning and content.
By an activist interpretation most of the fundamental rights, especially the
right to equality (Art. 14) freedom of speech and expression [Art. 19( 1 )(a)] and
right to life and personal liberty (Art. 21) have been converted into a regime of
positive human rights unknown in previous constitutional diction. By an affirmative
action the courts are trying to force the government to create favourable
conditions for effective realisation of the new individual, collective, diffuse rights.
The Supreme Court held that to be a fundamental right it is not necessary
that a right must be specifically mentioned in a particular Article. Even if it is not
mentioned in any of the Articles specifically, it may be a fundamental right if it is
an integral part of a named fundamental right or partakes of the same basic
nature and character as that fundamental right. Every activity which facilitates the
exercise of the named fundamental right may be considered integral part of that
right and hence be a fundamental right.
The expression 'personal liberty’ covers a wide variety of rights which go to
constitute the personal liberties of a man other than those which are already
included in Art. 19. The courts are inclined to give the widest amplitude to the
expression. On account of the liberal interpretation, Art. 21 has now come to be
invoked almost as a residuary right, even to the extent which the founding fathers
never dreamt of [Narinderjit Singh Sahni v UOI (2002) 2 SCC 210],
The discoveries of egalitarian goals in the fundamental rights have resulted in
the explosion of rights Thus, Art. 21 of the Constitution has sprung up a whole lot
of human rights jurisprudence. For example, it has been held that these are
fundamental rights under Art. 21, though not specifically mentioned.
(i) Right to speedy trial (M.H. Hoskot v State of Maharashtra, AIR 1978 SC
1548, Hussainara Khatoon v State of Bihar, AIR 1980 SC 1819; Raj
Deo Sharma v State of Bihar (1998) 7 SCC 507)),
P r ot e c t i o n o f L i f e a n d P e r s o n al L i be r t y 145

(ii) Right to travel abroad (Maneka Gandhi’s case),


(iii) Right to dignity (Maneka Gandhi, Francis Coraiie cases),
(iv) Right to privacy (Govind v State of M.P.) ,
(v) Right to clean environment (/W.C. Mehta v Union of India),
(vi) Right to livelihood (Olega Teilis case),
(vii) Right to education (Mohini Jain and Unni Krishnan cases),
(viii) Right to marriage (Lata Singh v State of U.P. AIR 2006 SC 2522),
(ix) Right against torture (Sunil Batra v Delhi Admn., AIR 1950 SC 1979;
Jolly Varghese v Bank of Cochin, AIR 1950 SC 470; Khatri v State of
Bihar (1951) 1 SCC 623;,
(x) Right against bondage (Bandhua Mukti Morcha case), People’s Union
For Democratic Rights v Union of India, (1952) 2 SCC 235; Neeraja
Chaudhary v State of M.P (1954) 3 SCC 243),
(xi) Right to legal aid (Sheela Barse v Union of India, (1956) 3 SCC 632;
Suk Das v V.T. of Ar. Pradesh, (1956) 2 SCC 401,
(xii) Right to food [PUCL v UOI 2000 (5) SCALE],
Similarly the freedom of speech and expression guaranteed under Art. 19(1)(a)
includes the right to know, right to information, right to reply etc. Freedom of press
is inferred from the freedom of speech and expression. Some of these unspecified
or implied fundamental rights are discussed below.
In State of M.P. v Ram Kishan Balothia (AIR 1995 SC 1195), held that
anticipatory bail cannot be granted as a matter of right. It is essentially a statutory
right. Similarly, the 'right to vote’ is a statutory right and not a fundamental/
common law right. The right to vote is subject to the limitations imposed by the
statute, and the challenge to any provision in the statute prescribing the nature of
right to elect cannot be made with reference to a fundamental right in the
Constitution. Thus, Prisoners/Under trials cannot claim a right to vote (Anukul
Chandra Pradhan v Union of India AIR 1997 SC 2514).
‘Right to life’ under Art. 21 do not include ‘right to die’ (suicide) or ‘right to be
killed’ (mercy-killing or euthanasia).
Right to life’ under Art. 21, as interpreted by the Supreme Court, includes:
Right to dignity, Right to privacy, Right to clean environment, Right to livelihood,
Right to education, Right to marriage, Right against torture, Right against
bondage, Right to legal aid, Right to health and medical care, Right to food, etc.
‘Right to personal liberty’ under Art. 21, as interpreted by the Supreme Court,
includes: Right to speedy trial, Right to travel abroad, Right against bondage,
Right to legal aid, etc.
Some other examples are:
(i) Handcuffing should be resorted to only when there is a clear and present
danger of escape.
(ii) Long delay in execution of death sentence may evoke the protection of
Art. 21. Death sentence would be commuted to life imprisonment. No
fixed period has been declared. The court will decide from case to case
(iii) In suitable cases the court may grant compensation to the victim who
XVI
Constitutional Law o f India- II

has been deprived of his/her right e.g. victims of torture, rape, custodial
death, fake encounter, etc.
(iv) Capital punishment is not violative of Arts. 14, 19 and 21.
(v) Hanging as a mode of execution is fair and just and does not offend
against Art. 21.

Right to Dignity
In Maneka Gandhi case, the court held that the right to iive is not merely confined
to physical existence but it includes within its ambit the right to live with human
dignity. In Francis Coralie case, held that the right to live includes the right to live
with human dignity and all that goes along with it, namely, the bare necessities of
life, such as adequate nutrition, clothing and shelter, and facilities for reading and
writing, freely moving about and mixing with fellow human being.
In Jolly Varghese v Bank of Cochin (AIR 1980 SC 470), the court held that
imprisonment of a poor person for non-payment of debts amounted to deprivation
of his personal liberty without fair procedure and was, therefore, violative of Art.
21. To be poor is no crime, said Krishna Iyer, J. In Vikram Deo Singh Tomar\/
State of Bihar (AIR 1988 SC 1782), it was brought to the notice of court that the
female inmates of the Care Home Patna were compelled to live in inhuman
conditions. Held that the right to live with human dignity is a fundamental right and
the State is under duty to provide at least the minimum conditions ensuring
dignity. Similar observations were made in Upendra Baxi v State of U P. (1986) 4
SCC 106.
In P.U.D.R. v UOI (AIR 1982 SC 1473), held that non-payment of minimum
wages to workers employed in various Asiad Projects in Delhi was a denial to
them of their right to live with basic human dignity and violative of Art. 21. Rights
and benefits conferred on the workmen under various labour laws are “clearly
intended to ensure basic human dignity to workmen and if they are deprived of
these rights and benefits that would clearly be a violation of Art. 21." Thus, non-
implementation by the private contractors and non-enforcement by the State
authorities of the provisions of various labour laws violate the fundamental right of
workers to “live with human dignity”.
In Vishaka v State of Rajasthan (AIR 1997 SC 3011), the Apex Court’s
attention was focussed towards prevention of sexual harassment of working
women in all work-places. Held that it resulted in violation of the fundamental
rights of “Gender Equality” and the “Right to Life and Liberty" enshrined in Arts.
14, 15 and 21. It was also held to be violation of the victim's fundamental right
under Art. 19(1 )(g) to practise any profession or to carry on any occupation, etc.
as a “safe” working environment is needed for that. The court observed: “Gender
equality includes protection from sexual harassment and right to work with dignity,
which is a universally accepted basic human right. In the absence of suitable
domestic legislation in this sphere, international conventions/norms, so far as they
are consistent with the constitutional spirit, can be relied on, viz. Convention on
the Elimination of All Forms of Discrimination against Women.” The court laid
P r ot e c t i o n o f L i f e a n d P e r s o n al L i be r t y 147

down some guidelines/ norms to be strictly observed by the


employers/responsible persons in work places and other institutions, until a
suitable legislation were enacted to occupy the field. The guidelines are to be
applicable to both public and private sector.
In Apparel Export Promotion Council v A.K. Chopra (AIR 1999 SC 625), the
court rejected the appellant’s contention that he merely attempted to molest her
but had not actually molested her. Held that the appellant’s act was wholly against
moral sanctions, decency and was offence to female subordinate’s modesty and
undoubtedly amounted to sexual harassment and hence the punishment of
dismissal from service imposed on him was commensurate with the gravity of his
objectionable behaviour and, thus, vaiid. An action or gesture, directly or by
implication having tendency to outrage modesty of women constitutes “sexual
harassment”; even actual touch is not required, an ‘attempt’ also will do for the
purpose.37
In R.D. Upadhayay v State of A.P. (AIR 2006 SC 1946), the Apex Court
showing serious concern regarding the plight of children living in jails with
prisoner mothers issued detailed directions for their interest regarding food,
shelter, medical care, clothing, education and recreation facilities which are
declared to be child’s right. The court directed that the child above 6 years not to
be kept with female prisoners. Birth of child in prison is not to be recorded as
‘prison’ in Birth Certificate.
In State of Bihar v Lai Krishna Advani (AIR 2003 SC 3357), the Apex Court
made it clear that one was entitled to have and preserve one’s reputation and one
also had a right to protect it.
Comments - Total and continuing destitution and impoverishment exposes people
to a loss of their humanity. In no society that takes human rights seriously should
there be allowed a state of affairs where human beings become sub-human i.e.
they do not have their basic needs (food, shelter, clothing, health, education)
fulfilled, where people sell their wives, children or themselves (in bonded labour)

37 A Bill (No. XI of 1998) to provide for the prevention of sexual harassment of


women employees at their work places has been introduced in the
Parliament. Sec. 2(c) of the Bill defines “sexual harassment” as including
any unwanted verbal or gestrual sexual advances, sexually implicit and
derogatory statements or remarks, avoidable physical contacts, touching or
patting, suggestive remarks, sexually slanted and obscene jokes, comments
about physical appearance, compromising invitations, use of pornographic
material, demands for sexual favours, threats, innuendoes, physical assault
and molestation of and towards women workers by their male superiors,
colleagues or anyone who for the time being is in a position to sexually
harass the women workeis.
Thus, a very wide and comprehensive definition of ‘sexual harassment' has
been provided for in the Bill. Further, Sec. 3 prescribes punishment for
sexual harassment - imprisonment up to 5 years or fine up to Rs. 25,000 or
both. Sec.
4 lays down that the onus of proving the innocence shall be on the accused
and the sexually harassed woman shall’ve the right to lead evidence in
rebuttal.
XVI
Constitutional Law o f India- II

in order to survive, and where the holders of public and private power find it
possible to use torture as a rational means of achieving certain policy objectives.
The expression 'human rights' presupposes a level at which biological
entities are bestowed with the dignity of being called human. The bearer of human
rights must have an implicit right to be and remain human, allowing them some
autonomy of choice in their survival. Further, there should be immunity against the
loss of dignity through the wilful infliction of physical and mental torture (U. Baxi).
The Supreme Court in the above mentioned cases has emphasised similar
points. However, it remains to be seen how far the right to live with human dignity
would be made available to the masses by those (i.e. the holders of public and
private power) for whom the term dignity has a rather sophisticated connotation
(for them, the term implies not the bare necessities but the luxuries of life) and
whose enjoyment of dignity consists in treating others with indignity and making
them indigent.

Right to Privacy
In Kharak Singh v State of U.P (AIR 1963 SC 1295) the court held that the term
life as used in Art. 21 meant something more than mere animal existence. The
inhibition against its deprivation extends to all those limits and faculties by which
life is enjoyed. An unauthorised intrusion into a person's home (by way of
domiciliary visits of policemen) and the disturbance caused to him is the violation
of personal liberty of the individual. But, in Govind v State ofM.P. (AIR 1975 SC
1379), the court held that Police Regulations authorising domiciliary visits were
constitutional. As regards the right of privacy, the court said that right to privacy
would necessarily have to go through a process of case by case development.
In Malak Singh v State of Punjab (AIR 1981 SC 760), the right of privacy of
a citizen as a fundamental right was emphasised by the court. In State of
Maharashtra v Madhulkar Narain (AIR 1991 SC 207), held that the right to privacy
is available even to a woman of easy virtue and no one can invade her privacy.
_ Ina landmark judgment (The Hindustan Times, 3.12.93), the Madras High Court
held that a minor girl had the right to bear a child. In this case, a 16-year old minor
girl became pregnant and wanted to have the child against the opposition from
her father. The public prosecutor, on behalf of the girl, argued that she had the
right to bear the child under the broader right to privacy. Even a minor had a right
to privacy under Art. 21. The Constitution does not make any distinction between
minor and major in so far as fundamental rights are concerned. The court held
that in the case of a mature and understanding minor, the opinion of
parent/guardian was not relevant
In R. Rajagopal v State of T.N. (1994) 6 SCC 632, held that the right to
privacy or the right to be let alone is included in Art. 21 and a “citizen has a right to
safeguard the privacy of his own, his family, marriage, procreation, motherhood,
child bearing and education among other matters”. None can publish anything
concerning the above matters without the person’s consent, whether truthful or
otherwise and whether laudatory or critical. If he does so he would be violating the
right of privacy of the person concerned and would be liable for action for
P r ot e c t i o n o f L i f e a n d P e r s o n al L i be r t y 149

damages However, this principle would not apply if the citizen were a busy body
vvith a penchant for throwing himself into controversies. Moreover, whatever is on
public records can be published. And the right to privacy or the remedy of action
for damage is simply not available to public official as long as the criticism
concerns the discharge of their public duties.
In People’s Union for Civil Liberties (PUCL) v UOI (AIR 1997 SC 568), the
petitioners challenged the constitutional validity of Sec. 5 of the Indian Telegraph
Act, 1885 which authorises the Central or State government to resort to phone
tapping in the circumstances mentioned therein. The writ petition was filed in the
wake of the report on “Tapping of phones of Politicians” by the CBI. The Supreme
Court held that in the absence of ‘just and fair procedure’ for regulating the
exercise of power under Sec. 5(2) of the Act, it is not possible to safeguard the
rights of citizens guaranteed under Arts. 19(1)(a) and 21 of the Constitution. The
Court observed: “The right to privacy - by itself - has not been identified under the
Constitution. As a concept it may be too broad and moralistic to define it judicially.
Whether right to privacy can be claimed or has been infringed in a given case
would depend on the facts of said case. But the right to hold a telephone
conversation in the privacy of one’s home/office without interference can certainly
be claimed as "right to privacy". Telephone conversation is a part of modern
man’s life and an important facet of his private life. Telephone tapping would, thus,
infract Art. 21 unless it is permitted under the procedure established by law.”
In Mr. 'X' v Hospital ‘Z’ (Dr. Tokugha Yepthomi v Apollo Hospital) (AIR 1999
SC 495), the appellant (Mr. ‘X’, a doctor) was to marry a girl, but the marriage was
called off on the ground of blood test conducted at the respondent's hospital (‘Z’)
in which the appellant was found to be HIV positive. The appellant moved the
Supreme Court for violation of his ‘right of privacy’. The court rejected the
appellant’s contentions by holding that right to privacy is not absolute. It may be
lawfully restricted for the prevention of crime, disorder or protection of health or
morals or protection of rights and freedom of others In the instant case, another
person (appellant’s would-be-bride) was saved in time by the disclosure (‘right to
be informed’). Moreover, where there is a clash of two fundamental rights, namely,
the appellant’s right to privacy as part of right to life and his fiancee’s right to lead
a healthy life which is her fundamental right under Art. 21, the right which would
advance the public morality or public interest, would alone be enforced through
the Process of court, for the reason that moral considerations cannot be kept at
bay and the Judges are not expected to sit as mute structures of clay in the hall
known as courtroom, but have to be sensitive, “in the sense that they must keep
their fingers firmly upon the pulse of the accepted morality of the day.”
In Surjit Singh Thind v Kanwaljit Kaur (AIR 2003 P & H 353), the Punjab
and Haryana High Court held that allowing medical examination of a woman for
her virginity amounts to violation of her right to privacy and personal liberty
enshrined under Art. 21 [See also, Sharda v Dharmpal, 2003 SCW 1950],
Comments - Privacy has been defined as the claim of individuals, groups or
institutions to determine for themselves when, how and to what extent information
about them is communicated to others. In other words, privacy can be defined as
control over knowledge about oneself.
XVI
Constitutional Law o f India- II

In the United States, the right of privacy came to be accepted as


fundamental right guaranteed under various amendments of the Constitution after
considerable hesitation and evolution. In India, the Supreme Court has recognised
the right of privacy as a fundamental right but with considerable reservation and
hedged by conditionalities. Thus, according to the Supreme Court, this right can
be curtailed in the interest of national security, public safety or the economic well-
being of the country, for the prevention of disorder or crime, for the protection of
health or morals or for the protection of the rights and freedoms of others.
There can be no doubt that invasion of privacy violates freedom of speech
and expression and freedom to move freely throughout the territory of India as
guaranteed under Art. 19(1 )(a) and 19(1 )(d) respectively besides the right to
personal liberty as provided under Art. 21. The right which is violated by tapping of
telephones is obviously the right of privacy of an individual. Even a minor, a
prostitute, and a prisoner need the right of privacy A homosexual, too, need this
right.
Thus, we see that the right to privacy has a very broad content. The need of
the hour is to strike a balance between the right of privacy of a citizen and
society’s claim for law and order.

Right to Travel Abroad


In Satwant Singh v Asst. Passport Officer, New Delhi (AIR 1967 SC 1836), it was
contended that right to leave India or travel abroad was part of one’s personal
liberty which could be restricted only by authority of law The petitioner who was a
citizen of India had to travel abroad frequently for business purposes. The
Government ordered him to surrender his passport. The contention of the Union
Government was that a passport was a political document to which no one had a
legal, much less a constitutional right. The^ourt rejected this contention and held
that personal liberty in Art. 21 takes in the right of locomotion - to go where and
when one pleases, and the right to travel abroad is included in it.
In Maneka Gandhi's case also, the court held that right to travel abroad is
included in the personal liberty under Art. 21.

Right to Food
In PUCL v Union of India [2000 (5) SCALE], the Supreme Court recognizing the
right to food’ has held that the people who are starving because of their inability to
purchase food grains have right to get food under Art. 21 and therefore they ought
to be provided the same free of cost by the States out of surplus stock lying with
the States The people entitled in such situation are those who are aged, infirm,
disabled, destitute women/men, pregnant and lactating women and destitute
children.
Right to Marriage
The 'right to marry’ is an essential element of right to privacy but is not absolute.
Marriage is the sacred union, legally permissible, of two healthy bodies of
P r ot e c t i o n o f L i f e a n d P e r s o n al L i be r t y 151

opposite sexes. Every system of matrimonial law provides that if a person is


suffering from venereal disease in a communicable form it will be open to the
other partner in the marriage to seek divorce Moreover, so long as the person is
not cured of such disease, the right to marry cannot be enforced through a court
of law and shall be treated to be a "suspended right” (Mr. 'X' v Hospital Z‘ AIR
1999 SC 495).
In Mr. ‘X’ v Hospital ‘Z’ (AIR 2003 SC 664), held that "it was open to the
hospital or the doctor concerned to reveal information about the 'AIDS status’ of a
person to persons related to the girl whom he intended to marry and she had a
right to know about the HIV positive status of the appellant.”
In Lata Singh v State of U.P (AIR 2006 SC 2522), it was held that the right
to life includes 'right to marriage’ A major boy or girl undergoing inter-caste or
interreligious marriage are to be protected from harassment, threats or acts of
violence. Stern action should be taken against person who give threats or harass
or commit violence.

Right to Livelihood (Right to Work)


In Olega Tellis v Bombay Municipal Corpn. (AIR 1986 SC 180), (also known as
pavement dwellers' case), the petitioners challenged the validity of Bombay
Municipal Corporation Act, 1888, which empowered municipal authorities to
remove their huts from pavement and public places on the ground that their
removal amounted to depriving them of their right to livelihood, and hence it was
violative of Art. 21. The court held that the world life in Art. 21 includes the right to
livelihood also, because no person can live without the means of livelihood. If it is
not so, the easiest way of depriving a person of his right to life would be to deprive
him of his means of livelihood. The court, however, held that right to livelihood can
be curbed or curtailed by following just and fair procedure. The restrictions placed
on the right of livelihood of slum dwellers are reasonable because it is in the
interest of general public. Public streets are not meant for carrying on trade or
business.
In Sodan Singh v Delhi Municipal Committee (AIR 1989 SC 1988), the court
held that Art. 21 is not attracted in a case of trade and business. The petitioners,
hawkers doing business on the pavement of roads in Delhi, were rightly refused
by the municipal authorities to carry on business.
In Delhi Development Horticulture Employee’s Union v Delhi Admn. (AIR
1992 SC 789), it was contended that right to life includes the right to livelihood and
therefore, right to work. The petitioners who were employed on daily wages in the
Jawahar Rozgar Yojana’ filed a petition for their regular absorption as regular
employees. Held that although broadly interpreted and as a necessary logical
corollary the right to life would include the right to livelihood and so right to work,
but this country has so far not found feasible to incorporate right to livelihood as a
fundamental right. Because the country has so far not attained the capacity to
guarantee it (gigantic resources needed for it). Advisedly, therefore it has been
placed under directive principles, Art. 41 of which enjoins upon the State to make
effective provision for securing the same, within the limits of its economic capacity
XVI
Constitutional Law o f India- II

and development.
In D.K. Yadav v J.M.A. Industries (1993) 3 SCC 258, held that the right to
life includes right to livelihood and therefore before terminating the service of an
employee or workmen fair play requires that a reasonable opportunity should be
given to him to explain his case. The procedure prescribed for depriving a person
of livelihood must meet the requirement of Art. 14 i.e. it must be right, just and fair.
Art. 21 clubs life with liberty, dignity of person with means of livelihood without
which the glorious content of dignity of person would be reduced to animal
existence.
In LIC of India v Consumer Ed. & Res. Centre (1995) 5 SCC 482, held that
the ‘right to life and livelihood’ under Art. 21 includes the ‘right to life insurance
policy of LIC’ and, such policies, therefore, must be within the paying capacity and
means of the insured. The Preamble, Chapters on Fundamental Rights and
Directive Principles accord right to livelihood as essential for meaningful life.
Social security and disablement benefits are integral schemes of socio-economic
justice to the people, in particular to the middle class and lower middle class. The
LIC must evolve their policies subject to such reasonable, just and fair
terms/conditions accessible to all segments of the society.
In Mr. X of Bombay, Indian Inhabitant v M/s ZY (AIR 1997 Bom 406), it was
held by the Bombay High Court that a HIV-positive person could not be rendered
“medically unfit” solely on that ground so as to deny him the employment. The
petitioner, in this case, was found fit for his normal duty of loading of drums in
trucks and he did not pose any threat or risk to any person at workplace and not
likely to develop symptoms of AIDS for 8 to 10 years. Deleting his name from the
panel of casual labourers to be regularised, was held arbitrary, unjust, unlawful
and violative of Arts. 14 and 21. The Supreme Court agreed with the High Court
and held that right to Government service cannot be denied to person suffering
from AIDS (1998) 8 SCC 296.
Comments - The right to livelihood or right to work has various economic
implications. First, all fundamental rights are justiciable and as such, any citizen
would be free to move a court to claim compensation in the event of work not
being available to him. Second, many voluntary organisations and/or political
activists can organise their legal cells to secure relief for the unemployed. Third,
the quantum of relief to be provided can become a subject of dispute before the
courts. The moot question is: Is it desirable for the government to provide
unemployment relief and thus develop a dependency psychology among the
people or encourage a pattern of investment which promotes more employment?
The right to work at the present stage of development of our economy with
its accumulated distortions cannot straightaway lead to waged employment.
Neither the pattern of allocation of public resources, nor the decision making
motives and roCesses nor the command of the State over the resources nor the
pattern of ^ernand ana availability of technologies are such that organized sector
wage employment be 9iven t0 everyone. This requires not only massive additional
resource mobilisation, but also a massive restructuring of the economy in order to
clear it of distortions and biases.
P r ot e c t i o n o f L i f e a n d P e r s o n al L i be r t y 153

Therefore, it wouid be more desirable to keep the right to work as a directive


principle of the Constitution and give it top priority in our planning process so that
the entire plan frame is woven round this supreme goal. This would require new
priority areas to be determined which enlarge employment, more especially in the
rural sector. Dr. L.C. Jain rightly put it: You can create right to work in the
Constitution, but the Constitution will not create work. For that purpose, we have to
develop a wage goods model with emphasis on full employment.
In Air India Statutory Corpn. v United Labour Union (AIR 1997 SC 645), the
Supreme Court observed: “Due to economic constraints, though right to work was
not declared as a fundamental right, right to work of workman, lower class, middle
class and poor people is a means to development and source to earn livelihood.
Though, right to employment cannot, as a right, be claimed but after the
appointment to a post/office, be it under the State or private entrepreneur, it is
required to be dealt with as per public element and to act in public interest
ensuring equality (a genus of Art. 14) and ali other concomitant rights to make
their right to life and dignity of person real and meaningful”. In this case, the
question was whether after abolition of contract labour system, the contract
labourers who were earlier having regulatory protections would be rendered
persona non grata and would be thrown out from the establishment and told off
the gates? The court said that in such a case the remedy of abolition of contract
labour would be worse than the disease. Without employment/appointment, the
workmen will be denuded of their means of livelihood and resultant 'right to life'.

Right to Shelter
in Chameli Singh v State of U.P. (1996) 2 SCC 549, it was held that the right to
shelter is a fundamental right under Art. 21. Right to live guaranteed in any
civilized society implies the right to food, water, decent environment, education,
medical care and shelter. These are basic human rights known to any civilized
society.
In Ahmedabad Municipal Corpn. v Nawab Khan Gulab Khan (AIR 1997 SC
15
2), the problem of unauthorised encroachment of pavements by slum-dwellers
was in issue. The Court, following the Olga Tellis case (1985) 3 SCC 545, held that
while n° one has a right to encroach upon public footpaths, pavements or roads,
the State/ Municipal Corporation has constitutional as well as statutory duty to
provide residential accommod3tion to the poor and indigent weaker sections of the
society by utilising “te excess urban vacant land available under the Urban Ceiling
Act. But in all cases of ejectment of the encroachers, it is not obligatory on the part
of the State/Corporation *° Provide alternative accommodation. No absolute
principle can be laid down in this re9ard arid it would depend upon the facts of
each case. The Court observed: Right
to life under Art. 21 would include right to live with human dignity. Due to want of
facilities and opportunities, the right to residence and settlement is an illusion to
the rural and urban poor. The directive principles positively charges the State to
distribute its largess to ihe weaker sections of society (envisaged in Art 46) to
make socioeconomic justice a reality, meaningful and fruitful so as to make life
worth living with dignity of person and equality of status
XVI
CIn
o nShiv
s t i tSagar
ution a l L va w
Tiwari o f ofI nIndia
Union d i a (AIR
- I I 1997 SC 2725), the Supreme
Court while recognizing the ‘right to shelter’ emphasised the need to prevent or
regulate “illegal shelter". The Court discussed the issues like, should there be
any out-ofturn allotment of Govt, residential quarters and if so how should it be
regulated and what should be its limits; should private citizens (viz. journalists,
freedom fighters, artists, etc.) be accommodated in government quarters; should
political parties and other organisations be given allotment of Govt.-quarters;
should President, Prime Minister be accommodated in Govt, quarters after
demitting of office by them; etc.

7
Right to Speedy Trial
In Raj Deo Sharma v State of Bihar (1998) 7 SCC 507, the Supreme Court held
that the legal position adumbrated by it (5-Judge Bench) in Abdul Rehman
Antuiay v R.S. Nayak (1992) 1 SCC 225, that the "right to speedy trial” flows
from Art 21 and encompasses the stages right from the date of registration of the
FIR and onwards, remains unaltered. The court fixed a time-limit in this regard,
to dispose of cases in that time, so that the accused persons do not languish in
jails, in “Common Cause" A Registered Society v Union of India (1996) 6 SCC
775, the Apex Court also laid down some guidelines.
In Raj Deo Sharma (II) v State of Bihar (1999) 7 SCC 604, a 3-Judge
Bench laid down the exact period after which an accused must be discharged or
acquitted. The matter was reconsidered by a 7-Judge Bench in P. Ramchandra
Rao v State of Karnataka (2002) 4 SCC 578 and it was decided that:
(a) The dictum in Antuiay is correct and still holds good.
(b) Fixing the period of limitation cannot be done by judicial directive.
(c) The guidelines laid down in Antuiay are not exhaustive but only
illustrative. Their applicability would depend on the facts of each case
The delay has to be decided by the courts having regard to the totality
of circumstances.
(d) The decision of smaller benches fixing a time-limit were overruled.
Comments - In the last two decades or so the judiciary has acted as the
watchdog of under trials (especially poor). However, the judiciary has avoided,
and very rightly, laying down any straight-jacket formula to be applied in all
cases of delayed

7. X was sentenced to death but his death sentence was not executed for
ten years. X contends that to take away his life after keeping him in jail for
ten years entitles him to demand the quashing of his death sentence
under Art. 21 of the Constitution. Examine X’s contention [/.AS.-97]

trials. Because while determining whether undue delay has occurred, one must have
regard to all the attendant circumstances, including nature of offence, number of
accused and witnesses, the workload of the court concerned, prevailing local conditions
and so on-what is called, the systemic delays. It is neither advisable nor practical to fix
any time-limit for trial of offences.
It is true that it is the obligation of the State to ensure a speedy trial and State
includes judiciary as well, but a realistic and practical approach should be adopted in
such matters instead of a pedantic
P r ot e cone.
t i o nEach
o f Land
i f e every
and Pdelay
e r s odoes
n al Lnot
i benecessarily
rty 155
prejudice the accused. Some delays may indeed work to his advantage. The benefit of
delayed trials should be given only to those accused persons who are not party to it.
as delay is a known defence tactic (since the burden of proving the guilt of the
accused lies upon the prosecution, delay ordinarily prejudices the prosecution).

Right to Legal Aid


Art. 22(1) of the Constitution provides that no person who is arrested shall be denied
the right to consult a legal practitioner of his choice. Further, the State is under a
constitutional mandate (as implicit in Art. 21) to provide free 'legal-aid’ to an indigent
or poor person (a detainee or an accused person). The right of the arrested person to
have a counsel of his choice is fundamental and essential to fair trial.
In Hussainara Khatoon v Home Secy., State of Bihar (1980) 1 SCC 98, the
Supreme Court observed: “The right to free services is dearly an essential ingredient
of ‘reasonable, fair and just' procedure for a person accused of an offence and it must
be held implicit in the guarantee of Art. 21.”
In a later decision in Khatri (ii) v State of Bihar (1981) 1 SCC 627, the Supreme
Court further clarified that the State cannot avoid its constitutional obligation to provide
free legal services to indigent accused persons by pleading financial or administrative
inability. The court held that the constitutional obligation to provide free legal services
to indigent accused does not arise only when the trial commences but also attaches
when the accused is for the first time produced before the magistrate. Because, it is
at that stage that he gets the first opportunity to apply for bail and obtain his release
as also to resist remand to police or jail custody.
It has been held that the constitutional right of legal aid cannot be denied even
if the accused failed to apply for it. It Is now therefore clear that unless refused, failure
to provide legal aid to an indigent accused would vitiate the trial [Suk Das v U.T. of Ar.
Pradesh (1986) 2 SCC 401]. Further, the accused persons who, because of peculiar
circumstances and security restraints, cannot have free access to the outside world,
can avail of the legal aid e.g. in the Indira Gandhi Assassination Case, the assassins
were provided with all legal aid and the lawyer of their choice. Similar would be the
case of accused terrorist.

V
Right to Clean Environment
38
In recent times, the judiciary in India has extended to new dimensions, the
concepts 'right to life’ and 'procedure established by law’ in Art. 21. The Supreme
Court, in several cases, interpreted the right to life and personal liberty to include
the right to a wholesome environment. The High Courts of the States like
Rajasthan, Himachal Pradesh and Kerala too, have observed that environmental
degradation violates the fundamental right to life
In Dehradun Quarrying Case (AIR 1988 SC 2187), the Supreme Court
evolved a new right to environment without specifically mentioning it. The case

38 Does the right to life and personal liberty in Art. 21 embraces a right to
clean, healthy and livable environment, using environment in its broadest
sense as interpreted in contemporary constitutional jurisprudence? Discuss
with reference to decided cases. [I.A.S.-96/2000\[L.C.II-
200S\ i
X Vwas filed under C Art.
I
o n32
s tof
i t the
u t i Constitution
o n a l L a wand
o forders
I n d i were
a - I Igiven with emphasis
on the need to protect the environment. According to a committee of experts
appointed by the court, mining of limestone in certain areas was found
dangerous and damaging ecological balance.
Similarly, while interpreting Art. 21, in Ganga Pollution (Tanneries) Case
(AIR 1988 SC 1037), Justice Singh justifying the closure of polluting tanneries
observed:' “We are conscious that closure of tanneries may bring unemployment,
loss of revenue, but life, health and ecology have greater importance to the
people"
Also in Shriram Gas Leak Case (AIR 1987 SC 1086), the court evolved the
principle of absolute liability of compensation through interpretation of the
constitutional, provisions relating to right to live and to the remedy under Art. 32
for violation of fundamental rights. The premises on which the decision is
rendered is clear and unambiguous - the fundamental right to a clean and
healthy environment The court said that the State had power to restrict
hazardous industrial activities for the purpose of protecting the right of the people
to live in a healthy environment. Further, the right to live contains the right to
claim compensation for the victims of pollution hazards.
While the apex court was reluctant for a short period to confer specifically
a right to a clean and humane environment under Art. 21 of the Constitution,
various High Courts in the country went ahead and enthusiastically declared that
the right to environment was included in the right to life concept in Art 21. In
comprehending the right to environment, the High Courts were more specific and
direct. In T. Damodhar Rao v Municipal Corpn. of Hyderabad (AIR 1987 A P.
171), the court held that the enjoyment of life and its attainment and fulfillment
guaranteed by Art. 21 embraces the protection and preservation of nature’s gifts
without which life cannot be enjoyed. There can be no reason why practice of
violent extinguishment of life alone should be regarded as violative of Art 21. The
slow poisoning by the polluted atmosphere caused by environmental pollution
should also be regarded as amounting to violation of Art. 21.
In Kinkri Devi v State of H P. (AIR 1988 HP 4), the court held that if a just
balance is not struck between development through tapping of natural resources and
the protection of ecology and environment, there will be a violation' of fundamental
rights conferred by Arts. 14 and 21 of the Constitution. In Attakoya Thangai v Union
of India (1990 KLT 580), the Kerala High Court held that right to life in Art 21 contains
the right to have dean water. The right to sweet water, and the right to free air, are
attributes of the right to life, for these are the basic elements which sustain life itself.
The first time when the Supreme Court came close to almost declaring the right
to environment in Art 21 were in 1990, in Chhetriya Pradushan Mukti Sangarsh Samiti
v State of U P. (AIR 1990 SC 2060). Chief Justice Sabyasachi Mukherji observed:
"Every citizen has a fundamental right to have the enjoyment of quality of life and
living as contemplated in Art. 21 of the Constitution." In Subhash Kumar v State of
Bihar (AIR 1991 SC 420), the court held the right to live includes the right of
enjoyment of pollution-free water and air for full enjoyment of life. If anything
endangers or impairs that quality of life in derogation of laws, a citizen has right to
have recourse to Art. 32.
In Bangalore Medical Trust v B S. Muddappa (1991) 4 SCC 54, an open space
which was reserved for pubiic park was allotted to a private person for the purpose of
constructing a hospital The residents of the locality challenged the allotment under
Art. 32. The court held that a nursing home could neither be considered an amenity
nor it could be considered improvement over necessity like a public park. A park is a
necessity not a mere amenity. For maintaining ecology in urban areas and for clean
environment, open space and park is necessary.
In Vellore Citizen's Welfare
P r ot eForum
c t i o n vo fUOI
L i f(AIR
e a n1996
d P e rSC
s o n2715),
al L i the
be rSupreme
ty 157
Court held that the remediation of the damaged environment is part of the process of
“sustainable development” and as such polluter is liable to pay the cost to the
individual sufferers as well as the cost of reversing the damaged ecology (‘Polluter
pays principle’). A similar view was taken by the Supreme Court in A.P. Pollution
Control Board v Prof. M.V. Nayudu (1999) 2 SCC 718.
In Murli S. Deora v UOI (2001) 8 SCC 765, realizing the gravity of the situation
and considering the adverse effect of smoking on smokers and passive smokers, the
court directed and prohibited smoking in public places and issued direction to •he
Union of India, State Governments and UTs to take effective steps to ensure
Prohibiting smoking in public places.
In Intellectuals Forum, Tirupathi v State of Andhra Pradesh (AIR 2006 SC
1
350), the Supreme Court has held that under Art. 21 and Art. 51-A, it is the
constitutional obligation of the Government to protect and preserve the environment.
In
the name of urban development, the environment (fresh water resources in the
Present case) cannot be destroyed Emphasising the need of sustainable
development, *he court held that the right of shelter is not so pressing need so as to
outweigh al1 environmental considerations.
In Milk Men Colony Vikas Samiti v State of Rajasthan (2007) 2 SCC 413, 'he
Supreme Court held that the ‘right to life’ means clean surroundings which lead to
healthy body and mind. It includes right to freedom from stray cattle and animals in
urban areas.
Comments - The Indian judiciary has shown unprecedented dynamism by expanding
the scope of Art. 21 by including in it ‘right to clean and wholesome environment'’
This feat is remarkable insofar as even some of the developed countries have yet to
achieve such distinction.
Generalist judges have a broader vision of national policies and interests than
the parochial stand of specialist government agencies charged with establishing
large, environmentally disruptive, industrial and developmental projects. Further,
judicial process frequently compels the defendant project authority to increase
environment-related spending. For example, if the location of a project is challenged
in court by an environmental group, the promoters frequently spend more funds on
environmental impact appraisal and pollution abatement, than they otherwise might.
However, it should be remembered that the courts can only respond to the
cases that come before them. They can rarely effect systemic or society-wide
changes in environmental conditions. Judicial action must be seen as only one of
many tools to bring about environmental improvement. In many situations, lobbying,
public education and political action by mobilising people, are far more effective
mechanisms than the bringing of a lawsuit. Nevertheless, there is a great need of the
people like M.C. Mehta who used environmental laws provision to get the court to
take stern action against wilful polluters.

Right to Education [Art. 21A]39


The following Article has been added by the Constitution 86th Amendment Act, 2002
in the Constitution of India (taking into consideration the 165th Report of the Law
Commission of India and the recommendations made by the Standing Committee of

39 Explain how fundamental right to education was evolved. What limitations are
now placed on this right by the Supreme Court? Discuss with reference W
decides cases. [M.S.-S*
X VParliament):
I
Constitutional Law o f India- II
“Art. 21-A: The State shall provide free and compulsory education to all
children between the age of 6 and 14 years in such manner as the State
may, bylaw, determine” (Fundamental Right).40
It casts a duty on the State to provide free and compulsory education to all children
between the age of 6 to 14 years. To implement this right the State will enact
appropriate laws. Education being a concurrent subject laws may be enacted either
by the Union or the States.
In Mohini Jain v State of Karnataka (AIR 1992 SC 1858), the two-judge bench
of the Supreme Court held that every citizen has a ‘right to education’ under the
constitution. The framers of the Constitution made it obligatory for State to provide
education for its citizens. The right to education is concomitant to the fundamental
rights. Thus, right to freedom of speech and expression cannot be fully enjoyed uniess a
citizen is educated and conscious of individualistic dignity. Without education, dignity
of the individual can’t be assured. Art. 21 includes the right to live with human dignity
and all that goes along with it. The 'right to education’ flows directly from the right to
life because of its inherent fundamental importance (in the life of an individual).
The court further observed: The State is under a constitutional mandate to
provide education at all levels and thus establish educational institutions at all levels
(including professional education like medicine, engineering) for citizens (either State
owned or State-recognised). The ‘Capitation fee' (charging amount beyond what is
permitted by law i.e. in excess of prescribed fee) brings to the fore a clear class
basis, and makes the availability of education beyond the reach of poor. Admission of
non-meritorious students by charging capitation fee strikes at the very root of the
Constitutional scheme and our educational system. Education in India has never been
a commodity for sale.
In Unni Krishnan v State of A.P. (“Capitation Fee Case”) (AIR 1993 SC 2178),
the five-judge bench, by 3-2 majority held that admission to all recognised private
educational institutions particularly medical and engineering, shall be based on merit,
but 50% of seats in all professional colleges be filled by candidates prepared to pay a
higher fee. The system devised by us, the court said, would mean correspondingly
more financial burden on affluent students; whereas in the system prevalent in
Andhra Pradesh, the financial burden is equally distributed among all the students, as
a result of which a poor meritorious student often unable to pay the enhanced fee
prescribed by the government for such colleges. This is unjust and violative of Art.
14.
The judgment of the court differs from that in Mohini's case on following
points:
(a) Conferring unconditional and unqualified right to education at all levels to
every citizen involving a constitutional obligation on State to establish
institutions either directly or through State agencies is not warranted by the
Constitution besides being unrealistic and impractical.
The citizens of this country have a fundamental right to education which
flows from Art. 21. This right is, however, not absolute-arid its content and
parameters have to be determined in the light of Arts. 45 and 41. In other
words, every child/citizen has right to free education until he completes the
age of 14 years. Thereafter his right to education is “subject to the limits of
economic capacity and development of the State."

40 Point out the constitutional enactment with respect to “right of education."


[L.C.11-200/1
(b) When the Government grants recognition to private educational institutions it
does not create an Pagency
r ot e c to
t i ofulfil
n o fitsLobligation
ife and P under
e r s othe
n alConstitution,
L i be r t y and 159
there is no scope to import the concept of agency in such a situation.
In T.M.A. Pai Foundation v State of Karnataka (AIR 2003 SC 2086), the
^°nstitution Bench of the Supreme Court overruled the Unni Krishnan case partly, ^as
held that the scheme relating to admission and the fixing of fee were not rrect and to
that extent, they are overruled.
In Ashoka Kumar Thakurv UOI (2008) 6 SCC 1, the court highlighted the
significance of Art. 21A. It observed: The Union of India should appreciate in proper
Con
perspective s t the
that i t u root
t i o ncause
a l L of
aw o f and
social I n deducational
ia- II backwardness is poverty All
efforts have to be made to eradicate this fundamental problem. If we want to really
help the socially, educationally and economically backward classes, we need to
earnestly focus on implementing Art. 21 A. We must provide educational opportunity
from day one. Only then will the casteless/classless society be within our grasp.
Once children are of college-going age it is too late for reservation to have much of
an effect. The most important fundamental right may be Art. 21A; education stands
above other rights, as one’s ability to enforce one’s fundamental rights flows from
one’s education.
The State is obligated under Article 21-Ato implement free and compulsory
education in toto. A 'carrot and stick’ approach i.e. financial incentives for ensuring
school attendance and strict criminal penalties for preventing on not ensuring school
attendance, appears to be the best way to implement Article 21-A.
The court said that a time-frame/deadline is to be set by which Art 21-A is to
be fully implemented. The court gave detailed recommendations: (1) enactment of
comprehensive Central legislation pertaining to compulsory education that penalizes
employers who prevent, and parents who do not ensure that children attend school
regularly, both monetarily and with imprisonment (though importation of practice of
imposing fines upon the children themselves, as followed in U.S., cautioned against);
(2) provision of financial incentives to poor families for regular school attendance by
their children, and imposition of criminal penalties on those parents availing the
incentives but not sending children to school; (3) continual and consistent increases
in budgetary allocation for primary and secondary education and provision of material
and infrastructural support by State until full compliance with Art. 21-A is achieved;
and (4) ensuring that quality education is provided.
The court emphasised that the giving of financial incentives for ensuring
school attendance was the most important aspect of implementing Art. 21-A, given
that in India the single most important reason why poor parents did not send children
to school was that without the children working there was not enough to eat for the
family.
Comments - Right to education, like right to work, is of inherent fundamental
importance. In fact, the other fundamental rights cannot be enjoyed unless a person
is educated. And the fundamental rights which are justiciable, are often rendered non-
justiciable by a general lack of education among the people.
Education facilitates the participation of the people in making decisions which
affect them. This in turn will strengthen the claim for material basic needs. The courts
in India have made the right to education a fundamental right. But, would this right be
of any real significance when there is a lack of even basic education^ facilities in the
country. Further, the standard of education is not uniform in the country. A student
from a government school often feels himself less educated comparison to a student
from a private school. Thus, a right to education should imply a right to quality
education.

m
Right to Medical Aid and Health
In Parmananda Katara v UOI P r ot(AIR
e c t i o1989
n o f SC
L i f e2039),
a n d Pit ewas
r s o nheld
al L ithat
be r tity is the 161
professional obligation of all doctors (government or private) to extend medical aid to
the injured immediately to preserve life without legal formalities to be complied with
by the police. Art. 21 casts the obligation on the State to preserve life. It is the
obligation of those who are in charge of the health of the community to preserve life
so that the innocent may be protected and the guilty may be punished. No law or
State action can intervene to delay the discharge of this paramount obligation of the
members of the medical profession.
In Paschim Bang Khet Mazdoor Samiti v State of W.B. (1996) 4 SCC 37, the
court followed the Parmananad Katara’s ruling and held that denial of medical aid by
government hospitals to an injured person on the ground of non-availability of beds
amounted to violation of right to life under Art. 21.
In Consumer Education & Research Centre v Union of India (AIR 1995 SC
922), the Supreme Court has held that the right to health and medical care is a
fundamental right under Art. 21 as it is essential for making the life of the workmen
meaningful and purposeful with dignity of person. ‘Right to life' in Art. 21 includes
protection of the health and strength of the worker. The expression life’ includes right
to livelihood, better standard of life, hygienic conditions workplace and leisure and
traditional and cultural heritage of a person. The court laid down some guidelines to
be followed by all asbestos industries - to make health insurance of worners, worker
suffering from occupational health hazard entitled for compensation of Rs.1 lakh,
maintenance of health record of every worker, etc.

Right to Die41
Is individual capable of taking decis'on to end his life? This question was before the
Bombay High Court in case of State of Maharashtra v Maruti Sripati Dubai (1987,
Cr.L.J. 549). The court held that the right to live guaranteed by Art. 21 includes also a
right to die. The judges felt that the desire to die is not unnatural but merely abnormal
and uncommon. They listed several circumstances in which people may wish to end
their lives, including disease, cruel or unbearable condition of life, a sense of shame
or disenchantment with life They held that everyone should have the freedom to
dispose of his > ? and when he desires.
In Rathinam v Union of India (1994 VOL II No. 7674, the Supreme
Court held that a person has a right to die and declared unconstitutional Sec. 309,
1
PC., which makes attempt to commit suicide a penal offence. The right to live in Art.
21 includes the right not to live i.e. right to die or to terminate one’s life. A person
can’t be forced to enjoy right to life to his detriment, disadvantage or disliking. The
court made it clear that an act of suicide couldn’t be said to be against religion,
Morality or public policy and an act of attempted suicide has no baneful effect on

[/.AS.-95]

41 “Right to life includes the right to commit suicide ’ Examine.


X Vsociety. Further,C suicide
I
o n s t i tor
u t attempt
ional L toa commit
w o f Iitn causes
d i a - I Ino harm to others and,
therefore, the State’s interference in the personal liberty of concerned person is not
called for. But, the court rejected the plea that euthanasia (mercy killing) should be
permitted by law. There is a distinction between an attempt of a person to take his life
and action of some others to bring to an end the life of a third person
In Gian Kaurv State of Punjab (1996) 2 SCC 648, a five judge Constitution
Bench of the Supreme Court overruled the P. Rathinam's case and held that 'right to
life’ under Art. 21 does not include 'right to die’ or ‘right to be killed’. The ‘right to die’
is inherently inconsistent with the ‘right to iife’ as is ‘death with life ’ The Court
accordingly held that Sec. 309 of IPC is not violative of Art. 21. The Court said that
the desirability of bringing about a change (regarding Sec 309) is the function of the
legislature which may by enacting suitable law provide adequate safeguards to
prevent any possible abuse.
The court observed: “Any aspect of life which makes it dignified may be read
into Art. 21 but not that extinguish it and is, therefore, inconsistent with the continued
existence of life resulting in effacing the right itself.” “Right to life" is a natural right
embodied in Art. 21 but suicide is an unnatural termination or extinction of life and
incompatible with the concept of “right to life.” Referring to protagonists of
euthanasia’s view that existence in persistent vegetative state was not a benefit to
the patient of terminal illness being unrelated to principle of “sanctity of life", the court
said that this argument was of no assistance to determine the scope of Art. 21 for
deciding whether the 'right to life’ includes the ‘right to die.’ The court made it clear
that the ‘right to life’ including the right to live with human dignity would mean the
existence of such a right up to the end of natural life. This also includes the right to a
dignified life up to the point of death including the dignified procedure of death. But,
the ‘right to die' with dignity at the end of life is not to be confused with the ‘right to
die’ an unnatural death curtailing the natural span of life.
Comments - 'Right to die' can never be and should not be permitted by law. Every
person has a ‘right to live’ (howsoever unfavourable the circumstances are) until the
last hope of his/her life has not gone. At some point of time in life, one may have to
live a 'forced life’ but that does not mean that one should end his life at that point of
time. Suicide is really a call for help and not a call for punishment.

Right to Know
42
In Stale of U.P v Raj Narain (AIR 1975 SC 865), the court held that the people have a
right to know every public act, everything that is done in a public way, by their public
functionaries. They are entitled to know the particulars of every public transaction in
all its bearing. In S.P. Gupta v Union of India (AIR 1982. SC 149), trie court
recognised the right to know to be implicit in the right to free speech and expression
i.e Art. 19(1)(a).
In Reliance Petrochemical Ltd. v Proprietors of Indian Express Newspapers
Bombay Pvt. Ltd. (AIR 1989 SC 190) the court recognised the right to know as
emanating from the right to life i.e. Art. 21. People at large have a right to know in
order to be able to take part in a participatory development in the industrial life and
democracy. The right puts greater responsibility upon those who take upon the
responsibility to inform.
In Dinesh Trivedi v UOI (1997) 4 SCC 306, the Government of India appointed
a committee to examine the nexus between criminals and politicians, bureaucrats,

42 Assess how far the 'right to know’ is a fundamentaj right under the Constitution
of India. [I.A.S.-89]
media persons and some members of the judiciary. Thus, the Vohra Committee
Report tabled in Parliament.P rAotwrit
e c t ipetition
o n o f Lwas
i f e filed
a n d praying
P e r s o nfor
al direction
L i be r t y to the 163
Government to make public the Report along with its annexure, memorials and
written evidence that were placed before the Committee, to reveal names of all those
against whom there was tangible evidence, etc. Held that Report tabled in Parliament
was genuine, authentic and unabridged and also a 'public document’. The full-scale
disclosure of “supporting material” would be against the public interest, and thus
need not be directed. The Court observed: In modern constitutional democracies, it is
axiomatic that citizens have a right to know about the affairs of the government
which, having been elected by them, seeks to formulate sound policies of
governance aimed at their welfare. However, as no right is absolute, in transactions
which have serious repercussions on public security, secrecy can be claimed
because it would then be in the public interest that such matters are not publicly
disclosed or disseminated.
Comments - The Supreme Court judges have derived the right to know from two
distinct constitutional sources: Art. 19(1 )(a) and Art. 21. There is no specific
enactment in India imposing a duty on the government to supply information to an
individual seeking it. Non-disclosure of information is the norm in India; openness is
the exception. Democracy expects openness and openness is a concomitant of a
free society.
Public access to government information enables citizens to exercise their
political choice meaningfully. Secrecy erodes the legitimacy of elected governments.
The right to be informed of public acts helps check the abuse of executive power.
The right to know also strengthens participatory democracy. Armed with information
on government programmes, citizens may influence decision-making through
representations, lobbying and public debate. The right to know is especially critical in
environmental matters. For example, government decisions to site dams may
displace thousands of people and deprive them of their lifestyles and livelihood.

QUESTIONS AND ANSWERS

Q. 1 Examine critically the scope of 'procedure established by law’ as used in


Art. 21 in the light of decided cases.[LC.//-95; L.C.1-93/94]

In Maneka Gandhi v Union of India (AIR 1978 SC 597). Bhagwati J.


observed: “The concept of reasonableness must be projected in the
procedure contemplated by Art. 21 having regard to the impact of Art
14 on Art. 21.” Do you think the Supreme Court by adopting the above
technique has brought, at least, the procedural due process’ from
America to India through the back door.
[CL C.-95L.C 1-95/2007]

“True our Constitution has no 'due process clause but after Maneka
Gandhi's case the consequence is the same." Discuss. [/.AS.-97]

Critically examine the statement that though Maneka Gandhi case has
made a great advancement towards understanding and application of
the Fundamental Rights in our Constitution, it has not been able to
make much difference either to the preventive detentions or towards
ensuring the due process of law. [I.A S.-2001]
A. 1 Article 21 says that: "No person shall be deprived of his life or personal liberty
except according to the procedure established by law.”
Scope of ‘procedure established by law ’ - The term procedure established by law' is
a narrower term in comparison to the term ’due process of law’, because the former
X Vincludes only procedural
I
C o n s t i tlaw (C.P.C.,
utio n a l L aCr.
w P.C.
o f Ietc.)
n d i while
a - I I the latter includes both
substantive law (I.PC., Hindu Marriage Act, etc.) and procedural law The phrase ’due
process of law’ is used in the American Constitution.
The U.S courts have gradually adopted a liberal view and held that if a law is
not just, fair or reasonable then the requirement of due process is not satisfied. Due
process is not limited to procedural safeguards. So in the U.S. it protects the
individual from the legislature as well as the executive
Pointing out the difference in the use of words our Supreme Court held in
Gopalan that the difference is deliberate and material In effect, Art. 21 only requires
that there must be a valid law. The court will not examine the content of the law and
will not judge its fairness or reasonability. This view is supported by the history of the
Article. In the draft Constitution the clause «vas as under. 'No person shall be
deprived of his life or liberty without due process of law' After discussion the
expression ‘without due process of law’ was substituted by the expression 'except
according to procedure established by law.’
In Maneka Gandhi case, Bhagwati J. did not applied the concept of ‘due
process” as such in the context of Art 21. when he stated that the protection secured
by Art. 21 is a limited one in as much as Art. 21 safeguards the right to 'personal
liberty’ against executive interference which is not supported by law, and law here
means ‘enscted law’ or ‘State law’. However, he read ‘procedure’ in Art. 21 not as
prescribing any specific procedure, but only that procedure which conforms to the
principles of natural justice i.e. procedure has to be ‘fair, right and just'. It was
explained that the principle of reasonableness which is an essential element of
equality pervading Art. 14, must also apply with equal force to the procedure’
contemplated by Art. 21, so procedure must not be arbitrary, or oppressive. Thus
‘due process' has been imported and established as a part of the Constitution.
Further, Krishna Iyer J., in his separate but concurring judgment, extended the
guarantee in Art. 21 not only to ‘procedure’ as being fair and just, but also to 'law' as
being ‘reasonable’, it meant in a way reading 'procedure established by law’ as
almost amounting to ‘due process of lav/ in the American sense. The change in the
judicial attitude may be attributed to the experience gained during the emergency
imposed in 1975.
In Sunil Batra v Delhi Administration (AIR 1978 SC 1675), Krishna Iyer, J said:
"True our Constitution has no due process clause, but after Cooper and Maneka
Gandhi, the consequence is the same. Art. 21 is the counterpart of the procedural
due process in the United States."
Art. 22 which provides for preventive detention is not a complete code, and a
law relating to preventive detention must satisfy requirements of Arts. 14, 19 and 21,
thus procedure prescribed under the preventive detention law must be reasonable,
just and fair The principles of natural justice in so far as they are compatible with
detention laws find place in Art. 22 itself.

Q. 2(a) “X" commits an offence of atrocity on a scheduled caste. He claims


anticipatory bail on the ground that anticipatory bail is an essential
ingredient of Art. 21. Will he succeed?

(b) A licencee of video games is prohibited from admitting students in


uniform (during school hours) to the video-games He contends that the
regulation is arbitrary and capricious and thus violative of Arts. 14 and
21. Decide.
A. 2(a) ‘‘X’’ will not succeed. In State of M.P. v Ram Kishan Balothia (AIR 1995 SC
1198), held that anticipatory bail cannot be granted as a matter of right. It is
essentially a statutory right. It cannot be considered as an essential
ingredient of Art. 21PArid
r ot eits
c t non-application
i o n o f L i f e a ntod aPcertain
e r s o n special
al L i becategory
rty of 165
offences (offences of atrocity on SC/ST) cannot be considered as violative
of Art. 21. These offences form a distinct class by themselves and cannot
be compared with other offences.
(b) The restriction is in the public interest of education of students, therefore,
neither unreasonable nor capricious nor violative of Art. 21. In M.J. Sivani v
State of Karnataka (AIR 1995 SC 1770), held that right to life under Art. 21
does protect livelihood, but no one has the right to play with the credulity of
the general public or career of young impressive age school or college
going children by operating unregulated video games.

Q. 3 Explain the meaning of the expression “life’’ in Art. 21 as interpreted by


the Supreme Court. [C.L.C -2006]

In Maneka Gandhi v Union of India. Bhagwati J said: “The correct way


of interpreting the provisions of Part III is that attempt of the court
should be to expand the reach and ambit of the fundamental rights
rather than to attenuate their meaning and content.” Thus, Arts.
19(1 )(a) and 21 have sprung up a whole lot of “human rights
jurisprudence”. Comment with reference to some of the unspecified/
implied fundamental rights. Do you think this is a welcome trend? What
might be the dangers involved in this kind of judicial activism?

“The frontiers of Art. 21 are still expanding and its new dimensions are still
being explored by the courts in view of the concept of ‘dignified life’
guaranteed by Art. 21 seem to be inexhaustible in range and scope." Do
you agree with this? [L.C.1-2006]
A. 3 Implied fundamental rights - ‘Right to life' under Art 21, as interpreted by the
Supreme Court, includes: Right to dignity, Right to privacy, Right to travel
abroad, Right to clean environment, Right to livelihood, Right to shelter,
Right to education, Right to marriage, Right against torture, Right to legal
aid, right to speedy trial, right to know, Right to health and medical care,
Right to food, etc. Some of these are discussed below:
(i) Right to dignity - In Maneka Gandhi v Union of India, the court held that
the right to live is not merely confined to physical existence but it
includes within its ambit the right to live with human dignity. In Francis
Coralie v Union Territory of Delhi, the court held that the right to live
includes the right to live with human dignity and all that goes along
with it, namely, the bare necessities of life, such as adequate nutrition,
clothing and shelter, and facilities for reading and writing, freely
moving about and mixing with fellow human being.
(ii) Right to privacy - In R. Rajagopal v State of T.N., held that the “right to
privacy” is included in Art. 21 and a “citizen has a right to safeguard
the privacy of his own, his family, marriage, procreation, motherhood,
child bearing and education among other matters". In State of
Maharashtra v Madhulkar Narain, it was held that this right is available
even to a woman of easy virtue, and no one can invade her privacy.
Right against “sexual harassment in work places” has been recognized in Vishaka v
State of Rajasthan, and, in Apparel Export Promotion Council v A.K. Chopra.
(iii) Right to livelihood - In Olga Tellis v Bombay Municipal Corpn., the
court held that the world life in Art. 21 includes the right to livelihood
also, because no person can live without the means of livelihood.
(iv) Right to clean environment - The Supreme Court (M. C Mehta cases)
XVI andC oHigh
n s t Courts
i t u t i oinn India
a l L have
a w ointerpreted
f I n d i a the
- I Iright to life and personal
liberty to include the “right to a wholesome environment." In Subhash
Kumar v State of Bihar, the court held the right to live includes the right
of enjoyment of pollution-free water and air for full enjoyment of life.
(v) Right to education - In Mohini Jain v State of Karnataka, held that
every citizen has a ‘right to education' under the Constitution. The right
to education is concomitant to the fundamental rights. Right to
education is implicit in right to life because of its inherent fundamenta1
importance (in the life of an individual). The State is under a constitutional
mandate to provide
P r ot education
e c t i o n o fatLall
i f eleveis
a n dfor
P ecitizens.
r s o n al L i be r t y 167
In Unni Krishnan v State of A.P., held that admission to all recognized private
educational institutions particularly medical and engineering, shall be based on merit
(“free seats”), but 50% of seats in all professional colleges be filled by candidates
prepared to pay a higher fee ("payment seats”). In T.M.A. Pai Foundation v State of
Karnataka, the Constitution Bench of the Supreme Court overruled the Unni Krishnan
case partly. It was held that the scheme relating to admission and the fixing of fee
were not correct and to that extent, they are overruled.
(vi) Right to health and medical cam - In Consumer Education & Research
Centre v Union of India, the Supreme Court has held that the "right to
health and medical care” is a fundamental right under Art. 21 as it is
essential for making the life of the workmen meaningful and purposeful
with dignity of person. ‘Right to life’ in Art. 21 includes protection of the
health and strength of the wor ker.
(vii) Right to die - In Rathinam v Union of India, the Supreme Court held
that a person has a right to die and declared unconstitutional Sec.
309,1.P.C., which makes attempt to commit suicide a penal offence.
The right to live in Art. 21 includes the right not to live i.e. right to die. A
person can't be forced to enjoy right to life to his detriment,
. disadvantage or disliking.
In Gian Kaur v State of Punjab, a five judge Constitution Bench of the Supreme Court
overruled the P. Rathinam's case and held that ‘right to life’ under Art. 21 does not
include ‘right to die' or ‘right to be killed’. The ‘right to die’ is inherently inconsistent
with the ‘right to life’ as is ‘death with life’. The Court accordingly held that Sec. 309 of
IPC is not violative of Art. 21.
(viii) Right to food - In PUCL v UOI, recognizing the ‘right to food’ the
Supreme Court held that the people who are starving because of their
inability to purchase food grains (viz. aged, infirm, disabled, destitute
men/women, pregnant and lactating women and destitute children)
ought to be provided the same free of cost by the States out of surplus
stock lying with the States.
(ix) Right to marriage - In Lata Singh v State of U.P., it was held that the
right to life includes ‘right to marriage’ A major boy or girl undergoing
inter-caste or inter-religious marriage are to be protected from
harassment, threats or acts of violence. Stem action should be taken
against person who give threats or harass or commit violence.
Second part of question - This is a welcome trend, it would certainly be able to
ir,
culcate a sense of accountability in public authorities discharging public duties
towards the people and particularly towards the weaker sections of society. The new
dispensation has produced astonishing results unthinkable a decade ago. degraded
bonded labourers, tortured prisoners, humiliated inmates of protective Hornes have
been liberated through court orders.
However, it must be realised that in the judicial crusade for enlarging justice the
justices inevitably engage themselves in the delicate task of mediating between social
actualities and social change. And ignorance in this regard may result in mass production
of rights and entitlements arousing very high expectations many of which (e.g. right to
work, education, etc.) cannot easily be fulfilled given the level of political and economic
development in the country. The overload of judicial commitments has resulted in massive
inflow of justice seekers demanding relief from ali kinds of miseries and distresses.
Nevertheless, the social effects of the symbolic presence of liberative judicial
messages are far more important than the direct effect of the pronouncements on affected
parties. The educational effects produced by the decisions might regulate the relations
between
XV I the jail C
authorities
o n s t i t uand
t i o prisoners,
n a l L a wcreditor-master
o f I n d i a - and
I I the bonded labour, the
principal and inmates of protective homes, municipal commissioner and pavement
dwellers, and so on.
We should, therefore, look to the courts as symbolic transmitters of the normative
images of a legitimate society dramatizing the significance, dignity and seriousness of
human dignity and liberty. This new trend of interpreting the provisions of the Part III of the
Constitution would go a long way in protecting fundamental right of citizens and securing a
social order where individual shall be free from inhuman and barbarous treatment.

Safeguards against Arbitrary Arrest and


Detention
(Article 22) ARTICLE 22 [SAFEGUARDS AGAINST ARBITRARY
43
ARREST AND DETENTION]

Article 22 prescribes the minimum procedural requirements that must be included in


any law enacted by the legislature in accordance with which a person may be deprived
of his life and personal liberty. Art. 22, thus, sets out certain limitations upon the
powers of the legislature. If a law contravenes the conditions or limitations prescribed
by Art. 22 the law would be a nullity. Art. 21 has to be read as supplemented by Art.
22.
Art. 22 is not a complete code, and a law relating to ordinary arrest or a law
relating to preventive detention must satisfy requirement of Arts. 14, 19 and 21, thus
Procedure prescribed under the ordinary law/preventive detention law must be
reasonable, just and fair. Art. 22 is designed to give protection against the act of
executive or order of non-judicial authorities and applies to a person who has been
a
ccused of a crime or of offence of criminal or quasi-criminal nature or some act
Prejudicial to the State or public interest.

(A] Rights of Arrested Person under Ordinary Law


an
Clauses (1) and (2) - guarantees four rights to the persons who are arrested under
ordinary law:
(a) the right to be informed 'as soon as may be’ of ground of arrest,
(b) the right to consult and to be represented by a lawyer of his own choice,

’ Write a short note on: Constitutional protection against arrest and detention.
lL.C.II-2006/2007]
Constitutional Law o f India- II

(c) the right to be produced before a Magistrate within 24 hours of his


arrest (excluding the time of journey), and
(d) no detention beyond 24 hours except by the order of Magistrate
Rights guaranteed under clause (1) and (2) are available to both citizens and
non. citizens.
Clause (3) - provides two exceptions to rule contained in clause (1) and (2) It says
that rights under clause (1) and (2) are not available to (a) an enemy alien, and,
(b) a person detained under a Preventive Detention law. An enemy alien may,
however, seek the protection under clauses (4) and (5) of Art. 22 if arrested under
a law of preventive detention, but subject to the law made by the Parliament.
In Joginder Kumar v State of U P (1994) 4 SCC 260, the Supreme Court has
laid down guidelines governing arrest of a person during the investigation. The
court held that a person is not liable to arrest merely on the suspicion of
complicity (involvement) in an offence. There must be some reasonable
justification in the opinion of the police officer effecting the arrest that such arrest
was necessary. The court laid down the following guidelines:
(1) An arrested person being held in custody is entitled to have one friend,
relative or other person who is known to him or likely to take an
interest in his welfare told as far as is practicable that he has been
arrested and where he is being detained.
(2) Police Officer shall inform the arrested person, when he is brought to
police station, of this right.
(3) An entry shall be required to be made in the police diary as to who was
informed of the arrest.
In D. K. Basu v State of W. B (AIR 1997 SC 610), the Supreme Court took
serious note of custodial violence and death in police lock-up. It laid down
various guidelines to be followed in all cases of arrest or detention.

Arts. 21-22 and Police Atrocities: Custodial Violence

LEADING CASE: D.K. BASU v STATE OF W .B.


(AIR 1997 SC 610)
In this case, the Supreme Court took a serious note of custodial
violence and death in police lock-up. The matter was brought before
the court by Dr. D.K. Basu, Executive Chairman of the Legal Aid
Services, a NGO, W.B through a PIL. He addressed a letter to the
Chief Justice drawing his attention to certain news items published in
the newspapers regarding deaths in police lock-ups and custody. This
letter was treated as a writ petition by the Court.
The Supreme Court (Kuldip Singh and Dr. A S. Anand J J.)
observed as follows:-
(i) Custodial violence (torture, rape, death in police custody/ lock-up)
is a matter of deep concern. It infringes Art. 21 as well as basic
human rights and strikes a blow at rule of law. It is aggravated by
S af e g u ar d s ag a i ns t A r bi t r a r y A r r e s t a n d D e t e n t i o n 1 7 9

the fact that it is committed by persons who are supposed to be


protectors of the citizens, in the four walls of a police station or
lock-up, the victim being totally helpless.
(ii) The word torture today has become synonymous with the darker
side of human civilisation. Torture involves not only physical
suffering but also mental agony. It is a naked violation of human
dignity and destructive of human personality.
(iii) The precious right guaranteed by Art. 21 cannot be denied to
convicts, under trials, detenus and other prisoners in custody,
except according to the procedure established by law. Any form of
torture or cruel, inhuman or degrading treatment would fall within
the inhibition of Art. 21, whether it occurs during investigation,
interrogation or otherwise.
(iv) The right to interrogate the detenus, culprits or arrestees in the
interest of the nation, must take precedence over an individual’s
right to personal liberty. The Latin maxim satus poputi suprema
lax (safety of the people is the supreme law) and salus republicae
suprema lax (safety of the State is the supreme law) co-exist and
lie at the heart of the doctrine that the welfare of an individual
must yield to that of the community. The State’s action, however,
must be “right, just and fair". Using any form of torture for
extracting any kind of information would neither be right nor just
nor fair. State terrorism is no answer to combat terrorism A
balanced approach is needed to meet the ends of justice (Society
expect that police must deal with the criminals in an effective
manner and bring to book those who are involved in the crime).
(v) To check the abuse of police power, transparency of public action
and accountability are two possible safeguards. Further, the
police force needs to be infused with basic human values and
made sensitive to the constitutional ethos. With a view to bring in
transparency, the presence of counsel of the arrestee at some
point of time during the interrogation may deter the police from
using third-degree methods.
The Supreme Court laid down the following guidelines (as preventive measures)
to be followed in all cases of arrest or detention till legislative measures are
taken:-
XVI
Constitutional Law o f India- II

(a) The police personnel carrying out the arrest and handling the
interrogation should bear an accurate and clear identification and
name tags with their designations. The particulars of such police
personnel must be recorded in a register
(b) The police officer carrying out the arrest shall prepare a “memo of
arrest” at the time of arrest and such memo must be attested by at
least one witness (a member of arrestee's family or a respectable
person of the locality from where arrest is made). It shall also be
countersigned by the arrestee and shall contain the time and date
of arrest.
(c) A person who has been arrested or detained shall be entitled to
have one friend/relative/other person known to him or having
interest in his welfare being informed as early as possible.
(d) The time, place of arrest and venue of custody of an arrestee must
be notified by the police where the next friend/relative of arrestee
lives outside the district/town through the Legal Aid Organisation in
the District and the police station of area concerned telegraphically
within a period of 8 -12 hour s after the arrest
(e) The arrestee must be aware of his right to have someone informed
of his arresVdetention as soon as he is put under arrest/detained.
(f) An entry must be made in the diary at the place of detention
regarding the arrest of the person which shall also disclose the
name of the next friend of the arrestee who has been informed of
the arrest and the name and particulars of the police officials in
whose custody the arrestee is.
(g) The arrestee should, where he so requests, be also examined at
the time of his arrest and major and minor injuries, if any present
on his/her body, must be recorded at that time. The "Inspection
Memo” must be signed by the arrestee and the police officer and
its copy provided to arrestee
(h) The arrestee should be subjected to medical examination by a
trained doctor every 48 hours during his detention in custody.
(i) Copies of all the documents including the memo of arrest should
be sent to the area Magistrate for his record
(j) The arrestee may be permitted to meet his lawyer during
interrogation, though not throughout the interrogation
The requirements, referred to above, flow from Arts. 21 and 22(1) and need to
be strictly followed. These requirements are in addition to the constitutional and
statutory safeguards.]

Safeguards against Arbitrary Arrest and Detention | 73

[0] Preventive Detention Laws


44

44
clauses (4) to (7) of Art 22 provide the procedure which is to be followed if a p erson
is arrested under the law of Preventive Detention.
Preventive Detention means detention of a person without trial. There is no
3uthoritative definition of the term “preventive detention” in Indian law. The word

•preventive” is used in contradistinction to the word “punitive”. It is not a punitive but a


preventive measure. It is a precaution taken by the executive. While the object of the
punitive detention is to punish a man for having done something, the object of
preventive detention is to intercept him before he does it and to prevent him from
doing it- No offence is proved nor is any charge formulated. The sole justification
of such detention is suspicion or reasonable probability of the detenu committing
s0me act likely to cause harm to society or endanger the security of the

Government, and not criminal conviction which can only be warranted by legal
evidence.
Thus, 'Preventive detention’ is the detention of a person where the evidence
against him is not sufficient for his conviction by a court but is sufficient to justify his
detention for reasons of defence, foreign affairs, security of State, maintenance of
public order, and, maintenance of supplies and services essential to the
community. Thus, the Preventive Detention Act, 1950 was passed with the
aforesaid objects in view.
Preventive detention laws are repugnant to democratic Constitutions.
Preventive detention laws are not found in any of the democratic countries of the
world. It was resorted to in England during the war time. The Indian Constitution,
however, recognises preventive detention in normal times also [A.K. Gopalan v
State of Madias (1950) S.C.R. 88],
India is a peculiar country having a multiethnic, multi-religious and
multilingual society. Caste and communal violence is still a common feature in
Indian society. In recent times, terrorist and anti-national activities are on the
increase. Thus, various preventive detention laws have been enacted e.g.
Preventive Detention Act, 1950 (Repealed); MISA (Maintenance of Internal
Security Act, 1971) (Repealed); Prevention of Black-marketing and Maintenance
of Supplies of Essential Commodities Act, NSA (National Security Act, 1980);
COFEPOSA (Conservation of Foreign Exchange and Prevention of Smuggling
Activities Act, 1974); TADA, POTA (Prevention of Terrorism Act, 2002); LiAPA
(Unlawful Activities (Prevention) Act; etc.
Anti-terror laws like TADA and POTA were rolled back after complaints of
a
buse Amendments to provisions of the UAPA have been suggested to strengthen

Are provisions relating to preventive detention ‘undemocratic and harsh”?


Discuss constitutional safeguards available to a detenu under preventive
detention law, especially his processual rights in proceedings before Board.
[L.C.II-95; I.A.S.-93]
Preventive detention is the only exception carved out to the generally
accepted Proposition that no person shall be deprived of his personal liberty
except in accordance with the procedure established by law.'' Discuss the
safeguards evolved by the judiciary for the protection of personal liberty of a
person detained under the law of Preventive Detention. [I. A.S.-
2003]
Constitutional Law o f India- II
the legal system against terrorist acts. Some of the proposed amendments
to UAP/\ curtail rights of citizens guaranteed by the Constitution. The National
Investigation Agency (NIA) Bill, 2008 calls for establishing a Federal Agency with
exclusive powers to investigate terror crimes.
In T.P. Moideen Koya v Govt, of Kerala (2004) 8 SCC 106, it was held that in
law there is no bar to passing a detention order even against a person who is
already in custody if the detaining authority is subjectively satisfied from the
(cogent) material placed before it that there is an imminent possibility of such a
person being released.

Constitutional Safeguards against Preventive Detention Laws


The preventive detention laws because of their nature are capable of being
abused by authorities. The abolition of TADA and POTA is a case in the point.
Thus, safeguards against these laws are necessary to mitigate their harshness,
by placing fetters on legislative power conferred on the Legislature. It is for this
reason that Art. 22 have been given a place in the chapter on “guaranteed rights”.
Clauses
(4) to (7) provide the following safeguards (which are available even to enemy
aliens):
(a) Review by Advisory Board.
(b) Composition and procedure of Advisory Board.
(c) Grounds of Detention and Representation.

(a) Review by Advisory Board


Clause (4) (prior to 44th Amendment) - No law providing for preventive detention
shall authorise the detention of a person for a longer period than 3 months unless,
(a) an advisory board consisting of who are, or have been, or are qualified
to be appointed as Judges of a High Court has reported before the
expiration of said period of 3 months that there is in its opinion sufficient
cause for such detention.
(b) such person is detained in accordance with the provisions of any law
made by parliament under (a) and (b) of clause (7).
Clause (4) (after 44th Amendment) - The Amendment reduced the period from 3
months to 2 months i.e. period for which a person may be detained without
obtaining the opinion of Board. The Amendment made the Advisory Board an
independent body i.e. free from executive control. The Amendment abolished the
provision for preventive detention without reference to an Advisory Board 3S
provided in sub clause (a) of clause (7).
Clause (7): (a) Parliament may by law prescribe the circumstances under which a
person may be detained for a period longer than 3 months, without obtaining the
opinion of an Advisory Board [Sub clause (a) now deleted].
S af e g u ar d s ag a i ns t A r bi t r a r y A r r e s t a n d D e t e n t i o n 1 7 9
(b) Parliament may by law prescribe the maximum period for which a person
may be detained under any law providing for preventive detention.
(Note: 44th Amendment has not yet been brought into force.)
If the Advisory Board reports that detention is not justified the Government is duty
bound to revoke the detention order. If the Board reports that detention is justified,
then only the detaining authorities determine the period of detention, however the
Board cannot express any opinion as to how much longer than 3 months the
detenu should be kept in detention. If the Board fails to submit report before
expiration of said 3 months, the detention would become illegal.

(b) Composition and Procedure of Advisory Board


The Constitution of the Advisory Board has to be in accordance with the
recommendation of chief justice of the appropriate High Court. The chairman of
the Board has to be a serving judge of High Couit and other members have to be
serving or retired judges of High Court.

(c) Grounds of Detention and Representation


Clause (5) of Art 22 gives two rights to the detenu:
(a) the authority making the order of detention must “as soon as may be”
communicate to the person detained the grounds of his arrest i.e. the
grounds which led to subjective satisfaction of the detaining authority,
(b) to give the detenu “the earliest opportunity” of making a representation
against the order of detention i.e. to be furnished with sufficient
particulars to enable him to make a representation.
Art. 22(5) cast a duty on detaining authority to inform the detenu while serving the
order of detention that he has a right to make representation against the order of
detention and also a right to be heard by the Advisory Board. The 'materials and
documents’ relied on in the order of detention must be supplied to the detenu
along with ‘grounds’. It is important to note that the grounds of detention must be
existent a< the time of making the order. No part of such ground can be held back
nor can new ground be added thereto (State of Bombay v Atma Ram AIR 1954
SC 157).
In Jaynarairt Sukul v State of W.B. (AIR 1979 SC 675), Court has
enunciated Allowing four principles to be followed in regard to the representation
of detenu: (i) l,le authority is bound to give an opportunity to detenu to make
representation and to ^sider it as early as possible, (ii) the consideration of
representation by authority is entirely independent of any action by Advisory
Board, including the consideration of ^Presentation by the Board, (iii) there should
not be any delay in matter of consideration, ' lv) the appropriate Government is to
exercise its opinion and judgment on the ^Presentation before sending the case
along with detenu’s representation to Board.
. In Kamlesh Kumar Ishwardas Patti v Union of India (1995) 4 SCC 51, it has 6en
held that where an officer specially empowered by the Central Government nder
the COFEPOSA and PITNDPS Act has passed a detention order, the detenu has
a right to make a representation to the said officer and the said officer is obliged
to consider the said representation and the failure on his part to do so results in
denial of the right conferred on the detenu against detention order. This fight of
the detenu is in addition to his right to make representation to the State
XVI
Constitutional Law o f India- II

Government.

Preventive detention when vitiated


Kamla v State of Maharashtra (AIR 1981 SC 814) - In this case, the detention
order was declared void on the ground that the documents and materials were not
supplied along with the detention order and also there was an unexplained delay
of 25 days in disposing of the representation of the detenu. When the detaining
authority applies its mind to the documents and materials forming the basis of
detention, there could be no difficulty in getting Photostat copies of the same and
attaching the same along with the grounds of detention. However, these
requirements were not being complied with resuiting in the detention orders being
set aside, even though on merits they might have been justified in suitable cases.
The court also expressed great concern about the non-compliance of the
constitutional safeguards contained in Art. 22(5) by the detaining authorities. The
court suggested that whenever a detention order is struck down by the courts, the
authority/officer concerned must be held personally responsible.
State of TN. v Senthil Kumar (AIR 1999 SC 971) - In this case, the detenu was
given some additional material in support of the grounds of detention already
conveyed to him. However, there was no covering letter indicating as to who
furnished them and the purpose for which they were given to him. The Supreme
Court, quashing the detention order, held that the manner in which the documents
were served on the detenu did cause confusion to the detenu as he was kept in
dark about the purpose of furnishing the documents and far from giving him the
earliest opportunity to make an effective representation, it deprived him of the
chance of making representation The principle of natural justice of affording an
opportunity of being heard is embedded in Art. 22(5), albeit opportunity of being
heard is afforded after making the detention order but before confirming the same
as the authority has the power to revoke, cancel or confirm the order. A casual or
random approach in complying with procedural safeguards more often results in
infringement of the safeguard and vitiates the detention.
S M Jahubar Sathik v State of T.N. (AIR 1999 SC 1945) - In this case the detenu
alleged that he had made a representation to the Central Government, but it was
rejected after an inordinate delay of 32 days. The Supreme Court held that the
detention of the detenu is vitiated as representation against it is delayed by the
authorities by seeking unnecessary clarification. The governmental authorities
before disposing of the representation had sought clarification thrice. In fact the
clarification was sought to cover up the delay in prompt disposal of the
representation.
Rajammai v State of T.N (AIR 1999 SC 684) - In this case, held that a representation
under Art. 22(5) should be considered and disposed of at the earliest However, the
Court can consider whether the delay was occasioned due to permissible reasons
or unavoidable causes. It is for the authority concerned to explain the delay; even
longer delay can be explained. In the instant case, merely stating that the Minister
was on tour and hence he could not pass orders is not a justifiable explanation.
Absence of the Minister at the headquarters is not sufficient to justify the delay,
since the file could be reached the Minister with utmost promptitude in cases
involving the vitally important fundamental right of a citizen.
S af e g u ar d s ag a i ns t A r bi t r a r y A r r e s t a n d D e t e n t i o n 1 7 9
Tarannum v Union of India (AIR 1998 SC 1013) - In this case, apprehending that
the detenu (charged of looting gold and cash from a house) would be bailed out
from the prison and he would indulge in several criminal activities, authorities
passed an order of detention against the detenu under the National Security Act,
1980. This order was passed mainly on the ground of above-said act of looting.
The validity of this order was challenged before the Supreme Court on the ground
that a detention order under Sec. 3(2) of the Act could be passed only if the acts
of detenu are prejudicial to the maintenance of ‘public order 1, while an act of
looting relates merely to 'law and order’ problem. The Supreme Court accordingly
quashed the detention order. It observed: “If an act is confined to an individual
without directly or indirectly affecting the tempo of the life of the community, it may
be a matter of ‘law and order' only. But where the granting of the act is otherwise
and likely to endanger the public tranquility, it may fall within the ambit of ‘public
order’, viz., daredevil repeated criminal acts, open shoot-out, throwing bomb at
public places, etc Even certain murder committed by persons in lonely places with
the definite object of promoting the cause of the party to which they belong may
also affect the maintenance of public order."

Exception to Clause (5)


Clause (6) provides that disclosure of facts which are considered to be against
public interest may not be furnished by the authority to the detenu.
Subjective satisfaction of detaining authority - Such satisfaction can be challenged
on the following grounds, namely, mala fide or vagueness, and irrelevant or non-
existent grounds or mechanical application of mind of detaining authority.
However, the recent amendments in NSA have considerably limited the scope of
judicial review on the matter. The amendments provide that the validity of
detention order can't be challenged on ground that one of the several grounds of
detention is vague, or nonexistent or invalid. Thus, the amended Act, 1984
separates each of the grounds for detention and allows further detention of a
person whose earlier detention has either expired or had been removed.

Leading Case: a.k. roy V union of india (AIR 1982 SC


710)

In this case, popularly known as the "NSA (National Security Act) case,”
the Supreme Court by 4-1 majority upheld the constitutional validity of
the NSA and the Ordinance which preceded the Act.
Various arguments advanced on behalf of the petitioners can be
broadly classified under the following heads:
(a) Whether an ordinance is a ‘law’ within the meaning of Art. 21 - The
Court observed that the fact that any particular law has a temporary
character (e.g. ordinance) is immaterial for the purpose of Art. 21,
so long as the procedure prescribed by it is definite and reasonably
ascertainable. In fact, the Preventive Detention laws were in their
inception of a temporary character, since they had a limited
duration.
(b) Vagueness of the provisions of National Security Act - The Court
XVI
Constitutional Law o f India- II

held that Act was neither vague nor arbitrary in its provisions
providing for detention of persons on certain grounds, as acting in a
manner prejudicial to the 'defence of India’, ‘security of India’, and to
‘relations with foreign powers’. As these expressions are not
capable of a precise definition, courts must strive to give these
concepts a narrower construction than what the literal words
suggest. While construing laws of preventive detention like the
NSA, care must be taken to restrict their application to as few
situations as possible.
(c) Fairness and reasonableness of the procedure before Advisory
Board - It was argued that such procedure is unfair and unjust, and
the Act thereby violates Art.s 14, 19 and 21. The question that
whether a preventive detention law violates Arts. 14, 19 and 21, has
been decided in Hardhan Saha v State of W.B. (AIR 1974 SC
2154). In that case, a detenu, under the MISAAct, questioned the
validity of said Act on the grounds that law is unreasonable and
violates Art. 19; the law violates Art 21 because guarantee of a right
to be heard is infringed; and the law does not lay down the just
procedure for giving effect to Art. 22(5). The Court proceeded on
the assumption that preventive detention law may as well be tested
with regard to its reasonableness with reference to Art. 19. But if a
statutory provision excludes the application of any or all the
principles of natural justice (which are applicable under Art. 19),
then the courts do not completely ignore the mandate of the
legislature. The principles of natural justice in so far as they are
compatible with detention laws find place in Art. 22 itself and also in
the Act (i.e. MISA). Even if Art. 19 be examined in regard to
preventive detention, it does not increase the content of
reasonableness to be observed in respect of orders of preventive
detention.
It was argued further, in the present case, that the detenu is entitled to exercise
the trinity of rights before the Advisory Board: (i) the right of legal representation,
(ii) the right of cross-examination, and (iii) the right to
present his evidence in rebuttal. Court observed that the procedure of
Advisory Boards in which the detenu is denied the right of legal
representation before the Board is not unjust or unreasonable. Since the
Constitution itself contemplates that such a right should not be made
available to a detenu [exceptions to clauses (1) and (2) of Art. 22], it can’t
be said that denial of said right is unfair. Moreover, the detaining
authority or Government also do not have the facility of appearing before
the Board with the aid of a lawyer. Court, further, held that detenu has no
right to cross-examine, but has right to present his own evidence in
rebuttal of the allegations made against him.
The Court also made it clear that the question as to whether there
are any circumstances on the basis of which the detenu should be kept
in detention after the Board submits its report and how long, is for the
detaining authority to decide and not for the Board. The Board’s enquiry
is limited to find out as to whether or not there is sufficient cause for the
S af e g u ar d s ag a i ns t A r bi t r a r y A r r e s t a n d D e t e n t i o n 1 7 9

detention of the person concerned.


(d) Unreasonableness and harshness of conditions of
detention - The Court observed that the laws of preventive
detention cannot, by the back-door, introduce procedural
measures of a punitive kind. The court, thus, directed:
(i) that immediately after detention, his kith and kin must be
informed in writing about his detention and place of
detention;
(ii) the detenu must be detained in a place where he
habitually resides, unless exceptional circumstances
require detention at other place;
(iii) that detenu is entitled to his book and writing materials, his
own food, visits from friends and relatives;
(iv) he must be kept separate from those convicted;
(v) no treatment of a punitive character should be meted out
to him and he should be treated according to the civi'ised
norms of human dignity.
As observed in Sunil Batra’s case (AIR 1980 SC 1479), even within the
prison, no person can be deprived of his guaranteed rights, save by
methods which are fair, just and reasonable. The most important right of
the person which is imprisoned is to be integrity of his physical person
and mental personality.]
Attorney General of India v Amrit Lai, (1994) 5 SCC 54, a 9-judge Constitution
en
ch of Supreme Court held that during the period of emergency the President is
6rn
Powered to suspend fundamental rights of people and a detenu has no locus
[a
ndi to question the reasons or grounds of his detention. The court upheld the
allt
% of the two enactments of 1975 period - COFEPOSA and SAFEMA
(Smugglers
XVI
Constitutional Law o f India- II

and Foreign Exchange Manipulators Act), as they were passed to meet the threat
to the security of India and has to be implemented effectively. The court upheld
Sec. 12-A of COFEPOSA which had done away with the requirements of supply
of grounds of detention and the consultation with Advisory Board during the
emergency. By upholding the validity of the above Acts the court has struck a
heavy blow on economic offenders by depriving them of their ill-gotten gams.

QUESTIONS AND ANSWERS

Q. 1 The National Security Act (NSA) provides for 'preventive detention'


for reasons connected with Defence, Foreign affairs. Security of
India or a State, the maintenance of public order, or the
maintenance of supplies and services essential to the community.
A, who is detained on the ground of ‘acting in any manner
prejudicial to the maintenance of supplies and services essential to
the community’, challenges the validity of the Act as well as his
detention .on the ground of violation of Arts. 14, 21 and 22 of the
Constitution. His contentions are:-

(a) Preventive detention is undemocratic and basically impermissible


under the Constitution.

(b) The grounds of detention under the Act are vague; no detention can
be ordered on the said ground unless the law or the government
has notified the “supplies and services" which are considered
essential for the application of the ground.

(c) He is not permitted to be represented by a lawyer friend/law


professor before the Advisory Board.

(d) He is not allowed to cross-examine the detaining authority or the


witness. Also, he is not supplied the name of witness.

(e) Heis not permitted to present his evidence in rebuttal of the


allegations made against him.

(f) He is denied his right to live with human dignity in as much as he


has no access to reading materials, cannot wear his own clothes or
eat his own food and also he is denied visits from friends and
relatives. Rules also authorise the authority to shift him from place
to place.

(g) He was given the grounds of detention after 7 days of detention and
the detaining authority did not give any reasons for the delay. Will
he succeed? Decide. [C.L.C.-
92/94/95J
A. 1 Clauses (4) to (7) of Art. 22 provide the procedure which is to be followed
if a person is arrested under the law of preventive detention. These
S a f e g u a r d s a g a i n s t A r b i t r a r y A r r e s t a n d D e t e n t i o n ] 0]

clauses also provide the safeguards against preventive detention laws:


Review by Advisory Board, Grounds of arrest and representation, and,
Composition and procedure of Advisory Board.
Clause (5) of Art. 22 gives two rights to the detenus: (i) Communication to the
detenu the grounds of his arrest which led to subjective satisfaction of detaining
authority,
(ii) Giving the detenu the earliest opportunity of making a representation against
the order of detention i.e. to be furnished with sufficient particulars to enable him
to make a representation. It is to be noted that the consideration of representation
by detaining authority is entirely independent of any action by Advisory Board
including the consideration of representation by the Board {Jaynarain Sukul v
State ofW.B.). The ‘materials and documents’ relied on in the order of detention
must be supplied to the detenu along with ‘grounds’. Clause (6) to Art. 22,
however, provides that disclosure of facts which are considered to be against
public interest may not be furnished by the authority to the detenu. !t is important
to note that the subjective satisfaction of detaining authority can be challenged on
various grounds, namely, mala fide or vagueness, and irrelevant or non-existent
grounds or mechanical application of mind of detaining authority.
Now, coming to the case in question:-
(a) The law of preventive detention is authorised by our Constitution
presumably because it was foreseen by Constitution makers that there
may arise occasions in the life of nation when the need to prevent
citizens from acting in ways which unlawfully subvert or disrupt the basis
of an established order may outweigh the claims of personal liberty.
Moreover, because of various safeguards available against preventive
detention laws, such laws have found a place in the Chapter on
Fundamental Rights.
In A.K. Roy v Union of India, it was argued that the procedure before Advisory
Board is unfair and unjust, and the Act (NSA) thereby violates Arts. 14, 19 and 21.
The court observed that preventive detention law may well be tested with regard
to reasonableness with reference to Art. 14 or 19. But if a statutory provision
excludes the application of any or all the principles of natural justice (which are
a
pplicable under Art. 14 or 19) then the court does not completely ignore the
mandate of the legislature. The principles of natural justice in so far as they are
compatible with detention laws find place in Art. 22 itself and also in the Act.
(b) In A.K. Roy case, held that NSA was neither vague nor arbitrary in its
previsions providing for detention on grounds, as acting in a manner
prejudicial to the ‘defence of India’, 'security of India’, etc. These
expressions are not capable of a precise definition. However, the
expression 'acting in any manner prejudicial to the maintenance of
essential supplies and services’ is not so vague and the government
must notify which supplies and services are essential to the community.
(c) In A.K. Roy case, held that the right of legal representation before the
Board is not permissible (as contemplated by exceptions to Art. 22(1)
and
XVI
Constitutional Law o f India- II

(2) ). Art. 22(5) gives a right to make a representation against the


detention order, but that right does not carry with it the said right. The
detaining authority also do not have such right. If the Act disallows legal
representation to a detenu, the State also cannot take help of a lawyer.
But the court conferred upon detenu the right to appear through his
‘friend’ who in truth and substance is not a legal practitioner, and is also
not a comrade in profession of the detenu for which he is detained.
Therefore, even in the face of constitutional and statutory denial of legal
representation to a detenu, he is entitled to a common law right of
representation through a ‘friend’ (but that 'friend' should not be a lawyer,
law professor, etc.).
(d) & (e) In A.K. Roy case, the Supreme Court held that a detenu has no right to
cross-examine, but has right to present his own evidence in rebuttal of
the allegations made against him. Regarding right to cross-examination,
the court reasoned that firstly the question before the Board is not
whether the detenu is guilty of any charge but whether there is sufficient
cause for his detention and secondly, the witnesses would be most
reluctant to testify and often it may harm public interest to disclose their
identity [Art. 22(6)]. Thus, a detenu may not be supplied the name of
witnesses.
(f) In A.K. Roy case, the court laid down some guidelines regarding the
conditions of detention:
(i) The detenu must be detained in a place where he habitually
resides, unless exceptional circumstances require detention at
other place. Even in these exceptional circumstances he ought not
to be sent to any far off place which by the very reason of its
distance is likely to deprive him of the facilities to which he is
entitled
(ii) The detenu is entitled to his book and writing materials, his own
food, visits from friends and relatives.
(g) The grounds of detention are required to be communicated 'as soon as
possible' to the detenu. It acts as a check against arbitrary and
capricious exercise of power. And it gives him an opportunity to make
effective representation. The National Security Ad provides that the
detenu will be conveyed the grounds of the detention within 5 days and
in exceptional cases within 10 days of his detention. If there are
exceptional circumstances, the detaining authority is to record its
reasons in writing-
Decision of the case in question - The detenu, A, will succeed. He succeeds on
four grounds - (b), (e), (f), (g) and fails on the three - (a), (c) & (d).
7
Right against Exploitation (Articles 23-24)

ARTICLES 23-24 [RIGHTS AGAINST EXPLOITATION]

Articles 23 and 24 constitute a group under the head ‘Right against Exploitation.'
Exploitation is opposed to the dignity of the individual proclaimed in the Preamble
and to the provisions of Art. 39(e) and (f).
The two rights guaranteed under this head seem to supplement the 'right to
freedom', as the real object of these two rights is nothing more than to protect the
personal freedom of the citizens. One may partially agree with the view of Prof.
K.V. Rao' “Indeed, the Makers had displayed considerable ingenuity in coining a
name for them (Rights against Exploitation), for they confer no right on any one,
nor an enforceable punishment. They ought to have been in Part IV, for the
Parliament has to make a law prescribing a punishment which it could have done
under Arts. 15 and 19(6).”45

Article 23: Traffic in Human Beings, Begar, Forced Labour


Art. 23(1) says: “Traffic in human beings and begar and other similar forms of
forced labour are prohibited and any contravention of this provision shall be an
offence punishable in accordance with law.
(2) Nothing in this article shall prevent the State from imposing compulsory
service for public purposes, and in imposing such service the State shall not make
any discrimination on grounds only of religion, race, caste or class or any of
them."
Article 23(1) prohibits “traffic in human beings and begar and other forced
labour". Traffic in human beings and forced labour militates against human
dignity.

45 ■ J.C. Johari, The Constitution of India - A Politico-Legal Study, pp. 59-


60 (2000).

[183]
XVI
Constitutional Law o f India- II

Art. 23 protects the individual not only against the State but also private citizens.
Under Art. 35, the Pariiament is authorized to make laws for punishing acts
prohibited by this article.
'Traffic in human beings' - It means to deal in men and women like goods such as
to sell or let out or otherwise dispose them off. It includes immoral traffic in women
or girls or subjecting children to immoral or such like practices. For this sake, the
Suppression of Immoral Traffic in Women and Girls Act, 1956 had been put in
operation. The validity of this Act has been upheld by laying down that it is not
inconsistent with the fundamental right to carry on a business, trade or profession
(Shama v State of UP. AIR 1959 All. 57).
Devadasis are also covered under the term “traffic in human beings”.
Though 'slavery' is not expressly mentioned, there is no doubt that the expression
‘traffic in human beings’ would cover it (Dubarv Union of India MR 1952 Cal. 496).
It may be noted that under Sec. 370, IPC, whoever imports, exports, removes,
buys, sells or disposes off any person as a slave shall be punished with
imprisonment.
Children of the prostitutes have a right to equality of opportunity, dignity,
care, protection and rehabilitation so as to be part of the mainstream of social life
(Gaurav Jain v UOI AIR 1997 SC 3021). The case highlights the practice of
prostitution prevalent in the States of A.P., Karnataka and Maharashtra under the
veil of customary practice of Devdasis, Jogins and Venkatasins and its evil effects
Taking note of the relevant provisions of the Constitution, viz. Arts.
23,21,13,14,15,16,38,39(f),46,32,etc., and the provisions of Immoral Traffic
(Prevention) Act, 1956 and Juvenile Justice Act, 1986, the Supreme Court came
to the conclusion that prostitutes have a right to enter the social mainstream and
their children have right to equality of opportunity, dignity, care, protection and
rehabilitation so as to be part of the mainstream of social life without any pre-
stigma attached on them.
The Apex Court, in the above case, made suggestions like: Children of
prostitutes, including ‘child prostitutes’, should be treated as ‘neglected juveniles’
as defined in Juvenile Justice Act and no stigma should be attached to them;
They should be rescued from the “red-light areas” and shifted to “juvenile homes”
for a short stay to relieve them from the trauma they may have suffered, and
thereafter they should be rehabilitated in an appropriate manner; It is State’s
mandatory obligation to establish juvenile homes; and, Union of India and State
Governments were to evolve, in a Ministerial-level conference, procedure and
principles regarding rescue and rehabilitation of the prostitutes for efficacious
enforcing of their fundamental rights and human rights.
‘Begar and other forced labour' - ‘Begar1 means forcing a person to do some work
against his will and that on the basis of non-payment or grossly inadequate
payment. However, this condition shall not apply to a case where forced labour is
a part of punishment as in a prison house or some such work forms part of the
service conditions or agreement.
Under the old zamindari system, the tenants v/ere sometimes forced to
render free service to their landlords. This was called begar. The sagri or hali
system in
7

[185]
186 Constitutional Law o f India- II

g0,ne parts of Rajasthan is an example of forced labour. Under it, a creditor


gives a loan to a debtor on the condition that until the loan is repaid with interest,
the debtor (or any of his family members) is to render labour or personal service
to jfie creditor.
In Kahaosan Thangkhuiv Simirei Shailei (AIR 1961 Manipur 1), the custom
which seemed to prevail in the State was that each householder in a village would
have to offer one day’s free labour to the headman of the village. The custom was
^eid as being violative of Art. 23. It has been held that even if some remuneration
is paid, the labour may be a forced one (PUDR v UOI AIR 1982 SC 1473).
‘Bonded labour1 is a form of forced labour that is forbidden. In the ‘Asiad
case' (,P.U.D.R v UOI AIR 1982 SC 1473), the Supreme Court gave a wide
meaning to the word ‘force.’ Force is not mere physical or legal force but also
force arising from the compulsion of economic circumstances. The person in want
has no choice. He may be compelled to work for a wage less than the minimum.
He may even agree to pay a part of his wages to a middleman.
It may be noted that, as a result of Art. 23, as many as 12 Acts sanctioning
forced labour, under certain circumstances, became void on the enactment of the
Constitution. The Bonded Labour System (Abolition) Act, 1976, has brought
freedom within the reach of many persons who were being forced to work,
alongwith their family in some cases, by contractors.
Compulsory service for public purposes - It may be noted that Art. 23(2) do not
prohibit the State from imposing compulsory service for public purposes e.g.
social or military service in times of emergencies ljj<e war, floods, etc. or assisting
the police in trying circumstances. It has been held that conscription for social
service, as for instance spread of literacy, is a public purpose (Stata v Jorawar
AIR 1953 A.P. 18).
Even when the State undertakes famine relief work it cannot pay less than
the minimum wage. The State cannot take advantage of their helplessness. If it
does so it would be violative of Art. 23 (Sanjit Roy v State of Rajasthan AIR 1983
SC 328).
In State of Gujarat v Hon'ble High Court of Gujarat (1993) 7 SCC 392, held •hat
exaction of hard labour from convicted prisoners is not forced labour, but they are
entitled to equitable wages. Thomas, J. held that if exaction of hard labour is
Without payment of wages or even payment of wages but below the minimum
w
ages, it would amount to forced labour prohibited under Art. 23(1). But protection
Ur
>der cl.(1) is subject to exception contained in cl.(2) of Art. 23 regarding
‘imposition compulsory service by the State for public purpose: Since according to
modern . Inking, main objective of punishment is ‘reformation’, which is a public
purpose, position of hard iabour on the convicted prisoner on payment of
minimum wages °uld serve the public purpose of his reformation and rehabilitation
and hence . °uid be saved under cl.(2). But under-trials, persons sentenced to
simple Prisonment and those detained under a preventive detention Act cannot
be asked manual work. Such persons may be permitted to do any work of their
Right against Exploitation 187

choice.
Wadhwa, J. (partly dissenting) held that putting a convicted prisoner to harg
labour cannot be equated with ‘begar’ or ‘other similar forms of forced labour’ ar^
there is no violation of Art. 23(1); nor ,can it be said to be in the nature of
compulsory service imposed by the State for public purpose within cl.(2). Hence
even no^ payment of any wages for the hard iabour done by the prisoner as part
of sentence of rigorous imprisonment would not be violative of Art. 23. Wages are
payable only under the provisions of the Prisons Act. Though prison reforms are a
must and prisoners doing hard labour are now being paid wages, but the
message must be loud and clear that 'crime does not pay’.

Article 24: Prohibition on Employment of Children


Art. 24 of the Constitution provides that “No child below the age of 14 years shall
be employed to work in any factory or mine or engaged in any other hazardous
employment”. This provision read with Art. 39(e) and (f) (directive principles),
provides for the protection of the health and strength of children below the age of
14 years. Several Acts also give effect to the provisions of the Constitution e.g.
the Employment of Children Act, 1938, the Factories Act, 1948, the Mines Act,
1953, the Child Labour (Prohibition and Regulation) Act, 1986, etc. This is in
keeping with the human rights concepts and United Nations/!.L.O. norms,
international Convention on the Rights of the Ch'ld was ratified by India on 20-11-
1989.
In P.U.D.R. v UOi (AIR 1983 SC 1473), held that under Art. 24 no child
below the age of 14 years can be employed in the construction work even if
construction industry is not specified in the Schedule to the Employment of
Children Act, 1938
In M.C. Mehta v State of T.N. (AIR 1997 SC 609), held that children below
the age of 14 years cannot be employed in any hazardous industry, mines or
other works and has laid down exhaustive guidelines how the State authorities
should protect economic, social and humanitarian rights of millions of working
children. The matter was brought before the Court by way of PIL, regarding the
plight of the children engaged in Sivakasi cracker factories.
The Supreme Court directed setting up of 'Child Labour Rehabilitation
Welfare Fund’ and asked the offending employer to pay for each child a
compensation of Rs.20,000 to be deposited in the Fund.*The Court made it clear
that employer’s liability would not cease even if he would desire to disengage the
child presently employed, and asked the Government to ensure that an adult
member of the child's family geta job in a factory or anywhere in lieu of the child, in
those cases where it would not be possible to provide jobs, the appropriate
Government would deposit Rs.25,000 in the Fund for each child engaged in
factory/mine or any other hazardous employment. 1° the case of getting
employment for an adult, the parent/guardian shall have to withdraw the child
from the job. Even if no employment would be provided, the parent shall have to
see that his child is spared from the requirement of the job as an alternative
188 Constitutional Law o f India- II

source of income - interest income from deposit of Rs.25,000 - would become


available to the child’s family till he continues his studies up to the age of 14 years.
All the above benefit® would cease if the child is not sent for education by
parents.
To start with, the Court said, the work couid be taken up regarding
hazardous empl°yments and then to be followed by comparatively less hazardous,
and so on. The Court identified nine industries where the work could be taken up,
namely - match industry in Sivakasi (T.N.); diamond-polishing industry in Surat
(Gujarat); precious stone polishing industry in Jaipur (Rajasthan); glass industry in
Firozabad, prass-ware industry in Moradabad, hand-made carpet industry in
Mirzapur, lock- making industry in Aligarh (U.R); slate-industry in Mankaour
(Andhra Pradesh); and slate-industry in Mandsaur (M.P), for priority action. In so
far as the non-hazardous jobs are concerned, the Inspector shall have to see that
working hours of child are not more than 4-6 hours a day and it receives
education at least for two hours each day. The cost of education shall be brrne by
the employer.
In Bandhua Mukti Morcha v Union of India (AIR 1997 SC 2218), the court
observed: “The basic cause for child labour being poverty, instead of its total
abolition which will have adverse effect, it should be banned progressively in a
planned manner starling from the most hazardous and intolerable activities like
slavery, bonded labour, trafficking, prostitution, pornography and dangerous forms
of labour, etc. Thus, other simultaneous alternatives to the child should be
evolved including providing compulsory education, health care, nutrient food,
shelter and other means of livelihood with self-respect and dignity of person’’, in
this case, a PIL alleged employment of children aged beiow 14 years in Carpet
Industry in U.P.
The Supreme Court observed: “The child of today cannot develop to be a
responsible and productive member of tomorrow's society unless an environment
which is conducive to his social and physical health is assured to him. Neglecting
the children means loss to the society as a whole. Their employment - either
forced or voluntary - is occasioned due to poverty; exploitation of their childhood
thus (in particular the poor/deprived sections) is detrimental to democracy and
social stability, unity and integrity of the nation”.
The Court further observed: “Various welfare measurers made by
Parliament/ State Legislatures are only teasing illusions and a promise of unreality
unless they are effectively implemented and the ‘right to life’ to the child driven to
labour is made a reality, meaningful and happy. Child labour must be eradicated,
through well- planned, poverty-focussed alleviation, development and imposition
of trade actions in employment of the children, etc.

Right to Freedom of Religion


(Articles 25-28)
Right to Freedom of Religion 189

Secularism in India
46
The Preamble to the Constitution from its inception proclaimed that its purpose is
to secure to all its citizens 'Liberty of thought, expression, belief, faith and
worship.’ This read with Articles 25 to 28 guaranteed equality in the matters of
faith and religion. The 42nd amendment, 1976, tried to make it more explicit by
adding the word ‘secular" in the Preamble. It has not altered the constitutional
position and was perhaps the result of political expediency. This step made the
Constitution look like a party manifesto.
’Secular” has many shades of meanings, in the Constitution it is used in
two different senses. In the Preamble it denotes that the State does not accept
any religion as the basis of State action. The State treats all religions with strict
neutrality. The second meaning of the word secular is worldly or mundane. Art
25(2){a) uses the word in this sense. There it means those activities which are
not essential religious functions but are of a worldly character e.g. investments or
use of the offerings, wages to be paid to the employees of religious institutions.
In India the ethos is Hindu. The Hindus constitute the majority. Respect of
all manners of thought, belief and worship is ingrained. Even without the
constitutional guarantees the minority religions would have enjoyed full freedom.
This is in contrast to our neighbours Pakistan, Bangladesh, Afghanistan and the
middle east Iran, Iraq, Jordan, Saudi Arabia, etc. which being a Muslim majority
country have declared

46 “Indian secularism is neither anti-religious nor is it based on total neutrality


towards religion but is based on equal respect for all religions.” Explain and
illustrate. [I A. S.-
97\
“Secularism is neither anti-God nor pro-God. It eliminates God from the
matters of State and ensures that no one shall be discriminated on the
grounds of religion.” In the light of the above observation discuss the true
import of freedom of religion guaranteed under the Indian Constitution. [/
A.S.-2005]
190 Constitutional Law o f India- II

themselves as Islamic Republic or Kingdom and have by law and practice denied
religious freedom to Hindus.47
‘Secularism’ means that the State shall observe an attitude of neutrality and
impartiality towards all religions. Articles 25 and 26 embody the principles of religious
tolerance that has been the characteristic feature of Indian civilisation from the start
of history. Besides they serve to emphasize the secular nature of Indian democracy
i.e. equal respect to all religions.
A secular State does not mean an irreligious State, it only means that in
matters of religion it is neutral, the State can have no religion of its own, and the State
protects all religions but interferes with none. In a secular State, the State is only
concerned with the relation between man and man; it is not concerned with the
relation of man with God. It is left to the individual’s conscience. If a parson is a
devout Hindu he does not cease to be secular. The word 'secularism', however, is
vague as it might be used as an instrument of unrestrained communalism or bigotry
or even anti-religionism.
In Santosh Kumar v Secy., Ministry of Human Resources Development AIR
1995 SC 293, held that the introduction of Sanskrit language as a subject in the
Central Board of Secondary Education (CBSE) is not against secularism as it is the
‘mother of all Aryan languages." The court said that without learning Sanskrit
language it is not possible to “decipher’ Indian philosophy, culture and heritage. All
the classes of Hindu religion such as Vedas, Puranas, and Upanishads are written
in Sanskrit. The court directed the CBSE within 3 months to make necessary
amendments in the syllabus to make Sanskrit an elective subject for nurturing our
cultural heritage; not conceding this status to Arabic and/or Persian, would not in
any way militate against the basic tenets of secularism.
The court said that “State tolerance of religion, does not make it either a
religious or a theocratic State.” Secularism represents faith borne out of the rational
faculties and it enables to see the imperative requirements for human progress in all
aspects. Secularism is neither ante-God nor pro-God, as it treats alike the devout,
antagonistic and the atheist. In this sense, the Indian Constitution embodies the
concept of positive secularism and has not accepted the American doctrine of
secularism i.e. the concept of erecting a wall of separation between Religion and
State. The concept of positive secularism separates spiritualism with individual faith
(Ramaswami, J. in Bommai case, below).
In S.R Bommai v Union of India J.T. 1994(2) SC 215, held that the
dismissal of the BJP Government in M.P., Rajasthan and H P. in the wake of
theAyodhya incident of Dec. 6, 1992 was valid and imposition of the President’s
rule in these States was not unconstitutional. The court held that 'secularism' is a
basic feature of the Constitution and any State Government which acts against
that ideal can be dismissed by the President. It was held, that in matters of State,

47
B.K Sharma, pp 101-102.
Secularism owes its birth to State sponsored programs against those
denominations which were not acceptable to the State. After prolonged
bloodshed France invented secularism It aimed at keeping the State and
Church apart. Among Hindus there is no Church hence the State was always
secular. In Islam the State and religion are united. Hence barring a few, all
Muslim majority States are proclaimed Islamic States. See Dr. Babasaheb
Ambedkar, Writings snd Speeches, Education Department, Government of
Maharashtra, Vol. 8, pp. 233-234 (1990). In B.K. Sharma, p. 46.
Right to Freedom of Religion 191

religion has no place. No political party can simultaneously be a religious party


and politics and religion can not be mixed. If any party or organisation seeks to
fight the elections on the basis of a plank which has the proximate effect of
eroding the secular philosophy of Constitution, it would certainly be guilty of
following an unconstitutional course of action.
In M. Ismail Faruqui v Union of India (“Ayodhya Case") (AIR 1995 SC 605),
the court observed: “It is clear from the constitutional scheme that it guarantees
equality in the matter of religion to all individuals and groups. The concept of
secularism is one facet of the right to equality woven as the central golden thread
in the fabric depicting the pattern of the scheme in Constitution. Moreover, any
step taken to arrest escalation of communal tension and to achieve communal
accord and harmony can, by no stretch of argumentation, be tgrmed non-secular
or anti-secular. Secularism is a creed of the Indian people embedded in the
ethos”.
In Aruna Royv UOI (AIR 2002 SC 3176), a value based education to
school children based on basis of all religions was challenged as anti-secular. It
was held that study of religions in school education is not against the secular
philosophy of the Constitution. Justice Dharmadhikari said that from the
experience of the working of Constitution for more than 50 years it is clear that
the complete neutrality towards religion and apathy for all kinds of religious
teachings in the State institutions has not helped in removing mutual
misunderstanding and intolerance inter se between sections of the people of
different religions, faiths and beliefs. Secularism therefore is susceptible to a
positive meaning that is developing understanding and respect towards different
religions (“religious pluralism”). The essence of secularism is non-discrimination
of people by the State on the basis of religious differences. Secularism can be
practised by adopting a complete neutral approach towards religions or by
positive approach by making one section of religious people to understand and
respect religion and faith of another section of people.
It may be noted that the insertion of the word ‘secular’ by the 42 nd Amendment
in the Preamble has only served to create confusion despite the Apex Court’s
efforts to clarify the meaning of secularism. The political parties have taken this
opportunity to define their own version of secularism. Some parties think that it is
the duty of the State to conceal truth if it is disliked by a community. Some others
would giv® a sort of veto to minority groups. What ever is not approved by them
is not secular (the Supreme Court has to clarify the picture in Pannalal Pitti v
State of A.P. AIR
1996 SC 1023).3 Yet others regard hostility to religion as the secular creed.
mischief generated by the insertion continues.

3. The Supreme Court has stated that while Arts. 25 and 26 granted religi° liS
freedom to minority religions like Islam, Christianity, etc., they do not intend
to deny the same guarantee to the Hindus.

Even before the 42nd Amendment the Supreme Court had observed, on the
b
sis of the provisions contained in the Constitution, that secularism is a basic . ature of
192 ^Constitutional Law o f India- II
the Constitution (Indira Gandhi v Raj Narain AIR 1975 SC 2299). It is ^ell-known
rule of interpretation that what is contrary to the Constitution cannot z justified
with reference to the Preamble. In Santosh Kumar case (above), the Supreme
Court observed: The Preamble does not control the meaning of the or0visions of the
Constitution. The word secular used in the Preamble cannot override the clear
provisions of Arts. 25-30, or, Art. 351.
In 1977, a Committee (formed by the Union Government) considered the
proposal to omit the words 'secular’ and ‘socialist’. It concluded that there is an
ernotional content attached to these expressions so they should be left untouched.
Ip Bommai case, B.P. Jeewan Reddy, J. observed that the expressions ‘socialist’
and ‘secular’ are not capable of precise definition.
48

ARTICLE 25 [FREEDOM OF CONSCIENCE AND FREE PROFESSION


PRACTICE AND PROPAGATION OF RELIGION]
regarded by those who profess religion to be conducive to the future well-being
Latham C.J. observed in Adelaide Co. of Jehovah’s Witnesses Incorporated v
Commonwealth (1943) 67 CLR 116, that 'what is religion to one is superstition to
another’.
Religion is not identical to Dharma. In AS. Narayan Deekshitulu v State of
A.P. AIR 1996 SC 1765, Justice Hansaria has observed: “Very often the words
'religion' and 'dharma’ are used to signify one and the same concept or notion; to
put it differently, they are used inter-changeably. This, however, is not so, the
word 'religion’ as used in Arts. 25 and 26 of the Constitution cannot be confined,
cabined or crabbed, to what is generally thought to be religion... The English
language having had no parallel word to Dharma, the word 'religion' was used in
these two articles. The signs and symptoms of Dharma are that it has no room for
narrow mindedness, sectarianism, blind faith and dogma... A sectarian religion is
open to a limited group of people whereas dharma embraces all and excludes
none. This is the core of our dharma, our psyche.”5
Conscience is the inner freedom of a person to mould his relation with
God, human beings and other creatures in the manner he pr efers. Freedom of
conscience connotes a person’s right to entertain beliefs and doctrines
concerning matters which are regarded by him to be conducive to his spiritual
well-being.
To'profess' a religion means to declare freely and openly one’s faith and
belief. He has right to practice his belief by practical expression in any manner he
likes. To ‘practice’ religion is to perform the prescribed religious duties, rites and
rituals.
To 'propagate' means to spread and publicise his religious views for the

Religion has not been defined in the Constitution, and, it is not susceptible to a
ec
ise definition. It is a matter of personal faith and belief and comprises of
various and observances which are considered as an integral part of that
religion by those 0 follows it. Religion undoubtedly has its basis in a system of
5 . B.K. Sharma,jsp. 103-104. beliefs which are
Right to Freedom of Religion 193
edification of others. In other words, it means the right to communicate a person's
beliefs to another person or to expose the tenets of that faith. The right to
propagate one’s religion does not give a right to convert another person to one’s
own religion, as that would impinge on the “freedom of conscience” guaranteed to
all persons.
In Commr., HRE v Sri L. T. Swamiar of Sri Shirur Mutt (AIR 1954 SC 282)
it was observed that Art. 25 secures to every person a freedom not only to
entertain such religious belief, as may be approved of by his judgment and
conscience but also to exhibit his belief in such outward acts as he thinks proper
and to propagate or disseminate his ideas for the edification of others. It was aiso
observed that what is protected is the propagation of belief, no matter whether
ihe propagation takes place in a church or monastery or in a temple or parlour
meeting.

Restrictions on Freedom of Religion


Article 25, Clause (1) - In the name of religion no act can be done against public
order, morality and health of the public. For example, in the name of religion
‘untouchability or traffic in human beings’ e.g. system of devadasis cannot be
tolerated.
In Gulam v State of U P. (AIR 1981 SC 2198) held that the Muslims have
no right to stop the procession of a community on the ground that the music being
played
operates as a nuisance or offends against their sentiments. In Gulam Abbas v State
of U.P- (1984) 1 SCC81, it has been held that the direction given by the Supreme
Court for shifting a property connected with religion to avoid clashes between two
religious communities or sects does not affect religious rights, being in the interest of
‘public order.’
In Gulam Kadar A. Menqn v Surat Municipality (AIR 1998 Guj 234), the
petitioners had challenged the validity of Sec. 212 of the Bombay Provincial
Municipal Corporation Act, 1949 under which ,the Municipal Corporation had ordered the
demolition of some parts of two mosques situated in main road of Surat district. The
Court held that the acquisition of a religious place or a part thereof is not prohibited
by the Constitution and therefore can be acquired in the public interest for widening
the road. The Court, however, cautioned that the State is expected to treat a place of
worship with reverence of special consideration and treatment which other private
properties may not command. This is the Indian concept of secularism.
The freedom of religion is also subject to the “other provisions of this Part” e.g,
right to freedom of speech and expression, freedom of assembly and association,
etc. In Moulana Mufti Sayeed Mohd. Noorur Rehman Barkariq v State of W.B. (AIR
1999 Cal. 15), the Calcutta High Court held that restrictions imposed by State on the
use of microphones and loudspeakers at the time of Azan is not violative of right
under Art. 25. ‘Azan’ is certainly an essential and integral part of Islam, but use of
microphones and loudspeakers are not an essential and integral part Microphone is
a gift of technological age; its adverse effect is felt all over the world - a source of
[C.L.C.-2006]
194 ^Constitutional Law o f India- II
pollution and health hazards. Tr • tionally and according to the religious order, Azan
has to be given by the Imam cr the person in-charge of the mosques through their
own voice and this is sanctioned under the religious order.
Article 25, Clause (2)(a)49 - The freedom to practice extends only to those activities
which are the essence of religion. It does not cover secular, political or commercial
activities associated with religious practice. Thus, the right to manage a temple can
be controlled.
In M.H. Quareshi v State of Bihar (AIR 1985 SC 731) it has been held that the
sacrifice of cow on the Bakrid day was not an essential part of Mohammedan
religion, and hence could be prohibited by the State under clause (2)(a) of Art, 25. In
State of W.B. v Ashutosh Lahiri (1995) 1 SCC 189, the respondents challenged the
validity of the exemption of slaughter of cows from the operation of the West Bengal
Animal Slaughter Control Act, 1950, on Bakrid day. The State of W.B. exempted from
the operation of the Act the slaughter of cows on the occasion of krid on the ground
that such exemption was required to be given for the religious Purpose of Muslim
community. The Supreme Court held that slaughtering of cows ° n the Bakrid day is
optional and not obligatory. It is not essential or required for re|igious purpose of
Muslim. Art. 25 deals with ‘essential' religious practices. Exemption UndertheAct can
be granted only for subserving an essential religious, medicinal 0r research purpose.

o
Write
5 . B.K. a short note
Sharma,jsp. on: Essential and non-essential religious practices
103-104.
Constitutional Law o f India- II
In Fazru v State of Haryana (AIR 1998 P & H 134), it was held that Muslims are
not immune from the law which disqualifies a person from being Sarpanch if having
more than two children and such a law is not violative of Art, 25. A person has a right
to profess and propagate his religion but it is subject to public order, morality and
health. The health can be of either spouse. Further, it is settled law that personal law
is not a fundamental right The fundamental right only is to practice and propagate
religion. Thus, Rule 21 of the Central Civil Services Conduct Rules, 1964 provide that
no government servant can enter into a contract of marriage with a person having a
spouse living, it is applicable to all persons professing any religion. Likewise, Haryana
Panchayati Raj Act is a special law and shall apply to all persons irrespective of their
caste and religion. Further, it does not interfere in professing their religion, practice or
propagating the same. The Supreme Court similarly held in Javed v State of Haryana
(AIR 2003 SC 3057). 50
In Surjeet Singh Chhabra v UOI (AIR 1997 SC 2560), the petitioner, a Sikh,
was alleged to be illegally importing the gold in the form of kara, a symbol of
religious wear by Sikh community. It was held that the gold cannot be exempted
from confiscation under the Foreign Exchange Regulation Act and the Customs Act.
The protection of Art.
25 cannot be availed of by the petitioner, as Art, 25 is subject to regulations by the
State.
In M. Ismail Faruqui v UOI (AIR 1995 SC 605), the question was whether the
provision in Sec. 7 of Acquisition of Certain Area atAyodhya Act containing the
mandate to maintain the status quo existing at the disputed site Ram Janms Bhoomi
Babri Masjid as on 7th Jan., 1993 is a slant in favour of the Hindu community
intended to perpetuate an injustice done to the Muslim community by demolition of
mosque on 6th Dec., 1992 and, therefore, it amounts to an anti-secular or
discriminatory act. Held that the right to practice, profess and propagate religion
under Art. 25 does not necessarily include the right to acquire or own or possess
property. Places of religious worship like mosques, churches, temples, etc. can be
acquired by the State. Such acquisition does not violate Art. 25 or Art. 26. Similarly,
this right does not extend to the right of worship at any and every place of worship.
The protection under these Articles is to religious practices which forms an essential
and integral part of religion. While offer of prayer or worship is a religious practice, its
offering at every location where such prayers can be offered would not be an
essential or integral part of such religious practices unless the place has a particular
significance for that religion so as to form an essential or integral part thereof51

50 In the light of Arts. 25 and 26 of the Constitution, discuss the legal position of
the following: Restriction on number of children and marriage for contesting a n
election is challenged on the ground that Muslim personal law permits
polygamy for the purpose of procreation. [C.L.C.-2007]
51 In this case, the Minority view was: Secularism is absolute; the State may no*
treat religions differently on the ground that public order requires it. When the
religious majority makes a claim upon and attacks the place of worship of
another religion and creates conditions that are conducive to public disorder, it
is the constitutional obligation of State to protect that place of worship. It is
impermissible for the State to acquire that place of worship to preserve public
order.
Right to Freedom of Religion
In Acharya Jagdishwaranand v Commissioner of Police, Calcutta ('Anand
Marga’ Case) (1984) 4 SCC 522, the Supreme Court held that the ‘Tandava dance’
in procession or at public places by Ananda Margis carrying lethal weapons and
human skulls was not an ‘essential’ religious rite and hence the order prohibiting
(under Sec. 144, Cr. P.C.) such procession in the interest of ‘public order’ and
‘morality’ was not violative of the right of petitioners under Arts. 25 and 26. Even
accepting that Tandava dance is prescribed as a religious rite for every follower of
Anand Marga it does not necessarily imply that they have right to perform it in public
places.

LEADING CASE: COMMR. OF POLICE v ACHARYA


JAGADISHWARANANDA [(2004) 12 SCC 809]

In this case, the Apex Court (majority opinion) held that the performance of
Tandava dance in public procession by Ananda Margis is not an essential
or integral part of their religion and, thus, not a matter of religion within the
meaning of Arts. 25 and 26.
The court observed: The protection guaranteed under Arts. 25 and
26 is not confined to matters of doctrine or belief but extends to acts done
in pursuance of religion, and therefore, contains a guarantee for rituals,
observances, ceremonies and modes of worship which are essential or
integral part of religion. What constitutes an essential or integral part of
religion has to be determined with reference to its doctrines, practices,
tenets, historical background, etc. of the given religion. Essential part of a
religion means the core beliefs upon which a religion is founded. Essential
practice means those practices that are fundamental to follow a religious
belief. It is upon the cornerstone of essential parts or practices that the
superstructure of a religion is built, without which a religion will be no
religion. The test to determine whether a part or practice is essential to a
religion is to find out whether nature of the religion will be changed without
that part or practice.
The court further observed: There cannot be additions or
subtractions to essential or integral parts or practices of a religion as they
are the very essence of that religion and alterations will change the
fundamental character of the religion concerned. It is such permanent
essential parts which are protected by the Constitution. Nobody can say
that an essential part or practice of one’s religion has changed from a
particular date or event. On facts, Ananda Marga was in existence from its
founding in 1955 to 1966 without the practice of Tandava dance. Tandava
dance was introduced as practice only in 1966. Hence, it was not the “core"
upon which the Ananda Marga was founded. Had it been so, then without
Tandava dance Ananda Marga could not have existed ]
Article 25, Clause (2)(b) - Under this clause State can eradicate social practices and
dogmas which stand in the path of the country’s onward progress. Such laws do not
affect the essence of any religion. Moreover, social evils cannot be tolerated
in 196
the name of religion.
C For
o n sexample,
t i t u t i o nbigamy
a l L a and
w o polygamy
f I n d i a are
- I Inot an essential part
of the Hindu religion, therefore, cannot be tolerated and can be regulated by a law
(State of Bombay v Narasu AIR 1953 Bom. 84). Prohibition of civil practices such as
sati or the system of devadasis has been held to be justified under this clause This
sub-clause ensures an individual's legal right to enter into a temple unobstructed
irrespective of his caste, untouchability, social inequality, etc.

ARTICLE 26 [FREEDOM TO MANAGE RELIGIOUS AFFAIRS]

Subject to public order, morality and health, every religious denomination or any
section thereof shall have the right:
(a) to establish and maintain institutions for religious and charitable purposes,
(b) to manage its own affairs in matters of religion,
(c) to own and acquire movable and immovable property, and,
(d) to administer such property in accordance with law.

Religious Denominations9
While, right guaranteed by Art. 25 is an individual right, the right guaranteed by Art.
26 is the right of an ‘organised body'. The words ‘religious denomination' in Art 26
must take their colour from the word ‘religion’ and so it must satisfy three conditions:
(1) It must be a collection of individual who have a system of beliefs which
they regard as conducive to their spiritual well-being i.e. a common faith
(2) It must have a common organisation.
(3) It must be designated by a distinctive name.
Thus, in a large sense, 'Hinduism', 'Muslims', ‘Christians’ are such
denominations. In limited sense, the various philosophers governing the Hindu
society such as Advaitas, Saivites, etc. are also denominations. Among
Mohammedans, Hanafi, Shia or Chisti sects are separate denominations. Also,
maths or the spiritual fraternity (a section of denomination) represented by it comes
within this article (LT Swamiar’s case).
In S. P. Mittal v Union of India (AIR 1983 SC 1), the Aurobindo Society, formed
in 1961, which preached and propagate the ideas and teachings of Sri Aurobindo,.
set up an international cultural township, Auroville in Pondicherry, by receiving large
funds as grants from Government and different organisations. The Government
received complaints about the mismanagement of the affairs of the society and so
enacted the said Act, providing for taking over the management of Auroville for a
limited period. Held that the Society and Auroville were not religious institutions or
denomination, so government’s taking over of the Ashram did not infringe the
society’s rights under Arts. 25 and 26.
The court observed: The teachings of Sri Aurobindo only represented his
philosophy and not a religion. The opinions of theologians, professors, reference
books and news agencies, treating the teaching of Sri Aurobindo as religious are not
conclusive. Nor it is sufficient that followers have a common organisation, a distinct
9. Write a short note on: Religious denominations. [C. L.C.-2006]
Right to Freedom of Religion 197

name after the founder, have specially prepared mantras, a special symbol for
identification and a sanctified place of pilgrimage or that there is uniqueness or
innovations in the philosophy and teachings. The lack of exclusivity and
distinctiveness that the membership of the society was open universally to any one
subscribing to its aim and objects without losing his own religion, militates against,
the plea that it is a religion. The words “religious denomination'’ in Art. 26 must
satisfy the condition that it must be a collection of individuals who have a system of
beliefs which they regard as conducive to their spiritual well being i.e. a common
faith.
Further, it was held that even assuming that the society and Auroville were a
religious denomination (in limited sense, the various philosophies governing the
Hindu society, such as Advaitas, Saivities, can also be termed as denomination,
likewise Ananda Marga held to be a denomination), the impugned Act did not
infringe its right under Art. 25 or 26. The Act has not taken away the right of
management in “matters of religion” of a religious denomination under Art. 26(b). It
has only taken away the right of management of property of Auroville e.g. in respect
of its secular matters which can be regulated in accordance with law, under Art.
26(d).
In Bramchari Sidheswar Shai v State of W.B. (1995) 4 SCC 646
("Ramakrishna Mission case”), held that the followers of Ramakrishna, who are
collection of individuals, and who adhere to a system of beliefs as conducive to their
spiritual well being, who have organised themselves collectively and who’ve an
organisation of definite name 'Ramakrishna Math or Mission’ can be regarded as a
religious denomination within Hindu religion as they satisfy the tests regarding a
denomination as “religious denomination,” and would therefore be entitled to claim
rights under Art. 26.
Art, 26 does not create rights, it merely safeguards and guarantees the
continuance of a right which a denomination/section had. If the denomination never
had the right to manage property in favour of a denominational institution as per
reasonable terms on which the endowment was created, it cannot be said to have it.
Further, the right to establish and maintain institutions for religious/charitable
purposes or to administer property of such institutions in accordance with law was
protected only in respect of such religious denomination or section thereof which
appears to extend help equally to all and religious practice peculiar to such small or
specified groups or section thereof as part of the main religion from which they got
separated. The denominational sect is also bound by the Constitutional goals and
law [Sri Adi Visheshwara of Kashi Vishwanath Temple v State of U.R (1997) 4 SCC
606].
XVI
Constitutional Law o f India- II

Art. 26(a) and (b)


The word “establish and maintain" in Art. 26(a) must be read together and therefore
it is only those institutions which a religious denomination establishes which it can,
claim to maintain it. Thus, held that Aligarh University was not established by the
Muslim minority and so it could not claim the right to ‘maintain’ it. It was established
under the statute by the Parliament (Aziz Basha v UOI AIR 1968 SC 662).
Under Art. 26(b), the court has a right to determine whether a particular rite or
ceremony is regarded as essential by the tenets of a particular religion. The right is
also subject to the regulatory powers of State under clause (2)(b) of Art. 25. The
Constitution requires not only that the State will be away from religion but also
imposes a positive duty on the State to free many aspects of our life from the control
of religion. The State is not expected to remain a mere passive spectator when the
freedom of religion is being used against the interest of the society and the nation as
a whole e.g. in recent times, the places of worship are being used for hiding
criminals or carrying on anti-national activities.
Art. 26(c) and (d)
The right of a religious denomination to manage its property [Clause (c) and (d) of
Art. 26] has been placed on a different footing from the right to manage its own
affairs in matters of religion. The latter is a fundamental right which can't be taken
away except on grounds mentioned in Art. 25, the former can be regulated by laws
which the Legislature can validly impose e.g. general property law. Therefore, if the
question merely relates to administration of properties belonging to a religious group,
it is not a matter of religion' to which Art. 26(b) would apply. Also, Art. 26(c) is to be
read subject to State’s power to acquire private property under Art 3T. Thus,
property of a denomination can be acquired by State. That means, Art. 26(c) is not a
part of basic feature of the Constitution.
Arts. 26(c) and (d) protects the right of the denomination and not of pujari or
shebaits. When the administration and management of temple property was
entrusted to Board, it does not mean divesting the denomination of its right of
administration.
In Rati Lal v State of Bombay (AIR 1954 SC 388) and Commr. HRE v L.T.
Swamiar(AIR 1954 SC 282), held that the law must leave the right of administration
of property to religious denomination itself subject to such regulation as to improve
the administration. A law which took away right of administration altogether from
religious denomination and vested it in other secular authority (Commissioner of
Endowments), held to be violative of Art. 26(d) (unless it is shown that such right
never vested in denomination). Thus, where a mahant could be deprived of his right
to administer at any time by the Commissioner under an Act. even if there is no
negligence or maladministration on the mahant's part then the Act is violative of Art.
26(d).
gase Law: Arts. 25 and 26
Regarding Aricles 25 and 26, two main principles have emerged from the decisions
in various cases. The first is that the protection of these Articles is not limited to
matters of doctrine or belief, but also extend to acts done in pursuance of religion e g.
rituals, ceremonies and modes of worship which are integral parts of religion. The
Right to Freedom of Religion
second is that what constitutes an essential part of a reiigion or religious practice has
to be decided by the courts with reference to the doctrine of a particular religion and
includes practices which are regarded by the community as a part of its religion.

Appointment of Archakas (Pujari)/Priest52

Leading C ASE : seshammal V state of Tamil nadu [(1972) 2 SCC 11]

In this case, a writ petition was filed by the hereditary Archakas and
Mathadhipatis of some ancient Hindu temples in Tamil Nadu challenging
the validity of the Tamil Nadu Hindu Religious and Charitable Endowments
(Amendment) Act, 1970 on the ground that it violates their freedom of
religion secured to them under Arts. 25 and 26 of the Constitution. The
Amendment Act does away with the hereditary right of succession to the
office of Archaka even if the Archaka was qualified under Rule 12 of the
Madras Hindu Religious Institutions (Officers and Servants) Service Rules,
1964. it is claimed on behalf of the petitioners that the effect of the
amendment is as follows:
(a) The freedom of hereditary succession to the office of
Archaka is abolished although succession to it is an
essential and integral part of the faith of the Saivite and
Vaisnnavite worshippers.

52 in the light of Arts. 25 and 26 of the Constitution, discuss the legal position of
the following- Appointment of a non-brahmin as ‘archaka’ or pujari of a temple
where Brahmins traditionally carried out functions of priest. [C.L.C.-2007]
The State of Tamil Nadu enacted “T.N. Hindu Religious and Charitable
Endowments Act, 1970” (relating to administration and governance of Hindu
Religious and Charitable Endowments). The Act was enacted to check
malpractices associated with Jiereditary priesthood in religious institutions arid
as a step towards social reform. The Act while abolishing the system of
hereditary rule prohibits discrimination on the ground of caste, race or
denomination in the matter of appointment of Archakas and Pujaris. Z
challenges the validity of the above law on the ground of violation of rights
under Arts. 25 and 26. Decide. [C.LC.-95]
[Hint, in the present case, the purposes of the Act are essentially secular, and
not intended to effect a change in the rituals and ceremonies followed in
temples Thus, Z cannot succeed.]
XVI
Constitutional Law o f India- II

(b) It is left to the Government to prescribe or not to provide such


qualifications as they may choose to adopt for applicants to this
religious office while the Act itself gives no indication whatever of the
principles on which the qualifications should be based,
(c) The Amendment Act gives the right of appointment for the first time to
the trustee who is under the control of the Government. This is the
very negation of freedom of religion and the principle of non-
interference by the State as regards the practice of religion and the
right of a denomination to manage its own affairs in the matter of
religion.
It was contended on behalf of the petitioners that the appointment of a person to a
religious office in accordance with the hereditary principle is itself a religious usage
and amounted to a vital religious practice and hence falls within Arts. 25 and 26.
Priests, who are to perform religious ceremonies, may be chosen by a temple on
such basis as the temple chooses to adopt. It may be election, selection,
competition, nomination, or hereditary succession. The right to select a priest has an
immediate bearing on religious practice and the right of a denomination to manage
its own affairs in matters of religion.
The court observed: It is true that a priest or an Archaka when appointed has
to perform some religious functions but the question is whether the appointment of a
priest is by itself a secular function or a religious practice. In a saivite or vaishnavite
temple, the appointment of Archaka will have to be made from a specified
denomination, sect or group in accordance with the usage or practices in these
temples. The question, however, remains whether the trustee, while making
appointment from specified denominations, will be bound to follow the hereditary
principle as a usage peculiar to temple. The real question is whether such a usage is
secular or a religious usage [legislation permissible under both via Art. 25(2)(a) and
Art. 25(2)(b)]. In other words, whether the appointment of a priest is by itself a
secular function or a religious practice?
The Archaka has never been regarded as a spiritual head of any institution
(like the Shankaracharya). He may be an accomplished person, well versed in the
Agamas and rituals necessary to be performed in a temple but he does not have the
status of spiritual head. It has been held in K. Seshadri Aiyangar v Ranga Bhattar,
ILR 35 Mad 631, that even the position of the hereditary Archaka of a temple is that
of a servant subject to the disciplinary power of the trustee. As a servant he is
subject to the discipline and control of the trustee as recognized by the un-amended
Sec. 56 of the Principal Act which provides "all office-holders and servants attached
to a religious institution or in receipt of any emolument or perquisite therefrom shall,
whether the office or service is hereditary or not, be controlled by the trustee...”
That being the position of an Archaka, the act of his appointment by the
trustee is essentially secular. He owes his appointment to a secular authority. The
Shebaits and Managers of temples exercise essentially a secular function in
choosing and appointing the Archaka. Thus the appointment of an Archaka is a
secular act and the fact that in some temples the hereditary principle was followed in
making the appointment would not make the successive appointments anything but
secular. It would only mean that in making the appointment the trustee is limited in
Right to Freedom of Religion 201

respect of the sources of recruitment, instead of casting his net wide for selecting a
proper candidate, he appoints the next heir of the last holder of the office. That after
his appointment the Archaka performs worship is no ground for holding that the
appointment is either a religious practice or a matter of religion.
The court further observed: Rituals and observances, ceremonies and modes
of worship considered by a religion to be its integral and essential part are secured
by Art. 25. What constitutes an integral and essential part of a religion or a religious
practice has to be decided by the courts with reference to the doctrine of a particular
religion and include practices regarded by the community as part of its religion. The
Act nowhere gives the indication that one of the purposes of the Act is to effect a
change in the rituals and ceremonies followed in the temples. On the other hand,
Sec. 107 of the Principle Act emphasizes that nothing contained in the Act would be
deemed to confer any power or impose any duty in contravention of the rights
conferred on any religious denomination by Art. 26.]

L EAD ING C AS E : N . AD IT HAYAN v T R AVANC O R E

DEVASWOM BOARD

[(2002) 8 SCC 106]

In this case, the issue was whether the appointment of a person, who is not a
Malayala Brahmin, as “Santhikaran” or poojari (priest) of the temple in question is
violative of the constitutional and statutory rights of the appellant. It was held that
there is no right that a Brahmin alone can perform the rites and rituals in a temple.
Properly trained and qualified persons may be appointed as poojari regardless of
caste.
The appellant claims himself to be a Malayala Brahmin by community and a
worshipper of the Siva Temple in question. The administration of the temple vests
with Travancore Devaswom Board, a statutory body created under the Travancore-
Cochin Hindu Religious Institutions Act, 1950. The Board had opened a Vedantha
School for the purpose of training Santhikarans; the School was open to all students,
irrespective of their caste/community. From 1969 onwards, persons who were non-
Brahmins but successfully passed out from the School were being appointed and the
worshippers - the public - had no grievance or grouse whatsoever.
The appellant contended that the appointment of a non-Brahmin Santhikaran
for the temple offends and violates the alleged long-followed mandatory custom and
usage of having only Malayaia Brahmins for performing poojas in the temple and this
denies the right of the worshippers to practise and profess their religion in
accordance with its tenets and and manage their religious affairs as secured under
Arts. 25 and 26 of the Constitution. It was contended that merely because such a
religious practice, which was observed from time immemorial, involves the
appointment of a Santhikaran or priest, it would not become a secular aspect to be
dealt with by the Devaswom Board dehors the wishes of the worshippers.
The appellant placed reliance, among other cases, on the decision in Commr.,
HRE v Sri L.T. Swamiar of Sri Shirur Mutt (AIR 1954 SC 282). In that case, whiie
elaborating the meaning of the words “its own affairs in matters of religion" in Art.
26(b), it has been observed that in contrast to secular matters relating to
XVI
Constitutional Law o f India- II

administration of its property the religious denomination enjoys complete autonomy


in deciding as to what, rites and ceremonies are essential according to the tenets of
the religion they hold and no outside authority has nay jurisdiction to interfere with
their decision in such matters.
The court placed reliance on various cases:
In Bhuri Nath v State of J&K (AIR 1997 SC 1711), the validity of the J&K Mata
Vaishno Devi Act, 1988 which provided for the better management of temple and its
endowment was challenged on the ground that it was violative of the petitioner’s
rights guaranteed under Arts. 25/ 26. The Act abolished the hereditary post of priest
in temple and made provision for appointment of priests by the State. Further, the
Government can also abolish his customary share in the offerings of the deity. It was
held that the service of the priest is a secular activity and that may be regulated by
the State under Art. 25(2). The court made distinction between ‘religious service’ and
the ‘person who performs service.’ Though performance of the ritual ceremonies was
an “integral part” of the religious freedom, but securing the service of a priest, who
perform ritual ceremonies was not. He is the holder of an office of a priest in the
temple and is subjected to discipline on par with other members of the establishment
In A S. Narayana Deekshitulu v State of A.P. (Balaji Temple case) (AIR 1996
SC 1765) also, it was held that the appointment of archaka (priest) is a secular
activity which could be regulated by law. Archakas are employees of temple-Balaji,
Tirupathi.
In Sri Adi Visheshwara of Kashi Vishwanath Temple v State ofU.P (1997) 4
SCC 606, the U.P Kashi Vishwanath Temple Act, 1983. which divested the Pandas
of their right to manage the temple and receive offerings was challenged as violative
of their rights under Arts. 21,25 and 26. The Supreme Court upheld the validity of the
Act. Temple is not of any religious denomination or section thereof, nor the believers
of Shaiva form of worship belong to a denomination or sect. The Supreme Court
observed and held as follows:
(i) Religious freedom guaranteed by Arts. 25 and 26 is intended to be a
guide to a community life and ordain every religion to act according to
its cultural and social demands to establish an egalitarian social order.
Arts. 25 and 26, therefore, strikes a balance between the rigidity of
right to religious belief and faith and their intrinsic restrictions in
matters of religion, religious beliefs/practices and guaranteed freedom
of conscience to commune with his Cosmos/Creator and realise his
spiritual self.
(ii) Sometimes, practices religious or secular, are inextricably mixed up.
Among Hindus, human actions from birth to death and most of the
individual actions from day-to-day are regarded as religious in
character in one facet or the other However, traditional religious
model is different from constitutional religious modei. The legitimacy
of the true categories is required to be adjudged strictly within the
parameters of the right of the individual and the legitimacy of the State
for social progress, well-being and reforms, social intensification and
national unity.
(iii) Law is a tool of social engineering and an instrument of social change
evolved by a gradual and continuous process. All secular activities
Right to Freedom of Religion 203

which may be associated with religion but which do not relate or


constitute an essential part of it may be amenable to State regulations
but what constitutes the essential part of religion may be ascertained
primarily from the doctrines of that religion itself according to its
tenets, historical background and change in evolved process, etc.
(iv) The concept of essentiality is not itself a determinative factor, it is one
of the circumstances to be considered in adjudging whether the
particular matters of religion/religious practices or belief are an
integral part of the religion. It must be decided whether the practices
or matters are considered integral by the community itself. Though not
conclusive, this is also one of the facets to be noticed.
(v) The protection of Arts. 25 and 26 is not limited to matters of doctrine
or belief. They extend also to acts done in furtherance of religion and
therefore, they contain a guarantee for rituals and observances,
ceremonies and modes of worship which are integral parts of the
religion.
However, every mundane and human activity is not intended to be
protected under the Constitution in the grab of religion. Arts. 25 and
26 must be viewed with pragmatism
(vi) By the very nature of things it would be extremely difficult, if not
impossible, to define the expression “religion” or “matters of religion’’
or “religious beliefs or practices’’. Nevertheless, the right to manage a
temple or endowment is not an integral part of religion or reiigious
practice, such activities are secular activities and the State could
regulate them by appropriate legislation. The right to receive offerings
is not a vested right of the Pandas but a right coupled with duty to
render service, which could be regulated by the State by law
In the present case, the court observed: The popular Hindu religion of modern times
is not the same as the religion of the Vedas though the latter are still held to be the
ultimate source and authority of ali that is held sacred by the Hindus. In course of its
development, the Hindu religion did undergo several changes, which reacted on the
social system and introduced corresponding changes in the social and religious
institution (Justice B.K. Mukherjea in Tagore Law Lectures on Hindu Law of
Religious and Charitable Trust).
It is now well settled that Art. 25 secures to every person, subject to public
order, health and morality and ether provisions of Part III, including Art. 17 (abolition
of all distinctions based on caste and creed), freedom to entertain and exhibit by
outward acts as well as propagate and disseminate such religious belief according to
his judgment and conscience for the edification of others. The right of the State to
impose such restrictions as are desired or found necessary on grounds of public
order, health and morality is inbuilt in Arts. 25 and 26 itself Art. 25(2)(b) ensures the
right of the State to make a law providing for social welfare and reform besides
throwing open of Hindu religious institutions of a public character to all classes and
sections of Hindus and any such rights of the State or of the communities or classes
of society were also considered to need due regulation in the process of harmonizing
the various rights. The vision of the founding fathers of the Constitution to liberate
the society from blind and ritualistic adherence to mere traditional superstitious
XVI
Constitutional Law o f India- II

beliefs sans reason or rational basis has found expression in the form of Art. 17.
If traditionally or conventionally, in any temple, all along a Brahmin alone was
conducting poojas or performing the job of Santhikaran, it may not be because a
person other than the Brahmin is prohibited from doing because he is not a Brahmin,
but those others were not in a position and, as a matter of fact, were prohibited from
learning, reciting or mastering Vedic rites, etc. and thereby acquire the right to
perform the ritualistic forms of worship in temples.
In the present case also there has been no proper plea or sufficient
proof of any specific custom or usage; the temple also does not belong to
any denominational category with any specialized form of worship peculiar
to such denomination or to its credit. Further, any custom or usage
irrespective of any proof cannot be countenanced as a source of law to
claim any rights when it is found to violate human rights, dignity, social
equality and the specific mandate of the Constitution and law made by
Parliament.)
|n shri Jagannath Temple Puri Mangt. Committee v Chintamani Khuntia (1997) 8
SCC 422, it was held that installation of Hundis in the temple for putting therein the
offerings (tiade by the pilgrims/devotees to the deity and disentitling the Sevaks of
the temple to any portion of the money placed in Hundis was not violative of Sevak’s
rights under /\rts. 25/ 26. Rijht of the Sevaks to get a share of the offerings as
recognised in the record of rights for or connected with seva puja of the temple is not
a religious right. Sevaks are servants of temple and duties they perform including
sweeping and collection of offerings after seva puja are ‘secular’ activities even if
associated with religious practice. The court allowed the payment of a small
percentage of the money, given in the offerings, to the Sevaks by way of their
remuneration.
A disposition towards making gift for charitable or religious purposes may be a
pious act but the same cannot be said to be an integral part of any religion. Only
because charity and compassion are preached in every religion, the same by itself
would not be part of the ‘religious practice' within the meaning of Art. 25 [John
Valumattom v UOI (2003) 6 SCC 611]

Right to Propagation and Religious Conversion

Leading Case: rev. stainislaus V state of m.p.53


(AIR 1977 SC 908)

In this case, the validity of the M.P. and Orissa Act (related to freedom of
religion) was challenged, on the grounds that -

■ A law prohibits religious conversions brought about by use of fraud, force, or


allurement and makes it a punishable offence. In a writ petition it is alleged that
the law is violative of right to propagate religion guaranteed under Art. 25 of the
Constitution. Decide giving reasons. [I.A.S.-99]
“The right to freedom of religion is not an absolute right but it includes right to
Propagate religion without allurement and coercion ” Examine.
Right to Freedom of Religion 205

(i) they violate the right to propagate one’s religion under Art.
25(1), and,
(ii) the State legislature lacked the legislative competence to
engct, as the matter of religion fell within List I (Union List).
These Acts were passed to prohibit forcible conversion of any person to
one’s own religion. These Acts were passed after a commission headed
by Justice K.C. Niyogi found that Christian Missionaries were indulging in
conversions by force and fraud. Stanislaus, a missionary, challenged the
validity of these Acts and submitted that the Acts infringed his rights under
Arts. 25 and 26. The Supreme Court rejected his pleas.
The Court held that Art. 25(1) by giving right to propagate one's
religion does not grant the right to convert another person to one’s own
religion, but to transmit or spread one’s religion by an expr ession of its
tenets. If a person purposely, undertakes the conversion of another person
to his religion, that would impinge on the “freedom .of conscience”,
guaranteed to all the citizens alike.
It may however be argued that the freedom of conscience is not
confined to the right to adhere to one’s own religion but also to abandon
one’s religion and adopt another according to his conscience. The latter may
be done either on his own volition or at the persuasion of another so long as
objectionable methods are not employed in doing so.
The court held that impugned Acts fell within the purview of List II or
State List (Entry 1 - public order) as they were meant to avoid disturbances
to the public order. If an attempt is made to raise communal passions e.g.
on the ground that someone has been “forcibly" converted to another
religion, it would in all probability, give rise to an apprehension of a breach
of public order, affecting the community at large.]

Right to Excommunicate
In Saifuddin Saheb v State of Bombay (AIR 1962 SC 853), the petitioner, the head of
the Dawoodi Bohra Community, challenged the constitutionality of the Bombay
Prevention of Ex-Communication Act, 1949, on the ground that it infringed his rights
guaranteed in Arts. 25/26. The Act purported to prevent excommunication. The
petitioner claimed that as the head of the said community he had the right to ex-
communicate a member and this power was an integral part of the religious faith and
belief of the said community.
The Supreme Court (by a 4:1 majority) struck down the impugned provisions of
the Act as violative of Arts. 25/26, as the ex-communication was itself based on
religious grounds such as lapse from religious creeds or breach of some essential
religious practices. The exercise of power of ex-communication will form part of the
management of the community through its religious head ‘of its own affairs in matters
of religion’ [Art. 26(b)]. The said Act is not a law providing f°r social welfare and
reform within Art. 25(2)(b). However, C.J. in his dissenM judgment said that the right of
ex-communication was not a purely religious matt ef and that it makes the ex-
communicated person a sort of “untouchable" in h|S community which is contrary to
Art. 17 of Constitution (which abolishes untouchabilW'’
According to majority judgment, excommunication on religious grounds
permitted but not on non-religious grounds. The judgment has been criticize^
jurists. In fact the right has been often used to curb criticism and reform and to oe
freedom of speech to the Bohra community. The case has been referred to the
Constitution Bench which may refer it to a 7-Judge Bench for reconsideration [Central
Board of Dawoodi Bohra Community v State of Maharashtra (2005) 2 SCC
673)].
Right to Freedom of Religion 207

In P.M.A. Metropolitan v Moran Mar Marthma (AIR 1995 SC 2005), held that
the ex-communication of Cathlico by Patriarch was not in accordance with law/
religious cannons. One of the effects of ex-communication is that the person
concerned is deprived of the right of worship. Under our Constitution, it is a
fundamental right. Any interference with it or its deprivation can be challenged in a
court of law.

ARTICLE 27 [FREEDOM
NOT TO PAY TAXES FOR
RELIGIOUS PROMOTION]

Art. 27 says that no person shall be compelled to pay any taxes for expenses on
promotion or maintenance of any particular religion or religious denomination i.e.
there could be no objection if the taxes were used for the promotion of all religions.
Thus, the object is to protect the secular characteristics of the Constitution
which prohibits the promotion or maintenance of any particular religion by the State
or at State’s expenses. Therefore, if such a tax is imposed, no person can be
compelled to pay it.
Where the payment was demanded under the Government Act for the purpose
of meeting the expenses of the Commissioner and his office which was the
machinery set up for the due administration of the affairs of the religious institution
concerned, it was held that the annual contribution so imposed was in the nature of a
'fee’ and not a ‘tax’ and thus Art. 27 were not attracted (Sri Jagannath v State of
Orissa AIR 1954 SC 400). Similarly, where the Bengal Wakfs Act required a
contribution towards the education of poor Muslim boys and girls by the Wakf
management it was held to be a fee realized for a secular purpose for the proper
management of the Wakf property; there was no religious affair involved, thus.. Art.
27 were not attracted (Nasima Khatun v State of W.B. AIR 1981 SC 302).
Art. 27 not only prohibit the imposition of a tax but it also prohibits the
utilization of public funds for the promotion of a particular religion or denomination.
However, reconstruction of the religious places damaged during communal riots at
the cost of the Government had been held to be valid. Similarly, the acquisition of
la
nd for construction of public temple had been upheld as not violative of Art. 27.
In Sri Divi Kodandarama Saran v State of A.P. (1S97) 6 SCC 189, it was held
^at on abolition of hereditary rights the status of Archaka is on a par with other
Se
cular staff of the temple. Hence, a scheme containing a provision for their
re,
renchment in the case of fali of income of temple was not improper. Further,
provision in welfare scheme for delinking the pay scales of Archakas from that of
government servants was not improper (separate service rules and packages of
emoluments framed for them to suit the Endowment Institutions).
The Court held that the payment of salary to the government servants
depends upon diverse factors like cost of index, etc Art. 27 of the Constitution
prohibits spending of any expenses for promotion/maintenance of any particular
religion/religious denomination from out of the funds flowing from the public
XVI
Constitutional Law o f India- II

exchequer. Resultantly, constitutional prohibition stands in the way of extending to


the Archakas and other office-holders such benefits as are granted to the
government servants. Secondly, the salary has a linkage with the income derived by
the charitable/religious institutions or endowments.

ARTICLE 28 [FREEDOM
NOT TO ATTEND RELIGIOUS
INSTRUCTIONS]

Art. 28(1) prohibits religious instructions in educational institution wholly maintained


by the State. In case of other institutions recognized and aided by the State, there
will be freedom for eveiy person not to participate in religious instruction or worship.
If such person is a minor, his guardian’s consent is required for participation [Art.
28(3)],
Clause (2), an exception to Clause (1), provides that the prohibition contained
in Clause (-1) would not apply to an educational institution which is administered by
the State but has been established under any endowment or trust which requires
that religious instruction shall be imparted in such institution. Thus, there is no
prohibition against imparting of religious instructions in such institutions.
In St. Xavier College v State of Gujarat (AIR 1974 SC 1389), the Supreme
Court held that if such a person is minor, consent of his guardian would be required
for requiring him to attend the religious instructions.
In D A V. College, Jullundur v State of Punjab (AIR 1971 SC 1737), it was
held that a provision for an academic study of the life and teachings or the
philosophy and culture of any great saint of India (Guru Nanak, in the present case)
in relation to, or their impact on the Indian and world civilization could not be taken
as providing for religious instructions relating to a particular religion. Thus, the Guru
Nanak University Act, 1969, which provided for the establishment of Guru Nanak
University at Amritsar with a view to make provision for the study and research on
the life and teachings of Guru Nanak, did not imply that religious instructions would
be imparted therein.
In Aruna Roy v Union of India (2002) 7 SCC 368 (“The Textbook Case”) (“New
Education Policy Case"), the Supreme Court upheld the validity of National
Curriculum Framework for School Education (NCFSE) rejecting the petitioner's
contention that it was an attempt to “saffronise” education by the BJP led N.D.A.
Government. It held that an emphasis in, about requirement of “education about
religions”, their basics, the values inherent therein and also comparative study of the
philosophy of all religions is neither violative of Art. 28 nor offends against
secularism.
The court observed: The entire emphasis in Art. 28 is against imparting
'religious instructions’ (teaching rituals, customs, etc.). There is no prohibition for
having ‘study of religious philosophy and culture,’ particularly for having value-based
social life in a society which is degenerating for power, post or property. Secularism
is the basic structure of the Constitution. “Secularism” has a positive meaning that is
Right to Freedom of Religion 209

developing understanding and respect towards different religions (“religious


pluralism”). The essence of secularism is non-discrimination of people by the State
on the basis of religious differences. Art. 28(1) does not prohibit introduction of study
of religions in the State educational institutions including those wholly or partially
aided by the State. After the National Education Policy of 1986, a shift by the
impugned National Education Policy, 2002 towards teaching of religions in the
schools to educate children to understand common factors in all religions, is not a
non-secular step.
It may be noted that, according to Dr. B.R. Ambedkar, Art. 28 were enacted to
ensure that the peaceful atmosphere of educational institutions should not be
disturbed by the controversies with regard to the truthful character of any particular
religion and the erroneous character of the other. It was to provide for a path of
complete safety.
In P.M. Bhargava v University Grants Commission (AIR 2004 SC 3478), it was
held that introduction of Jyotir Vigyan as a course of study by the UGC did not mean
teaching religion. It would not result in propagation of a particular religion merely
because the subject had got its basis or origin traceable to some cult.

Q UESTIONS AND A NSW ERS

Q. 1 “The Salute to National Flag and Singing of National Anthem Act”


enacted by State of Gujarat, requires all the schools in the State to
hold an assembly before the start of the day The Head of the school,
then will unfurl the National Flag and all the students and other
employees of the school present will salute the flag. Thereafter the
National Anthem will be sung and all will participate in the singing.

“X", a student (belonging to the 'Jehovah’s witnesses' of the Christian


community), whose religion does not permit singing in praise of
anyone other than God refuses to sing National Anthem and salute
the flag but stands in respect when National Anthem is sung and when
the flag is unfurled. The school authorities expel “X” for violating the
Act.

He challenge the law and his expulsion on the grounds of violation of


his rights under Arts. 19(1 )(a) and 25 of the Constitution. Decide.
[C.L C.-93/94/95; I.A.S-93]
“The right to freedom of religion is not an absolute right but it includes
right to keep silence." Examine. [D.U.-2008]
A. 1 Art. 19(1 )(a) guarantees the freedom of speech and expression. Art. 25(1)
guarantees to every person the freedom of conscience and the right to
profess, practice and propagate religion. However, this clause is subject to
public order, morality and public health. Art. 25(2)(a) ensures that the
freedom to practice extends only to those activities which are the essence
of religion. The State can regulate any economic, financial, political or other
secular activity which may be associated with religious practice.
‘Religion’ is essentially a matter of personal faith and belief. ‘Freedom of
XVI
Constitutional Law o f India- II

conscience’ connotes a person's right to entertain beliefs and doctrines concerning


matters which are regarded by him to be conducive to his spiritual well-being. ‘To
profess' a religion means to declare freely and openly one’s faith and belief. He has
right to practice his belief by practical expression in any manner he likes.

Leading Case: bijoe emmanuel V state of kerala


(“NATIONAL ANTHEM CASE”)
[(1986) 3 SCC 615]
In this case, three children belonging to the ‘Jehovah’s witnesses' of
Christian community were expelled from school for refusing to sing National
Anthem. The Kerala High Court held that it was their Fundamental Duty
under the Constitution to sing the National Anthem. Their conduct in not
showing unqualified respect to the National Anthem would endanger the
security of the nation in that the same will develop among the citizens a
tendency to ignore the mandates of Constitution. If the religious practices
run counter to public order, morality, health or a policy of Government to
uphold the sovereignty, integrity and unity of nation then the said religious
practices must give way for the benefit of people and the nation as a whole.
On appeal, however, the Supreme Court reversed the High Court's
decision and held that there is no legal obligation for a citizen to sing
national anthem. The freedom of speech and expression under Art.
19(1)(a) also includes the ‘freedom of silence'. Moreover, the Act was not
enacted in the interest of security or sovereignty of State, public order, etc.
so as to come under Art 19(2).
The court said that by standing up while the anthem was being sung,
the children had shown proper respect to national anthem and had thus not
violated the fundamental duty - “to abide by the Constitution and respect its
ideals and institutions, the national flag and anthem by standing up when
the anthem is being sung” (Art. 51 A). They haven’t committed an offence
under the Prevention of Insults to National Honour Act, 1971, as they didn't
prevent the singing nor caused disturbances to the assembly irvthe singing
of anthem.
The Supreme Court relied heavily on the decisions of Australian
and American Supreme Courts. In Minersville School v Gobitis and West
Virginia Board of Education v Barnette, the Jeohavh’s witnesses claimed
that they could not be compelled to salute the flag of U.S. while reciting
pledge of allegiance. In Gobitis’ case, court observed that government
can as a supposed educational measure and to foster a sentiment of
national unity amongst children, compel affirmations which violate their
religious conscience. In Barnette case, however, court observed that the
compulsory flag salute and pledge requires affirmation of a belief and an
attitude of mind.
To sustain the compulsory flag salute we are required to say that a Bill
of Rights which guards the individual's right to speak his own mind, left it
open to public authorities to compel him to utter what is not in his mind.
The censorship or suppression of expression of opinion is tolerated by
Right to Freedom of Religion 211

our Constitution only when the expression presents a clear present


danger to State, and remaining passive during a flag salute presents no
such danger. Moreover, it is not clear whether the regulation
contemplates the pupils forego any contrary convictions of their own and
become unwilling converts to prescribed ceremony or whether it will be
acceptable if they simulate assent by words without belief and by a
gesture barren of meaning.
Art. 25 is an Article of faith in the Constitution, incorporated in
recognition of the principle that the real test of a true democracy is the
ability of even an insignificant minority to find its identity under
Constitution. As observed in Shirur Mutt case, "the Constitution protects
religion within community organized under a Constitution, so that the
continuance of such protection necessarily assumes the continuance of
the community so organized.” This view makes it possible to reconcile
religious freedom with ordered Government. The courts will have the
responsibility of determining whether a particular law can fairly be
regarded as a law to protect the existence of community, or on the other
hand, it is a law for prohibiting free exercise of any religion. However,
the question is not whether a particular religious belief or practice
appeals to our reason or sentiment, but whether the belief is genuinely
and conscientiously held as part of the profession or practice of religion.
Thus, held that no person can be compelled to sing the national anthem
“if he has genuine conscientious religious objection”.
The Supreme Court’s judgment is criticised by some jurists as it
has taken care about the individual’s rights more than the national
interests. Religion should not be given preference over the national
interests. As observed in Donald v Board of Education, Hamilton, “a
command to join the flag salute or singing of national anthem, would be
command not to join in any enforced religious exercise, but to pay
respect to a nation and country which stands for religious freedom."]
Decision of the case in question - “X” will succeed in his action against the school
au
thorities on grounds of violation of his rights under Arts. 19(1)(a) and 25.
Q. 2 ‘Constitution of India accepts secularism but there is no rigid
separation between State and freedom of religion given to the
individual.’ Critically examine. [I.A.S.-91]
A. 2 A secular State means that in matters of religion the State is neutral. The
State can have no religion of its own, and the State protects all religions but
interferes with none.
Various restrictions on freedom of religion guaranteed under Arts. 25 and 26
show that there is no rigid separation between State and freedom of religion given to
the individual:
(i) In the name of religion no act can be done against public order, morality
and health of public - Art. 25(1).
(ii) The freedom to practice extends only to those activities which are the
essence of religion. It does not cover secular, political or commercial
activities associated with religious practice -Art. 25(2)(a).
(iii) The State can eradicate social practices and dogmas associated with a
XVI
Constitutional Law o f India- II

religion which stand in the path of the country’s onward progress - Art.
25(2)(b).
(iv) The freedom to establish and maintain religious institutions and to manage
its own affairs in matters of religion is subject to public order, morality and
health - Art. 26(a) and (b).
(v) The right of a religious denomination to manage its property [Art. 26(c) and
(d) ] can be regulated by laws which the legislature can validly impose
e.g. general property law.

Rights to Minorities (Cultural and


Educational Rights) (Articles 29-30)

Minorities54
The word "minority” is not defined in the Constitution. Whether a section of
citizens constitutes a “minority” for the purpose of Arts. 29 and 30 in a State must
be determined by reference to the entire population of the State. It refers to any
community which is numerically less than 50% of the entire population of State. A
minority could not be determined in relation to entire population of the country.
The State is to be regarded as the unit for determining both “linguistic
minority" as well as “religious minority.” The question whether a sect or
denomination of a religion can claim minority status even though followers of that
religion are in majority was left unanswered [T.M.A. Pai Foundation v State of
Karnataka (2002) 8 SCC 481], Again, in PA. Insmdarv State of Maharashtra (AIR
2005 SC 3226), the court held: ’Minority,' whether linguistic or religious, is to be
determined by the demography of a State.
in Bal Patil v Union of India (AIR 2005 SC 3172). the Apex Court examined
the scope and definition of the term “Minorities”. It observed that though not
specifically defined, “minority” as understood from the constitutional scheme
signifies an identifiable group of people or community who were seen as
deserving protection from a likely deprivation of their religious, cultural and
educational rights by other communities who happened to be in a majority and
likely to gain power in a democratic form of government based on eiection. The
constitutional ideal, which can be gathered from the group of articles in the
Constitution under the chapters of fundamental rights and fundamental duties, is
to create social conditions where there remains no necessity to shield or protect
rights of a minority or majority.
In Frank Anthony Public School Employees Association v UOI (AIR 1987
SC 311) the Supreme Court observed: ‘The idea of giving special rights to
minorities

What is the meaning of “minorities" as used in Art. 30 of the Constitution of


India? [L.C ll-2007][D.U.-
2008]
*

Rights to Minorities 213

is not to have a kind of a privileged or pampered section of the population but to


give them a sense of security and feeling of confidence.’’
Art. 30 was enacted as a protection against legislative encroachment on the
rights of minorities. It was to allay the fear that the majority may suppress or deny
the minorities the right to preserve their culture and language. Majority and
minorities must be treated equally. However, a feeling is developing that
minorities have been conferred more rights than the majority.
It would be useful to recall what Dr. Ambedkar said in the Constituent Assembly:
“It is wrong for the majority to deny the existence of minorities. It is
equally wrong for the minorities to perpetuate themselves. A solution
must be found which will serve a double purpose, it must recognize the
existence of the minorities to start with. It will also be such that it will
enable majorities and minorities to merge some day into one.
Pandit Nehru spoke in the same vein:
I do not think it will be a right thing to go the way this country has gone
in the past by creating barriers and by calling for protection. As a matter
of fact nothing can protect such a minority or a group less than a barrier
which separates it from the majority. It makes it a permanently isolated
group and it prevents it from any kind of tendency to bring it closer to
the other groups in the country." 2

Articles 29 and 30
Articles 29 and 30 both confer certain rights on the minorities Art. 30 specifically
mentions the right of all minorities to establish and administer educational
institutions.

ARTICLE 29 [PROTECTION OF INTEREST OF MINORITIES]

(1) Any section of the citizens residing in the territory of India or any part
thereof having a distinct language, script or culture of his own shall
have the right to conserve the same.
(2) No citizen shall be denied admission into any educational institution
maintained by the State or receiving aid out of State funds on grounds
only of religion, race, caste, language or any of them.
Art. 29(1) is aimed to safeguard the cultural rights of the minorities. If a section of
the citizens possess a distinct language, script or culture then such citizens shall

2. B.K. Sharma, p. 111.


214 Constitutional Law o f India- II
Rights to Minorities 215

have the right to conserve it. The language, script and culture can be effectively
conserved through educational institutions. Thus, it flows from this that there is a right to
establish and maintain educational institutions. This right is expressly conferred by
Art. 30(1). But this right is subject to the limitations contained in Art. 29(2).
Right to conserve the language of citizens includes the right to agitate for the
protection of language (Jagdev Singh v Pratap Singh AIR 1965 SC 183). Unlike Art.
19(1), Art. 29(1) is not subject to any reasonable restrictions.
Art. 15 also, like Art. 29(2), prohibits discrimination on grounds of religion, etc.
But the scope of two Articles is different:55
(i) Art. 15 protects all citizens against the State whereas the protection of Art.
29(2) extends to the State or anybody else who denies the right conferred
by it.
(ii) Art. 15 protects all citizens against discrimination generally, while Art. 29(2)
is a protection against a particular species of wrong, namely, denial of
admission into educational institutions of specific kind.
(iii) The specific grounds on which discrimination is prohibited is not the same in
two Articles. ‘Place of birth’ and ‘sex’ do not occur in Art. 29(2), while
‘language’ is not mentioned as a ground in Art. 15. Thus, under Art. 29(2), a
girl student can be denied admission in educational institutions.
Art. 29(2) is quite general and applies to all citizens, whether they belong to majority
or minority groups. Therefore, a school run by minority, if it is aided by State funds,
cannot refuse admission to boys of other communities. Also, the State cannot direct
such school to restrict admissions to their own community. However, nothing in Art.
29(2) shall prevent the State from making reservations for backward classes under
Art, 15.
In State of Bombay v Bombay Educational Society {AIR 1954 SC 561), court
struck down an order of Bombay Government banning admission of those whose
language was not English into schools having English as a medium of instruction,
because it denied admission solely on the ground of language.
Art. 29(2) would not be attracted where a person is refused admission because
he does not possess the requisite qualifications or has not secured prescribed
percentage of marks or has not applied within the prescribed time.
ARTICLE 30 [RIGHT OF
MINORITIES TO ESTABLISH AND ADMINISTER
EDUCATIONAL INSTITUTIONS]56

(1) All minorities, whether based on religion or language, shall have the right to
establish and administer educational institutions of their choice.

55 Art. 29(2) and Art, 15(1) of the Constitution both prohibits discrimination on
grounds of religion, race, caste, sex or place of birth. Discuss their points of
difference. [C.L.C.-2007]
56 Explain briefly the scope and content of right of minorities to establish and
administer educational institutions of their choice. [C.LC.-20Q6] [D.U.-2008]
If the words “All minorities .......... ” in Art. 30 are replaced by the words “AH
sections of citizens having distinct language or religion ... ", what fundamental
rights of the minorities would be affected? [I. A. S.-2002]
216 Constitutional Law o f India- II

(2)The State shall not, in granting aid to educational institutions, discriminate


against any educational institution on the ground that it is under the
management of a minority
The minorities derive their right to establish and administer educational institutions
from Art. 30(1). The right is conferred by this clause on two types of minorities -
religious and linguistic. The choice is left to the minorities to determine the type of
institution they would like to establish. The power of the State to determine the
medium of education (i.e. the language in which lessons will be given) cannot take
away the right of the minorities to give education through their language (D.A. V.
College v State of Punjab AIR 1971 SC 1731).
However, there is no violation of Art. 29 or 30 where a State makes the mother-
tongue the medium of instruction at the primary level and at higher stages implements
the 3-language formula by making the State language compulsory [English Medium
Parents Asscn. v State of Karnataka (1994) 1 SCC 550], The State may by law
require the minority to teach the State language. Such law is reasonable and does not
violate minority rights [Usha Mehta v State of Mahaiashtra (2004) 6 SCC 264]
The word "establish" indicates the right to bring into existence, while the right to
“administer” means the right to effectively manage and conduct the affairs of
institution. An educational institution which has not been established by a religious
minority cannot claim the right to administer it. in Naresh Agarwal v Bharat (2006), the
petitioners (Hindu Students) were denied admissions to Post Graduate Medical
Courses in Aligarh Muslim University (A.M U.) for the 2005-06 Session. They
challenged the validity of rule which declared the A.M.U. a minority institution and
allowed 50 per cent reservation to Muslim students. The Allahabad High Court held
thatA.M.U. was not a minority institution. The court struck down the amendment made
to this effect in the statute of A.M.U. for reservation to Muslim Students. The court
followed the Aziz Basha v UOI (AIR 1968 SC 662) wherein the Supreme Court had
held that Aligarh University was established by an Act of Parliament, not by any
Muslim 57

57 Can an educational institution set up under an Act of Parliament be considered to


be a minority institution? [L.C.11-2007]
Can minority status be conferred by legislation? [D.U -2008]
\
\
I
I
Rights to Minorities 217

It is not necessary that the institution is for the benefit of the minority exclusively. It
can admit persons belonging to non-minority community also.

Relationship Between Art. 29(1) and Art. 30(1) 6


In St. Xaviers College v State of Gujarat (AIR 1974 SC 1389), the extent and scope
of Art. 30(1), and its relationship with Art. 29(1), was discussed by the Supreme Court.
On behalf of the State, it was contended that protection to minorities as under Art. 30(1)
was not available to this college because it was not founded for the conservation of
language, script or culture as mentioned in Art. 29(1). Held that Art. 30(1) covers
institutions imparting general secular education. The object of this Article is to enable
children of minorities to go out in the world fully equipped. Arts. 29 and 30 create two
separate rights though it is possible that the rights might meet in a given case. The
court pointed out the following distinctions between two Articles:
(i) Art. 29(1) confers right on any section of citizens, thus including majority; Art.
30(1) confers right only on religious or linguistic minorities.
(ii) Art. 29(1) is concerned with only three subjects viz. language, script or
culture; Art. 30(1) deals with minorities based on language or religion.
(iii) Art. 2.9(1) is concerned with right to conserve language, etc.; Art. 30(1) deals
with right to administer educational institutions.
(iv) Art. 29(1) does not deal with education as such; Art. 30(1) deals only with
establishment and administration of educational institutions.
Thus, conseivation of language, script or culture under Art. 2.9(1) may be by means
wholly unconnected with the educational institutions and similarly, establishment and
administration of educational institutions under Art. 30(1) may be unconnected with
any motive to conserve language, script or culture. It is also to be noted that Art. 30(1)
does not say that minorities based on religion should establish institutions for teaching
religion or that linguistic minorities should establish institutions for teaching their own
language alone.

Whether Minority Institutions have a Fundamental Right to Affiliation


ln
St. Xaviers College v State of Gujarat, held, that there is no such fundamental ri9ht
to affiliation, but to deny affiliation or recognition except upon certain terms and
c
°nditions amount to the surrender of their right to administer under Art. 30(1). When a
minority institution applies to a university for affiliation, it expresses its choice to
Participate in the system of general education. Affiliation is for regulating the
ed
ucational character and content of minority institutions. These regulations are not

Discuss the relationship between Art. 29 and Art. 30 of the Constitution.

only reasonable in interest of general secular education but also improve the[I.A.S.-2001]
strength of
minority institutions. Therefore, measures which regulate the course of study, the
qualifications and appointment of teachers, the conditions of employment of teachers,
health and hygiene of students, facilities for libraries and labs, are all compared in matters
germane to affiliation of minority institutions. These regulatory measures for affiliation are
for uniformity, efficiency and excellence in educational courses and do not violate rights
XVI
under Art 30. Constitutional Law o f India- II
In N. Ammad v Emjay High School (1998) 6 SCC 674, it was held that a school
which is otherwise a minority school would continue to be so whether the government
declared it as such or not. When the government declared the school as a minority school
it has, recognised a factual position that the school was established and is being
administered by a minority community. The declaration is only an open acceptance of a
legal character which should necessarily have existed antecedent to such declaration.
In Re Kerala Education Bill, 1967 (AIR 1958 SC 956), the court did not uphold the
validity of clause 14 and 15 in the Kerala Education Bill, 1957, which authorised the
Government to take over any aided school under certain circumstances. Those clauses
amounted to expropriation of the schools. The schools were recognised on condition that
they submitted to those clauses. Such submission amounted to surrender of right under
Art. 30. The State cannot do indirectly what it cannot do directly.58
In State of T.N. v Melapalayam Muslim MegalirK.alvi Sangam (AIR 1998 Mad 91), it
was held that Minorities could not be asked to maintain their educational institutions
without State aid. The educational institutions are not business houses. They do not
generate wealth and therefore they could not survive without public funds or private aid.
The Minorities could not be saddled with the burden of maintaining educational institutions
without grant-in-aid. The paucity of finance could not be a valid ground to deny aid to
minority institutions. The same view was held in Re Kerala Education Bill case.

Power of Government to Regulate Minority run-lnstitutions59


The right conferred by Art. 30(1) is not an absolute right. The State may step in to maintain
the standard of education. It can give security and protection to teachers and other staff (by
laying down terms and conditions of service). Regulations maV' be made in the interest of
discipline, health, sanitation, morality, public order, etc. The Industrial Disputes Act which
is a social security measure designed for the welfa re of the labour applies to all and
minority institutions are no exception

58 Discuss the validity of the following act of the State vis-S-vis right of religioU*
minority: Taking over of the management of the minority educational institution on
seeking State-aid. [C,L,C.-200'>
59 What is the scope of State power to regulate administrative aspects of institutions?
Discuss in the light of recent decisions. [LC.H-ZW
Rights to Minorities 219
An institution established for commercial purpose can lay no claim for being
protected under Art. 30(1). The regulatory measures should not abridge the right of
administration. The minority has the right to choose the members of the governing council,
and the teachers. The regulations imposed must be reasonable and must regulate the
educative character of the institution.
The power of the government to take over management of an institution which has
failed to carry out the directions of the government was declared void on the ground that it
robbed the minority of its right.
In St. Xaviers College v State of Gujarat, the Government Act provided that every
college shall be under the . management of government body which shall include amongst
its members, a representative of the University nominated by Vice- Chancellor and
representatives of teachers, students, etc. It also provides for a selection committee,
consisting of representatives nominated by vice-chancellor, for recruitment of the Principal
and staff members. These provisions in effect displace the managing committee of college.
Further, no member of the teaching/non-teaching staff of affiliated college shall be
dismissed or reduced in rank except after an inquiry and he be given a reasonable
opportunity of being heard and to make representation against penalty, and the penalty is
approved by Vice-Chancellor.
The court held that these provisions abridged the right of minority to administer
educational institutions of their choice and, therefore, did not apply to minority institutions.
Under Art. 30(1), the “right to administer" is the right to ‘conduct’ and ‘manage’ the affairs
of institution. The right to administer is said to consist of 4 principal matters: right to choose
its managing or governing body; right to choose its teachers, who have compatibility with
the aims and aspirations of institution; right to admit students of their choice subject to
reasonable regulations about academic qualifications; and right to use its properties and
assets for benefit of institution. Thus, the management must be free of control so that
founders of their community can mould the institution as they think fit in accordance with
their ideas of how the interest of community in general and institution in particular will be
served.
However, right to administer is not an absolute right. Just as regulatory measures
are necessary for maintaining educational character and content of minority institutions,
similarly regulations are necessary for ensuring efficient and sound administration. The
right to administer is not the right to maladministration fvide re Kerala’ case).
The right to administer implies a correlative duty to good administration. The
regulatory measures should not restrict the right of administration but facilitate it through
instrumentality of management of minority institution. In the present case, the said
provisions have the effect of displacing the management and entrusting it to a different
agency. The autonomy in administration is lost. Autonomy is very hecessary to administer
effectively the affairs of the institution.
LEADING CASE: FRANK ANTHONY PUBLIC SCHOOL
EMPLOYEES ASSOCIATION v UNION OF INDIA (AIR 1987 SC 311)
Facts and Issues - The Delhi School Education Act was passed to provide better
organisation and development of school education in Delhi. Sections 8 to 11 deals with
“terms and conditions of service of employees of recognised private schools”. Sec. 10
requires that scales of pay and other conditions shall not be less than those of the
employees of government schools of Delhi. But Sec, 12 makes these beneficial provisions
of Secs. 8 to 11 inapplicable to 'unaided' minority schools.
The scales of pay and other conditions of service of teachers and employees of
Frank Anthony School, Delhi compare very unfavourably with those of their counterparts of
Delhi administration schools, thus their association sought a declaration from court that
Sec. 12 was unconstitutional as being violative of Arts. 14, 21 and 23 The management of
school contended that Sec. 12 is valid, as Sec, 8 to 11 interfere with the right of
management vested in minorities under Art, 30(1). The payment of salary was the part of
the right of management to appoint members of staff, especially in an unaided institution.
XVI
Constitutional Law o f India- II
Observations - The Supreme Court observed that “the idea of giving special rights to
minorities is not to have a kind of a privileged or pampered section of the population but to
give them a sense of security and feeling of confidence”. The words “of their own choice” in
Art. 30(1) indicate that the extent of the right to be determined, not with reference to any
concept of State necessity and general interest of society, but with reference to
educational institutions themselves i.e. making institutions effective vehicles of education
for the minorities or other persons who resort to them. The regulatory measures aimed at
making minority institutions effective instruments for imparting education, without nullifying
management’s right in substantial measures, are permissible.
In St. Xaviers College case, the court observed that although disciplinary control
over teachers of a minority institution would be with the management, regulations can be
made for ensuring proper conditions of service of teachers and for securing a fair
procedure in the matter of disciplinary action against teachers. In All Saints High School v
State of
A. P. (AIR 1980 SC 1042), court said that for maintenance of educational standard of
institution it was necessary to ensure that it was competently staffed, and therefore,
service conditions viz. minimum qualifications, pay scales, other benefits of service and
safeguards which must be observed before their removal or dismissal, were permissible
measures of a regulatory character.
Decision - The Supreme Court held that the statutory measures regulating terms and
conditions of service of teachers and other employees of

minority educational institutions, for maintaining educational standards and


excellence, are not violative of rights of minorities under Art. 30(1). Secs.
8 to 11, as regulatory in nature, do not violate Art. 30(1). The excellence of
educational institutions would directly depend upon the excellence of
teaching staff, which in turn, would depend upon the quality and contentment
of teachers.
The management of a minority institution cannot be permitted under
the guise of Art. 30(1), to oppress or exploit its employees which would
bound to lead to discontent and adversely affect the object of making the
institution an effective vehicle of education for minorities or other persons.
Also, mere prescription of scales of pay and other service conditions would
not jeopardise the right of the management to appoint teachers of their
choice. Thus, it was held that Sec. 12 was discriminatory and violates Art. 14
and Art. 39(d) (‘equal pay for equal work’).
This decision will be able to bring minority institutions in the main
stream of nation and provide social justice to employees working in them.]
in St. Stephens College v University of Delhi (AIR 1992 SC 1630), the validity of
admission programme and the preference given to Christian students (a 10%
relaxation for them) by the St. Stephen’s College, was challenged by Delhi University
and Student’s Union, as violative of Delhi University circulars for admission to
B.A./B.Com./
B. Sc. courses. It was contended that the college was bound to follow University
rules for admission. The Supreme Court held that the college, being a minority
institution,, was not bound to follow the directions of the university. It may select
students for admission by an interview and may not abide by the university rule to
select students on the basis of marks obtained in the written examination. It also laid
down that up to 50% seats may R ibe
g hreserved
ts to M fori n o r i t i ebelonging
students s 221
to the Christian
community. Other students may be admitted purely on merits. This has now been
overruled by the Supreme Court in T.M.A. Pai Foundation case (discussed later).
In St. Stephens case, also held that a college after affiliation does not lose its
minority character. The court also observed that the fact that Art. 29(2) applies t°
minorities as well as non-minorities does not mean that it was intended to nullify 'he
special right guaranteed to minorities under Art. 30(1). Art. 29(2) deals with non-
discrimination and it is available only to individuals. The general equality by non-
discrimination is not the only goal of minorities. Protection of interests and institutions
ar|
d advancement of opportunity are just as important. Differential treatment that
distinguishes them from the majority is a must to preserve their basic characteristics.
Il
's necessary to mediate between Art. 29(2) and Art. 30(1), between letter and sPirit of
these Articles, between traditions of past and the convenience of the Person, between
society’s need for stability and its need for change.
In St. John’s Teachers Training Institute v State of T.N. (1993) 3 SCC 594, e
appellant challenged the validity of the Recognition Rules made by the Government
_nder the T.N. Minority Schools (Recognition and Payment of Grants) Rules 1977 s
amended by the Order of 1991 on the ground that they were violative of Arts.

L
30(1) and 14. The Recognition Rules provided for the extent of land sizes of class
rooms, cost of library with 10,000 books, number of bathrooms, furnitures and lab
equipments, sports, music equipments, minimum qualifications for teaching and
non- teaching staffs, hostel, etc. The High Court dismissed the writ petition holding
that these conditions were regulatory in nature and framed with a view to promoting
excellence of educational standard and ensuring security of the services of teachers
and other employees. The minority institutions must be fully equipped with
educational excellence to keep in step with other institutions The Supreme Court
agreed with the High Court.
However, in N. Ammad v Emjay High School (1998) 6 SCC 674, the
Supreme Court relying on Ahmedabad St. Xavier's College case held that the
management's right to choose a qualified person as the Headmaster of the school is
weil insulated by the protective cover of Art.30(1) of the Constitution and it cannot
be chiselled out through any legislative act or executive rule except for fixing up the
qualifications and conditions of service for the post. Selection and appointment of
Headmaster in a school (or Principal of a college) are of prime importance in
administration of that educational institution. If management of the school is not
given very wide freedom to choose the personnel for holding such a key post,
subject of course to the restrictions regarding qualifications to be prescribed by the
State, the ‘right to administer’ the school would get much diminished.
The right to choose the principal is an important part of the right to
administer. Even if the institution is aided there can be no interference. The minority
has the right to select the principal [Secretary, Malankar Syrian Catholic College v T
Jose, 2006 (13) SCALE 1],
In State of Bihar v Syed Asad Mirza (AIR 1997 SC 2425), it was held that
creation of a post in a minority institution without prior sanction of the State
governmentA/ice-chancellor, is a valid exercise of power keeping in view the rights
given to the Minorities by Art. 30 of the Constitution. The persons so appointed
would be entitled to grant-in-aid in view of Art. 30(1).
XVI
Constitutional Law o f India- II
In Lilly Kurian v University Appellate Tribunal (1997) 2 SCC 240, Secs. 60(7)
and 61 of the Kerala University Act, 1974 were in question. In the case of a dispute
between management and teacher of a private college relating to conditions of
service, Sec. 60(7) conferred power on University Appellate Tribunal to pass such
order as it may deem fit including order of reinstatement, and Sec. 62 conferred a
right of appeal on the teacher in respect of past disputes. It was held that Secs.
60(7) and 61 were violative of the rights of minority educational institutions under
Art. 30(1). The Court observed: Conferment of a right of appeal to an outside
authority took away the disciplinary power of a minority educational authority,
particularly because the appellate power was unlimited and undefined. The
Appellate Tribunal can even order reinstatement of a dismissed teacher. Such an
unguided and uncanalised power which could be exercised in appeal constituted
interference with the right of a minority institution to administer its own institutions. It
could not be construed merely as a check on maladministration.
Leading Case: t.m.a. pai foundation V state
Rights to Minorities 223
OF KARNATAKA
[(2002) 8 SCC 481]
In this case, an 11-Judge Constitution Bench (6:5 majority) of the Supreme Court
held that the right to establish and administer educational institutions is guaranteed
to all citizens [Arts. 19(1)(g) and 26] and to minorities specifically under Art. 30.
These rights are not limited to minorities and are available to all persons The State
is to be regarded as the unit for determining both “linguistic minority” as well as
"religious minority." The question whether a sect or denomination of a religion can
claim minority status even though followers of that religion are in majority was left
unanswered. The question as to what is the criteria for treating an 'educational
institution’ as minority institution was left open.
The rights of minorities, under Art. 30, cover professional institutions. Minority
communities have an untrammelled right to establish and administer unaided
educational institutions, while the institutions which receive State aid could be
subject to Government rules and regulations. In respect of ’unaided’ institutions, the
only regulation a State Government or University can make is regarding the
qualifications and minimum conditions of eligibility of teachers and principal (and
also in regard to the service conditions) in the interest of academic standards. The
State can make no laws regarding admissions or fees in such institutions.
Conditions of recognition and affiliation by or to a Board or University have to be
complied. But the appointment of teaching and non teaching staff and control over
them will vest in the management.
A minority institution does not cease to be so, the moment grant- in-aid is
received An aided institution will be entitled to have the right of admission of
students belonging to the minority group. At the same time, it would be required to
admit a reasonable extent of non-minority students, so that the rights under Art.
30(1) are not substantially impaired and further, citizen’s rights under Art. 29(2) are
not infringed The State can provide that in aided institutions consideration is shown
to weaker sections.
A minority institution may have its own procedure and method of admission
but the procedure must be fair and transparent. Selection in professional and higher
education colleges should be based on merit (that also applies to minority group
admissions). In case of aided professional institutions the State may prescribe that
only those persons may be admitted who have passed a common entrance test.
The Apex Court also ruled that the basic ratio in the St. Stephen’s case
(reservation of 50 per cent of the seats in aided institutions to the management’s
discretion) is correct, however, rigid per centage cannot be stipulated. The Court
empowered States to fix quotas for minority students taking into account the type of
institution, population and educational needs of the minorities. The Court reiterated
that the constitutional rights conferred on minorities to establish and administer
educational institutions of their choice is not absolute or above other laws.
The court also held as “unconstitutional” the Scheme framed by the Apex
Court in the Unnikrishnan’s case viz. fixing ‘free’ and ‘payment’ seats. The court
lifted the regulation on fees to be charged by unaided institutions (i.e. they can
charge any fees) with the proviso that there would not be “profiteering”. No
institution can charge capitation fee. Reasonable surplus to meet the cost of
expansion and augmentation of facilities does not, however, amount to profiteering.

LEADING CASE: ISLAMIC ACADEMY OF EDUCATION v STATE OF KARNATAKA


[(2003) 6
SCC 697]
A five-judge Bench of the Supreme Court in this case clarified the doubts read in
T.M.A. Pai Foundation case (majority decision). In this case, the following questions
XVI
arose for consideration: C o n s t i t u t i o n a l L a w o f I n d i a - I I
(1) Whether the educational institutions are entitled to fix their own fee
structure?
(2) Whether minority and non-minority educational institutions stand on
the same footing and have the same rights?
(3) Whether private unaided professional colleges are entitled to fill in
their seats, to the extent of 100%, and if not, to what extent?
(4) Whether private unaided professional colleges are entitled to admit
students by evolving their own method of admission?
Regarding the first question, the court held that each institute will be entitled to have
its own fee structure. But there can be no profiteering and capitation fees cannot be
charged. Imparting of education is essentially charitable in nature. Thus the
surplus/profit that can be generated must be only for the benefit/use of that
educational institution. It cannot be used for personal gain or for any other business
or enterprise. As, at present, there are statutes/regulations which govern the fixation
of fees and as this court has not yet decided the validity of those
statutes/regulations, it is directed that in order to give effect to the judgment, in TMA
Pai’s case, the respective State governments shall constitute a separate committee
in each State, to be headed by a retired judge of the High Court (to be nominated by
the Chief Justice of that State), to approve the fee structure of the institutions
conducting medical and engineering courses.
Regarding the second question, the court held that laws,
regulations, etc. cannot be such that they favour majority institutions
over minority institutions. Non-minority educational institutions would
have the same rights as those conferred on minority educational
institutions by Art. 30, but they do not have the protection of Art. 30.
Even though the principle behind Art. 30 is to ensure that minorities are
protected and are given an equal treatment yet the special right under
Art. 30 do give them certain advantages. For instance, they have a
preferential right to admit students of their own community/language.
Regarding the third and fourth questions, the court held that the
committee mentioned above would ensure that the tests conducted for
admission by these institutions (i.e. private unaided professional
colleges) were fair and transparent. In fixing percentage of quota for
students to be admitted by the Management of unaided minority
professional colleges, the State would keep in mind, apart from local
needs, the paramount interest/need of that community in the State.]
In Modern School v UOI (AIR 2004 SC 2236), it was held that in determination of
the fee structure, unaided educational institutions exercise a great autonomy as
they are entitled to generate a reasonable surplus for development of education
and expansion of the institution itself . What is prohibited is commercialization of
education/ profiteering and the diversion of the profit/surplus for any other use.
Hence a balance has to be struck between autonomy of such institutions and
measures to be taken to prevent commercialization of education.

Leading Case: p.a. inamdar V state of Maharashtra60 (AIR 2005 SC


3226)

60 Does the rights of minorities to establish and administer educational


institutions of their choice include right to regulate admissions and fee
structure? Define minority for this purpose. [L.C.II-
2006]
Rights to Minorities 225

In this case, the Apex Court (7-Judge Bench) clarified some issues
raised in T.M.A. Pai case (2002) 8 SCC 481 and Islamic Academy case
(2003)
6 SCC 697, relating to rights of professional unaided (minority and non-
minority) educational institutions. The court held:
(i) ‘Minority,’ whether linguistic or religious, is to be
determined by the demography of a State. ‘Minority
educational institutions’ may be classified into 3
categories: (i) those which do not seek aid or recognition,
(ii) those which want aid, and (iii) those which want only
recognition and no aid. Category (i) may exercise their
right without interference. Category (ii) and (iii) may be
subjected to restrictions and conditions.
XVI
Constitutional Law o f India- II

(ii) Each minority institution is entitled to have its own fee structure
subject to the condition that there can be no profiteering and
capitation fees cannot be charged- To check profiteering and
capitation fees, the method of admission has to be regulated so
that admissions are based on merit and is transparent and the
students are not exploited. Hence it is permissible to regulate
admission and fee structure for achieving the same.
(iii) Minority institutions stand on a better footing than non- minority
institutions. Minority educational institutions have a guarantee or
assurance to establish and administer educational institutions of
their choice. State legislation cannot favour non-minority institutions
over minority institutions. The difference arises because of Art. 30,
the protection whereunder is available to minority institutions only
(iv) The unaided professional institutions will have full autonomy in their
administration, but the principle of merit cannot be sacrificed, as
excellence in professions is in the national interest. Right to
administer does not include right to mal- administer.
(v) Without interfering with the autonomy of unaided institutions, the
object of merit-based admissions can be secured by insisting on it
as a condition to the grant of recognition and subject to the
recognition of merit (via conduct of a common entrance test by a
centralized mechanism), the management can be given certain
discretion in admitting students. The State can provide for
reservation in favour of financially or socially backward sections of
the society. The plea that each minority unaided institution can hold
its own admission test cannot be sustained (to avoid great hardship
to hapless students).
The court, however, clarified that neither can the policy of reservation be enforced
by the State nor can any quota or percentage of admissions be carved out to be
appropriated by the State in a minority or non-minority unaided educational
institution. There is nothing in T.M.A. Pai case (2002) 8 SCC 481 or in Kerala
Education Bill case (1959 SCR 995) (approved by Pai Foundation), which permits
the same. Else it would amount to nationalization of seats, which has been
specifically disapproved in T.M.A. Pai case.
Such State imposition would constitute a serious encroachment on the right
and autonomy of private professional institutions and also cannot be held to be a
regulatory measure in the interest of the minority within the meaning of Art. 30(1),
or, a reasonable restriction under Art. 19(6) [Right to establish an educational
institution is an occupation protected by
Art 19(1 )(g)]. The judgment in T.M.A. Pai case only mentioned the possible
consensual arrangements, which could be reached between the minority
institution and the State. In T.M.A. Pai it has been very clearly held at several
places that unaided institutions should be given greater autonomy in
determination of admission procedure and fee structure. State regulation should
be minimal and only with a view to maintain fairness and transparency in
Rights to Minorities 227

admission procedure and to curb capitation fees.


(vi) The prescription for percentage of seats, that is allotment of
different quotas such as management seats, State’s quota,
appropriated by the State for allotment to reserved categories, etc.
has to be done by the State in accordance with the “local needs”
and the interests/needs of that minority community'in the State,
both deserving paramount consideration.
(vii) The merit criterion in the opinion of Sinha J., was required to be
associated with the leval of education - to be judged like a pyramid.
At the Kindergarten, primary, secondary levels, minorities may
have 100% quota. At this level, the merit may not have much
relevance at all but at the level of higher education and in
particular, professional education and postgraduate-level
education, merit indisputably should be a relevant criterion.
Services of doctors, engineers and other professionals coming out
from the institutions of professional excellence must be made
available to the entire country and not to any particular class or
group of people.
(viii) Limited number of seats, not exceeding 15%, may be made
available to Non Resident Indians (NRIs).
(ix) There is nothing wrong in holding entrance test for a group of
Institutions. It would be permissible to regulate by a single window
procedure.
(x) The right to establish and administer an institution, the phrase as
employed in Art. 30(1), comprises the following rights: (a) to admit
students; (b) to set up a reasonable fee structure; (c) to constitute a
governing body; (d) to appoint staff (teaching and non-teaching);
and (e) to take action if there is dereliction of duty on the part of
any of the employees.
However, in regulating minority professional educational institutions, a balance
has to be struck between the two objectives: (a) that of ensuring the standard of
excellence of the institution, and (b) that of preserving the right of the minority
under Arts. 30 and 29. Subject to the reconciliation of the two objectives, any
regulation accompanying affiliation/recognition
228 Constitutional Law o f India- II Rights to Minorities 229
or grant of aid must satisfy four tests: (1) Reasonableness and rationality; The opening of a medical or engineering college has no relation with preservation
(2) Effective vehicle of education; (3) Maintenance of excellence: and (4) of culture. Non-religious and non-cultural education cannot be segmented into
Protection of minority character. minority and majority. It is not conducive to the growth of a secular and integrated
The regulations framed in the national interest (based on the society. Behind the facade of minority educational institutions lies a whole world
considerations of public safety, national security and national of elements engaged in amassing wealth by way of capitation fee and other
integration) must necessarily apply to all educational institutions, under-the-table payments. Education which is an industry being run with a profit
whether unaided or aided, unrecognized or unaffiliated. Such limitations motive has no claim to minority protection Even non-citizens residing in India are
must necessarily be read into Art. 30. entitled to the benefit of Art. 30 which was never the intention of the Constitution
The Bench still felt that some of the questions remained makers.12
unanswered after Pai Foundation and such questions need to be
QUESTIONS AND ANSWERS
answered by a laiger bench than Pai Foundation, but left it to posterity.]

Q. 1 The Association of Minorities claims, among others, the following


General Comments rights under Art. 30(1) of the Constitution:
Mature nations are deriving strength by promoting the “melting pot” phenomenon
(a) to have their own managing committees (the University Rules, on
and encouraging minorities to join the mainstream. Exhorting the Minorities
the other hand, provide that the managing committee of college
Commissions of the Centre and of States to prevent “generating feelings of multi-
will-have among others a nominee of the Vice-Chancellor, a
nationalism in various sections of people of Bharat", the Supreme Court recently representative of teaching staff, a representative of karamcharies
in Bat Patil v Union of India (2005) 6 SCC 657, speaking through Dharmadhikari, and a representative of student’s union. It also provides for a
J. has said: “We have to develop such enlightened citizenship where each citizen selection committee, consisting of representatives nominated by
of whatever religion or language is more concerned about his duties and Vice-Chancellor, for recruitment of Principal and staff members).
responsibilities to protect rights of the other group than asserting his own rights.”
Speaking a note of warning he added: “Differential treatments to linguistic (b) to prescribe qualifications and other conditions of services including
salary for the recruitment of teachers and other employees (the
minorities based on language within the State is understandable but if the same
University Rules, on the other hand, provide that the university will
concept for minorities on the basis of religion, is encouraged, the whole country,
prescribe qualifications and other conditions of services for the
which is already under class and social conflicts due to various divisive forces, will recruitment of teachers. The Public Service Commission will select
further face division on the basis of religious diversities. Such claims to minority candidates for appointment as teachers).
status based on religion would increase in the fond hope of various sections of
people getting special protections, privileges and treatment as part of (c) to have their rules for disciplinary actions against employees (the
constitutional guarantee. Encouragement to such fissiparous tendencies would be University Rules, on the other hand, provide that no disciplinary
a serious jolt to the secular structure of constitutional democracy.” 61 action against an employee will be taken without observing
A feeling is developing that minorities have been conferred more rights principles of natural justice e.g. giving him a reasonable opportunity
than the majority. Minorities had been made more equal - to borrow a phrase from of being heard. Further, no dismissal or revision will be effective
unless approved by the Vice-Chancellor or by any other officer duly
George Orwell. Even some denominations of the majority Hindu community had
authorised by the Vice-Chancellor).
been compelled to lay claim to minority status The Preamble proclaims the ideal
of unity and integrity of the nation. Art. 14 guarantees equality. Art. 15 prohibits (d) to prescribe qualifications for admission of students, and, to
discrimination on the ground of religion. Yet we had a situation where the prescribe their own procedure for admission of students (the
minorities had a vested interest to remain separate and away from the national University Rules, on the other hand, provide that the college will
mainstream. They had privileges which were denied to the Hindu majority. Unity follow the principles and procedures laid down by the university for
cannot be achieved by division. 62 admission of students).
In a plural society all cultures and languages must be allowed to flourish.
(e) to reserve 75% seats for students belonging to their respective
communities.
61 S. Malik (Ed.), Supreme Court Yearly Digest 2005, Eastern Book Co., p.
241 (2006).
62 B.K. Sharma, p. 106. 1 2 . B.K. Sharma, pp. 110-111.
XVI
Constitutional Law o f India- II

Examine the validity of the above claims. [C.LC.-92/93/94/95]


In the light of the right of minorities to establish and administer
educational institutions of their choice, examine the validity of the
following:

(i) A State law providing for appointment of all teachers in its coileges
with prior approval of State.

(ii) A State law allotting 50% of all seats for non-minority students in
aided minority colleges in the State.

(iii) A State iaw providing uniform and common syllabus in engineering


studies in all colieges throughout the State. [C.LC.-
2006]

A minority educational institution (unaided) is required by the


Education Code framed by the Government to pay to its teachers,
salary at the prescribed-fate. The institution shows its inability and
contends that the enforcement of the Code amounts to interference
with its constitutionality guaranteed right of administration. Decide
giving reasons. [I.A.S.-99]
A. 1 Article 30(1) provides that all minorities, whether based on religion or
language, shall have the right to establish and administer educational
institutions of their choice.
The 'right to administer' means the right to effectively manage and conduct
the affairs of institution The right to administer is said to consist of four principal
matters: right to choose its managing or governing body, right to choose its
teachers who have compatibility with the aims and aspirations of institution., right
to admit students of their choice subject to reasonable regulations about
academic qualifications, and, right to use its properties and assets for benefit of
institution (St. Xaviers College v State of Gujarat).
However, right to administer is not an absolute right. Just as regulatory
measures are necessary for maintaining educational character and content of
minority institutions similarly regulations are necessary for ensuring efficient and
sound administration. The right to administer is not the right to maladminister (Re
Kerala Education Bill case).
The regulatory measures aimed at making minority institutions effective
instruments for imparting education, without nullifying management’s right in
substantial measures, are permissible (Frank Anthony Public School Employees
Association v UOI). In St. Xavier College case, the court observed that the
distinction is between a restriction on the right of administration and a regulation
prescribing the manner of administration. To regulate is not to restrict, but to
facilitate effective exercise of the very right.
The following regulatory measures are held to be permissible: measures
which regulate the educational standards, the course of study, the qualifications
and
Rights to Minorities 231

appointment of teachers, the conditions of the employment of employees,


safeguards which must be observed before removal of employees, the academic
qualifications for admission of students, health and hygiene of students, facilities
for libraries and labs, etc.
In St. John’s Teachers' Training Institute case, the conditions for grant of
recognition provided for the class rooms size, library with 10,000 books, number
of bathrooms, furnitures and lab equipments, sports, minimum qualifications for
teachers and other employees, etc. Held that these conditions were regulatory in
nature and framed with a view to promoting excellence of educational standard
and ensuring security of the services of teachers and other employees. The
minority institutions must be fully equipped with educational excellence to keep in
step with other institutions.
In PA. Inamdar v State of Maharashtra (AIR 2005 SC 3226), the court
observed: The right to establish and administer an institution, the phrase as
employed in Art. 30(1), comprises the following rights: (a) to admit, students; (b)
to set up a reasonable fee structure; (c) to constitute a governing body; (d) to
appoint staff (teaching and non-teaching); and (e) to take action if there is
dereliction of duty on the part of any of the employees.
The court further observed: However, in regulating minority professional
educational institutions, a balance has to be struck between the two objectives:
(a) that of ensuring the standard of excellence of the institution, and (b) that of
preserving the right of the minority under Arts. 30 and 29. Subject to the r
econciliation of the two objectives, any regulation accompanying
affiliation/recognition or grant of aid must satisfy four tests: (1) Reasonableness
and rationality; (2) Effective vehicle of education; (3) Maintenance of excellence;
and (4) Protection of minority character. The regulations framed in the national
interest (based on the considerations of public safety, national security and
national integration) must necessarily apply to all educational institutions, whether
unaided or aided, unrecognized or unaffiliated. Such limitations must necessarily
be read into Art. 30.
Now, coming to the first case in question:
(a) The University Rules, in effect, displace the managing committee of
College and entrust it to a different agency. The autonomy in
administration is lost. Autonomy is very necessary to administer
effectively the affairs of the institution. The regulatory measures should
not restrict the right of administration but facilitate it through
instrumentality of management of minority institution. A minority
institution have a right under Art. 30(1) to have its own managing
committee (St. Xavier College case).
(b) & (c) In All Saints High School v State of A.P, the Supreme Court held that for
maintenance of educational standard of institutions it is necessary to
ensure that it is competently staffed, and therefore, service conditions
viz. minimum qualifications, pay scales, other benefits of service, and
safeguards which must be observed before their removal or dissmissal,
are permissible measures of regulatory character.
XVI
Constitutional Law o f India- II

In Kerala Education Bill case, the court upheld the conditions designed to
protect ill-paid teachers who were rendering service to the nation, as permissible
regulations.
In Frank Anthony School Employees Association case, held that the
management of a minority institution can’t be permitted under the guise of Art.
30(1), to oppress or exploit its employees which would lead to discontent and
adversely affect the object of making the institution an effective vehicle of
education for minorities or other persons. Also, mere prescription of scales of pay
and other service conditions would not jeopardise the right of the management to
appoint teachers of their choice.
(d) !n PA. Inamdar case, the Apex Court observed: Without interfering with
the autonomy of unaided minority institutions, the object of merit-based
admissions can be secured by insisting on it as a condition to the grant
of recognition and subject to the recognition of merit (via conduct of a
common entrance test by a centralized mechanism), the management
can be given certain discretion in admitting students. The State can
provide for reservation in favour of financially or socially backward
sections of the society. The plea that each minority unaided institution
can hold its own admission test cannot be sustained (to avoid great
hardship to hapless students).
(e) In T.M.A. Pai case, The Apex Court ruled that the basic ratio in the St.
Stephen’s case (reservation of 50 per cent of the seats in aided
institutions to the management’s discretion) is correct; however, rigid
per centage cannot be stipulated. The court empowered States to fix
quotas for minority students taking into account the type of institution,
population and educational needs of the minorities. In PA. Inamdar
case, also, the Apex Court observed: The prescription for percentage of
seats, that is allotment of different quotas such as management seats,
State's quota, appropriated by the State for allotment to reserved
categories, etc. has to be done by the State in accordance with the
“local needs” and the interests/needs of that minority community in the
State, both deserving paramount consideration.

Second case in the question


(i) and (Ii) are invalid regulations; (iii) is a valid regulation by the State.

Q. 2 Discuss - Statutory regulation of educational standards and conditions


of employment in minority educational institutions - with reference
to decided cases. [/.AS.-90]
A. 2 Please refer to answer to the Question 1 above.
io
Right to Constitutional Remedies
(ARTICLE 32)
judicial Review: A Basic Feature
In order to maintain the supremacy of Constitution, there must be an independent
and impartial authority to decide disputes between units of federation. The
Supreme Court under our Constitution is such an arbitrator. It is the final
interpreter and guardian of Constitution, which keep the democratic structure
intact, by preventing the arbitrary use of governmental authority and safeguarding
the rights of citizens.
Philosophy of judicial review is rooted in the principle that Constitution is the
fundamental law, all governmental organs must not do anything which is
inconsistent with the provisions of Constitution; and the theory of limited
government. Thus, judicial review makes Constitution legalistic.
Judicial review means that the judiciary can declare a law or legislation as
unconstitutional if it is beyond the competence of legislature according to the
distribution of powers (under Art. 246), or it is in contravention of fundamental
rights or any of the mandatory provisions of the Constitution (e.g. Art. 301, 304).
Even in the absence of such express constitutional provisions, the court can
invalidate a law which contravenes any right or is ultra vires, for such power of
judicial review follows from the very nature of the Constitutional law
The ‘reasonable restrictions' in case of fundamental rights are subject to
court’s supervision. Judicial review is thus 'the interposition of judicial restraint on
the legislative as well as executive organs of the Government. Judicial review is a
‘basic feature’ of the Constitution and cannot be amended.
In L. Chandra Kumar v UOi (AIR 1997 SC 1125), held that the jurisdiction
conferred upon the High Courts under Arts. 226/227 and upon the Supreme Court
u
nder Art. 32 is part of the inviolable basic structure of the Constitution of India
[For a detailed discussion, see under the Chapter ‘Amendment of the
Constitution’].
The Supreme Court has always considered the power of judicial review
vested in the High Courts and in the Supreme Court under Arts. 226 and 32
re
spectiveiy, enabling legislative action to be subjected to the scrutiny of superior
234 Constitutional Law o f India- II
r Right to Constitutional Remedies 235

courts, to be integral to our Constitutional scheme. While several judgments have There is no need to resort to Art. 226 before approaching the Supreme
made specific references to this aspect [Beg, J. and Khanna, J. in Keshavananda Court under Art. 32. Thus, a person may directly move the Supreme Court. As
Bharati case; Chandrachud, CJ. and Bhagwati, J. in Minerva Mills; etc.] the rest regards the enforcement of the orders and direction of the Supreme Court, Art.
have made general observations highlighting the significance of this feature. 144 says that, “All authorities civil and judicial in the territory of India shall act in
Therefore the power of judicial review is an integral and essential feature of the aid of the Supreme Court. If any of these authorities fail to carry out the orders of
Constitution' constituting part of its basic structure. the Court, the Court can punish them for the contempt of the court.”

Appropriate Proceedings
ARTICLE 32 [RIGHT TO CONSTITUTIONAL REMEDIES] The Supreme Court has to be moved by appropriate proceedings. It means
proceedings which may be appropriate having regard to the nature of order,
direction or writ which the petitioner seeks to obtain from the court (Daryaov State
of U,P. AIR 1961 SC 1457).
Importance of Art. 32/ 226 The Court has been extremely liberal and favourable to the petitioner who
A right without a remedy is but a worthless declaration. A right becomes valuable approaches it. When it relates to the enforcement of fundamental rights of the
when there is an effective means to implement it. The existence of a right is felt poor, disabled or ignorant even a letter addressed by him to the Court can
only through the pronouncements of a court. Dr. Ambedkar regarded Art. 32 as legitimately be regarded as an appropriate proceeding. The letter need net be in
“the most important article of the Constitution without which this Constitution any particular form. It may not be addressed to the Court or the Chief Justice.
would be a nullity. It is the very soul of the Constitution and the very heart of it.” It Postcards addressed to any judge have been entertained as appropriate
is a substantive right not a mere procedural right. proceedings. It has come to be known as epistolatory proceedings.
Art. 32(1) states: “The right to move the Supreme Court by appropriate Appropriate proceedings may not be adversary proceedings which are
proceedings for the enforcement of the rights conferred by this Part is aimed at or against a particular person [Bandhua Mukti Morcha v UOI (1984) 3
guaranteed." Art. 32(2) deals with Supreme Court’s power to issue order or writs, SCC 161]. They may be inquisitorial where the court wiil make general inquiries
for the enforcement of fundamental rights. to find out whether a fundamental right is being trammelled.
The jurisdiction conferred on the Supreme Court by Art. 32 is an important The Supreme Court cannot refuse relief under Art. 32 on the ground that
part of the ‘basic structure’ of the Constitution because it is meaningless to confer the aggrieved person may have his remedy from other court; the disputed facts
fundamental rights without providing an effective remedy for their enforcement have to be investigated before the relief given; and that petitioner has not asked
when they are violated. Parliament can, however, empower the Supreme Court for proper writ applicable to his case. It is necessary that the petitioner is presently
with such a power under Art. 139. affected by the impugned order or the like i.e. his fundamental rights are either
The right to move the Supreme Court where a fundamental right has been threatened or violated. It is not necessary for him to wait till the actual threat has
infringed is itself a fundamental right. It is a constitutional remedy which has been taken place; however, his rights should be in imminent danger of being evaded.
guaranteed by the Constitution. Art. 32, thus, provides an expeditious and The maintainability of a petition will depend on the facts as they were on the date
inexpensive remedy for the protection of fundamental rights from legislative and of the petition and rot on subsequent events.
executive interference.
Effect of Guarantee
Article 226 (Concurrent Jurisdiction of High Courts) The effect of the use of the word guaranteed in Art. 32(1) is that the right to move
Article 226 enables the High Courts to issue writs for the enforcement of fundamental the Supreme Court where a fundamental right has been infringed is itself a
rights and also other ordinary legal rights. Thus, Arts. 32 and 226 provides the right fundamental right. The following consequences flow from this guarantee:
to constitutional remedies. The power of the High Court is wider than the power (i) The right cannot be suspended except as provided in Art. 359.
conferred by Art. 32 on the Supreme Court. However, a petition under Art. 32 (ii) Supreme Court is the guarantor of fundamental rights and it is the duty
may be filed to challenge the validity of a law with reference to a provision other of the Supreme Court to grant relief under Art. 32 where a fundamental
than those involving fundamental rights, provided it inevitably causes a restriction right has been violated.
on the enjoyment of fundamental rights.
It may be noted that the power of the High Court to issue writs cannot be in
derogation of the Supreme Court’s power. In other words, an order under Art. 32
will supersede the orders of the High Court previously passed.
(iii) An aggrieved person may approach the Supreme Court in the first instance.
XVI He need notC move
o n sta
i t High
u t i oCourt,
n a l Lexcept
a w o inf service
I n d i a -matters
II (where a person
has to go to the High Court).
(iv) Availability of alternate remedy is no bar to relief under Art. 32.
(v) There is no territorial limitation to the powers of the Supreme Court,
(vi) The grant of a writ is not discretionary but as a matter of right.
(vii) The right cannot be abridged or taken away by legislation.
Though Art. 32 is itself a guaranteed right, it could not be contended that the Supreme
Court does not have discretion to deny relief. Undoubtedly, Art. 32 guaranteed the right
to approach the Supreme Court but that does not restrict the Court's discretion to grant
relief. One of the considerations relevant for the exercise of such discretion is laches
(discussed below).

Res judicata and Art. 32


Res judicata is a rule of public policy that there shoild be finality to binding decisions of
courts of competent jurisdiction and parties to the litigation should not be vexed with the
same litigation again [Sec.11, C P.C.]. Thus, where the matter had been ‘heard’ and
‘decided’ by the High Court under Art. 226, the writ under Art. 32 is barred by the rule of
res judicata and could not be entertained. Similarly, if a question has been once
decided by the Supreme Court under Art. 32 the same question cannot be re-opened
again under Art. 226. Howe ver, a petition under Art. 32 for habeas corpus is an
exception to this general rule.
Thus, the Supreme Court cannot oe moved more than once on the same facts. It
has been held that in the absence of new circumstances arising since the dismissal of
the petition filed in Supreme Court under Art. 32, a fresh petition under Art. 32 on the
same matter cannot be filed in the Supreme Court [Lakhgnpal v UOI AIR 1967 SC 908],
It is to be noted that a petition filed in the Supreme Court under Art. 32 and dismissed
by it on suit by a speaking order will also be operative as res judicata, even though the
order has been made ex parte [Virudhgnagax Mills v Govt, of Madras AIR 1968 SC
1196],
If a writ petition is filed for the violation of a fundamental right in thoj High Court
under Art, 226 and it is dismissed by the High Court on the ground that the
contravention of the fundamental rights is constitutionally justified and thereafter, the
petitioner files writ petition (on the same facts and grounds) under Art. 32 in the
Supreme Court, instead of filing a regular appeal to the Supreme Court, trie decision of
the High Court will operate as res judicata and the writ petition ■will not be entertained
by the Supreme Court [Titokchand Motichandv H.B. Munshi A\R 1970 SC 898; Daryao
v State of U.P. AIR 1961 SC 1457],
However, for' this purpose the writ petition under Art. 226 must have been
dismissed by the High Court on merit, if it has been dismissed not on merit but some
preliminary grounds viz. on the ground of laches (delay in filing the petition) or on the
ground of alternative remedy available to the petitioner, it will not operate as res judicata
and, thus, will not bar petition under Art. 32 [Joseph v State of Kerala AIR 1974 SC
1514],
The principle of res judicata does not apply in the case of petition for habeas
corpus. InGhulam Sarwar v UOI (AIR 1967 SC 1335), a writ petition was filed in the High
Court under Art. 226 challenging the detention of the petitioner but it was dismissed by
the High Court on merit, the petitioner then moved the Supreme Court for the issue of
same writ. The Supreme Court entertained the petition and decided it on merit, although
the petition was dismissed on some other ground. In Sunil Dutt v UOI (AIR 1982 SC
53), the petitioner’s father was detained under the COFEPOSA Act. A petition for the
writ of habeas corpus was dismissed by the Supreme Court in limine (summarily, at the
initial stage). After sometime a petition was filed in the Supreme Court seeking release
of the detenu. The Supreme Court entertained it and observed that the earlier petition
would not be operative as res judicata.
Right to Constitutional Remedies 237
Desirability of res judicata - The right given to the citizen to move the Supreme Court by
a petition under Art. 32 and claim an appropriate writ .against the unconstitutional
infringement of his fundamental rights itself is a matter of fundamental right. However,
the rule of res judicata itself embodies a principle of public policy which in turn is an
essential part of the rule of law, thus, the objection that the rule cannot be invoked
where fundamental rights are in question may lose much of its. validity. The doctrine of
res judicata is not a technical doctrine applicable only to records; it is a fundamental
doctrine of all courts that there must be an end of litigation. Thus, the general rule of res
judicata cannot be treated as irrelevant or inadmissible even in dealing with
fundamental rights in petition filed under Art.32 [Daryao v State of U.P AIR 1961 SC
1457],

Laches/Delay and Art. 32


The Limitation Act is not applicable but the Supreme Court has held that a party seeking
relief from the Court must come to the Court with utmost despatch. A party coming to
the Court after undue delay may not be given relief. Stale claims will not be entertained.
it is fundamental principle of administration of justice that the courts will help
those who are vigilant about their rights and who do not sleep on their rights. The courts
will refuse to exercise their jurisdiction in favour of a party who comes to the court after
a considerable delay and is otherwise guilty of laches. Thus, the remedy under Art. 32
must usually be sought within a reasonable time, unless the explanation given for the
delay is convincing and acceptable.
What is laches or undue delay would be decided on the facts of each case
(Tilokchand Motichand v H.B. MunshiMR 1970 SC 898). In this case, the petitioner fled
the petition in the Supreme Court after the lapse of 10 years after his petition under Art.
226 was dismissed by the High Court. The Supreme Court rejected the Petitioner’s writ
petition on the ground of delay. Hidayatullah, C.J., felt that no hard ahd fast rule can be
adopted in this matter. He said, “the question is one of discretion f°r this Court to follow
from case to case. In a suitable case this Court may entertain

such a petition even after a lapse of time. It will all depend on what the breach of
the fundamental right and the remedy are and why the delay arose.”
Again, in K. Prasad v UOI (AIR 1988 SC 535), it was held that there is no
inviolable rule that whenever there is delay the Court must necessarily refuse to
entertain the petition. Each case is to be decided on its facts and circumstances.
Where the petitioners challenged seniority rules after 16 years the Court refused
to grant relief on the ground of inordinate delay (Rabindra Nath v UOI AIR 1970
SC 470).

Locus Standi and Public interest Litigation (PIL) 1


The traditional rule of locus standi is that a petition under Art. 32 can only be
filed by a person whose fundamental right is infringed This rule has now been
considerably relaxed by Supreme Court in its recent rulings. The Court now
permits ‘Social action litigation' or ‘public interest litigation’ (pro bono publico
litigation) at the instance of ‘public-spirited citizens', for the enforcement of any
constitutional
XVI or legal Cright
o n soft iany
t u t person
i o n a l or
L agroup
w o of
f Ipersons
n d i a - who
I I because of
their poverty or socio-economic disadvantaged position or otherwise (e.g. being
in custody) are unable to approach the court for relief.
The grievance in a public interest action is about the content and conduct
of Government action in relation to the Constitutional or statutory rights of
segments of society and in certain circumstances the conduct of Government
policy. The doctrine of PIL applies to any case of public injury arising from -
(a) the breach of any public duty, or
(b) the violation of some provision of the Constitution, or
(c) of the law.
The relief to be granted looks to the future and is, generally, corrective rather
then compensatory which, sometimes it also is. Public interest litigation involves
collaboration and cooperation between the Government and its officers, the Bar
and the Bench, for the purpose of making human rights meaningful for the
weaker sections of the community.
The traditional doctrine of standing means that judicial redress is available
only to a person who has suffered (or is likely to suffer) a legal injury by reason
of violation of his legal right or legally protected interest. However, there are a
few exceptions to it, which have been evolved by the courts over the years.
Thus, a tax payer of a local authority is accorded standing to challenge an illegal
action of local authority. The reason for liberalisation of the rule in the case of a
tax payer of a municipality is that his interest in the application of the money of
municipality is direct and immediate. The statute itself may expressly recognise
the locus standi of an applicant, even though no legal right of the applicant has
been violated resulting in legal injury to him.

1. Examine and elucidate the scope and significance of Art. 32 of the


Constitution in the context of Public Interest Litigation as expounded by
the Supreme Court in M.C. Mehta v UOI AIR 1987 SC 1086 [/.AS. -
20071

For example, in J.M. Desai v Roshan Kumar (AIR 1976 SC 518), the court
noticed that the Bombay Cinematograph Act 1918 and the Bombay Cinema Rules,
1954 made under that Act, recognised a special interest of persons residing, or
concerned with any institution such as a school, temple, etc. located within a distance
of 200 yards of the site on which the cinema house is proposed to be constructed and
held that as the petitioner, a rival cinema owner, did not fall within the category of such
persons having a special interest in the locality, he had no locus standi to maintain a
writ petition.
In Sunil Batra v Delhi Administration (AIR 1980 SC 1579), the court accepted the
habeas corpus petition of a prisoner complaining of a brutal assault by a head waiden
on another prisoner, in this case, the Supreme Court broadened the scope of habeas
corpus by holding that this writ can be issued not only for releasing a prisoner from
illegal detention, but also for protecting prisoners from inhuman and barbarious
treatment. In Dr. Upendra Baxi v State of Uttar Pradesh (1983) 2 SCC 308) the
Supreme Court permitted the petitioner, a law professor, to raise infringement of Article
21 on behalf of the inmates of the Agra Protective Home, who were living in inhuman
and degrading conditions.
Representative Proceedings Why Necessary?
The Constitution-makers did not lay down any particular form of proceeding [the
expression 'appropriate proceeding in clause (1) of Article 32 is with reference to
clause (2) i.e. nature of order or writ], nor did they stipulate that such proceeding
should conform to any rigid patternR i g hort at ostrait-jacket
C o n s t i t uformula...
t i o n a l because
R e m e d ithey
e s knew that 239
in a country like India with so much of poverty, ignorance and exploitation, any
insistence on rigid formula of proceeding would be self-defeating. The procedure is but
a hand-maid of justice and cause of justice may never be allowed to be wasted by any
procedural technicalities (S.P. Gupta v UOI AIR 1984 SC 802).
In the aforesaid case, the Apex Court observed: The broadening of the rule of
locus standi has been largely responsible for the development of public law, because it
is only the availability of judicial remedy for enforcement which invests taw with
meaning and purpose, or else the law would remain merely a teasing illusion and a
promise of unreality. This broad rule gives standing to any member °f the public who is
not a mere busybody or a meddlesome interloper but who has sufficient interest in the
proceeding. Also, it would have to be decided from case to case that whether person
has sufficient interest and has not acted, for personal Sains or political motives, or
mala fides. If such a person moves the court, it would amount to abuse of public
interest litigation. Moreover, the court may confine this strategic exercise of jurisdiction
to cases where legal injury is caused to a determinate ctass or group of persons, and
not entertain cases of private or individual wrong 0r injury at the Instance of a third
party, where there is an effective legal aid Or9anisation which can take care of such
cases.
The public interest litigation is absolutely necessary for maintaining the rule law,
furthering the cause of justice and accelerating the pace of realisation of °nstitutional
objectives. There can be no doubt that the risk of legal action against

the State or public authority by any citizen will induce the State or such authority to act
with greater responsibility and thereby improving the administration of justice
There is also another reason why the rule of locus standi needs to be liberalised.
Today we find that the law is being increasingly used as a device of organised social
action for the purpose of bringing about socio-economic change. An enormous increase
in developmental activities has created more and more a new category of rights in favour
of large sections of people and imposed a new category of duties on the State officials
with a view to reaching the social justice to the common man. Individual rights and
duties are giving place to meta-individual, collective ‘diffuse’ social rights and duties of
classes or groups of persons.
The conferment of socio-economic rights (under the Directive Principles) and
imposition of public duties on the State authorities for taking positive actions generates
situations in which single human action can be beneficial or prejudicial to a large
number of people, thus making entirely inadequate the traditional scheme of litigation as
merely a two-party affair. For example, the discharge of effluents in a a lake or river
may harm all who want to enjoy its clean water; defective or unhealthy papkaging may
cause damage to all consumers of goods and so also the illegal raising of railway or bus
fares may affect the entire public which wants to use the railway or bus as a means of
transport. In these cases, the duty which is breached giving rise to the injury is owed by
the State authority not to any specific or determinate class of persons, but to the
general public. In other words, the duty is one which is not correlated to any individual
rights; public injury is one caused to an indeterminate or unspecified class of persons.
In Bandhua Mukti Morcha case, the court explained the nature and purpose of
public interest litigation: It is a challenge and an opportunity to government to make
basic human rights meaningful to deprived sections and assure them socio-economic
justice. Government must welcome PIL, because it provides them an accession to
examine whether poor are getting their socio-economic entitlements or not. The court
did not want to tilt at executive authority or usurp it, but to protect poor against violation
of their basic human rights, thus court is merely assisting in the realisation of
Constitutional
XVI objectives.
Constitutional Law o f India- II
The court observed: It is not at all necessary that an adversarial procedure,
where each party produces his own evidence tested by cross-examination by other
side, and the judges playing a passive role, must be followed in a proceeding under Art.
32. A strict adherence to adversarial procedure may lead to injustice, particularly when
the parties are not evenly balanced in social or economic strength. Art. 32 does not
merely confer power on the court to issue a writ, but it also lays a constitutional
obligation to protect fundamental rights, and for that purpose the court has all incidental
and ancillary powers ... to forge new remedies and fashion new strategies to enforce
fundamental rights, to make fundamental rights meaningful for the large masses of
people.
In M.C. Mehta v UOI (AIR 1987 SC 1086), the Supreme Court further widened the
scope of public interest litigation under Article 32:
(i) Held that a poor can move the court, by writing a Tetter (even without an
affidavit) to any judge (instead of the entire court), as they might not know the
proper form of address to the court.
(ii) The court has power to grant remedial relief, in the form of compensations,
where violation of fundamental right is “gross and patent” and “affects persons
on a large scale”.
(iii) The court can appoint socio-legal commission or device any procedure and
forge any tools, for the enforcement of fundamental rights under Article 32
(vide Bandhua Mukti’s case).
The 5-judge bench of the court in this landmark judgement opened the doors of the
highest court of the nation for the oppressed, the exploited and the down-trodden in
villages or urban slums. The court has brought legal aid to the door steps of the
teeming millions of the India which the executive has not been able to do despite that a
lot of money is being spent on the legal aid schemes operating at the Central and State
level.

Misuse of PIL
It is now clearly and firmly established that any member of the public having 'sufficient
interest’ can approach the court for enforcing the rights of other persons and redressal
of a common grievance (‘public injury’). The liberal view of permitting PIL does not
mean that locus standi is completely dead and buried The court does not allow a
person to abuse the legal process pretending to be a public spirited person. The court
scrutinizes the proceeding to find out whether it will serve a public purpose.
Public interest litigation has been criticised on a number of grounds, viz. that it
can be misused for private motive or political ends, that it would result in the
tremendous increase in the litigation, that it would develop uncertainty as to the
admission of the petition for hearing. It is said that there is no guideline as to the cases
which should be admitted and the cases which should not be admitted. Due to this, the
PIL has become unpredictable. Moreover, the court has no capacity to enforce its
orders and in many cases the conditions have not changed.
Some judges and jurists have sounded a note of caution against PIL. It
increases the work load so that regular matters are delayed. PIL is a sort of interference
by the courts in matters which strictly fall in the domain of the executive. This leads to
friction between the different organs of the State. The court has no means to enforce its
orders. But none of this has deterred the courts in their commitment to help the
disadvantaged. PIL has generally been welcomed. It has been able to secure relief for
those who did not even know that there is a law to protect them and look after their
welfare. Public authorities are now more circumspect in exercising their powers. A
measure of accountability has come to all those who wield power.
The Supreme Court has sounded a note of warning. It observed, “there is ... a
feeling ... that public interest litigation is now tending to become ‘publicity interest
litigation’ or ‘private interest litigation’... PIL is not a pill or panacea for all wrongs, (it)
was not meant to be a weaponRtoi gchallenge
h t t o C the
o n sfinancial
t i t u t i oor
n aeconomic
l R e m e ddecisions
ies which 241
are taken by the Government ...”
The tide of PIL has not yet subsided. The Supreme Court has stated that time
has come to weed out the petitions, which though titled as public interest are in essence
something else. The Court has cautioned that because of false PIL innumerable days
are wasted. PIL is a weapon which has to be used with great care. The judiciary has to
be extremely careful to see that behind the beautiful veil of PIL an ugly private malice,
vested interest and/or publicity seeking is not lurking. It should be aimed at redressal of
genuine public wrongs or public injury.

Existence of Alternative Remedy and Art. 32


In K.K. Kochuni v State of Madras (AIR 1959 SC 725). the Supreme Court held that
though the existence of an adequate legal remedy was a thing to be taken into
consideration in the matter of granting prerogative writs, this was not an absolute
ground for refusing a writ under Art. 32, because the powers given to the Supreme
Court under Art. 32, were much wider and were not confined to the issue of prerogative
writs only.
It follows that the existence of^an alternative relief is no ground for the refusal to
the grant of remedy under Art. 32. However, the Supreme Court ordinarily insists that
the alternative remedy should be availed of unless it is of.no avail to the petitioner (Y.
Thedammav UOI AIR 1987 SC 1210).
In S.A. Khan v State of Haryana (AIR 1993 SC 1152) it was held that when a
statutory alternative remedy was available to the petitioner against the order of
suspension, the writ petition under Art. 32 was not maintainable. 63

NATURE AND SCOPE OF RELIEF UNDER ART. 32

Writs, Directions or Orders


Article 32(2) provides: “The Supreme Court shall have power to issue directions or
orders or writs, including writs in the nature of habeas corpus, mandamus, prohibition,
quo warranto and certiorari, whichever may be appropriate, for the enforcement of any
of the rights conferred by this Part.”

[a] Writs
A writ is a quick remedy against injustice, a device for the protection of the rights of
citizens against any encroachment by the governmental authority. Writs originated in
Britain where they were King’s or Queen’s ‘prerogative’ writs and were commands to the
judicial tribunals or other bodies to do or not to do something. In India, the power to
issue writs has been vested in the Supreme Court and the High Courts. It is a speedy
remedy and is made available without going into avoidable technicalities. It is an
extraordinary remedy which can be expected in special circumstances.
The Supreme Court has been empowered to issue writs in the nature of habeas
corpus, mandamus, prohibition, certiorari and quo warranto for protecting the

63 Narendra Kumar, Constitutional Law of India, Pioneer Publications, pp. 292' 293
(1997).
fundamental rights (Art. 32). Similar power has been conferred on the High Courts via
Art. X226
V I of the Constitution. The High Court can issue the above writs for protecting the
Constitutional Law o f India- II
fundamental as well as statutory and common law rights. The High Courts can issue
writs to any governmental authority outside its territorial jurisdiction, provided the cause
of action arises within their territorial jurisdiction. A writ is a discretionary remedy and
the High Court can refuse it on the ground of acquiescence, laches (delay), available
alternative remedy and no benefit to the party.
Under Art. 226, writs can be issued to “any persons or authority” (any person or
body performing public duty). There are five well-known writs:
(i) Habeas Corpus - (a) Habeas Corpus It literally means ‘a demand to produce
the body (whether dead or alive) in the Court’. The issuance of the writ means
an order to the detaining authority or person to physically present before the
Court the detained person and show the cause of detention so that the Court
can determine its legality or otherwise (However, the production of the body of
the person alleged to be unlawfully detained is not essential). If the detention
is found to be illegal, the detained person is set free forthwith.
While the Supreme Court can issue the writ of habeas corpus only against the State in
cases of violation of fundamental rights, the High Court can issue it also against private
individuals illegally or arbitrarily detaining any other person.
(ii) Mandamus - (b) Mandamuslt is a command to act lawfully and to desist from
perpetrating an unlawful act. Where A has a legal right which casts certain
legal obligations on B, A can seek a writ of mandamus directing B to perform
its legal duty. Mandamus may lie against any authority, officers, government
or even judicial bodies that fail to or refuse to perform a public duty and
discharge a legal obligation. Mandamus is a judicial remedy in the form of ‘an
order to do or to forbear from doing some specific act’ which that agency is
obliged to do or to refrain from doing under the law.
(iii) Certiorari - (c) Certiorari'Certiorari’ is a Latin word meaning ‘to inform’. It was
essentially a royal demand for information. ‘Certiorari’ may be defined as a
judicial order directed to any constitutional, statutory/ non-statutory body/
person, requiring the records of any action to be certified by the court and
dealt with according to law. Certiorari can be issued to quash judicial, quasi-
judicial as well as administrative actions.
Grounds for the issue of certiorari are: (1) Lack of jurisdiction; or the authority declining
jurisdiction where it legally belongs to it. (2) Excess of jurisdiction. (3) ^buse of
jurisdiction. (4) Violation of the principles of natural justice. (5) Error of aw apparent on
the face of the record. The jurisdiction of the High Court to issue a Writ of certiorari is a
supervisory jurisdiction.
243 Constitutional Law o f India- II

(iv) Prohibition - (d) ProhibitionProhibition is a judicial order to the agencies


(constitutional, statutory or non-statutory) from continuing their proceedings
in excess or abuse of their jurisdiction or in violation of the principles of
natural justice, etc. In India, prohibition is issued to protect the individual
from arbitrary administrative actions.
Prohibition has much in common with certiorari- both are issued by a superior court
against an inferior court; however, prohibition is issued while administrative process is
in motion to prevent it from proceeding further, certiorari is issued to quash the
proceedings and is therefore issued when the administrative process has ended in a
decision.
(v) Quo-Warranto - (e) Quo-Warrantolt is a question asking 'with what authority
or warrant’. The writ may be sought to clarify in public interest the legal
position in regard to claim of a person to hold a public office.

[b] Directions and Orders


Article 32(2) is of wide amplitude. It does not confine the power of the Supreme Court
to the issuance of the named writs, but the Court may issue “any direction or order,”
whichever may be appropriate for the enforcement of the fundamental rights (M.C.
Mehta v UOI AIR 1987 SC 1080) In Rupa Ashok Hurrra v Ashok Hurra (AIR 2002 SC
1771), held that in order to rectify gross miscarriage of justice in its final judgment
which cannot be challenged, again the Court will allow curative petition by the victim to
seek a second review of the final order of the Court.
The power of the court under Art. 32 is not only injunctive in ambit i.e.
> preventing the infringement of a fundamental right, but it is also remedial in scope
and empowers the court to grant relief against a breach of a fundamental right already
committed (Rudai Shah v State of Bihar AIR 1983 SC 1086). Thus, in the. exercise of
power conferred by Art. 32(2), the Supreme Court has:
(i) directed the State to pay compensation and exemplary costs for the violation
of the fundamental rights (Sebastian M. Hongray v UOI AIR 1984 SC 1026);
(ii) directed the State to ensure minimum wages to contract labourers (People’s
Union for Democratic Rights v UOI AIR 1982 SC 1473);
(iii) directed the Company manufacturing hazardous and lethal chemicals and
gases posing cjanger to health and life of workmen and people living in their
neighbourhood, to take necessary safety measures before reopening the
plant (M.C. Mehta v UOI AIR 1987 SC 965);
(iv) directed the State to rehabilitate under-trial victims [Khatri v State of Bihar
(1983) 2 SCC 266];
(v) ruled that it was a paramount obligation of every member of medical
profession to give medical aid to every injured person brought for treatment
immediately without waiting for procedural formalities to be complied in
order to avoid negligent death (Parmanand Katara v UOI AIR 1989 SC
2039);
(vi) directed to provide better facilities to inmates of government protective
homes [Upendra Baxi v State of U.P. (1986) 4 SCC 106];
(vil) laid down the guidelines with a view to assisting rape victims for their
rehabilitation and compensation [Delhi Domestic Working Women’s
Forum v UOI (1996) 1 SCC 14];
(viii) held that the writ of habeas corpus could be issued not only for
releasing a person from illegai detention but also for protecting
prisoners from inhuman and barbarous treatment (Sunil Batra v Delhi
Administration MR 1980 SC 1579);
(ix) laid down the guidelines on which compensation was to be awarded
unaer Arts. 32 and 226 to the victims of State action (Nilabati Behera v
Right to Constitutional Remedies 244

State of Orissa AIR 1993 SC 1960);


(x) awarded compensation to the victims of police activities [Arvinder Singh
Bagga v State of U.P. (1994) 4 SCC 602].3

Art. 32(3): Powers of the Supreme Court and Any Other Court
Article 32(3) empowers Parliament to confer by law, all or any of the powers,
exercisable by the Supreme Court under Art. 32, on any other Court. The words
“any other court” refers to a court other than the High Courts. It is because the
High Courts are expressly vested with such power under Art. 226 as is
exercisable by the Supreme Court under Art. 32(2).

Art. 32(4): Suspension of Rights Conferred by Art. 32


Article 32(4) provides that “the right guaranteed by this article shall not be
suspended except as otherwise provided for by this Constitution.”
Thus, the right to move the Supreme Court under Art. 32 cannot be taken
away or whittled down by an ordinary law and that it is beyond the challenge of
the Legislature and a fortiori of the Executive. The jurisdiction is conferred on the
Supreme Court by the Constitution under Art. 32 and therefore limitation cannot
be placed on it except by the Constitution itself.
Article 359 refers to a situation when the right guaranteed by Art. 32 can be
suspended. It is when a Proclamation of Emergency made under Art. 352 is in
operation. Under such a circumstance, Art. 359 empowers the President to
suspend by order the enforcement of the fundamental rights. Art. 33 empowers
Parliament to modify the application of Fundamental Rights to the Armed Forces
or forces charged with maintenance of public order, etc. in the interest of
discharge of duties and maintenance of discipline. Under Art. 34, Parliament may
by law indemnify any Person for anything done in contravention of fundamental
rights for maintenance of order during the operation of martial law.

Directive Principles and Fundamental


Duties (Articles 36-51)

The Fundamental Rights and the Directive Principles have a common origin. The
Nehru Report of 1928 which contained a Swaraj Constitution of India
incorporated some fundamental rights. These included such rights as the right of
elementary education. The Sapru Report of 1945 clearly divided the fundamental
rights into two categories- justiciable and non-justiciable. Sir B.N. Rau,
Constitutional Adviser to the Constituent Assembly advised that the individual

3 . Narendra Kumar, pp 290-291.


11
rights should be divided into two categories. Those which can be enforced by a
court and those which are not so enforceable. The latter he though are ‘moral
precepts' for the authorities of the State. His suggestion was accepted by the
Drafting Committee. Our Constitution makers followed the model of the
Constitution of Ireland which sets forth certain principles of social policy for the
guidance of the State but which are not cognizable by any court.1

ARTICLES 36-51 [DIRECTIVE PRINCIPLES]

The Directive Principles of State Policy contained in Part !V of the Constitution


set out the aims and objectives to be taken up by the States in the governance of
the country. The idea of welfare State envisaged by our Constitution can only be
achieved if the States endeavour to implement them with a high sense of moral
duty. The real importance of Directive Principles is that they contain positive
obligation of State towards its citizens. They are the ideals which the Union and
State Governments must keep in mind while they formulate or pass a law.
The Directive Principles constitute a very comprehensive political, social
and economic programme for a modern democratic State. The main object in
enacting them appear to have been to set standards of achievements before the
legislature and the

1. B.K. Sharma, p. 12.5.

executive, the local and other authorities, by which their success or failure can be
judged. Sanctions behind the Directive Principles are based on sound
constitutional and moral obligations. Constitutional obligation are secured by
having Directive Principles as an integral part of Constitution. The biggest moral
force is public opinion which can enforce the Directive Principles and ensure
government’s accountability at the time of elections. Thus, sanctions behind
Directive Principles are political.
Art. 37 of the Constitution lays down that it shall be the duty of State to
apply these Directives in making laws. Articles 38 to 51 contain 17 Directive
Principles. Arts. 355 and 365 of the Constitution can be applied for enforcing
implementation of Directive Principles. The Constitution contains directives in
[246j
places other than Part IV. These directives are equally important. Article 335
states that in making appointments to services and posts in the government the
claims of Scheduled Castes and Tribes shall be taken into consideration,
248 Constitutional Law o f India- II

consistently with the maintenance of efficiency of administration. Article 350A


enjoins the State and every local authority to provide facilities for instruction in the
mother-tongue at the primary stage for linguistic minorities. Article 351 says that it
shall be the duty of Union to promote the spread of Hindi language, to develop it
and to secure its enrichment.

Classification of the Directives64 (A)


Social and Economic Charter
(1) Social order based on justice - Art. 38(1) provides that the State shall
strive to promote the welfare of people by securing and protecting a
social order in which justice - social, economic and political - shall inform
all the institutions of national life. Further, the State shall strive to
minimise inequalities in income and endeavour to eliminate inequalities
in status, facilities and opportunities.
(2) Economic justice - Art. 38 specifically requires the State to direct its
policy towards securing the following principles -
(a) Equal right of men and women to adequate means of livelihood.
(b) Distribution of ownership and control of materiai resources of the
community to the common good.
(c) To ensure that the economic system should not result in
concentration of wealth and means of production to the common
detriment.
(d) Equal pay for equal work for both men and women.
(e) To protect health and strength of workers and tender age of
children and to ensure that they are not forced by economic
necessity to enter avocations unsuited to their age or growth,
(f) That children are given opportunities and facilities to develop in a
healthy manner and in condition of freedom and dignity and that
childhood and youth are protected against exploitation and against
moral and material abandonment.
In Randhir Singh v Union of India (AIR 1982 SC 879), held that the principles of
‘equal pay for equal work’ (Art. 39(d)) though not a fundamental right is certainly a
constitutional goal and, therefore, capable of enforcement through constitutional
remedies under Art. 32.
Arts. 38 and 39 embody the principle of 'distributive justice’ which connote
the removal of economic inequalities rectifying the injustice resulting from
transactions between unequals in society In State of T.N. v Abu Kavar Bai (1984)
1 SCC 516, the Court upheld the validity of a law enacted for the nationalisation
of transport services in the State on the ground that it was for giving effect to
directives contained in Art. 39(b) and (e).

64 Directive Principles of State Policy have played a significant role in the


interpretation of the Constitution, Explain and illusrate your answer. [I.A.S.-
98]
249 Constitutional Law o f India- II

(B) Social Security Charter


(1) Participation of workers in management of industries - Art 43-A.
(2) Right to work, education and public assistance in cases of
unemployment, old age, sickness and disablement ~ Art. 41
(3) Just and human conditions of work - Art 42.
(4) Living wage for workers - Art. 43. The concept of living wage includes in
addition to the bare necessities of life (food, shelter and clothing),
provisions for education of children and insurance, etc.
(5) Free and compulsory education for children until they complete the age
of 14 years - Art. 45.
(6) Duty to raise the standard of living and improvement of health - Art. 47.
It includes, in particular, the prohibition of liquor.
(7) Promotion of educational and economic interest of weaker sections - Art.
46.
(8) Equal justice and free legal aid to economically backward classes - Art.
39-A.
In Centre of Legal Research v State of Kerala (AIR 1986 SC 1322), it has been
held that in order to achieve the objectives in Art. 39-A, the State must encourage
and support the participation of voluntary organisations in operating the legal aid
programme. In a recent judgment in State of Maharashtra v Manubhai Pragaji
Vashi (1995) 5 SCC 730, held that under Art. 21 read with Art. 39-A casts on the
State a duty to afford grants-in-aid to recognised private law colleges similar to
other faculties e.g. Art, Science, etc. 'Free legal aid’ and speedy trial’ guaranteed
under Art. 21 read with Art. 39-A are fundamental rights. In order to enable the
State to afford free legal and speedy trial a vast number of lawyers are needed.
This can only be done if adequate number of well equipped law colleges are
established to train sufficient number of persons in various branches of law. This
will, in turn, enable the State to provide free legal aid and opportunities for
securing justice.
Directive Principles and Fundamental Duties 24 9

In a democratic polity, governed by rule of law, it should be the main concern


of the State to have a proper legal system. The principle contained in Art. 39-A
are fundamental and cast a duty on the State to secure that the operation of legal
system promotes justice, on the basis of equal opportunities and further mandates
to provide free legal aid in any way - by legislation or otherwise so that justice is
not denied to any citizen by reason of economic or other disabilities.

(C) Community Welfare Charter


(1) Uniform Civil Code - Art. 44.
(2) Organisation of agriculture and animal husbandry - Art. 48.
(3) Protection and improvement of forest and wild life - Art. 48-A.
(4) Protection of monuments, etc. - Art. 49.
(5) Separation of judiciary from executive - Art. 50.
(6) Promotion of international peace and security - Art. 51. Pursuant to the
direction enshrined in Art, 51, Parliament passed the Protection of
Human Rights Act, 1993 which provides for the setting of a National
Human Rights Commission and Human Rights Courts to meet the
growing concern for human rights in the country and abroad.
(7) Organisation of village panchayats - Art. 40. The object of this provision
is to introduce democracy at the grass roots.

Implementation and Utility of Directives65


Opinions differ on how far the Directives have been implemented in actual
practice. All principles not completely translated into action, but also not totally
ignored. In keeping with Directives, the main object of 5-year plans is to achieve a
balanced economic development and to raise living standard of masses. As for
the implementation of specific Directives, good progress has been made -
(i) Land reforms, abolition of intermediaries and zamindari system.
(ii) Organisation of village panchayats; prom ion of cottage industries;
welfare of SC, ST, etc.
(iii) Compulsory primary education achieved in most States.
(iv) Community Development Programmes successfully launched for raising
the standard of living of rural population.

65 “Directive Principles are not enforceable in the court of law. nevertheless,


they are fundamental in the governance of the country.” Discuss. [I.A.S.-
2006]
Explain and elucidate the significance of the various strategies adopted for
the implementation of the ‘Directive Principles' enshrined in Part-IV of the
Indian Constitution. [/.AS.-200S]
XVI
Constitutional Law o f India- II

(v) Minimum wages for workers; modernisation of labour laws.


(vi) Nationalisation of major banks of country.
(vii) Hindu Marriage Act and Hindu Succession Act - for securing uniform civil
code.
True, India is far from having become a full-fledged welfare State as the goals of
full employment, social security and free education for children have yet to be
achieved. There is a concentration of wealth and the benefits not reaching to the
common man. However, in a country committed to democratic procedures,
progress is bound to be slow; political power is in the hands of political parties
which are unwilling or unable to make structural changes. There is no doubt that
the goal of economic and social justice has been accepted and the country is
moving in the chosen direction.
The incorporation of Directive Principles in Constitution has been justified
by a consensus of opinion, as well as working of Constitution since 1950. The
Directives are moral precepts for the authorities of State. They have an educative
value for the citizens of India and are helpfui to effect social change. They provide
a middle way between individual liberty and public good.
They are ‘instrument of instructions’ upon all future governments,
irrespective of their party creeds. The governmental policies and programmes
have been shaped by Directives.They are fundamental in the governance of the
country.
For lifting the standard of living of the people specially in the rural areas
many schemes have bee” launched e.g. Jawahar Rojgar Yojana, integrated Rural
Development Programme, Indira Awas Yojana, Desert Development Programme,
Education Guarantee Scheme, Sarva Shiksha Abhiyan, etc
Prohibition (Art. 47) of intoxicating drinks and drugs was an important
element in Gandhiji’s view. Immediately after Independence, many States
introduced prohibition. But dismal failure in implementation gave rise to rampant
corruption without in any way reducing the bad habit. Haryana imposed prohibition
in 1966 and lifted it in 2000. But gradually all such States barring a few were
forced to repeal prohibition laws. It is the failure of the people to follow the
Gandhian ideals. Prohibition failed in India as it did in the U.S.A.
Many States have enacted laws to prohibit slaughter of cows, bullocks and
calves in view of the Art. 48. The numerous Acts pertaining to labour e.g.
Minimum Wages Act, Workmen’s Compensation Act, Maternity Benefit Act, The
Factories Act, etc. are all aimed at implementing Aits. 41, 42, 43,43A, etc. The
Water Pollution, Air Pollution, Environmental Pollution Acts, The Forest Act, and
Wild Life Protection Act passed by the Parliament are in furtherance of the
directive contained in Art 48A.
Judicial interpretation of the Constitution has ensured implementation of
Directives (e.g. in making a law to ensure minimum wages to workers, evolving of
right to education and right to livelihood, etc.). The Directives are thus being
enforced indirectly by the courts.
In determining the Constitutional validity of a legislation, they are given
Directive Principles and Fundamental Duties 24 9

precedence over the Fundamental Rights They are relevant to consider what are
reasonable restrictions under Art. 19.
Uniform Civil Code
The State, according to Art. 44, has been enjoined to take steps for establishing a
uniform civil code for citizens of India. While the State has tried to reform and
codify the personal law of the Hindus (which is also applicable to Sikhs, Jains and
Buddhists), no attempt has been made to bring the Muslims, Christians and
Parsees under the purview of a common civil code.
One-objection to the promulgation of a uniform civil code has been that it
would violate Art. 25 (right to religious freedom). This objection is misconceived as
Art. 25(2) specifically saves secular activities associated with religious practices
from the guarantee of religious freedom mentioned in Art. 25(1). Another objection
to uniform civil code even raised by som<= secularists is that it could be tyrannical
to minorities A modernised uniform civil code drafted on rational principles will
come into clash with the sacramental laws of all communities including Hindus,
not merely minorities. It will surely contradict the existing gender bias and
patriarchal notions of Hindu law regarding marriage, family and succession as
those found in the existing personal law of the Muslims or the Christians.
In Sarla Mudgal v Union of India (AIR 1995 SC 1531), the Supreme Court
strongly recommended the need for a uniform civil code, declaring illegal the
second marriage by a Hindu husband after conversion to Islam. This judgment
came in the wake of four petitions filed by Hindu women who had been deserted
by their husbands after their opportunistic conversion to Islam. A two-judge bench
requested the Government through the Prime Minister to have a fresh look at Art.
44 of Constitution and expedite the promulgation of a common civil code which,
according to the court, is imperative for both protection of the oppressed and
promotion of national unity and integrity. The court directed the Union government
to ask the lav/ commission to draft a comprehensive legislation incorporating the
present day concept of human rights for women in consultation with the Minority
Commission.
The judges argued that religious practices, violative of human rights and
dignity and sacredotal suffocation of essentially civil and material freedoms, are
not autonomy but oppression. The judges upheld the view that unified,
progressive civil law would promote national unity and solidarity and safeguard
the interests of the weaker and oppressed sections. The court declared that when
more than 80 percent of the citizens have already been brought under the codified
personal law, there is no justification to keep in abeyance any more the
introduction of the code. The court observed that Art. 44 sought to divest religion
from social relations and personal laws. Further, no community could claim to
remain a separate entity on the basis of religion. Marriage, succession and like
matters of a secular character cannot be brought within the guarantees under Arts
25 and 26 of Constitution.
It is legislation, not religion which has the final authority to determine the
nature of personal law of citizens. The legislature is within its rights to supersede
or supplement this personal law by a uniform civil code. Mr. Justice Sahai
XVI
Constitutional Law o f India- II

cautioned: "The sentiments and emotions have to be cooled and tempered by


sincere efforts. The judgment envisages a common code which incorporates the
best of all the personal laws”.
The apex court’s judgment has elicited different responses The need is to
draw a balance; to safeguard the civil rights and the identity of minorities while
refusing to pander to religious fundamentalism of any colour.
The State system is now under stress. The modem secular State was
expected to be a pace-setter in Indian society. Its failure to give us a modernised,
uniform civil code based on universally recognized principles of social justice and
gender equality demonstrates that the Indian State’s capability to function
effectively on socially progressive and modernist-rational lines has been close to
zero.
The same penal laws and laws of evidence are applicable to all irrespective
of religion. Time has come to apply the same set of laws in regards to marriage,
divorce, adoption and succession to all citizens irrespective of their religious
persuasion. Without a uniform set of laws the people of India will not amalgamate
to form an integrated nation. Secularism, a basic feature of the Constitution, can
be achieved only by implementing the provisions of the Constitution of which Art
44 is a vital part.
It would be apt to recall what Justice M.C. Chagla former Chief Justice of
Bombay High Court had to say on this; ‘‘Th3t (Art. 44) is a mandatory provision
binding on the government... The Constitution was enacted for the whole country,
it is binding on the whole country and every section and community must accept
its provisions and directives.”66

ARTICLE 51-A [FUNDAMENTAL DUTIES]67

Fundamental Duties did not form part of the Constitution as originally adopted.
Fundamental Duties are incorporated in Part IV-A, which consists of only one Art.
51-A, added by 42nd Amendment, 1976.
None of the major democracies i.e the U.S.A., Australia, Canada, France,
Germany, etc. contain a table of duties. However, the Constitution of socialist
countries lay great emphasis on the citizens' duties. Art. 29(1) of the Universal
Declaration of Human Rights makes a succinct statement regarding duties. It
states: “Everyone has duties to the community in which alone the free and full
development of his personality is possible.”
Art. 51-A says that it shall be the duty of every citizen of India -

4. B.K. Sharma, pp 128-129.


67 “Fundamental Duties are only ethical and moral duties and should not form a
part of the fundamental law.” Comment. [/.A S.-
20061
Directive Principles and Fundamental Duties 24 9

(a) to abide by the Constitution and respect its ideals and institutions, the
National Flag and National Anthem,
(b) to cherish and follow the noble ideals which inspired our national
struggle for freedom,
(c) to uphold and protect the sovereignty, unity and integrity of India,
(d) to defend the country and render national service when called upon to
do so,
(e) to promote harmony and the spirit of common brotherhood amongst all
the people of India transcending religious, linguistic and regional or
sectional diversities; to renounce practices derogatory to the dignity of
women,
(f) to value and preserve the rich heritage of our composite culture,
(g) to protect and improve the natural environment including forests, lakes,
rivers and wild life, and to have compassion for living creature,
(h) to develop the scientific temper, humanism and the spirit of inquiry and
reform,
(i) to safeguard public property and to abjure violence, and,
(j) to strive towards excellence in all spheres of individual and collective
activity so that the nation constantly rises to higher levels of endeavour
and achievements. «
(k) [who is a parent or guardian to provide opportunities for education to his
child or, as the case may be, ward between the age of six and fourteen
years- Inserted by the Constitution (86th Amendment) Act, 2002].
Significance of Duties- Rights and Duties are correlative. The fundamental duties
(code of conduct) are intended to serve as a constant reminder to every citizen
that while the Constitution specifically conferred on them certain fundamental
rights, it also requires citizens to observe certain basic norms of democratic
conduct and democratic behaviour. The fundamental duties seek to limit the
operation of fundamental rights - a countervailing factor and a warning to reckless
citizens against anti-social activites like destroying public property, burning
national flag, and the like.
Some critics held the view that there was too heavy an emphasis on the
rights in Constitution. A society in which everyone is conscious of his rights and
not of his duties is bound to find himself in state of anarchy, sooner or later.
However, the Constitution has incorporated the duties - the preamble emphasises
the duties, “justice, social, economic and political". Further, the State is
empowered to impose reasonable restriction and curtail the rights in the interest of
society.
Further, some critics question the utility of these duties amidst starvation
and poverty. A poor and unemployed cannot be expected to perform his duties
towards the society if the society fails to discharge its obligation towards
individuals The ‘right to work’ should, therefore, be guaranteed to every citizen,
who are expected to do certain duties to the nation
Comparison with Directive Principles - Fundamental Duties are comparable to the
Directive Principles. The duties are addressed to the citizens. The country expects
XVI
Constitutional Law o f India- II

them to perform certain duties as citizens. They are not spectators but active
participants in attainment of national goals. The Directive Principles are goals set
up by the Constitution for all governments. It is the duty of the State to apply these
principle in making laws (Art. 37). But in case a State does not implement a
Directive Principle it cannot be penalized. There is no legal sanction attached.
The citizen must introspect and endeavour to perform these duties. The sanction
should be self imposed. They cannot be enforced by a court. 6
In Union of India v Navin Jindal (2004) 2 SCC 410, the Supreme Court
observed: For the purpose of interpretation of the constitutional scheme and for
the purpose of maintaining a balance between the fundamental/legal rights of a
citizen vis-a-vis, the regulatory measures/restrictions, both Parts IV and IV-A of
the Constitution can be taken recourse to.
Enforcement of Duties- Parliament may, by law, provide penalties to be imposed
for failure to fulfil fundamental duties and obligations. The success of this
provision would, however, depend much upon the manner in which and the
person against whom these duties would be enforced.
For the proper enforcement of duties, it is necessary that it should be
known to all. Most of the people of this country are illiterate and not politically
conscious of what they owe to society and country. Thus, what is needed is a
systematic and intensive education of the people. In M.C. Mehta v Union of India
(!983) 1 SCC 471, held that under Art. 51-A(g) it is the duty of the Central
Government to introduce compulsory teaching of lessons at least for one hour in
a week on protection and improvement of natural environment in all educational
institutions.
A court in determining the constitutional validity of any law seeking to give
effect to duties, may consider law to be ‘reasonable’ in relation to Art. 14 or 19.
The courts will uphold as void any law which prohibits an act which is violative of
the duties. Fundamental duties provide a valuable guide and aid to interpretation
of the Constitution. The Supreme Court has held that the duties must be used as
a tool to control State action drifting from constitutional values [AIIMS Students’
Union v AIIMS (2002) 1 SCC 428],
Fundamental Duties are a mixed bag of expectations and exhortations - A
number of duties (clauses a, c and d) are enforceable today without specific
incorporation in the Constitution.

Q UESTIONS AND A NSW ERS

Q. 1 It is fallacious to think that Directive Principles and Fundamental


Rights are opposed to each other in their ultimate objectives. They
are, in fact, “complementary and supplementary to each other, both
striving to secure socio-economic welfare by ensuring a social
order in which justice and individual liberty are safeguarded".

Discuss the relationship between the Fundamental Rights and the


Directive Principles in the context of judicial decisions and
constitutional amendments.[L C.1-93/94/95/2004/2005/2007;L. C.
ll-93/94][IA. S. -20001
Make
D i r e cat icritical
v e P rappraisal
i n c i p l eof
s legal
a n d status
F u n dof
am Directive
e n t a l Principles
D u t i e s 2vis-&-
4 9
vis Fundamental Rights as established by judicial decisions and
constitutional amendments. [L.C.1-
2006]

Write a short note on: The scope of the constitutional harmony and
balance between Fundamental Rights and Directive Principles.
[I.A.S.-2008]
A. 1 Relation between Directive Principles and Fundamental Rights
The Directive Principles differs from Fundamental Rights in certain respects -
(1) The Directives are non-justiciable i.e. can’t be enforced by the courts;
they are declaratory. While Fundamental Rights are justiciable i.e. can
be enforced by courts; they are mandatory.
(2) The Directives are instrument of instructions to the Government, they
contains positive commands to the State to promote a social and
welfare State. The Fundamental Rights are limitations upon the State
actions, they contain negative injunctions to the State not to do various
things. Articles 14 and 21 are negatively worded.
(3) Unlike Fundamental Rights, the Directives are required to be
implemented by legislation.
(4) The courts cannot declare as void any law which is otherwise valid on
the ground that it contravenes any of the Directives. On the other hand,
the courts are bound to declare as void any law that is inconsistent with
fundamental rights.
(5) The scope of Directive Principles is larger. The Directives set the
guidelines, in the larger interest of community, for achieving socialistic
goals through democratic methods. They are social rights and pertain to
the economic field. The Fundamental Rights guarantees some basic
rights to individuals. They are mostly political rights which tend to
restrain the State.
The place the Directive Principles occupy in the Constitution vis-a-vis the
Fundamental Rights has been a subject-matter of controversy for a long time. The
controversy has now been set at rest by the Supreme Court.
(1) In State of Madras v Champakam Dorairajan (AIR 1951 SC 228), held
that the Directive Principles cannot override the Fundamental Rights
The Directives have to conform and to run as subsidiary to the
Fundamental Rights.
(2) In Re Kerala Education Bill (AIR 1957 SC 956), the court observed that
though the Directive Principles cannot override the Fundamental Rights,
nevertheless, in determining the scope and ambit of Rights the court
may not entirely ignore the Directives but should adopt the principles of
harmonious construction and should attempt to give effect to both as
much as possible.
(3) 25th Amendment, 1971 considerably enhanced the importance of
Directives. Art. 31-C, added by it, provided that a law for implementing
XVI
Constitutional Law o f India- II
Directives contained in Art. 39 (b) and (c) could not be struck down on
the ground that it contravened rights conferred by Art 14, 19 or 31.
(4) 42ndD Amendment,
i r e c t i v e P r1976
i n c iwidened
p l e s a nthe
d Fscope
u n d aofmArt.
e n t31-C
a l D so
u t ias
e sto2cover
4 9
all Directive Principles Thus it gave precedence to all the Directives
over the Fundamental Rights - Arts. 14, 19 or 31.
(5) In Keshavananda Bharati v Union of India (AIR 1978 SC 1461), the
court observed that the Fundamental Rights and Directive Principles
are meant to supplement one another. It can well be said that the
Directives prescribed the goal to be attained and Fundamental Rights
lay down the means by which that goal is to be achieved. Both are
equally fundamental though Directives are not directly enforceable by
(6) courts
In Minerva Mills v Union of India (AIR 1980 SC 1789), the court struck
down Art. 31-C as amended by 42nd amendment as unconstitutional on the
ground that it destroys the “basic features” of the Constitution. The majority
observed that the Constitution is founded on the bed rock of the balance between
Part III and IV. To give absolute primacy to one over the other is to disturb the
harmony of the Constitution which is the essential feature of the basic structure.
The goals set out in Part IV have to be achieved without the abrogation of the
means provided for by Part III.
The court heid that the unamended Art. 31-C is valid as it does not destroy the
basic features of Constitution. Art. 39(b) and (c) are vital for the welfare of people
and do not violate Arts. 14 and 19. Other Directive Principles could not override
Fundamental Rights.
(7) But in Sanjeev Coke Mfg. Co. v Bharat Coking Coal Ltd (1983) 1 SCC
147, the Supreme Court expressed doubt on the validity of its decision
in Minerva Mills case. It held that amended Art. 31-C is valid. The
extension of constitutional immunity to other Directives does not destroy
the basic structure of Constitution.
(8) The confusion created by the above judgment has been removed in
State of T.N. v Abu Kavur Bai (AIR 1984 SC 626). A five-judge Bench
held that although the Directives are not enforceable yet the court
should make a real attempt at harmonising and reconciling the
Directives and the Rights and any collision between the two should be
avoided as far as possible. The reason why the founding fathers of our
Constitution did not make these principles enforceable was, the court
said, perhaps due to the vital consideration of giving the government
sufficient latitude to implement these principles from time to time
according to capacity, situation and circumstances that may arise.
(9) In Uni Krishnan v State of A P. (1993) 1 SCC 645, the court has
reiterated the same principle that the Fundamental Rights and Directive
Principles are supplementary and complementary to each other and the
provisions in Part III should be interpreted having regard to the
preamble and Directive Principles.
Conclusions - There is no conflict or disharmony between Directive Principles and
Fundamental Rights, because they supplement each other in aiming at the same
goal of bringing about a social revolution and establishment of a welfare State As
described by Granvil Austin, the Fundamental Rights and Directive Principles are
XVI
Constitutional Law o f India- II
the “Conscience of our Constitution’
D i r e c t iof
The objective v eboth
P r iis
n csecuring
i p l e s a social,
n d F u economic
n d a m e n tand
al D 2 4 9 and
u t i e s justice
political
dignity and welfare of the individual Both are on the same level. In case of an
apparent conflict it is for the court to resolve it in exercise of its power of judicial
review. For example, the exercise of a Fundamental Right by a person seems to be
inconsistent with a Directive Principle - A butcher’s right to carry on his business and
Art. 48 prohibiting slaughter of cows, etc. Similarly, a legislation to give effect to a
Directive Principle infringes or abridges a Fundamental Right. A legislation fixing a
minimum wage violates the right to carry on a trade under Art. 19(1 )(g).
The court cannot declare a law which is inconsistent with a Directive principle
as void but the courts have upheld the validity of a law on the ground that it was
enacted to implement a Directive Principle. In recent times, certain directive
principles have been judicially enforced by imaginative and creative interpretation of
fundamental rights (viz. right to education, right to clean environment, etc.). 68

Q. 2 In view of the majority decision in Keshavanand Bharati’s case, is it


permissible for the Parliament to so amend the Constitution as to give
a position of precedence to Directive Principles over Fundamental
Rights'? Examine critically. [LC.//-95]
A. 2 In Keshvanand Bharati’s case, a 13-judge Bench by 7-6 majority overruled
the Golak Nath's case which denied Parliament the power to amend
fundamental rights of citizens.
Delivering the leading majority judgment Sikri C.J. said:”Reading the
Preamble, the fundamental importance of the freedom of the individual, its
inalienability and the importance of the economic, social and political justice
mentioned in the Preamble, the importance of Directive Principles ... an irresistible
conclusion emerges that it was not the intention to use the word “amendment” in the
widest sense. It was the common understanding that the fundamental rights would
remain in substance as they are and they would not be amended out of existence.
The expression “amendment of Constitution” in Art. 368 means any change in
any of the provisions of Constitution within the broad contours of the Preamble and
the Constitution to carry out the objectives in the Preamble and the Directive
Principles applied to Fundamental Rights. It would mean that while Fundamental
Rights cannot be abrogated, reasonable abridgements of Fundamental Rights can
be effected in the public interest".
'If this meaning is given”, the Chief Justice said “it would enable Parliament to
adjust Fundamental Rights in order to secure what the Directive Principles direct to
be accomplished, while maintaining the freedom and dignity of every citizen”.
Shelat and Grover, J.J. observed, “Fundamental Rights and Directive
Principles are meant to supplement one another. It can well be said that the directive
principles Prescribed the goal to be attained and the fundamental rights lay down the
means by which that goal is to be achieved
In this case, the majority upheld the validity of Art 31-C (as added by 25th
Amendment, 1971) which gave a precedence to directive principles contained in
Art 39(b) and (c) over the fundamental rights conferred by Arts. 14 and 19.
The picture that emerges from majority opinion in this case is that it is not

In recent times, certain directive principles have been judicially enforced by


imaginative and creative interpretation of fundamental rights. Do you agree?
Give reasons. [I.A. S. -97]
XVI
Constitutional Law o f India- II

permissible for Parliament to so amend the Constitution as to give precedence to


directive principles over fundamental rights, except regarding directives
contained in Art. 39 (b) and (c) The court has emphasised a balance between the
directive principles and fundamental rights.
Q. 3 For a developing country like India, the Directive Principles ought to be
more important than Fundamental Rights to bring about social justice.
Discuss Do you agree with the view that in certain situations, a
harmonious interpretation of the provisions in Part III and Part IV is just
not possible?
A. 3 For a developing country like India where the majority of the people suffer
from economic backwardness, the Directive Principles assume great
importance Decause it is only by implementing these that India can
achieve the declared goal of a 'welfare State' wherein everyone will be
assured of socio-economic justice. Undoubtedly Fundamental Rights
are important for the all round development of human faculties. But it
should be kept in mind that these rights are the privileges of the
individual. What is the significance of the right of freedom for an
individual who cannot afford one square meal a day?
The judiciary in India has correctly appreciated that being a part of the
Constitution, these Principles “represent not the temporary will of a majority but
the deliberate wisdom of the nation expressed through them to be fundamental in
the governance of the country". The Supreme Court has taken the view that
reasonable restrictions on fundamental rights can be made to implement the
Principles. The abolition of zamindari, for instance, has been held to have a
'legitimate' public purpose
It is apparent, therefore, that the courts have frequently been guided by the
Directive Principles in determining the precise scope of Fundamental Rights.
However, the judiciary does not have a free hand since, in the ultimate analysis,
it has to go by the letter of the Constitution when there is a conflict between
Rights and Principles Judicially, therefore, Directives must subserve the
Fundamental Rights. But in so far as they embody the economic and social
objecitves accepted by the Constitution-makers, the Directives represent the larger
interests of the community as a whole. In this sense, they have a higher sanction
than the Fundamental Rights which represent the interests of the individual
India is a land of social and economic inequalities. It is an uncontrovertible
fact that progress requires removal of lop-sided development and of regional
imbalances. It is for this that socialism has been adopted by our Constitution. In
the circumstances there is justification in giving pre-eminence to Directive
Principles.

12
Civil Servants
(Articles 308- 323)

The permanent civil service was regarded as the steel framework of the British
Empire in India. Even after Independence the framework continues though some
attempts are periodically made to reorient it with the new objectives of the Republic.
Tne law relating to Government Services in India is spread over in many
places. It is found in the Constitution (Articles 309-312), Acts of Parliament and State
legislature, Rules and Regulations made by the appropriate Government, directions
and clarifications issued by the authorities and judicial decisions. These have to be
combined with the principles of natural justice and fundamental and other rights
conferred by the Constitution.
The courts have a delicate task to perform. They must give an amount of
security to the employee yet ensure that he performs his duties with care. The
Government as employer must have the necessary power to discipline employees
who neglect their duties or are corrupt but the power has to be exercised in a
judicious manner and not arbitrarily.
Although matters concerning Government services could be normally
regulated by laws and the power to lay down detailed rules for recruitment and
conditions of service of the Union and State employees was left to the respective
legislatures (vide Entry 70 of List I and Entry 41 of List II), the Constitution-makers
deemed it most prudent to assure the services by providing some constitutional
guarantees and safeguards in the matter of recruitment, security of tenure,
procedure for disciplinary action, etc. In this connection, the Constitution also
provided for the setting up of an independent Public Service Commission. The
provisions for the Union and State Services applied to the whole of India except the
State of Jammu & Kashmir (Art 308).

Art. 309: Power to Frame Recruitment Rules


Art. 309 provides that “Acts of the appropriate Legislature may regulate the
recruitment, and conditions of service of persons appointed to public services and
posts in connection with the affairs of the Union or of any State.” Until the appropriate
Legislature makes such laws, the Proviso to Art 309 authorises the
President/Governor or such person as he may direct, to make rules for the aforesaid
purpose in relation to the services under the Union/State. The rules so made by the
President/Governor shall have effect subject to the provisions of any Act enacted by
the appropriate Legislature.
Article 309 provided for the regulation of recruitment and conditions of service
of Union and State Government services [2 6
by1 ]appropriate Legislatures. Until any such
laws were enacted, the services were to be regulated by rules made by the President
or the Governor. The power under the proviso was intended to be in the nature of an
interim power to be exercised by the executive till the legislature takes over. It has
XVI
Constitutional Law o f India- II

been held that the rule making power conferred by the proviso to Art. 309 is
legislative power (S.S. Yadav v State of Haryana AIR 1981 SC 561).
Art. 309, however, does not make it mandatory for the Parliament or a State
Legislature to make laws for the said purpose. Art. 309 is an enabling provisions
conferring power on the Legislature. It does not impose on the Legislature a duty to
enact regulations in regard to recruitment and conditions of service of persons
appointed to public services (State of W.B. v Nripen Bagchi AIR 1966 SC 447). Nor,
it is obligatory for the Executive to make rules for recruitment etc., before a service
can be constituted or a post created or filled Art. 309 does not limit the right of the
State under Art. 73 or 162. It can recruit persons in exercise of its executive power
(Ramesh Prasad v State of Bihar AIR 1978 SC 327). The Court or the Administrative
Tribunal, therefore, cannot issue a mandate to the Government to legislate under Art.
309.
If appointments are made in contravention of the statutory rules they cannot
be regularized in exercise of the power conferred by Art 162 [State of U. P. v Neeraj
Awasthi (2006) 1 SCC 667], Executive order cannot prevail over an Act or the Rules
made under Art. 309.
‘Executive instructions’ may be issued to fill in the gap or to clarify an
ambiguity or may supplement the Rules. But the instructions issued must not be
inconsistent with any Rule or law. An instruction cannot amend or alter a Rule (6. N.
Nagarajan v State of Mysore AIR 1966 SC 1942).
Article 313 says that if neither the appropriate Legislature has made a law nor
the Executive has framed rules, then all the laws in force immediately before the
commencement of the Constitution and applicable to any public service or any post
which continue to exist, shall continue in force so far as consistent with the
provisions of the Constitution.
Conditions of service - Article 309 speaks of rules for appointment and general
conditions of service. The expression conditions of service” is substantially wide in
amplitude and would inter alia include matters relating to salary, time scale of pay or
grades; provident funds; dearness allowance; tenure of service; termination of
service; eligibility for promotion; seniority; retirement; pensions; suspension; and the
like. The rules dealing with functions of the Public Service Commissions, Union or
State, are not rules relating to recruitment and therefore do not fall within the scope
of Art. 309
Civil Servants 263
The legislature has left the field to the executive - The Union and the State make
recruitment to a very large number of posts. The essential and desirable
qualifications and length of experience required vary from post to post. The mode of
recruitment- by open selection, deputation, promotion, limited departmental
examination, etc. is best left to the executive because it may conveniently change
the mode in the light of the experience gained. Under Art. 320, the Public Service
Commission (Union or State) has to be consulted on all matters relating to methods
of recruitment and the principles to be followed in making appointments The
Government (Executive) finds it convenient to consult the Commission while making
a rule but if it is a legislation the procedure may be cumbersome and time
consuming. There are few Acts governing recruitment to services and these Acts
only lay down a skeletal framework leaving the rest to be done by delegated
legislation.

Restraints on Power Conferred by Article 309


The legislative power, or, the rule-making power under Art 309 is subjected to the
provisions of the Constitution. The Acts or Rules relating to ‘conditions of service’
made under Art. 309 are subject to the following limitations:
(a) The Acts or Rules must not contravene fundamental rights especially those
secured by Articles 14, 15 and 16 (D.S. Nakara v UOI AIR 1983 SC 130).
(b) The Rules should also comply with the Directive contained in Art 39(d)
relating to “equal pay for equal work for both men and women” (Bhagwan
Dass v State of Haryana AIR 1987 SC 2049)
(c) The legislative power or rule-making power conferred by Art. 309, is
subjected to Art. 310 (1) which incorporates the “doctrine of pleasure.”
(d) Article 311 which contains procedural safeguards for civil servants imposes
a limitation on the power conferred by Art 309 in relation to “conditions of
service.” The rules must be fair, reasonable and just. Not only the rules but
the practice of the Government must not be arbitrary.
(e) Clause (3) of Article 320, through not mandatory in nature, requires
consultation with the appropriate Public Service Commission in all matters
relating to the recruitment to civil services and for civil post, in making
promotions and transfers and on all disciplinary matters affecting civil
services. The rule making power under Article 309 is, thus, limited by
Article 320(3).
(f) Separate provisions have been made for regulating the recruitment and
conditions of service of certain categories of public servants e.g. officers
and staff of Legislature Secretariats (Arts. 98 and 187), employees of the
Supreme Court and the High Courts (Arts. 146 and 299) and persons
serving the Indian Audit and Accounts Department (Art. 148). Art. 309 did
not apply to them.
Arts. 310-311: Doctrine of Pleasure and Constitutional Safeguards
69
Art. 310(1) provides: “Except as expressly provided by this Constitution, every
person who is a member of a defence service or of a civil service of the Union or

69 The Government has the right to regulate or determine the tenure of its
employees at pleasure, notwithstanding anything in their contract to the
contrary, provided that the mandatory provisions of Article 311. have been
observed. Discuss.
XVI
Constitutional Law o f India- II

of an all-India service or holds any post connected with defence or any civil post
under the Union, holds office during the pleasure of the President, and, every
person who is a member of a civil service of a State or holds any civil post under
a State, holds office during the pleasure of the Governor of the State ”
Art. 310(1) laid down the principle that every Government employee - in a
defence service or a civil service - held his office 'during the pleasure of the
President or the Governor.' Art. 310(1) embodies a rule of constitutional doctrine
pertaining to the tenure of service.
It was, however, possible to provide in special cases by contract to pay
compensation for early termination of service. Otherwise, the doctrine of pleasure
cannot be fettered or limited by a contract. Also, it cannot be fettered or limited by
Rules made under Art. 309 or by an Act of Parliament/State legislature. Article
309 is subject to Art. 310. Thus, pleasure is not fettered by ordinary legislation
(State of U.P. v Babu Ram Upadhya AIR 1961 SC 751).
In UOI v Tulsiram Patel (AIR 1985 SC 1416), held that the Acts or Rules
made in pursuance of Art. 309, were subject to the doctrine of pleasure contained
in Art. 310(1), and except to the extent that the pleasure had been restricted by
the “express provisions of the Constitution" such as Art. 311, no restrictions could
be placed on the exercise of pleasure by an Act or Rules framed under Art 309.

Doctrine of Pleasure
70
Article 310 derives its ancestry from the English (Common) Law where the rule is
that all service, military or civil, under the crown is held at the pleasure of the
Crown. The expression “doctrine of pleasure" is drawn from the Latin phrase
“durante bene placito" which means “during pleasure.” The services of a public
servant may be dispensed with and he may be dismissed at will. There is no need
to assign reasons. He could not even claim arrears of his salary due to him. Nor
he could claim any damages for his wrongful dismissal. The Common Law “rule of
pleasure" was, however, subject to one exception that it could be excluded by a
Statute of Parliament, for the Crown being a party to every Statute.
The “doctrine of pleasure" is based on public policy. That a public servant,
whose continuance in office is not or is against the public interest should be
reiieved of his office. The theory is that the Crown cou!d not fetter its future
executive action by entering into a contract in such matters concerning the
welfare of the country.
It is held that the doctrine is not based upon any special prerogative of the
Crown but upon public policy. The Government has a right to expect that each of
its servants will observe certain standards of decency or morality not only in his
public but also private life e.g. the Government may demand that none of its
servant would remany during the life-time of his first wife (Madho Singh v Stale of
Bombay AIR 1960 Bom. 285)

70 Explain doctrine of
pleasure [L.C.II-2007] [D.U.- 2008]
Civil Servants 265

Limitations on the “Doctrine of Pleasure”71


A public servant under the Constitution of India though works at the pleasure of
the President/Governor, but he does not work at his mercy (State of Bihar v Abdul
Majid AIR 1954 SC 245). In view of the opening words “except as expressly
provided by this Constitution”, in Art. 310(1), the “doctrine of pleasure" admits of
certain limitations and exceptions:
(1) Art. 310(2) exempts a person having special qualifications, not being a
member of a defence service or of an all-India service or of a civil
service of the Union/State, appointed to civil post for a fixed term.
(2) Art 311(2) controls the pleasure of the President/Governor, in regard to
the civil servants of the Government. It is meant to be a Constitutional
safeguard (discussed below).
(3) The pleasure of the President/Governor is controlled by the
Fundamental Rights. It cannot be exercised in a discriminatory or unfair
manner. In Kameshwar Prasad v State of Bihar (NR 1962 SC 1166),
Rule 4Aof the Bihar Government Servant’s Conduct Rules 1956, in so
far as it prohibited any form of demonstration’ was struck down by the
Supreme Court as being violative of the fundamental rights contained in
Art. 19(a) and (b) of the Constitution
(4) The pleasure is done away with as regards the holders of specified
offices. For example, the Judges of the Supreme Court and High
Courts, the Comptroller and Auditor General of India, the Members of
the Union and State Public Service Commissions, and the Chief
Election Commissioner, do not hold office during the pleasure of the
President or the Governor. The Constitution prescribes a secured tenure
and express procedure for the removal of such office holders. Under
their fixed tenure, the office or post is held during good behaviour and
not under the doctrine of pleasure.
(5) Art. 320(3)(c) restricts the pleasure in so far as, it requires that the
respective Public Service Commission shall be consulted on all
disciplinary matters affecting a person serving under the Government of
India or the Government of a State in a civil capacity
(6) it has been held that the doctrine of pleasure does not confer a power
on the President or the Governor to compel a government servant to
continue in service against his will after reaching the age of
superannuation,

71 Mention the offices/posts which are excluded from the doctrine of pleasure
under Art. 310 of the Constitution. [L.C.II-
2006]
264 Constitutional Law o f India- II Civil Servants 265

except where the services of such a person are required in the public employed by the defence services The Armed Forces are governed by Special
interest (Pratap Singh v State of Punjab MR 1964 SC 72). Acts e.g. The Army Act, the Navy Act, and, the Air Force Act.
(7) The pleasure of the President/Governor may be done away with by A ‘Civil Post’ means an appointment or office or employment on the civil
repealing Art. 310(1) by the Parliament in the exercise of its power to side of the administration. There must be a relation of master and servant
amend the Constitution under Art. 368. between the State and the person holding the post. A person employed in the
Police holds a civil post. But employees of statutory corporations or government
Delegation of Pleasure by the President/ Governor companies are not civil servants. A person procuring appointment on the basis of
The pleasure conferred by Art. 310(1) can be exercised by the President/ a false certificate is not a person holding a civil post.
Governor either with the aid and on the advice of the Council of Ministers or by The protection of Art. 311 was not mailable in case of civil employees if the
the authority specified in the Acts or Rules made under Art. 309. penalty was any other than dismissal, removal or reduction in rank. The Article,
In State of U.P. v Babu Ram Upadhya, the majority held that power to however, made no distinction between a person holding a temporary post and
dismiss a public servant subject to the provisions of Article 311, was not an one holding a permanent post (Parshottam Lai Dhingra v UOI AIR 1958 SC 36;
executive power under Art. 154 but a constitutional power and was not capable of State of Punjab v Ram AIR 1992 SC 2188).
being delegated to officers subordinate to the Governor. However, in Moti Ram Temporary employee and probationer- In Parshottam case, held that reasonable
Deka v N.E.F. Rly. (AIR 1964 SC 600), the Court overruled the majority view in opportunity is the right of temporary servants as it is of the permanent servants.
Babu Ram Upadhya case. But later many situations came before the Apex Court which necessitated
The question was considered in UOI v Tulsiram Fatel (AIR 1985 SC 1416), formulation of general rules. For example, if a government employs a person for a
and held that the pleasure of the President/Governor was not required to be specific project and terminates his services when the project comes to an end, no
exercised by either of them personally. It being an executive power within the hearing is required because of the nature of employment. The rules were
meaning of Article 53(1), 74(1), 77(1), 154(1), 163(1), 166(1), may be exercised formulated by the Supreme Court in State of Punjab v Sukh Raj Bahadur (MR
by the President/Governor acting on the aid and advice of the Council of Ministers 1968 SC 1089);
(1) The services of a temporary employee or of a probationer may be
72 terminated in accordance with the rules of employment. Such
Art. 311: Constitutional Safeguards to Civil Servants
termination does not call for an enquiry.
Article 311 sought to place certain limitations on the exercise of the ‘pleasure
principle’ in respect of civil seivants. it provides certain safeguards and protection (2) The circumstances preceding the termination have to be examined. The
to the government servants. It is to be noted that they are available only to - (a) a motive behind it is immaterial.
member of a civil service of the Union or of a State (b) a member of an All India (3) If the order visits the public servant with evil consequences or casts
Service, and (c) a person who holds a civil post under the Union or a State. They stigma on his character on integrity then the order is by way of
are not available to members of the Armed Forces. punishment. In such cases, Art. 311 is attracted even in case of a
probationer or a temporary servant.
No civil servant could be dismissed or removed by an authority subordinate
to the appointing authority [Art. 311(1)] and no civil servant could be dismissed or (4) If the order is in a simple form which does not cast any aspersion on the
removed or reduced in rank except after an enquiry informing him of the charges civil servant then Art. 311 is not attracted even though superior
against him and giving him a reasonable opportunity of being heard in respect of authorities conducted an enquiry to ascertain the suitability of the public
those charges [Art. 311 (2)].73 servant to conclude whether to retain him or not.
As stated above the safeguards can be claimed by a class of persons who (5) If there is an enquiry i.e. a charge-sheet is served, an enquiry officer is
hold “civil posts.” It is not available to defence personnel or even civil employees appointed and an explanation is called, Art. 311 will apply 74
Dismissed, removed or /educed in rank- A person who is dismissed is not eligible
for re-empioyment under the Government. A person who is removed does not
4. What constitutional safeguards are available to “Civil Servants” under the become disqualified and may be re-employed. Both are termination of services.
Constitution of India? [L.C.II- 2006] [I.A.S.- Art.
2003/2C06]
Do these safeguards override the “Doctrine of Pleasure" embodied in the
Constitution? [I A. S.- when it was proposed to impose a penalty. After the 42'“’ Amendment the
2003] penalty may be imposed on the basis of the evidence adduced during such
73 Prior to the 42 ^ Amendment Act, 1976, the opportunity of being heard was inquiry. There is no legal requirement that a second hearing be given. The
to be given twice. Firstly, in respect of the charges against him and secondly, lawSharma,
6. B.K. has been changed. See B K. Sharma, p 308.
pp. 308-309.
XVI
Constitutional Law o f India- II

311 treats both in the same manner. All termination of services may not be
removal or dismissal.
Both removal and dismissal are 'penalties’ awarded on the ground of
misconduct or inefficiency, etc. Both entail penal consequences which are
contained in the relevant rules. In both the cases, Art. 311(2) is attracted. Art 311
would not be attracted if there are no penal consequences like ioss of salary,
allowances or pension accompanying the action against him. Whatever the words
used , if these amount to removal or dismissal, the Article would apply.
I

'Reduction in rank’ means the degradation in rank or status of the officer


directed by way of penalty. The government servant is reduced to a lower post or
rank or to a lower stage in the pay scale. The form of the order is not conclusive.
In each case, the court must look to all the surrounding circumstances and
determine whether the order was given as a punishment.
A person holding a post in a substantive capacity cannot be brought down
to a lower post without following the Art. 311 procedure of enquiry, etc. But, if it is
reversion to the substantive post from an officiating one, it is not reduction in rank
for purposes of Art. 311 uniess there are any penal consequences (Parshottam v
UOI).
Suspension/ Compulsory Retirement - Suspension of a government servant is
neither dismissal nor removal. It is not a punishment. Hence an employee who
has been suspended cannot claim the protection of Art. 311.
Generally the Rules of employment provide for compulsory retirement of a
government servant after he has rendered the prescribed years of service.
Premature retirement is done in public interest. It does not cast a stigma on him. If
the decision to retire has been taken in a bona fide manner then its correctness
cannot be challenged in a court of law. Compulsory retirement does not attract
the safeguard of Art. 311(2), because it is not regarded as a punishment.
If the services were terminated in accordance with the terms of the contract
or on superannuating or by way of compulsory retirement as per procedure for the
same, Art. 311 would not provide any protection.

Procedural Requirements of Art. 311(2)75


Art. 311(2) gives to the Government servant a valuable right (‘reasonable
opportunity of being heard’). It has been held that the protection extended by Art.
311 is only procedural in nature and not substantive. No remedy may lie if all the
procedural requirements have been meticulously fulfilled
In the main these requirements are:
(a) that specific charges must be framed against a civil servant proposed to

75 Explain the requirement of “reasonable opportunity of being heard’’


prescribed under Art. 311(2) of the Constitution of India. Can such an
opportunity be taken away? [D.U.-2008]
What is the scope and content of the phrase "reasonable opportunity of
being heard in respect of those charges” in Art. 311(2) of the Constitution?
[/./VS.- 7996]
Civil Servants 267

be proceeded against;
(b) the charges (clear, precise and accurate) must be formally conveyed;
(c) he must be provided a reasonable opportunity of answering the charges;
(d) he should be informed of the evidence by which the charges are sought
to be substantiated;
(e) all evidence must be given in his presence;
(f) he must be given an opportunity of defending himself by cross-
examining the witnesses and adducing all evidence on which he relies;
(g) the decision in the matter must be based on the facts and materials
placed before the enquiring authority and no materials should be relied
upon without the civil servant concerned having an opportunity to
examine and explain them,
Some other requirements are: if the government servant demands a personal
hearing it must be provided. The employee has a right to argue his case. The
enquiry officer himself cannot be a witness.
The basic principle is that the enquiry must follow rules of natural justice
(Union of India v Verma AIR 1957 SC 882). The broad principles are: No one can
be a judge in his own cause; Both the parties must be heard; Reasonable notice
must be given; There should be no bias (viz. on the part of the enquiry officer).

Exceptions to Art. 311 (2): Exclusion of Inquiry and Opportunity of being


Heard76
The enquiry contemplated in Art. 311 may, however, be dispensed with in certain
cases like: (a) when the person has been guilty of misconduct resulting in
conviction or a criminal charge and on that ground he is sought to be dismissed or
removed or reduced in rank; (b) where the appropriate authority records in writing
reasons for the enquiry not being practicable; (c) when the President or the
Governor, as the case may be, is satisfied that in the interest of the security of the
State it is not expedient to hold such an enquiry Second Proviso to Art. 311(2)].

(a) Exception I: Conviction on a Criminal Charge [Proviso 2(a), Art. 311


(2)]
Clause (a) of the second proviso to Art. 311(2) provides that where a civil servant
is convicted on a criminal charge, he may be dismissed or removed or reduced in
rank without giving him an opportunity of hearing before he is so punished.
Conviction,

76 Explain the circumstances when a civil servant can be dismissed from


services without holding regular enquiries. [LC.II- 2006/2007] [I.A.S.
2006]

XVI
Constitutional Law o f India- II

for the purpose of this exception, must be under any law which provides for
punishment for a criminal offence, whether involving moral turpitude or other
crimes. However, conviction must be one which has been imposed upon the
delinquent servant during the course of and not prior to the appointment in
question
Conviction means final conviction i.e. conviction by the final court of appeal.
However, the passing of order of dismissal, removal or reduction in rank is not
barred on ground that sentence was suspended by appellate court or accused
was released on bail pending appeal/revision. But if the conviction is
subsequently set aside, the order of dismissal or removal made against the
delinquent would cease to have effect and he would be entitled to be reinstated
along with salary and allowances for the back period {State of U.P. v Mohd. A/oor
AIR 1958 SC 26).
In Roshan Lai Ahuja v S.C Jain (AIR 1987 SC 384), the petitioner working
as a Draftsman, was dismissed from service cn the ground of his conviction for
attempting to commit murder of his wife. Although, he was absolved of moral
turpitude, the Supreme Court held his dismissal under clause (a) of second
proviso to Art. 311(2), without giving him an opportunity of hearing, as neither
illegal nor improper.

Leading Case: state of m.p. V hazarilal (AIR 2008 SC


1300)

In this case, the respondent was a peon in a school. He is said to have


assaulted one Ram Singh. He was prosecuted for commission of the
said offence and was convicted under Sec. 323, IPC and sentenced to
undergo one month’s simple imprisonment. On an appeal, the sentence
was reduced to a fine of Rs. 500 only. However, a show cause notice
was issued to the respondent as to why disciplinary action shall not be
taken against him in view of the judgment of conviction passed against
him iri the said criminal case. By an order his services were terminated
by the Deputy Director, Public Education, Vidisha.
The State Administrative Tribunal, on an appeal filed by the
respondent, allowed his appeal and held that the appellant succeeds on
the ground that the punishment of removal from service is grossly
excessive because the punishment was only under Sec. 323, IPC and
the punishment does not involve any moral turpitude. Every power
vested in a public authority has to be exercised fairly, justly and
reasonably.
A writ petition was filed there against by the appellant (State)
before the High Court, but it was dismissed by reason of the impugned
judgment of the Tribunal. Before the Supreme Court, it was contended
on behalf of the appellant that the High Court failed to take into
consideration the fact that the Tribunal or the High Court could not have
interfered with the quantum of punishment.
Civil Servants 269

The Supreme Court observed: The case in hand appears to be a


gross one. The respondent had not committed any misconduct within
the
'

270 Constitutional Law o f India- II

meaning of the provisions of the M.P. Civil Services Rules, 1966. The
disciplinary authority has been empowered to consider the circumstances
of the case where any penalty is imposed on a Government servant on
the ground of conduct which has led to his conviction on a criminal
charge. But the same would not mean that irrespective of the nature of
the case in which he was involved or the punishment which has been
imposed on him, an order of dismissal must be passed. Such a
construction, in our opinion, is not warranted.
An authority which is conferred with a statutory discretionary power
is bound to take into consideration all the attending facts and
circumstances of the case before imposing an order of punishment. While
exercising such power, the disciplinary authority must act reasonably and
fairly. The respondent occupied the lowest rank of the cadre. Continuation
of his service in the department would not bring a bad name to the State
He was not convicted for any act involving moral turpitude. He was
involved in a matter for causing simple injury to another person. He was
not punished for any heinous offence. He was not even sent to prison and
only a nominal fine was imposed on him.
In Shankar Das v UOI (AIR 1985 SC 772), the appellant
(Government servant), under compelling circumstances, could not deposit
the money in question. He was not a previous convict. The Magistrate
observed that having regard to the circumstances of the case, he should
be dealt with under the Probation of Offenders Act, 1958. Despite the said
observation, he was dismissed from the service. The Supreme Court held
that: “It is to be lamented that despite these observations, the
Government chose to dismiss the appellant in a huff, without applying its
mind to the penalty which could appropriately be imposed upon him
insofar his service career was concerned." Referring to Art. 311, Proviso
2(a), the court observed that the right to impose a penalty carries with it
the duty to act justly. Considering the facts of the case, the penalty
imposed upon the appellant was whimsical.77
In the present case, the Apex Court observed: Furthermore the
legal parameters of judicial review has undergone a change. Wednesbury
principle of unreasonableness has been repealed by the doctrine of
proportionality. It is interesting to note that distinguishing between the
traditional grounds of judicial review and the doctrine of proportionality,
Lord Carswell in Tweed v Parades Commission for Northern Ireland
(2007) 2 All ER 1007, observed:
“The starting point is that there is an overlap between the traditional
grounds of review and the approach of proportionality. Most cases wouid
be decided in the same way whichever approach is adopted. But the

77 A civil servant is convicted of embezzlement but let off under the Probation of
Offenders Act. He is dismissed by the President without enquiry and hearing.
Is the dismissal valid? [D.U.-2008\
Civil Servants 271

intensity of review is somewhat greater under the proportionality approach


I would mention three concrete differences without suggesting that my
statement is exhaustive. First, the doctrine of proportionality may require
the reviewing court to assess the balance which the decision maker has
struck, not merely whether it is within the range of rational or reasonable
decisions. Secondly, the proportionality test may go further than the
traditional grounds of review inasmuch as it may require attention to be
directed to the relative weight accorded to interests and considerations.
Thirdly, even the heightened scrutiny test developed in R. v Minister of
Defence, Ex p Smith (1996) QB 517, is not necessarily appropriate to
the protection of human rights.”
In Coimbatore District Central Cooperative Bank v Coimbatore
District Central Cooperative Bank Employees Association (2007) 4 SCC
669, also, this court accepted the applicability of the doctrine of
proportionality [vide Ranjit Thakur v UOI (1987) 4 SCC 611; M.P
Gangadharan v State of Kerala (2006) 6 SCC 162],
The Court, thus, held in the present case, that the dismissal of the
respondent was not justified.]

(b) Exception II: Where Inquiry is not Reasonably Practicable [Proviso


2(b), Art. 311 (2)]
Article 311(2), proviso 2(b), provides that no inquiry need to be held for the
dismissal, removal or reduction in rank of a civil servant, if the disciplinary
authority records in writing that it is not reasonably practicable to hold such
inquiry.
In Satyavir Singh v UOI (AiR 1986 SC 555), a large number of employees
of RAW (Research and Analysis Wing which is the intelligence agency of the
government operating outside India) were indulging in various act of misconduct.
They had resorted to coercing and intimidating their co-workers. The Government
dismissed the said employees without an enquiry. It was stated that because of
coercion and threat no employee would come forward to give evidence. The
disciplinary authority concluded that it was not reasonably practicable to hold an
enquiry. The Court upheld the dismissal order. The Court protected the
Government from its own delinquent employees.
The decision to dispense with the inquiry, however, cannot be rested solely
on the ipse dixit of the concerned authority.
In Jaswant Singh v State of Punjab (AIR 1991 SC 385), the appellant was
dismissed form service, without holding an inquiry by invoking clause (b) of
second proviso and the reason for dispensing with the inquiry was that “he had
thrown threats that he with the help of other police employees would not allow
holding of any departmental enquiry against him and he and his associates would
not hesitate to cause physical injury to witnesses as well as the enquiry officer."
The dismissal order was quashed as not sustainable. The Supreme Court held
that the disciplinary authority did not furnish any particulars as to threats thrown
by the delinquent servant, nor any material was shown on which the concerned
'

272 Constitutional Law o f India- II

authority came to the conclusion that the delinquent had thrown threats.

(c) Exception III: Holding of Inquiry not Expedient in the Interest


of Security of State [Proviso 2 (c), Art. 311 (2)]
Clause (c) of second proviso to Art. 311(2) provides that no inquiry need to be
held before a civil servant of the government is dismissed, removed or reduced in
rank, where the President or the Governor, as the case may be, is satisfied that,
in the interest of the security of State, it is not expedient to give to the civil servant
such opportunity as contemplated in Art. 311(2).
It is now established that the satisfaction contemplated under Art. 311(2),
Proviso 2(c), of the President' Governor, is not his personal satisfaction but, on
the aid and advice of the Council of Ministers. It is because the power to take
disciplinary action against a government servant is the “executive power" of the
“Union or the State” as the case may be (Bakshi Sardari Lai v UOI AIR 1987 SC
2106).
In A.K. Kaul v UO! (AIR 1995 SC 1403), the Supreme Court explained the
true purport and the scope of Art. 311(2), Proviso 2(c):
(i) An order passed under Art. 311(2), Proviso 2(c) is subject to judicial
review and its validity can be examined by the court on the ground that
the satisfaction of the President/ Governor is vitiated by mala tides or it
is based on wholly extraneous or irrelevant grounds
(ii) The President/Governor while exercising the power has to bear in mind
the distinction between situations which affect the security of the State
and the situations which affect public order or law and order.
(iii) The Government is obliged to place before the court or tribunal the
relevant material on the basis of which the satisfaction was arrived at
subject to a claim of privilege under Sec, 123 and 124 of the Evidence
Act to withhold production of a particular document or record
(iv) Unlike clause (b), clause (c) of the second Proviso does not prescribe
for the recording of reasons for the satisfaction.
(v) The said claim of privilege would not extend to the disclosure of the
nature of the activities because such disclosure would not involve
disclosure of any information connecting the employee with such
activities or the source of such information.
(vi) In the instant case, the Supreme Court relied upon S.R. Bommai v UOI
(AIR 1994 SC 1918), and held that the satisfaction of the President;'
Governor under clause (c) of the second Proviso was subjected to the
judicial review within the limits laid down in that. case. Since the
satisfaction of the President while making a Proclamation under Art.
356(1), has been held to be justiciable to a limited extent, there is
nothing in the provisions
of clause (c) of the second Proviso which compels a departure from the
principles laid down in that case.10
Civil Servants 273

LEADING CASE: UNION OF INDIA v TULSIRAM PATEL


[(1985) 3 SCC 398]
In this case, the Supreme Court held that for a government servant to
discharge his duties faithfully and conscientiously, he must have a
feeling of security of tenure. At the same time, it would be in public
interest and for public good that government servants who were
inefficient, dishonest or corrupt or had become a security risk should
not continue in service and that the protection afforded to them under
the laws made under Art.
309 and by Art. 311 be not abused by them to the detriment of public
interest and public good.
(I) In the first case (appeal), the respondent was working as
an Auditor in the Regional Audit Office, M.E.S. He hit his
superior officer with an iron rod, which led to his
conviction under Sec. 332 I.PC., for causing head injury.
He was compulsorily retired under clause (a) of second
proviso to Art. 311(2) and the Service Rule 19(1). The
Supreme Court held the punishment imposed on the
respondent as neither excessive nor arbitrary in the fact
situation.
The Supreme Court laid down that the charges in the criminal case
must relate to a misconduct of such magnitude as would have
deserved the penalty of dismissal, removal or reduction in rank.
Further, a conviction on a criminal charge does not automatically entail
dismissal, etc. of the concerned government servant; the disciplinary
authority must consider whether his conduct which led to his conviction
was such as warranted the imposition of a penalty and if so what the
penalty should be.
However the authority would do this exercise ex parte and
without giving to the government servant a reasonable opportunity of
hearing. But the competent authority must exercise the power fairly,
justly and reasonably, while deciding the penalty to be imposed on the
delinquent servant. Therefore, if the court finds that the penalty
imposed is arbitrary, or grossly excessive, etc. the court would strike
down the order imposing penalty. It was illustrated that clause (a) of the
second proviso to Art. 311(2) did not contemplate that a government
servant who was convicted for parking his scooter in a noparking area
should be dismissed from service.
(II) In the second case (appeal), the members of the Central
Industrial Security Force were dismissed from service
after dispensing with holding of inquiry under Rule 37(b)
of the
C.I.S.F. Rules, 1969 read with Art. 311(2), proviso 2(b). The
respondents were pressing for recognition of their Association
XVI
Constitutional Law o f India- II

and in view of the negative attitude of the authorities, they


resorted to wilful and deliberate disobedience of orders of the
supervisory officers and “gherao" of such officers, hunger
strike, dhama, shouting of rebellious slogans and threats of
violence and bodily harm to supervisory officers and acts
tending to intimidate the officers, thus creating a riotous
situation completely breaking down discipline in the Force as a
result of which a Security Force itself became a security risk. All
these acts virtually amounted to a mutiny and the situation
became so grave that the Army had to be called out. The
Supreme Court held that no person with any reason or sense of
responsibility could say that in such a situation the holding of an
inquiry was reasonably practicable.
In the third case (appeal) decided in this case, the railway employees were
dismissed or removed from service, without holding any inquiry. These
employees had participated in an illegal all-India strike with the object of
forcing the Government to meet their demands. The court observed that in
seeking to have their demands which were for their private gain and in their
private interest, they caused untold hardship to the public and prejudicially
affected public good and public interest. Not only that these employees had
struck work, but also paralysed the railway services, assaulted and
intimidated the loyal workers and superior officers, held the country to ransom
In these circumstances, the Supreme Court held that it could not be said that
an inquiry was reasonably practicable
The Supreme Court laid down the following guidelines for the
application of Art. 311(2), proviso 2(b):
(a) It is the satisfaction of the disciplinary authority that “it is not
reasonably practicable to hold the inquiry" contemplated in Art.
311(2). The Supreme Court would not sit in judgment over the
relevancy of the reasons given by the authority like a court of
first appeal and that even in those cases where two views were
possible, the court would decline to interfere
(b) It is not a total or absolute impracticability which is required by
clause (b). What is required is that the holding of the inquiry is
not practicable in the opinion of a reasonable man taking a
reasonable view of the presenting situation.
(c) Tne disciplinary authority must not dispense with the inquiry
lightly or arbitrarily or out of ulterior motives or merely in order
to-avoid the holding of an inquiry or because the
Department’s case against the government servant is weak and
must fail.
(d) A situation which makes the holding of inquiry not reasonable
practicable may be existing at the initial stage or can also come
into existence subsequently during the course of inquiry. In
such a case also the authority would be entitled to apply clause
Civil Servants 275

(b) of second proviso because the word “inquiry” in that clause


includes part of an inquiry.
(e) The disciplinary authority is required to record 'reasons in
writing' for dispensing with the inquiry. There is no obligation to
communicate the reasons to the government servant. It would,
however, be better to do so, because such communication of
reasons would eliminate the possibility of an allegation being
made that the reasons have been subsequently fabncated It
would also enable the government servant to approach the
High Court under Art 226, or, in a fit case, the Supreme Court
under Art. 32.
(f) The reason for dispensing with the inquiry need not contain
detailed particulars, but the reason must not be vague or just a
repetition of the language of clause (b) of second proviso.
(g) The decision of the disciplinary authority is given finality by Art.
311(3). However, it is not binding upon the court so far as its
power of judicial review is concerned.
(Ill) In the fourth appeal decideo in this case, the petitioners
belonging to the M.P. District Police Force/M.P. SpecialArmed
Force were dismissed from service by applying Art. 311(2),
proviso 2(c). It was shown that the petitioners indulged in
violent demonstrations and riotous activities, attacked the police
station at a Mela ground, ransacked it and forced the release of
their colleagues who were arrested for an incident in which one
man was burnt alive in the Mela held at Gwalior. The petitioners
also carried on active propaganda against the Government by
holding secret meetings, distributing leaflets, and inciting the
constabulary to rise against the administration. After
considering these material facts, the Supreme Court held the
order of dismissal as justified. The Court observed that the facts
left no doubt that the situation was such that prompt and urgent
action was necessary and the holding of an inquiry into the
conduct of each of the petitioners, would not have been
expedient in the interest of the security of the State
Explaining the true purport and the scope of Art 311(2), Proviso 2(c), the
Court observed that the satisfaction of the President/Governor required by
Proviso 2(c) was with respect to “the expediency or inexpediency of
holding an inquiry in the interest of the security of State.” Expediency
involved matters of policy. Satisfaction might be arrived at as a result of
secret information received by the Government about the brewing of
danger to the security of State and like matters. The reasons for the
satisfaction reached by the President/Governor under clause (c) could
not, therefore, be required to be recorded in the order of dismissal,
XVI
Constitutional Law o f India- II

removal or reduction in rank, nor could they be made public. 78


When the principles of natural justice have been expressly
excluded by the second proviso, it cannot be imported by resorting to
Art. 14 of the Constitution. The Court overruled the decision in the
Challappans' case [(1976) 3 SCC 1990] where it held that imposition of
penalty without holding an inquiry was unconstitutional and illegal.
When the second proviso to Art. 311(2) applies, a public servant still
has two remedies available to him: (i) Departmental appeal, and (ii)
Judicial review. This would be a sufficient compliance with the
requirement of natural justice.
Comments - Tulsiram Patel is a watershed in the law of services. Till
then the Courts were always tilting the balance in favour of the
employee.
In Motiram v N.E.F Railway (AIR 1964 SC 600), the Supreme Court
virtually abolished tenure at pleasure. A government servant could
continue in service till superannuation. The judgment was criticized by
Seervai who posed the question what is the source of the right to hold
the post till superannuation? No law, rule or contract gives such a right.
In Tulsiram, the Supreme Court reversed the trend. The court said that
Art. 311 provided an exception to the doctrine of pleasure.
No Act or Rule can be framed to affect or limit the doctrine of
pleasure. The court made it clear that it is as much in public interest that
civil servants should have security of tenure as that the inefficient,
corrupt or those who are a security risk are not given protection of the
Constitution.
The doctrine of pleasure is not a thoughtless implant from the British
Law but public policy requires, public interest needs, and public good
demands that there should be such a doctrine. The court applied the
principle of Tulsiram in Satyavir Singh v UOI (1985) 4 SCC 252.]79

Art. 311(3): Finality Clause


Clause (3) of Art. 311 declares that if in respect of the delinquent servants
punished under Art. 311(2), Proviso 2(b), a question arises whether it is
reasonably practicable to hold such inquiry as is referred to in clause (2) of Art.
311, the decision thereon of the disciplinary authority shall be final.
However, it has been held in Tulsiram Patel case that the finality given to the
decision of the disciplinary authority by Art. 311(3) would not be binding upon the
court, so far as its power of judicial review was concerned.

Opportunity of Hearing at Punishing Stage [Proviso 1 to Art.


311(2)]

78 Narendra Kumar, pp. 563-567.


79 B.K. Sharma, pp. 306-307.
Civil Servants 277

Prior to the Constitution (42nd Amendment) Act, 1976, opportunity to be heard had
to be offered to the delinquent servant at the two stages i.e. at the inquiry stage and
at the punishing stage. 80
Thus, Art. 311 (2) as it stood prior to 1976, enabled the delinquent servant to
make his representation as to why the proposed punishment should not be inflicted
on him. This, second opportunity enabled the delinquent to plead that no case had
been made out against him, or that the conclusions drawn from the evidence taken
at the inquiry were not correct or that the punishment proposed to be imposed was
excessive. There was a plethora of case-law around this second opportunity of
hearing, and punishment was liable to be quashed for the non-compliance with this
requirement. With a view to expedite the disciplinary proceedings, the Constitution
(42nd Amendment) Act, 1976, cut down the procedural formalities, by abolishing the
second opportunity. It is now expressly provided that -
“Where it is proposed after such inquiry, to impose upon him any such
penalty, such penalty may be imposed on the basis of the evidence adduced during
such inquiry and it shall not be necessary to give such person any opportunity of
making representation on the penalty proposed.”
However, this amendment has not taken away the right of the delinquent to
approach the Court and challenge the order of the disciplinary authority at least on
the ground that the findings of the Inquiry Officer have not been established on the
materials adduced during the inquiry and also that the authority has not been
influenced by such findings while deciding the punishment to be imposed on the
delinquent servant.
The delinquent servant can challenge the order of punishment only when he
is furnished with a copy of the ‘inquiry report.’ The Supreme Court in UOI v Mohd
Ramzan Khan (AIR 1991 SC 471) held that deletion of the second opportunity by
the 42nd Amendment, 1976, had nothing to do with providing of a copy of the Inquiry
Report to the delinquent in the matter of making his representation. Though the
second stage of the inquiry in Art. 311(2) had been abolished, the delinquent was
still held entitled to represent against the conclusions of the Inquiry Officer. The
court, however,
restricted the right of the delinquent to the copy of the inquiry report only to the
cases where the disciplinary authority himself was not the inquiry officer. 81

LEADING CASE: managing director, electronic


CORPORATION OF INDIA (ECIL) v B. KARUNAKAR 82 (AIR 1994 SC 1074)

80 Art. 311(2) lays down that a civil servant cannot be dismissed, removed or
reduced in rank unless he has been given a reasonable opportunity to show
cause against the action proposed to be taken against him.” Critically
examine the statement with reference to the Constitution (42n Amendment)
Act, 1976.
[I.A.S.- 2000\
81 Narendra Kumar, pp. 569-570.
82 What is the scope and content of the phrase “reasonable opportunity of being
XVI
Constitutional Law o f India- II

A five-Judge Constitution Bench of the Supreme Court did a survey of


the law on the subject in this case and unanimously laid down the law as
follows:
Where the Inquiry Officer is not the disciplinary authority,
(i) the delinquent employee has a right to receive a copy of
the Inquiry Officer's report before the disciplinary authority
arrives at its conclusions with regard to the guilt or
innocence of the employee with regard to the charges
levelled against him. That right is a part of the employee's
right to defend himself against the charges levelled
against him. A denial of the copy of the report is a denial
of reasonable opportunity to prove his innocence and is a
breach of the principles of natural justice.
(ii) In regard to awarding of punishments other than
dismissal, removal or reduction in rank, the procedure to
be followed is laid down in the relevant service rules. The
law, laid down above, should apply to employees in all
establishments whether Government or non-Govemment,
public or private.
(iii) When for the non-furnishing of the report, the Inquiry is
set aside by the Court or Tribunal, the employee should
not, in all cases, be directed to be reinstated with back
wages.
The Court and Tribunal should cause the copy of the
report to be furnished to him and give the employee an
opportunity to show how his or her case was prejudiced
because of the non-supply of the report. The
Court/Tribunal should not mechanically set aside the
order of punishment on the ground that the report was not
furnished.
(iv) It is only if the Court/Tribunal finds that the furnishing of
the report would have made a difference to the result in
the case that it should set aside the order of punishment.
The Supreme Court thus reaffirmed the l?w laid down in Ramzan’s case
It was further emphasized that the reinstatement ordered as a result of
the

heard in respect of those charges" in Art. 311(2) of the Constitution? Does it


embrace a right to receive a copy of the Inquiry Officer’s report? Refer to the
case-law on the subject. [/.AS.-7996]
279 Constitutional Law o f India- II

setting aside of the inquiry for failure to supply the report should be
treated as a reinstatement for the purpose of holding the fresh inquiry
from the stage of furnishing the report and no more, where such fresh
inquiry was held.
The Supreme Court thus affirmed the ruling in Ramzan's case,
that the law laid down therein in regard to the furnishing of the report of
the Inquiry Officer, would have prospective application. Therefore, the
orders of punishment passed prior to the decision in Mohd Ramzan
Khan’s case (i.e. 20th November, 1990), without furnishing the report of
the Inquiry Officer should not be disturbed and the disciplinary
proceedings which gave rise to the said orders should not be reopened
on that account. It was considered desirable for administrative reality
and public interests.]16

Right of Government Employees to go on Strike

LEADING CASE: T.K. RANGARAJAN v STATE OF T.N.


(AIR 2003 SC 3032)

In this case, a two-Judge bench of the Supreme Court held that


Government servants have no right to go on strike. In the year 2002,
the Government of Tamil Nadu terminated the services of 2 lakh
employees under the TN Essential Services Maintenance Act, 2002;
the government employees had gone on strike for their demands.
The court said that “Government employees cannot hold society
to ransom by going on strike’’. If the employees felt aggrieved by any
Government action, they should seek redressal from the statutory
machinery provided under different statutory provisions. The court said
strike as a weapon is mostly misused which results in chaos and total
maladministration.
The court held:
(i) There is no fundamental right to go on stnke - Though
the trade unions have a guaranteed "right for collective
bargaining’’ on behalf of employees, they have no
fundamental right to strike.
No political party or organization can claim a right to paralyse economic
and industrial activities of a State or the nation or cause inconvenience
to the citizens. There cannot be any right to call or enforce a “Bandh”
which interferes with the exercise of the fundamental freedoms of other
citizens, in addition to causing national loss in many ways [vide

1 6 . Narendra Kumar, pp. 570-571.


XVI
Constitutional Law o f India- II

Communist Party of India (M) v Bharat Kumar (1998) 1 SCC 201],


In All India Bank Employees'Asscn v National Industrial Tribunal
(1962) 3 SCR 269. it was held that even very liberal interpretation of Art.
19(1)(c) of the Constitution cannot lead to the conclusion that trade unions
have a guaranteed right to an effective collective bargaining or to strike
either as part of collective bargaining or otherwise.
(ii) There is no legal/ statutory right to go on strike- There is no
statutory provision empowering the employees to go on
strike. Further, there is prohibition to go on strike under the
T.N. Govt. Servants Conduct Rules, 1973.
(iii) There is no moral/ equitable justification to go on strike - In a
society where there is large scale unemployment and a
number of qualified persons are eagerly waiting for
employment, strike cannot be justified on any equitable
ground.
For redressing their grievances, instead of going on strike, if employees do
some work honestly, diligently and efficiently, such gesture would not only
be appreciated by the authority but also by people at large. The reason
being, in a democracy even though they are Government employees, they
are part and parcel of governing body and owe duty to the society.
The court agreed with the suggestion of the Government Counsel
that the Government was agreeable to withdraw the dismissal orders
against the large number of employees except 6072 whose cases would
be resolved by a panel of three retired Judges. The court hoped that
reinstated employees would “take care in future in maintaining discipline.”]

All-India Services (Art. 312)


All India services are distinguished from Central and State services inasmuch as
members of Central services are concerned with only the affairs of the Union and
those of State services with State matters while members of the All-India services are
common to the Union and the States and serve by turns both the Union and State
Governments. An officer is allotted to a State cadre and serves the Union on
deputation. Two All India services - the Indian Administrative Service (IAS) and
Indian Police Service (IPS) - are mentioned in the Constitution itself. The officers are
appointed by the President.
Art. 312 lays down that if Rajya Sabha passes a resolution by two-thirds
majonty to the effect that it is necessary or expedient in national interest to create
one or more All-India services, including All-India Judicial Service, Parliament may by
law provide for such services. Parliament has under this Article enacted the All India
ServicesAct, 1951 creating certain All-India Services in addition to the IAS and the
IPS which had been already created in 1948 Art. 312 also empowers Parliament to
regulate the recruitment and conditions of service of persons appointed to All-India
Civil Servants 281
services.
Article 312 contemplates an ‘All India Judicial Service' alsc but no steps have
been taken to establish it. The All India Services enjoy a large degree of
independence because they are not under the control of the States even though,
working in it. Members of All India Services are posted in key posts in the States.
Some of the considerations which influenced the creating of All-India Services
are.
- In any administration, more so in a developing economy like India, the
services should have the best available talent in the country as a whole. It
requires that the recruitment to the public services should be on All-India
basis.
- It emphasizes the unity of India and encourages the development of a
national point of view. Also, it promotes uniform standard of administration
in the country.
- The All-India basis of recruitment also helps in securing greater inter-State
co-ordination for efficient implementation of all India policies.

Public Service Commissions (Arts. 315-323)


Article 315 lays down that there shall be a Public Service Commission for the Union
and a Public Service Commission for each State. Two or more States may opt for a
Joint Commission On request, the Union Public Service Commission may also agree
to serve some needs of a State.
The Public Service Commissions are envisaged as independent constitutional
institutions not subject to governmental or political interference or control and
charged with the responsibility of recruitment and management of public services.
Article 318 provides that the terms and conditions of service of a member of the
Commission cannot be varied to his disadvantage after his appointment. The
President or the Governor, as the case may be, may determine the number of
members and staff of the Commission and regulate their conditions of service. All
expenses of the Union and State Service Commissions are charged on the
Consolidated Fund of the Union or the State concerned (Art. 322).
The Chairman and members of the Public Service Commissions are appointed
by the President or the Governor as the case may be. At least half of the members
must have had ten years’ service in the Union or a State Government (Art. 316).
Members shall hold office for a fixed term of six years or until the age of 65 in case of
the Union and 62 in case of the State Commissions, whichever is earlier. A member
may himself resign from his office.
A Commission member is not removable from his office except in case of
insolvency, infirmity of mind or body (thus, unfit to continue in office, in the opinion of
the President), on engaging in another paid employment; or on being found by the
Supreme Court (on reference made by the President to the Supreme Court) guilty of
proved misbehaviour, for having an interest in a Government contract or sharing the
profits of any such contract or agreement (Art. 317) (U.P Public Service Commission
v Suresh AIR 1987 SC 1953; Hargovind v Raghukul AIR 1979 SC 11C9).
In Jai Shanker Prasad v State of Bihar (1993) 2 SCC 597, held that the
XVI
Constitutional Law o f India- II

“infirmity of body” must have arisen after appointment and not which existed at the
time of appointment. In this case, a blind qualified person was appointed as a
member of the Bihar State Public Service Commission. The court upheld his
appointment
Member of a Public Service Commission on expiration of his term of office is
ineligible for re-appointment to that office. They are also ineligible for any other
employment under the Central or State Governments. This provision is necessary to
ensure impartiality (Art. 319). Members of the Commission are not eligible for any
other appointment under the Government (Union of India v U.D. Dwivedi AIR 1997
SC 1313).
The Chairman of a State Commission can become a member or Chairman of
UPSC or Chairman of another State Commission. A member of the UPSC is eligible
to become chairman of UPSC or of a State Commission and a member of a State
Commission is eligible to become a member or Chairman of UPSC or any State
Commission (Art. 319).
The functions of a Public Service Commission are:
(i) to conduct examinations for appointment to the services of the Union/State;
(ii) to make recommendations to the Union/State Government for appointment
of persons to its services;
(iii) to be consulted in regard to method of recruitment, principles in matters of
appointments, promotions, transfers from one service to another, and
disciplinary matters of civilian employees;
(iv) on any claim for compensation in respect of injuries sustained by a person
while in service of the Government;
(v) to advise on any other matter ,that may be referred by the President/
Governor;
(vi) on request to assist two or more States in regard to schemes for joint
recruitment;
(vii) to present an annual report to the President/Governor who shall cause it to
be laid, before Houses of Parliament/State Legislature (together with a
memorandum as regards the cases where the advice of the Commission
was not accepted and the reasons for such non-acceptance) (Art. 323);
and
(viii) any other function that Parliament/State Assembly may by law assign (Art.
321).
The Supreme Court has held that the function of the Commission is purely advisory
and if the Government fails to consult it in any matter specified for consultation, a
public servant affected thereby cannot expect a remedy in a court of law under Art.
320. But, where law or regulation provides for consultation, it will constitute a legal
obligation [Keshav v U.R HESC (1985) 1 SCC 671; Neelima v State of Haryana MR
1987 SC 169]
The President and the Governors may make regulations specifying the matters
in which the Commission may not be consulted. Art. 320 (4) lays down that the
Commission need not be consulted as regards the reservation of post for Backward
Classes, SCs/ STs.
Amendment of the Constitution (ARTICLE 368)
2

The amendment of the Constitution, at times, becomes necessary to adapt to the


changing needs of national development and strength; to overcome the difficulties
which may encounter in future in working of the Constitution; and to realise any
popular demand for changing the political system e.g. State reorganisation,
provisions for SC/STs, lowering of age for voting, etc. The amendment of Constitution
should only be resorted to in cases of serious repercussions or emergent
circumstances or a special contingency.
However, the amendment of Constitution often been used to achieve political
purposes or to override judicial verdicts. Moreover, multiple and multifarous
amendments undermines the sanctity of Constitution as an organic instrument and
creates confusion. For example, the 42nd Amendment (‘Act of Revision’) effected
vital changes (the Congress government took advantage of its monolithic control) e.g.
fundamental rights devalued vis-a-vis directive principles, legislature override the
judiciary, etc. The 43rd and 44th Amendments by Janta Party government wiped out
many of the provisions of 42nd Amendment.
The procedure for amendment, instead of being rigid, has rather proved too
flexible, and more than ninety Amendments have been made so far. There is no
separate constituent body for amendment and the Parliament, the ordinary legislative
organ of Union, performs the function.
The fact that many amendments have been made just to amend the
amendments justifies the conclusion that on some occasions the Parliament had
acted in hasty and casual manner. In this respect the Parliament lost its credibility in
such a manner and to such an extent that the judiciary is forced to import the concept
of ‘unamendability of the basic features.’
Amendment of the Constitution 283
Art. 368: Power of Parliament to Amend the Constitution and Procedure
therefor83
(1) Notwithstanding anything in this Constitution, Parliament may in exercise of
its constituent power amend by way of addition, variation or repeal any
provision of this Constitution in accordance with the procedure laid down in
this Article.
(2) An amendment of this Constitution may be initiated only by the introduction
of a Bill in either House of Parliament, when the Bill is passed in each
House by a majority of the total membership of that House and by a
majority of not less than two-thirds of the members of House present and
voting, the Constitution shall stand amended.
Provided that if such amendment seeks to make any change in -
(a) Arts. 54, 55, 73, 162 or 241, or
(b) Chapter IV of Part V, Chpater V of Part VI, or Chpater I of Part XI, or
(c) any of the lists in Seventh Schedule, or
(d) the representation of States in Parliament, or
(e)the provisions of Art. 368 itself,
the amendment shall also require to be ratified by the legislature of not
less than one-half of the States.84
(3) Nothing in Art. 13 shall apply to any amendment made under this Article.
(4) No amendment of this Constitution (including the provisions of Part III),
made or purporting to have been made under this article [whether before or
after the commencement of Sec. 55 of Constitution (42nd Amendment) Act,
1976] shall be called in question in any court on any ground.
(5) For the removal of doubts, it is hereby declared that there shall be no
limitation whatever on the constituent power of Parliament to amend by
way of addition, variation or repeal the provisions of this Constitution under
this article.
(Note: Clauses (4) and (5) were held unconstitutional in Minerva Mills
case.)
For the purposes of amendment the provisions of Constitution fall under 3 categories:
(a) Amendment by Simple Majority - of each House of Parliament. It is like an
ordinary bill. Formation of new States, creation or abolition of Legislative
Councils (Arts. 4, 169 and 239-A) is made by such procedure. Thus,
amendment at the instance of the States, or amendment by State
legislatures, are included in such category. Amendments under this
category are expressly excluded from the purview of Art. 368.

1. Enumerate the various procedures of amending the Constitution. [/ AS.-93]


84 Mention three provisions of the Constitution which, if amendment is sought,
requires ratification by not less than one-half of the States. [L.C.H- 2007]
XVI
Constitutional Law o f India- II

Amendment by Special Majority i.e. majority of ‘total members of each House' and by
(b) a majority of at least 2/3rd 'present and voting' All amendments, other
than those referred to above, come within this category.
Amendment by Special Majority and Ratification by States - The States
jo) are given an important voice in the amendment of these matters.
Election of President, Extent of executive power of Union and States,
Supreme and High Courts, Distribution of legislative powers between
Centre and States, and three lists, Representation of States in Parliament
and Art. 368 itself. These are provisions which relate to the federal
structure of the Constitution and the powers, position and authority of the
constituent States.85

ARE FUNDAMENTAL RIGHTS AMENDABLE?86

In 1951, within a year of the Constitution coming into force, the Constitution
(First Amendment) Act was passed. This Act sought to curtail the right to
property guaranteed by Art. 31. Its constitutionality was questioned in
Shankari Prasad v UOI (AIR 1951 SC455).
The argument addressed by the petitioners was that Art. 13(2) prohibited the State
from making any law which takes away or abridges a fundamental right. The word
‘law’ used in Art. 13(2) includes all Acts including the Constitution Amendment Acts.
Rejecting the contention the Supreme Court held that an Act passed in exercise of
the power conferred by Art. 368 is not a law within the meaning of Art. 13(2). The
Court ruled that law in that article referred to an ordinary law but not to a Constitution
Amendment Act (Constituent law) The Court thus distinguished between the ordinary
legislative power and constituent power. Fundamental rights are subject, to the
amending power of the Parliament under Art. 368. In other words, ordinary laws
cannot amend the fundamental rights but constituent laws can.

85 “Amending power has been exclusively assigned to the Union Parliament


except when the amendment involves amendment of the federal provisions.”
Discuss.
[/AS.-92]
86 Discuss the limitations on the power of Parliament to amend, change, modify or
vary the provisions of Fundamental Rights with reference to decided cases and
constitutional amendments. [L.C I- 2006]
How much is it justified to claim that the judiciary has a say in the matter of
amendment of the Constitution? Elucidate the theory of basic structure pointing
out constitutional provisions. [L.C.II- 2006]
"All these years Art. 368 has been a tug of war between the constituent power
and power of judicial review.” Comment on and narrate this development in
detail. [I.A.S.-2005\
Amendment of the Constitution 285
In Sajjan Singh v State of Rajasthan (AIR 1965 SC 845), the Supreme Court
adhering to its decision in Shankari Prasad case, held that the words ‘amendment of
Constitution’ means amendment of all the provisions of Constitution.
(II) In Golak Nath v State of Punjab (AIR 1971 SC 1643), the Supreme Court
by a majority of 6 to 5 overruled its earlier decisions in Shankari Prasad
and Sajjan Singh cases and held that Parliament cannot amend the
Fundamental Rights, as these rights are assigned transcendental position
under our Constitution. It refused to distinguish between legislative power
and constituent power.
The Chief Justice rejected the argument that power to amend the Constitution was a
sovereign power and that it did not permit any implied limitations, and observed that
amendment is a legislative process and Art. 368 lays down merely the ‘procedure’ for
amendment of Constitution. An amendment is a ‘law1 within the meaning of Art. 13(2)
and therefore, if it violates any of the fundamental rights it may be declared void.
In answer to the question as to whether there would be any way to change the
structure of Indian Constitution or abridge the fundamental rights; J. Hidayatullah
referring to the amending process under the French and Japanese Constitutions,
explained that Parliament could amend Art. 368 to convoke another or new
Constituent Assembly. That Assembly might be able to abridge or take away-the
fundamental rights, if desired.
As to the Constitution (1st Amendment) Act, 1951, the Constitution (4th
Amendment) Act, 1955 and the Constitution (17th Amendment) Act, 1964, the
majority held that these amendments abridged the scope of fundamental rights. But
they took recourse to the “doctrine of prospective overruling” and upheld the validity
of these amendments.
The Court said that it was indisputable that the court could overrule its earlier
decisions. Also, there would not be any valid reason as to why the court should not
restrict its ruling to the future: The decision in this case, thus, would not invalidate the
amendments made so far to the fundamental rights (a large body of legislation had
been enacted bringing about agrarian reform in the country, pursuant to these
amendments). But in future, Parliament would, the court laid down, have no power to
abridge any of the fundamental rights.
(III) 24th Amendment, 1971 - In order to remove difficulties created by the
decision in Golak Nath’s case, Parliament enacted this Act. It not only
restored the amending power of Parliament but extended its scope by
adding the words in Article 368, “to amend by way of the addition or
variation or repeal any provision of this Constitution in accordance with the
procedure laid down in this Article.” A new clause added to Art. 13 which
provides that “nothing in this Article shall apply to any amendment of this
Constitution made under Art. 368.” Thus, Art. 13 would not be applicable to
Constitution amending Acts.
Therefore, the validity of a Constitution Amendment Act shall not be open to
question on the ground that it takes away or affects a fundamental right. The 24th
Amendment has been held to be valid in Keshavanand’s case (though subject to
some qualifications).
XVI
Constitutional Law o f India- II

(IV) Keshavanand Bharati’s case and Theory of Basic Structure87

LEADING CASE: keshavanand bharati v state OF kerela88 (AIR 1973 SC1461)


In this case, also known as ‘Fundamental Rights case1 the question involved
was as to what was the extent of the amending power conferred by Article 3687
A special bench of 13 judges was constituted to hear the case (the
case was heard for 5 months and the court gave the longest judgement
running into 595 pages). The views of Majority (7 judges - Sikri C.J., Shelat,
Hegde, Grover, Jagmohan Reddy, Mukherjea, J.J.; Khanna J. took a middle
path and tilted the balance in forming the majority) could be summarised as
follows:
(i) The Golak Nath’s case was overruled and the majority held that
Art. 368 even before 24th Amendment contained the power as
well as procedure of amendment The 24th Amendment merely
made explicit what was implicit in the unamended Art. 368.
Hence, limitations upon the amending power must be found
from Art. 368 itself.
(ii) The Parliament has wide powers of amending the Constitution
and it extends to all the Articles, but amending power is not
unlimited and does not include the power to destroy or
abrogate the basic feature’ or 'framework' of Constitution.
There are implied or inherent limitations on the power of
amendment under Art. 368. Within these limits Parliament can
amend every Article of Constitution,
(iii) Whether there are implied limitations on the amending power
or not would depend upon the interpretation of word

87 Are there features in the Constitution that arc basic or fundamental to it? Would
you subscribe to the proposition that the Parliament's competence to amend the
Constitution is subject to the basic or fundamental features of the Constitution?
Elucidate. [M.S.- 2007]
88 Is the power of amendment implicit in the Constitution of India and is it wide
enough to replace the present Constitution by a new one without the need of
revolution or referendum? Discuss by making reference to Keshavananda Bharati
case doctrine. [I.A.S.-91]
Judgment in Keshavananda Bharati case led to introduction of new constitutional
jurisprudence by the name of the ‘Doctrne of Basic Structure/Feature’. This has
introduced uncertainty with respect to limits of legislative power by the way of
constitutional amendment. Comment. [L.C.II- 2007]
[Note: Also see under the heading 'Doctrine of Basic Features as it Stands
Today', below.]
Amendment of the Constitution 287
‘amendment.’ Khanna J., said that the word ‘amendment' postulated
that the old Constitution must survive without loss of identity and must
be retained though in the amended form and, thus, the power does not
include the power to abrogate the basic structure.
Sikri, C.J. said that the word ‘‘amendment’’ must derive its colour from Art, 368 and
rest of constitutional provisions. Reading the Preamble, the fundamental importance
of freedom of individual, the importance of Directive Principles, and various other
provisions, a conclusion emerges that it was not the intention of Constitution makers
to use the word “amendment” in the widest sense The expression “amendment of
Constitution” in Art. 368 means any change in any of the provisions of Constitution
within the broad contours of Preamble and the Constitution to carry out the
objectives in Preamble and Directive Principles. Thus it would mean that while
fundamental rights cannot be abrogated, reasonable abridgements'of fundamental
rights can be effected in the public interest.
(iv) ‘Basic Structure’ - According to Sikri, C.J., the ‘basic structure’ was built
on the basic foundation i.e. the freedom and dignity of the individual;
the basic structure of the Constitution consists of following features:
(a) Supremacy of Constitution, (b) Republican and Democratic form of
Government, and sovereignty of country, (c) Secular and federal
character of Constitution, and (d) Separation of powers between
Legislature, Executive and Judiciary.
Shelat and Grover, J.J , also included the ‘fundamental rights' and ‘Directive
Principles' in the basic structure.
The Minority View - The remaining 6 judges (Ray, Palekar, Mathew, Beg, Divedi and
Chandrachud) held that there are no limitations, express oi implied, on the amending
power. The word “amendment" did not include the power to completely abrogating
the Constitution at one stroke, it was, however, wide enough to erode the
Constitution completely step by step so as to replace it by another Constitution.
Thus, fundamental rights can be abrogated.
The ‘doctrine of basic structure' has been criticised, on the ground that it lays
down a vague and uncertain test. However, the court observed that there are many
concepts of law which cannot be defined precisely, but they do exist and play very
important part in our law, for example, natural justice and negligence. The Chief
Justice said that the argument that because something cannot be cut and dried or
nicely weighed or measured and therefore does not exist is fallacious. If the historical
background, the Preamble, the entire scheme of Constitution and the relevant
provisions thereof including Art. 368 are kept in mind, there can be no difficulty in
determining what are basic elements of the basic structure of Constitution. The
doctrine of basic structure will act as a safety-valve against the arbitrary use of amending
power.
Comments - After Golak Nath no fundamental right could be taken away or
abridged but after Keshavanand it is for the court to decide whether a
fundamental right is a basic feature or not. If it is so, then it cannot be
abrogated]
(V) Application of the Doctrine of Basic Structure - In Indira Gandhi v Raj Narain
(AIR 1975 SC 2299), the Supreme Court added the following features as
‘basic features’ to the list of basic features laid down in Keshavanand's
XVI
Constitutional Law o f India- II

case: (i) Rule of Law; (ii) Judicial review; (iii) Democracy, which implies free
and fair elections; (iv) Jurisdiction of Supreme Court under Art. 32.
(VI) 42nd Amendment and Doctrine of Basic Structure89 - In Minerva Mills Ltd. v
UOI (AIR 1980 SC 1789), the constitutional validity of the Secs 4 and 55 of
42nd Amendment, 1976, which effected changes in the Art. 31-C and Art.
368 respectively, were challenged by the petitioner Minerva Mills.
(a) Sec. 55 of 42nd Amendment added two new clauses, Clauses (4) and
(5) to Article 368: Cl. (4) provided that “no amendment of Constitution
(including the provisions of Part III) made under Art. 368 (whether
before or after the commencement of 42nd Amendment), shall be
called in question in any court on any ground. Ci. (5) declared that
there shall be no limitation whatever on the constituent power of
Parliament to amend by way of addition, variation or repeal the
provisions of Constitution under this Article. Thus, it would mean that
even the “basic feature” of Constitution could be amended.
Clause (4) asserted the supremacy of Parliament. It was urged by the Union that
Parliament represents the will of people and if people desire to amend Constitution
through Parliament, there can be no limitations whatsoever on the exercise of this
power. The theory of 'basic structure’ as invented by Supreme Court is vague and
will create difficulties. It was argued that the amending body under Art. 368 has the
full constituent power. In other words, the Parliament acts in the same capacity as
the Constituent Assembly when exercising the power of amendment under Art. 368.
The Supreme Court by 4 to 1 majority struck down clauses (4) and (5) of Art.
368 on the ground that these clauses destroyed the essential feature of basic
structure of the Constitution. The Court held, that the following are the basic features
of Constitution:
(i) Limited power of Parliament to amend the Constitution
(ii) Harmony and balance between fundamental rights and directive
principles.
(iii) Fundamental rights in certain cases.
(iv) Power of judicial review in certain cases.
The judgment thus makes it clear that it is the Constitution that is supreme in
India, and not the Parliament. Parliament is a creature of the Constitution and
derives its power from Constitution, including the power to amend under Art. 368.
Laws passed by Parliament can be declared ultra vires the Constitution. There is a
clear distinction between the ordinary legislative power and the constituent power
(amending power) of Parliament. The amending power, therefore, cannot be
exercised in such a way so as to subvert or abrogate the Constitution. The court,
however, held that doctrine of basic structure is to be applied only in judging the
validity of amendments to the Constitution and it does not apply for judging the
validity of ordinary laws made by legislatures.
(b) Sec. 4 of 42nd Amendment amended Article 31-C, which further widened
the scope of Art. 31-C so as to cover all directive principles. It

89 Parliament’s power to amend the Constitution cannot be equated with the


power of the Constituent Assembly to frame a Constitution. Discuss. [/.AS.- 99]
r

Amendment of the Constitution 289


substituted for the words “directives contained in Art. 39(b) and
(c) ” the following words “all or any of the principles laid down in Part
IV."
Art. 31-C is meant for saving of certain laws for giving effect to certain directive
principles. This Article was first amended by 25th Amendment, 1971, which gave
primacy to directive principles in Art. 39 (b) and (c) over fundamental rights under
Arts. 14 and 19, as such laws for giving effect to directives cannot be challenged on
the ground that they infringe Arts. 14 and 19. It further provided that any law which
contained a declaration that it was put on statute book for giving effect to such policy
could not be called into question in any court that the new law did not give effect to
such policy. Thus, judicial review barred completely.
While the 25th Amendment, 1971 gave primacy to directives contained in Art.
39(b) and (c) only over the fundamental rights under Arts. 14, 19 and 31, the 42nd
Amendment gave precedence to all directive principles over fundamental rights
under Arts. 14, 19 and 31.
But, the Supreme Court by 4:1 majority held Art. 31-C (amended by 42nd
Amendment) unconstitutional on the ground that it destroys the “basic features" of
Constitution by totally excluding from challenge to any laws giving effect to directive
principles, even if they are inconsistent with rights under Arts. 14 and 19. The Indian
Constitution is founded on the bed-rock of the balance between Part III
(Fundamental Rights) and Part IV (Directive Principles). To give ‘absolute’
supremacy or primacy to one over the other is to disturb the harmony of the
Constitution. The harmony and balance between the two is an essential feature of
the basic structure of Constitution. The Directive Principles prescribes the goal to be
attained and the Fundamental Rights lay down the means by which the goal is to be
achieved. Thus, there is no conflict between the two, and they are meant to
supplement each other. The Court, however, held that Art. 31-C as originally
introduced by 25th Amendment is constitutionally valid.
Power of Judicial Review90: A Basic Feature

Leading Case: L. CHANDRA KUMAR v UNION OF INDIA (AIR 1997


SC 1125)
In this case the impact of Arts. 323-A and 323-B (under which the Tribunals
were created) on the power of the judicial review and superintendence of
the High Courts (under Art. 226) and of Supreme Court (under Art.32) was
in question. The clause 2(d) of Art.323-A and clause 3(d) of Art.323-B to the
extent they exclude the jurisdiction of the High Court and the Supreme
Court were held to be unconstitutional. It was held that the jurisdiction
conferred upon the High Courts under Arts.226/227 and upon the Supreme
Court under Art.32 is part of the inviolable basic structure of the Constitution
of India. While this jurisdiction cannot be ousted, other courts and Tribunals

90 Write a short note on: Meaning and basis of Judicial Review. [L C.II- 2006]
‘Judicial review is one of the basic features of the Constitution'? Do you agree
with this view? Cite relevant case laws [I.A.S.-2008]
XVI
Constitutional Law o f India- II

may perform a supplemental - as opposed to a substitutional - role in


discharging the powers conferred by Arts.226/227 and 32 of the
Constitution.
The Supreme Court held that "judicial review is a basic and essential
feature of the Constitution.” The Court observed as follows
(i) The origin of the power of judicial review of legislative action
may well be traced to the classic enunciation of the principle
by Chief Justice John Marshall of the US Supreme Court in
Marbury v Madison. So when the Framers of our Constitution
set about their monumental task, they were well aware that
the principle that courts possess the power to invalidate duly-
enacted legislations had already acquired a history of nearly
a century and a half.
(ii) Broadly speaking, judicial review in India comprises three
aspects: judicial review of legislative action, judicial review of
judicial decisions and judicial review of administrative action.
The judges of the superior courts have to ensure that the
balance of power envisaged by the Constitution is maintained
and that the legislature and the executive do not, in the
discharge of their functions, transgress constitutional
limitations. It is equally their duty to oversee that the judicial
decisions rendered by those who man the subordinate courts
and tribunals do not fall foul of strict standards of legal
correctness and judicial independence.
Amendment of the Constitution 291
(iii) The Constitution of India while conferring power of judicial
review of legislative action upon the higher judiciary,
incorporated important safeguards. An analysis of the
manner in which the Framers of our Constitution
incorporated provisions relating to the judiciary would
indicate that they were very greatly concerned with
securing the independence of the judiciary. These attempts
were directed at ensuring that the judiciary would be
capable of effectively discharging its wide powers of
judicial review.
(iv) The statement of Dr. Ambedkar in the Drafting Committee
of the Constituent Assembly that Art.32 was the “most
important” article and that “it is the very soul of the
Constitution and the very heart of it" has been specifically
reiterated in several Supreme Court’s decisions. So also
the power of constitutional courts in India in respect of
judicial review of legislative action has been affirmed in
several decisions.
(v) The Supreme Court has always considered the power of
judicial review vested in the High Courts and in the
Supreme Court under Arts. 226 and 32 respectively,
enabling legislative action to be subjected to the scrutiny of
superior courts, to be integral to our Constitutional scheme.
While several judgments have made specific references to
this aspect [Beg, J. and Khanna, J. in Keshavananda
Bharati case, Chandrachud, CJ. and Bhagwati, J. in
Minerva Mills', etc.] the rest have made general
observations highlighting the significance of this feature.
(vi) Therefore, the power of judicial review is an integral and
essential feature of the Constitution, constituting part of its
basic structure. Ordinarily, therefore, the power of High
Courts and the Supreme Court to test the constitutional
validity of legislations can never be ousted or excluded.
Further, the power vested in the High Courts to “exercise judicial
superintendence over the decisions of all courts arid tribunals within their
respective jurisdictions” is also part of the basic structure of the
Constitution.
This is because a situation where the High Courts are divested of all
other judicial functions apart from that of constitutional interpretation is
equally to be avoided ]

Other Basic Features of the Constitution


In B.R. Kapoor v State of T.N. (AIR 2001 SC 3435), the “Supremacy of the
Constitution” has been reiterated to be a basic feature of the Constitution. The
court ruled that the mandate of the people could not overrule the provisions of the
Constitution.
292 Constitutional Law o f India- II

!n People’s Union for Civil Liberties v UOI AIR 2003 SC 2363, the “Republican
and democratic form of Government” and “free and fair election” have been
reiterated to be basic features of the Constitution, Similarly, held in T.N. Seshan v
UOI (1995) 4 SCC 611.
In Special Reference No. 1 of 2002 (AIR 2003 SC 87), the apex court held that
“democracy” is a part of the basic structure of the Constitution and periodical, free
and fair election is the substratum of democracy, if there is no free and fair periodic
election, it is end of democracy. Whether any particular brand or system of
government by itself, has this attribute of a basic feature, as long as the essential
characteristics that entitle a system of government to be called democratic are
otherwise satisfied is not necessary to be gone into.
In State of Bihar v Bal Mukund Sah (AIR 2000 SC 12S6), it was held that the
concepts of “Separation of powers between the legislature, the executive and the
judiciary” and “Independent Judiciary" are now elevated to the level of basic structure
of the Constitution. In this case, the issue related to recruitment to District Judiciary
and to subordinate judiciary. It was held that the High Court could get consulted by
the Governor for forming appropriate rules governing recruitment, etc. under Arts.
233-234 of the Constitution. But so long as it is not done, the Legislature cannot, by
an indirect method, completely bypassing the High Court and exercising its
legislative power, circumvent and cut across the very scheme of recruitment and
appointment as envisaged by £he makers of the Constitution.
In Indra Sawhney v UOI (AIR 2000 SC 498), “Principle of equality” was
reiterated to be a basic structure of the Constitution. It was held that Parliament and
State Legislatures cannot transgress the feature of the Constitution, namely, the
principle of equality enshrined in Art. 14 of which Art. 16(1) is a facet. Whether
creamy layer is not excluded or whether forward castes get included in the list of
backward classes, the position will be the same, namely, that there will be a breach
not only of Art. 14 but of the basic structure of the Constitution.

Doctrine of Basic Features as it Stands Today91


After the re-affirmation and extension of the applicability of the doctrine of basic
structure .or features in the Minerva Mills case, it is now evident that so long as the
decision in Keshavariand’s case is not overturned by another Full Bench of the
Supreme Court .(which may come only as an extraordinary event), any amendment
of the Constitution is liable to be interfered with by the court on the ground that it
affects one or other of the basic features of the Constitution.
One post-Keshavananda development of the doctrine is that the court has
declined to foreclose the list of the basic features as suggested by different judges in
the Keshavananda case. In Indira Gandhi's case, it has been observed that the claim
of any particular feature of the Constitution to be a ‘basic feature’ would be
determined by the court in each case that comes before it So far, quite a multitude of
features have been acknowledged as ‘basic’ by different judges, individually, in
different cases, though there is no consensus as regards each of them, in particular.

91 “The doctrine of basic structure has established judicial supermacy in the area
of constitutional amendment”. Examine. [/.AS.-95]
Amendment of the Constitution 293
Conflict between Different Basic Features
Sometimes, an amendment may indirectly affect a basic feature of the Constitution
while promoting or protecting another basic feature of the Constitution. The following
such amendments have been held not to destroy or damage the basic structure of
the Constitution:
(i) The insertion of Art. 31A by the Constitution (1st Amendment) Act, 1951
(relating to agrarian reforms), because, instead of damaging the principle
of equality, these amendments strengthen the basic feature, by removing
inequalities in the matter of agricultural holdings (Waman v UOI AIR 1981
SC 271).
(ii) Even though Parliamentary democracy is a basic feature of the
Constitution, the rights and immunities provided to the members of
Parliament under Art 105(2) cannot be elevated into the status of
fundamental rights or basic features so as to invalidate a Constitutional
Amendment (by inserting the Tenth Schedule) for the purpose of
condemning defection (Kihota Hollohon v Zachilhu AIR 1993 SC 412).
(iii) The amendment of Art. 334 by the 45th Amendment Act, 1978, extending
the reservation for SCs, STs, and Anglo-Indians in the Legislatures
(Vicihtra v UOI AIR 1982 Raj. 297).
(iv) The insertion of Art. 323A (by the 42nd Amendment Act, 1976) which takes
away the jurisdiction of the High Courts and the Civil Courts to decide
'service matters', because that jurisdiction of the Courts is replaced by that
of an Administrative Tribunal which is “a complete substitute of the High
Court” being competent to decide any question and grant any relief which
the High Court could (Sampat v UOI AIR 1987 SC 386).
The addendum of Sampat to the doctrine of basic features is that the basic feature of
‘judicial review’ is not violated by an amendment of the Constitution which, instead of
totally excluding judicial review, substitutes “an effective alternative institutional
mechanism or authority for judicial review”, i.e , provides for the setting up of a
Tribunal with similar powers as that of the courts.92
(v) When a fundamental right itself is taken away by an amendment of the
Constitution, the Amendment Act cannot be challenged on the ground that
it has abridged judicial review - a basic feature of the Constitution
{Shankari Prasad v UOI AIR 1951 SC 458).
Vagueness and Uncertainty93
The doctrine of basic structure is criticised ort various grounds, for instance, that it lays
down a vague and uncertain test. Can anything be called ‘basic’ which is not prone to any
definite definition and even the creator of which is not sure about its contour? Further, the
basic rational behind an amending provision in any Constitution is to provide an
opportunity to the future generation to make suitable adjustments in it and thus, bypass
the fear of revolt /constitutional breakdown, if this is the position then how can it be
assumed that certain provisions (i.e. basic features) of the Constitution would never

92 D.D. Basu, Shorter Constitution of India, 12th Edn.


93 “The amending power of Parliament should not be subjected to the vague and
uncertain doctrine of basic structure.” Comment. [I.A.S.-97]
294 Constitutional Law o f India- II

require amendment? For instance, in recent times, in India, there is a talk about the
‘Parliamentary to Presidential system’, because of the decline in standards of
“Parliamentary democracy" (a basic feature). The question thus arises: if a basic feature
such as the Parliamentary system of Government may be replaced by the Presidential
system?
Further, what would happen if two or more basic features come in conflict? In
Turkey, ‘secularism’ and ‘democracy’ came in conflict. The conflict was resolved by
banning a fundamentalist party even though that enjoyed the support of more than 3/4th of
the population.
Nevertheless, the doctrine of basic features will act as a safety-valve against the
arbitrary use of the amending power by Parliament (Sikri, C.J. in Keshvananda case).

Replacement of the Existing Constitution


Three modes have been suggested to replace the existing Constitution or amend the
basic features: (a) Revolution (b) Parliament converting itself into a Constituent Assembly
(c) Referendum. The ‘Revolution’ is a very radical measure. In fact, it was to avoid
revolution that a provision for amendment is included in a written Constitution (Khanna J.
in Keshavananda case).
A ‘Constituent Assembly’ could be set up by amending Art. 368 itself. But the
majority in Minerva Mills case held that the limited nature of the amending power in Art.
368 is itself one of the basic features of the existing Constitution so that Parliament cannot
enlarge its own powers by making itself a new Constituent Assembly. Any such
amendment of Art. 368 or ordinary law made by Parliament would be unconstitutional.
As regards ‘Referendum’, the decision in Minerva Mills case would not stand in the
way, inasmuch as a referendum to the verdict of the people would be no enlargement of
the amending power of Parliament itself. Thus, referendum or public opinion could be
used as a mode of amending the Constitution itself or its basic features. “It is not the
Constitution itself, (rather) the belief that Constitution ought to be obeyed that is more
important” (Keisen).
Amendment of the Constitution 295
12
Judicial Review of Legislations included in Ninth Schedule
(Whether a law declared void by a Court may be included in the 9 th Schedule)
In Waman Rao v UOI (AIR 1981 SC 271), the court had clarified that the doctrine
of basic features will apply to Amendment Acts passed subsequent to 24-4-1973/
(when Keshavanand Bharati’s judgment was delivered) i.e. it will apply
prospectively and not retrospectively to earlier legislation.
The question whether a law which has been found by a court as
abrogating or abridging a fundamental right is saved by putting it in the Ninth
Schedule was referred to a larger bench in the below-discussed case.

LEADING CASE: I.R. COELHO (DEAD) BY LRS v STATE OF TAMIL


NADU (AIR 2007 SC 861)

In this landmark judgment, a 9-Judge Constitution Bench headed by


Chief Justice Y.K. Sabharwa!, held that any law placed in the Ninth
Schedule of the Constitution after April 24, 1973 (when Keshavanand
Bharati’s judgment was delivered) will be open to challenge. The court
observed that even though an Act is put in the Ninth Schedule by a
constitutional amendment, its provisions would be open to challenge on
the ground that they destroy or damage the basic feature or structure of
the Constitution, for instance, if the fundamental rights are taken away
or abrogated. Further, the power of judicial review which forms integral
part of basic structure cannot be abrogated by any Act. The Parliament
cannot, by inclusion in the Ninth Schedule, grant immunity and exclude
examination by the Court after the enunciation of the doctrine of basic
structure.
Parliament has the power to amend Part III. But if the amending law
destroys the basic structure it becomes invalid. Even if the law is
inserted
in the Ninth Schedule the result is the same. It remains void. The bench clarifying the
impact of Keshavanand pronounced that the doctrine of basic structure has become
an axiom. It is premised on the basis that every invasion of certain fundamental
rights needs to be justified. The invasion attracts the Basic Structure doctrine.
The Ninth Schedule, which was introduced through Art. 31-B by the First
Constitution (Amendment) Act, 1951, aimed to save land reform laws enacted by
various States from being challenged in the court. Later on, however, it became an
omnibus and every kind of law whether it related to elections, mines and minerals,
industrial relations, requisition of property, monopolies, etc. were included in it. No
principle underlies this selection of laws under the Ninth Schedule. In the present
case, the petitioners had challenged the validity of the various Central and State laws
put in the Ninth Schedule including the Tamil Nadu Reservation Act. The T.N. Act
provided for the reservation of 69% of seats in government jobs for the OBCs. The
said Act was added in the Ninth Schedule because of the Supreme Court’s ruling in
the Mandal case that overall reservation cannot exceed 50%.
The Apex Court said that the validity of any Ninth Schedule law has been
upheld by this court and it would not be open to challenge it again. But, if a law is
XVI
Constitutional Law o f India- II

held to be violative of fundamental rights, incorporated in Ninth Schedule after the


date of judgment in Keshavanand Bharati case, such a violation would be open to
challenge on the ground that it destroys or damages the basic structure of
Constitution. The court further said that this court is duty bound to uphold
constitutional values and enforce constitutional limitation. A constitutional
amendment is permissible subject to the limitation of the doctrine of basic structure.
The court held: The constitutional validity of the Ninth Schedule Laws on the
touchstone of basic structure doctrine can be adjudged by applying the direct impact
and effect test, i.e., rights test, which means the form of an amendment is not the
relevant factor, but the consequence thereof would be determinative factor. The
court concluded:
(i) A law that abrogates or abridges rights guaranteed by Part III of the
Constitution may violate the basic structure doctrine or it may not. If
former is the consequence of iaw, whether by amendment of any
Article of Part III or by an insertion in the Ninth Schedule, such law will
have to be invalidated in exercise of judicial review power of the Court.
The validity or invalidity would be tested on the principles laid down in
this judgment.
(ii) The majority judgment in Kesavananda Bharati’s case read with Indira
Gandhi’s case, requires the validity of each new constitutional
amendment to be. judged on its own merits. The actual effect and
impact of the law on the rights guaranteed under Part III has to be
taken into account for determining whether or not it destroys the basic
structure. The impact test would determine the validity of the
challenge.
(iii) All amendments to the Constitution made on or after 24th April, 1973
by which the Ninth Schedule is amended by inclusion of various laws
therein shall have to be tested on the touchstone of the basic or
essential features of the Constitution as reflected in Article 21 read
with Article 14, Article 19, and the principles underlying them. To put it
differently even though an Act is put in the Ninth Schedule by a
constitutional amendment, its provisions would be open to attack on
the ground that they destroy or damage the basic structure if the
fundamental right or rights taken away or abrogated pertains or pertain
to the basic structure.
(iv) Justification for conferring protection, not blanket protection, on the
laws included in the Ninth Schedule by Constitutional Amendments
shall be a matter of Constitutional adjudication by examining the
nature and extent of infr action of a Fundamental Right by a statute,
sought to be constitutionally protected, and on the touchstone of the
basic structure doctrine as reflected in Article 21 read with Article 14
and Article 19 by application of the “rights test” and the “essence of the
right" test taking the synoptic view of the Articles in Part lil as held in
Indira Gandhi's case. Applying the above tests to the Ninth Schedule
laws, if the infraction affects the basic structure then such a law(s) will
not get the protection of the Ninth Schedule.
(v) If the validity of any Ninth Schedule law has already been upheld by
Amendment of the Constitution 297
this Court, it would not be open to challenge such law again on the
principles declared by this judgment. However, if a law held to be
violative of any rights in Part
III is subsequently incorporated in the Ninth Schedule after 24th
April, 1973, such a violation/infraction shall be open to challenge on
the ground that it destroys or damages the basic structure as indicated
in Article 21 read with Article
14, Article 19 and the principles underlying thereunder.
(vi) Action taken and transactions finalized as a result of the impugned
Acts shall not be open to challenge. We answer the reference in the
above terms and direct that the petitions/appeals be now placed for
hearing before a Three- judge Bench for decision in accordance with
the principles laid down herein.]
Comments - In Ashoka Kumar Thakur v UOI (2008) 6 SCC 1. the court observed
(FterBhandari, J): To determine if a constitutional amendment violates the basic
structure, a Two-Step 'Effect Test1 as laid down in I.R. Coelho case (also known as
the Impact or Rights Test) is to be applied on a case-by-case
basis. Step One requires us to ask if the impugned amendment affects a
facet of the basic structure. If it does, then Step Two requires us to ask if
the effect on the facet of the basic structure is to such an extent that the
facet’s original identity has been altered. Applying the Effect Test amounts
to saying that the form of a constitutional amendment is irrelevant: it is the
consequence thereof that matters. A total deprivation of fundamental rights,
even in one limited area, may amount to abrogation of the basic structure.
In Ashoka Kumar Thakur case, the court further observed: The
“width test” asks if an amendment is so wide that in effect (actual or
potential), it goes beyond Parliament’s amending power. What is relevant is
that the real consequences can be taken into account while judging the
width of the power. The court cannot ignore consequences to which a
particular construction would lead. Thus, anticipatory challenge to an
enabling constitutional amendment on the ground that amendment
concerned created the potential for a violation of the basic structure is valid.
Constitutional amendments by their very nature are often enabling
provisions. If they clear the way for future legislation that would in fact
violate the basic structure, the court need not wait for a potential violation to
become an actual one. It can strike down the entire amendment ab initio.]

SCOPE OF AMENDING POWER OF PARLIAMENT94

94 Discuss the
limits of the amending power of the Parliament. [/.AS.-93]
Write a short note on: Scope of amending power of Parliament under Art. 368.
[C L.C.-96]
XVI
Constitutional Law o f India- II

Article 368 as it Stands Today


The Constitution, though expressly confers amending power on the Parliament, but it
is the Supreme Court, which is to finally interpret the scope of such power and to
spell out the limitations, if any, on such amending power. At present, any part of the
Constitution can be amended under Art. 368, the only limitations being the
procedural limitations (as in Art. 368 itself), and, the substantive limitations ('basic
features’).
(A) Procedural Limitation - The power conferred by Art. 368 is subject to the
procedure laid down in the Article [cl. (1)]. The court would be competent to
strike down an Amendment Act which did not comply with the procedural
requirements of Art. 368, such as - (i) that it has not been passed by the
special majority provided in Art. 368, (ii) that it has not been ratified by the
Legislatures of the specified number of States.
In Kihota Hollohon v Zachillu (AIR 1S93 SC 412), Para 7 of the Tenth Schedule to
the Constitution inserted by the Constitution (52nd Amendment) Act, 1985, excluding
the jurisdiction of all courts including the Supreme Court under Art. 136 and the High
Courts under Arts. 226 and 227, on the question of disqualification on the ground of
defection, was struck down as unconstitutional since it was not enacted complying
with the requirement of Proviso to Art. 368 (2) which required ratification of the
amendment by at least half of the State Legislatures.
It may be noted that while exercising the power under Art. 368, Parliament
would not be subject to the limitations which curb its legislative power to make laws
under Arts. 245-246, because the amending power conferred by Art.368 is
‘constituent power’ (Sasanka v UOI AIR 1981 SC 522).
(B) Substantive Limitation - In Keshavanand’s case, the Supreme Court, for the
first time, exercised the power of judicial review on the following
substantive grounds -
(i) That, by virtue of the power to ‘amend’ conferred by Art. 368,
Parliament could not alter the ‘basic structure’ or ‘fundamental
features’ of the Constitution because the word ‘amend’ implied that
even after amendment, by way of addition, alteration or repeal of
some of its provisions, the identity of the original Constitution must
remain.
(ii) The power to amend is vested by Art. 368 in Parliament (except where
ratification by State Legislatures is required). Hence, Parliament
cannot, directly or indirectly, abdicate or delegate that power to some
other body; if it does, the court would strike down that Constitution
Amendment Act as invalid.
In Keshavanand’s case, that latter part of Art. 31C was struck down by the majority,
on the ground that it not only authorised the legislature to make a law violative of
Arts. 14,19 and 31, but also conferred on it the power to make it immune from attack

Every legislation amending the Constitution of India is subject to judicial review


and the same would be unconstitutional if it goes beyond the Parliament’s
constituent power. Do you agree with this observation? Discuss the scope and
limits of the constituent power of the Parliament. [L.C.I- 2007]
Amendment of the Constitution 299
on that ground by itself inserting in that law a declaration. This constituted a
delegation of an effective power to amend the Constitution to the Legislature.

F URTHER Q UESTIONS

Q.1 Discuss the principles and philosophy underlying the doctrine of 'basic
structure’ as enunciated in Keshavananda Bharati's case. Which
features of the Constitution have been so far been declared as part of
its basic structure by the Supreme Court?
[C.L.C.-95/97]

Discuss the basic feature doctrine in light of recent pronouncements.


[L.C.II- 2007]
(a) Art. 356(1) provides for imposition of President’s rule in any State “If
the President, on receipt of a report from Governor of a State or
otherwise is satisfied that a situation has arisen in which the
government of the State cannot be carried on in accordance with
constitutional provisions ...’’

The Constitution Amendment Act, 1993, amends the above clause by


inserting words "or is likely to arise” after the words “a situation has
arisen.” Cl. (2) of the Amendment Act further provides that “no
Proclamation issued under this Article shall be called in question in
any court on any ground whatsoever.” Discuss the constitutional
validity of the Amendment Act with reference to the 'basic structure’
doctrine. [C.LC.-S3]

(b) Art. 13(2) says that State shall not make any law which takes away or
abridges the rights conferred by Part III (Fundamental Rights) and any
law made in contravention of it be void. By an Amendment, cl.
(5) inserted in Art. 13: 'Nothing in this Article shall apply to any law
enacted for giving effect to the Directive Principles of State Policy.'
The Amendment Act further provides that validity of this amendment
shall not be called in question in any court on any ground whatsoever.
Discuss the validity of the above amendment. [C.L.C.-
94]

(c) In view of the new economic policy adopted by the Government of


India since 1991 which seeks to promote market economy and
capitalist path of economic development, Parliament wants to amend
the Constitution so as to delete the word ‘Socialist’ from the Preamble.

As an advocate advise the Union Government on the constitutional


validity of the proposed amendments in the light of the principles laid
down in Kcshavananda Bharati case. [C.L.C.-97]
A.1 Doctrine of Basic Structure
In Raghunatharo Ganpathrao v UOI (AIR 19S3 SC 1267), it was held that an
amendment which is a change/alteration is only for the purpose of making the
Constitution more perfect, effective and meaningful. It should not result in abrogation/
XVI
Constitutional Law o f India- II

destruction of its basic structure or loss of its original entity or character and render it
unworkable.
It may be noted that every integral part of the Constitution is not necessarily
an essential feature of the Constitution. Both are totally distinct and qualitatively
different concepts. Therefore, destroying an integral part of the Constitution did not
amount to destroying the basic structure or framework of the Constitution (Madhav
Rao Scindia v UOI AIR 1971 SC 530, Raghunathrao Ganpathrao case).
For determining whether a particular feature of the Constitution is part of the
basic structure, it nas to be examined in each individual case keeping in mind the
scheme of the Constitution, its objects and purpose and the integrity of the
Constitution as a fundamental instrument for the country's governance [Ashoka
Kumar Thakur v UOI (2008) 6 SCC 1j.
Amendment of the Constitution 301
Various features held to be part of 'basic structure' are.
(1) Supremacy of the Constitution (K Bharati case; B.R. Kapoor v State of T.N).
(2) Republican, and Democratic form of government - free and fair elections (K.
Bharat/ case, Indira Gandhi v Raj Narain\ PUCL v UOI\ T.N. Seshan v UOI,
Special Reference No. 1 of 2002). Also, Parliamentary system of Government.
(3) Sovereignty (unity and integrity) of country (K. Bharati case, R.C. Poudyal v
UOI AIR 1993 SC 1804).
(4) Federalism and Securalism (S.R. Bommai case).
(5) Separation of Powers between Legislature, Executive and Judiciary (K. Bharati
case; State of Bihar v Bal Mukund Sah).
(6) Rule of law and Judicial Review [Indira Gandhi v Raj Narain, S.R. Bommai
case, P. Sambhamurthy v State of A P. AIR 1987 SC 663, L. Chandra Kumar
v UOI AIR 1997 SC 1125],
(7) Independence of judiciary (Supreme Court’s Advocates-on-Record Asscn. v
UOI AIR 1994 SC 268; State of Bihar v Bal Mukund Sah)' Judicial
Superintendence - Power of High Courts over decisions of all courts/ tribunals
within their respective jurisdictions (L Chandra Kumar v UOI AIR 1997 SC
1125); Powers of the Supreme Court under Arts. 32, 136, 141, 142 (Delhi
J.S.A. v State of Gujarat MR 1991 SC 2176).
(8) Jurisdiction of Supreme Court under Art. 32 (Indira Gandhi’s case).
(9) Harmony and balance between Fundamental Rights and Directive Principles
(Minerva Mill’s case). The ‘essence’ of FundamorrtaJJ^ights (Waman v UOI
MR 1981 SC 271).
(10) Fundamental Rights in certain cases (Minerva Mill’s case); Right to equality
(Indira Gandhi’s case; Indra Sawhney v UOI): Not every feature of equality,
but the quintessence of equal justice (Raghunathrao’s case).
(11) The concept of Soc:al and Economic Justice - to built a welfare State, Part
IV in toto (Bhim v UOI AIR 1981 SC 234).
(12) Limited power of Parliament to amend the Constitution (Minerva Mill's case).
(13) Un-amendability of ‘basic structure’ (Minerva Mill’s case, R.C. Poudyal’s case).
(14) Objectives mentioned in the Preamble to the Constitution (Indira Gandhi's
case).
(15) Freedom and dignity of the individual (Keshavananda's case, Bommai case).
(16) Effective access to justice (Central Coal Fields v Jaiswal Coal Co. AIR 1980
SC 2125).
It may be noted that ‘Right to Property' under Art.300-A is not a basic feature of the
Constitution [Jilubhai Nanbhai v State of Gujarat (1995) Supp.(1) SCC 596].
Decision of the cases in question
(a) Insertion of words “or is likely to arise” poses a great danger to the
Federalism (a basic feature of Constitution). Similarly, judicial review is a
part of the basic structure. Thus, the amendment is invalid on both counts.
(b) Harmony and balance between fundamental rights and directive principles
is part of the basic structure of the Constitution. The amendment insofar as
it gives primacy to directive principles over fundamental rights is invalid. It
may be noted that directives contained in Art. 39(b) and (c) (relating to
XVI
Constitutional Law o f India- II

nationalisation of resources) have been given precedence over the


fundamental rights (K. Bharati case).
The amendment is invalid on the count of judicial review also.
(c) Objectives mentioned in the Preamble to the Constitution are part of the
basic structure of the Constitution. Thus the amendment seeking to delete
the word 'Socialist' from the Preamble is constitutionally invalid.
REFERENCES
I. A. V.Dicey : The Law of the Constitution (10th ed.).

2. D.D. Basu : Commentary on the Constitution of India.


3. Bare Act : The Constitution of India, 1950.
4. H.M. Seervai : Constitutional Law of India.
5. J.C. Johari : The Constitution of India - A Politico-Legal Study.
6. J.N. Pandey Constitutional Law of India (2008 ed.).
7. B.K. Sharma : Introduction to the Constitution of India.
8. M.P. Jain : Indian Constitutional Law (5th ed., 2003).
9. Narender Kumar : Constitutional Law of India (1st ed., 1997).
10 S.C. Kashyap : Our Constitution (National Book Trust).
.
11 D.D. Basu : Shorter Constitution of India (14th ed., 2009).
.
12 S.N. Ray : Judicial Review and Fundamental Rights (1974
. ed.).
13 Subba Rao : Lectures on Constitutional Law.
.
14 T.K. Tope : Constitutional Law of India.
.
15 M.P. Singh : V.N. Shukla's Constitution of India (11th ed.,
. 2008).
16 S'. Malik (Ed.) : Supreme Court Yearly Digests (1994-2008).
.
17 Ashok K. Jain / : Landmark Judgments of 1997-1998.
.
18 N.H. Jhabvaia : The Constitution of India.
.
19 Faculty of Law, Delhi Univ.: Cases and Materials on 'Constitutional Law-II.'
.
20 Question Papers Referred : Delhi and Other Universities, IAS Exam.
.
The Government plans to enact a law which prohibits certain kinds of
advertisements, particularly those which make indecent depiction of
women. Discuss the constitutional validity of such legislation vis-a-vis Art.
19(1)(a).
[I.A.S.-2001]
10. A question based on the facts of this case.
(1) Subject to public order, morality and health and to the other provisions
of Part III of Constitution, all persons are equally entitled to freedom of
conscience and the right freely to profess, practice and propagate
Constitutional Law o f India- II vii

religion.
(2) Nothing in this Article shall affect the operation of any existing law or
prevent the State from making any law -
(a) regulating or restricting any economic, financial, political, or other
secular activity which may be associated with religious practice;
(b) providing for social welfare and reform or the throwing open of
Hindu. religious institutions of a public character to all classes and
sections of Hindus.
Explanation I - The wearing and carrying of Kirpans shall be deemed to be
included ln the profession of Sikh religion.
Explanation II- The expression “Hindu” includes Jain, Sikhs and Buddhists.
25
(1) guarantees to every person, and not merely to the citizens of India, the
eed
°m of conscience, etc.
12. Write a short note on: Validity of legislations included in Ninth Schedule
affecting basic structure. [L.C.II-
2007]
The Parliament enacts the "Restrictions on Press Act, 2008” imposing
certain restrictions on page number, price, new publications and
advertisements. The legislation is included in Ninth Schedule of the
Constitution by an amendment. Discuss the validity of the legislation.
[D.U.-
2008]
Assuming that the Parliament, under Art. 368, passes the following
constitutional amendments:
(i) Religion is a matter of personal faith. Its propagation in any form is
strictly prohibited.
(ii) Art. 16(4) of the Constitution empowering the State to make
reservation in services under the State in favour of backward class of
citizens is completely deleted.
Discuss whether these amendments are likely to be held valid. [I.A S.-2000]

You might also like